You are on page 1of 194

@

ACP&MKSA
,.ll

..,:.,1'

Medical Knowledge Self-Assessment Program@

Rheumatology

t
rACP American College of Physicians@
Leading lnternal Medicine, lmproving Lives
Welcome to the RheumatologY
Section of MKSAP 191
In these pages, you will flnd updated information on approaches to the patient with rheumatologic disease, principles of
therapeutics, rheumatoid arthritis, osteoarthritis, spondyloarthritis, systemic lupus erythematosus, systemic sclerosis,
infectious arthritis, and other clinical challenges. A11 ofthese topics are uniquely focused on the needs ofgeneralists and
subspecialists outside of rheumatologz.

MKSAP 19 strives to provide the clinical knowledge its learners need to navigate their longitudinal learning paths. MKSAP 19's
core content contains essential, newly researched information in 11 subspecialty areas of internal medicine-created by
dozens of expert generalists and subspecialists. Development of MKSAP 19's syllabus and its 1200 all new peer-reviewed,
psychometrically validated multiple-choice questions (MCQs) has been informed by ABIM Certification and Maintenance
of Certification (MOC) requirements, emerging internal medicine knowledge, and our learners' feedback. MKSAP 19 contin-
ues to include HighValue Core (HVC) recommendations and MCQs, based on the concept of balancing clinical benefit with
costs and harms. Hospital-based internists can continue to trust that MKSAP's comprehensive hospitalist content, integrated
throughout the syllabus, and hospitalist focused MCQs, specially designated with the blue hospitalist icon (El), continue to
align with the ABIM's Focused Practice in Hospital Medicine MOC exam blueprint and enhance learning for hospital-based
practitioners.

More than ever before, MKSAP 19 Digital focuses on individualized learning and convenience. In addition to custom quizzes
and interlinked questions and sy'labus sections, MKSAP 19 Digital's new learning dashboard enables users to create a self
directed learning plan, with topic-specific links to resources within MKSAP and ACP Online. Multimedia formats, including
whiteboard animations and clinical videos, will benefit our audiovisual learners, while MKSAP's Earn as You Go CME/MOC
feature now allows subscribers to earn CME/MOC as they answer individual questions. In addition to Extension Questions
and New Info Updates, MKSAP 19 Complete and Complete Green continue to offer Virtual Dx and Flashcards and now offer
brand new enhancements: MKSAP Quick Qs, a set of concise questions mapped to high-frequency/high-importance areas
of the ABIM blueprint mirroring boards-style MCQs, and an embedded digital version of Board Basics for easy access
exam prep.

Language can be imprecise and imperfect, but MKSAP 19's Editors and contributors commit to using language and images
that support ACP's commitment to being an anti-racist organization that supports diversity, equity, and inclusion through-
out health care and health education. ACP also continues to ensure diversity among MKSAP's physician-contributors. When
appropriate, the MKSAP Editors also rely on MKSAP 19 Digital's expanded use of multimedia enhancements, including video
and audio, to explore and more fully explain issues surrounding the presentation of MKSAP 19 clinical content as it relates to
race and ethnicity. MKSAP 19 users are encouraged to contact the Editors at mksap_editors6acponline.org to help us identifiz
opportunities for improvement in this area.

On behalf of the many internists and editorial staff who have helped us create our new edition, we are honored that you have
chosen to use MKSAP 19 to meet your lifelong learning needs.

Sincerely,

Davoren Chick, MD, FACP


Editor-in-Chief
Senior Vice President
Medical Educat ion Division
American College of Physicians
\
I
I

\
\
: Rheumatology

Committee Editor-in-Chief
Michael Pillinger, MD, EACE Section Editor Davoren Chick, MD, FACP
Professor of Medicine and Biochemistry and Molecular Senior Vice President. Medical Education
Pharmacologz American College of Physicians
Division of Rheumatologz Philadelphia. Pennsylvania
NYU Grossman School of Medicine
New York. New York
Senior Deputy Editor
Aryeh M. Abeles, MD Patrick C. Alguire, MD, EACP
Adjunct Associate Professor American College of Physicians
Division ol Rheumatologz Phi ladelphia. Pennsylvan ia
NYU Grossman School of Medicine
New York. New York
Deputy Editor
Gregory C. Gardner, MD, MACP
Janet A. Jokela, MD, MPH, EACP
Gilliland Henderson Professor of Medicine
Professor & Head, Department of Medicine
Division of Rheumatologz
Acting Regional Dean
University of Washington
University of Illinois College of Medicine
Seattle, Washington
Urbana, Illinois
Sharon L. Kolasinski, MD, FACP
Professor of Clinical Medicine
Division of Rheumatologz
Rheumatology Medicine Reviewers
University of Pennsylvania Perelman School of Medicine David Armstrong, DO, FACP
Ph iladelphia, Pen nsylvania Fawad Aslam, MBBS, MSc, FACP
Angelique Collamer, MD, FACP
Susan Faye Kroop, MD
Michael Guma, DO, FACP
Associate Professor of Medicine
Marie Kuchynski, MD
Division of Rheumatologz Ashima Makol. MD
Vanderbilt University Medical Center J. Suzanne Moore, MD, FACP
Nashville, Tennessee
Patricia J. Papadopoulos, MD
Vikas Majithia, MD, MPH, FACP Avis Ware, MD, FACP
Professor of Medicine Andrew L. Wong, MD, FACP
Division Director and Fellowship Program Director
Division of Rheumatologz
Hospital Medicine Rheumatology
University of Mississippi Medical Center
Jackson, Mississippi
Reviewers
Bernard Hildebrand, MD, MA, FACP
Bethany A. Marston, MD Julie Hildebrand, MD
Division Director, Pediatric Rheumatologz Kam A. Newman, MD, FACP
Program Director, Rheumatologz Fellowship Niluka Weerakoon, MD
Associate Professor of Medicine and Pediatrics
Pediatric Rheumatologr and Allergz/lmmunologz and
Rheumatologr Divisions
University of Rochester
Rochester, New York

llt
l

'!

Rheumatology ACP Editorial Staff Aryeh M. Abeles, MD


Consultantship
Suzanne Meyers, Medical Editor, Assessment and Education
Bristol-Myers Squibb, Johnson & Johnson
Programs
Beclg Krumm, Director, Assessment and Education Programs Vikas Majithia, MD, MPH, FACP
Jackie Twomey, Managing Editor, Assessment and Research G rants / Contracts
Education Programs GlaxoSmithKline, Bristol-Myers Squibb, Janssen
Consultonfship
Novartis
ACP PrincipalStaff
Aduisory Board
Davoren Chick, MD, FACP UCB
Senior Vice President, Medicql Educqtion
Bethany Marston, MD
Tabassum Salam, MD, MBA, FACP Re search G rants / Co ntracts
Vice President, Medical Educqtion Rheumatologr Research Foundation
Margaret Wells, EdM Michael Pillinger, MD, FACP
Vice President, Learning Assessment, Accreditation, Reseo rch Grants / Contracts
ctnd Research Hikma Pharmaceuticals, Horizon Pharmaceuticals
Patrick C. Alguire, MD, FACP Consultontships
MKSAP Senior Deputy Editor Horizon Pharmaceuticals, Sobi, Ampel Biosciences

BeckyKrumm Tabassum Salam, MD, MBA, FACP


Director, Assessment and Education Programs Consultantship
Johnson & Johnson
Jackie Twomey
Managing Editor

Iulia Nawrocki Acknowledgments


Digital Con ten f Asso ciate / Edito r The American College of Physicians (ACP) gratefully
acknowledges the special contributions to the development
Linnea Donnarumma
and production of the 19th edition of the Medical
Senior Medical Editor
Knowledge Self-Assessment Program' (Vt<SAp' 19) made
Amanda Cowley by the lollowing people:
Medical Editor
Graphic Design: Barry Moshinski (Director, Graphic
Sandy Crump Services), Raymond DeJohn (Designer, Graphic Services),
Medical Editor Tom Malone (Print/Mail Production Manager, Graphic
Services), Mike Ripca (Technical Administrator, Graphic
Georgette Forgione
Services).
Medicol Editor
P roduction / Systems: Dan Hoffmann (Vice President,
Beth Goldner
Information Technology), Scott Hurd (Manager, Content
MedicalEditor
Systems), Neil Kohl (Senior Architect), and Chris
Suzanne Meyers Patterson (Senior Architect).
liledical Editor
MKSAP 19 Digital: Under the leadership of Steven Spadt
Elise Paxson (Senior Vice President, Information Technology and
MedicalEditor Chief Technology Officer), the development of the dig-
ital version of MKSAP 19 was implemented by ACP's
Chuck Graver
Digital Products and Services Department, directed and
Finance and Operations Administrqtor
led by Brian Sweigard (Vice President, Digital Products
Kimberly Kerns and Services). Other members of the team included Dan
Administratiu e Coordinator Barron (Senior Web Application Developer/Architect),
Callie Cramer (Data Visualization/Web Developer),
Disclosures of relationships with any entity producing, Chris Forrest (Senior Web Application Developer),
marketing, reselling, or distributing health care goods or Kathleen Hoover (Manager, User Interface Design and
services consumed by, or used on, patients. Individuals not Development), Kara Regis (Director, Product Design
listed below have nothing to disclose. and Development), Brad Lord (Senior Web Application

tv
Developer/Architect), and John McKnight (Senior Web r Pass the ABIM Certification Examination
Developer). r Pass the ABIM Maintenance of Certification Examination
i
The College also wishes to acknowledge that many other
persons, too numerous to mention, have contributed to
the production of this program. Without their dedicated Target Audience
efforts, this program would not have been possible. o General internal medicine specialists, including primary
care physicians and hospitalists
o Internal medicine subspecialists who desire to remain up
MKSAP Resource Page to date in internal medicine
. Residents preparing for the ABIM Internal Medicine
The MKSAP Resource Page (wwwacponline.org/mksap19-
Certifi cat ion Examination
resources) provides access to MKSAP 19 online answer
. Physicians engaged in the ABIM Maintenance of
sheets for transcribing answers from the print eclition;
Certification Longitudinal Assessment Option, pre-
access to MKSAP 19 Digital; Board Basics '; information paring for the ABIM Internal Medicine Maintenance ol
on Continuing Medical Education (CME), Maintenance
Certitlcation Examination, or engaged with the ABIM
of Certification (MOC), and international Continuing
Focused Practice in Hospital Medicine program
Professional Development (CPD) and MOC; errata; and
other new information.

Earn CME Credits or MOC Points Online


lnternational MOC/CPD To earn CME credits or to apply for MOC points, MKSAP
Information and instructions on submission ol inter users need to answer at least one of two questions cor-
national MOC/CPD is available by accessing the CME/ rectly (earning a score of at least 50%) and click the Submit
MOC/CPD tab on the left navigation menu of MKSAP 19 CME button. Each single MKSAP 19 self-assessment ques-
Digital. tion qualifies for one quarter of a CME credit hour or ABIM
MOC point.

Continuing Medical Education MKSAP 19 Subscribers can enter their self assessment
question answers and submit for CME/MOC in two
The American College of Physicians is accredited by the
ways:
Accreditation Council for Continuing Medical Education
(ACCME) to provide continuing medical education for 1. Users of MKSAP 19 Complete who pref'er to use their
physicians. print books and a paper answer sheet to study and
record their answers can use the printed answer sheet
The American College of Physicians designates this enduring
at the back of this book to record their answers. The
material, MKSAP 19, for a maximum of 300 AMA PRA
corresponding online answer sheets, which are avail-
Category 1 CreditsrM. Physicians should claim only the
able on the MKSAP 19 Resource Page, may be used to
credit commensurate with the extent of their participation
transcribe answers onto the online answer sheets. Users
in the activity.
may then submit their answers to qualify for CME cred
Up to 24 AMA PRA Category 1 Creditdtu are available from its or MOC points (see below for information on Opting
January 31,2022, to January 31, 2025, for the MKSAP l9 in for MOC). Users who preler to record their answers
Rheumatology section. on a paper answer sheet should save their answer sheet
for future use. Users who study with MKSAP 19 print
can also submit their answers directly within MKSAP 19
Learning Objectives Digital by accessing the self assessment questions dash
The learning objectives of MKSAP 19 are to: board and selecting the preferred subspecialty section
to begin answering questions.
. Close gaps between actual care in your practice and
preferred standards of care, based on best evidence 2. Users of MKSAP l9 Digital can enter their answers within
. Diagnose disease states that are less common and sometimes the digital program by accessing the self-assessment
overlooked or confusing questions dashboard and selecting the preferred sub
. Improve clinical management decisions that affect patient specialty section to begin answering questions and
safety and quality of care clicking the Submit CME button once they qualify for
o Determine when to ref'er patients for care by medical CME and are ready to submit. Learners should keep in
subspecialists, surgeons, and other members olthe health mind their yearly CME and MOC deadlines when deter
care team mining the appropriate time to submit.

v
t

Learners' CME/MOC submission progress will be shown updated clinical care guidelines in developing content,
on the MKSAP 19 Digital CME/MOC/CPD page. when such evidence and guidelines were available' A1l
content underwent review by external peer reviewers not
on the committee to ensure that the material was balanced
Opting in for MOC and unbiased. Contributors' disclosure information can be
found with the Iist of contributors' names and those of ACP
MKSAP 19 users can opt in for simultaneous submission of
principal staff listed in the beginning of this book'
CME and MOC points as they answer self-assessment ques-
tions. To opt in, users will be required to complete a form
requesting their name, date of birth, and ABIM number' Language Reflecti ng Diversity,_Equity,
The MOC Opt in Form will be presented during a user's and lnclusion Within MKSAP 19
first CME submission and needs to be completed only once.
MKSAP 19's Editors and contributors commit to using
Ianguage and images that support ACP's commitment to
ABIM Maintenance of Certification being an anti racist organization that supports diversity'
Successful completion of the CME activity, which includes equiry and inclusion throughout health care and health
participation in the evaluation component, enables the par education. ACP also continues to ensure diversity among
ticipant to earn up to 300 medical knowledge MOC points MKSAP's physician-contributors. When appropriate,
in the ABIM's MOC program. It is the CME activity provider's the MKSAP Editors will also rely on MKSAP 19 Digital's
responsibility to submit participant completion information expanded use of multimedia enhancements, including video
to ACCME for the purpose of granting MOC credit. and audio, to explore and more fully explain issues sur-
rounding the presentation of MKSAP 19 clinical content as it
relates to race and ethnicity. MKSAP 19 users are encouraged
Disclosure Policy to contact the Editors at mksap editors6acponline.org to
It is the policy of the American College of Physicians help us identiSr opportunities for improvement in this area.
(ACP) to ensure balance, independence, objectivity, and \
scientific rigor in all ofits educational activities. To this
end, and consistent with the policies of the ACP and the
Hospital-Based Medici ne
Accreditation Council for Continuing Medical Education For the convenience ofsubscribers who provide care in
(ACCME), contributors to all ACP continuing medical hospital settings, comprehensive hospital-focused content
education activities are required to disclose all relevant aligned with the ABIM Focused Practice in Hospital
financial relationships with any entity producing, mar Medicine blueprint is integrated throughout the syllabus,
keting, re-selling, or distributing health care goods or and self-assessment questions that are specific to the
services consumed by, or used on, patients. Contributors hospital setting are specially designated with the blue
are required to use generic names in the discussion of hospitalist icon (E).
therapeutic options and are required to identify any unap
proved, off label, or investigative use of commercial prod-
ucts or devices. Where a trade name is used, all available
High Value Care Key Points
trade names for the same product type are also included. Key Points in the text that relate to High Value Care con-
If trade name products manufactured by companies with cepts (that is, concepts that discuss balancing clinical
whom contributors have relationships are discussed, benefit with costs and harms) are designated by the HVC
contributors are asked to provide evidence based citations icon [HVC].
in support of the discussion. The information is reviewed
by the committee responsible flor producing this content.
If necessary adiustments to topics or contributors' roles in
Educational Disclaimer
content development are made to balance the discussion. The editors and publisher of MKSAP l9 recognize that the
AII relevant relationships are mitigated. Readers of this development of new material offers many opportunities
content are asked to evaluate it for evidence of commercial for error. Despite our best efforts, some errors may persist
bias and send any relevant comments to mksap editors6r in print. Drug dosage schedules are, we believe, accurate
acponline.org so that future decisions about content and and in accordance with current standards. Readers are
contributors can be made in light of this information. advised, however, to ensure that the recommended dos
ages in MKSAP 19 concur with the information provided
in the product information material. This is especially
Mitigation of Conflicts important in cases of new, infrequently used, or highly
To mitigate all conflicts of interest and influences of vested toxic drugs. Application of the information in MKSAP 19
interests, ACP's content planners used best evidence and remains the professional responsibility of the practitioner

vl
The primary purpose of MKSAP 19 is educational. of MKSAP sold by unauthorized sellers (e.g., Amazon,
Information presented, as well as publications, technol- eBay), with whom ACP has no relationship. We do not
ogies, products, and/or services discussed, is intended to honor third party sales. CME credits and MOC points can
inform subscribers about the knowledge, techniques, and not be awarded to those purchasers who have purchased
experiences of the contributors. A diversity of professional the program from non authorized sellers.
opinion exists, and the views of the contributors are their
own and not those of the ACP. Inclusion of any material
in the program does not constitute endorsement or rec Unauthorized Use of This Book ls
ommendation by the ACP. The ACP does not warrant the Against the Law
safety, reliability, accuracy, completeness, or usefulness of Unauthorized reproduction of this publication is unlaw
and disclaims any and all liability for damages and claims ful. ACP prohibits reproduction of this publication or
that may result from the use of information, publications, any of its parts in any form either for individual use or for
technologies, products, and/or services discussed in this distribution.
program.
ACP will consider granting an individual permission to
reproduce only limited portions of this publication for
Publisher's Information his or her own exclusive use. Send requests in writing to
Copyright ,: 2022 American College of Physicians. A11 MKSAP" Permissions, American College of Physicians, 190 N
rights reserved. Independence Mall West, Philadelphia, PA 19106-1572, or
email your request to mksap editorspacponline.org.
This publication is protected by copyright. No part of
this publication may be reproduced, stored in a retrieval MKSAP 19ISBN: 978 1-938245-7s 6
system, or transmitted in any form or by any means, elec- Rheumatology ISBN: 978 I-938245 87 9
tronic or mechanical, including photocopy, without the Printed in the United States of America.
express consent of the ACP. MKSAP 19 is for individual
use only. Only one account per subscription will be per- For order information in the U.S. or Canada, call
mitted for the purpose of earning CME credits and MOC 800-ACP-1915. In all other countries, call 215-351-2600
points and for other authorized uses of MKSAP 19. (Monday to Friday, 9.4u-5 plr ET). Fax inquiries to
215-351 2799 or email to custserv@acponline.org.

Disclaimer Regarding Direct Purchases


from Online Retailers Errata and Revisions
CME and/or MOC for MKSAP 19 is available only to cus- Errata and Revisions for MKSAP 19 will be available
tomers who purchase the program directly from ACP. ACP through MKSAP 19 Digital at mksaplg.acponline.org as
will not set up MKSAP CME/MOC accounts for purchasers new information becomes known to the editors.

vll
Table of Contents

Approach to the PatientWith Rheumatologic Disease Uricosuric Agents 15


InflammatoryVersus NoninflammatoryPain . .. . . . . . . 1 Pegloticase 15
TheMusculoskeletalExamination ........... 1 Medications and Pregnancy 15

Arthritis .....1 Vaccination and Screening in Immunosuppression 15

Monoarthritis...... .....1 Nonpharmacologic and Nontraditional


Oligoarthritis ...........2 Management 15

Polyarthritis Physical and Occupational Therapy


.....2 15

Soft-TissueAbnormalities..... ......2 Complementary and Alternative Medicine . . . L7

Extra-Articular Manifestations of Rheumatologic RoleofSurgery .... 77

Disease ......2
Constitutionalsymptoms. .......2 Rheumatoid Arthiltis
Skinlnvolvement... ...........2 Pathophysiologr and Risk Factors. t7
Eyelnvolvement... .....2 Genetic Factors 17
InternalOrganlnvolvement.. ........... 2 Environmental Factors 17

LaboratoryStudies ..........2 Infectious Agents. t7


TestsThatMeasurelnflammation ........ 2 Hormones 17

AutoantibodyTests .. ...........5 Diagnosis 1B

ImagingStudies. .....5 ClinicalManifestations. . . . . . 18


Radiography. ...........5 Laboratory Studies 19
\t cT.... .........7 Imagrng Studies. t9
t
t MRI... .........7 Complications and Extra-Articular
Ultrasonography . . . . .7 Manifestations..... 20
I
..........
JointAspiration.... .........7 Joints. . 20

TissueBiopsy ........8 Skin... 20

MentalHealthScreening .....8 Eyes... 20


Lungs . 21.

Heart.. 2l
Principles of Therapeutics
Hematologic. 2t
Overview 8
Blood Vessels 2t
Anti-lnflammatory Agents. ,8
Management 2t
Glucocorticoids . . . . ,8
GeneralConsiderations. . . . . . 2l
NSAIDS ,9
Disease-Modi$zing Antirheumatic Drugs 22
Colchicine ,9
NSAIDS 22
Analgesics and Pain Pathway Modulators ,9
I Acetaminophen. . . . 10
Glucocorticoids.... 22
I

Surgery 23
I Tramadol 10
Pregnancy 23
Serotonin Norepinephrine Reuptake Inhibitors 10
:
Gabapentinoids.... 10
r
Disease-Modi$ring Antirheumatic Drugs. 10 Osteoarthritis
:
Nonbiologic Disease-Modi$ring Pathophysiolory .... 23
!
i
Antirheumatic Drugs 10 Epidemiolory and Risk Factors 23
i
I
Biologic Disease-Modifying Antirheumatic Drugs t2 Classification 24

Urate-Lowering Therapy 13 PrimaryOsteoarthritis. ... . 24


i
I Allopurinol. 13 Secondary Osteoarthritis 25
t Febuxostat t4 Diffuse Idiopathic Skeletal Hyperostosis 25

tx
Diagnosis 25 Diagnosis 46
ClinicalManifestations. . . . . . .. 25 General Considerations. . . . . 46
Laboratory and Imaging Studies 27 Laboratory Studies 46
Differential Diagnosis 27 Differential Diagnosis. . . . . . 48
Management 28 Management 48
Nonpharmacologic Therapy. . . . 29 Pregnancy and Childbirth Issues. 50
Pharmacologic Therapy 29 Prognosis 51
Surgical Therapy 30
Sjtigren Syndrome
Fibromyalgia Epidemiologz and Pathophysiologr 51
Epidemiologr and Pathophysiologz 30 Clinical Manifestations. . . . . . . . . . . 5t
Diagnosis 30 Diagnosis 51
Management 32 Management 52
Nonpharmacologic . 32 Prognosis 52
Pharmacologic..... 32

ldiopathic lnflammatory Myopathies


Spondyloarthritis Overview 53
Overview 32 Evaluation of the Patient With Muscle Pain
Pathophysiolory .......... en or Weakness. 53
Genetic Factors 33 Epidemiologr and Pathophysiologz 53
Environmental Factors . 33 Clinical Features 54
Tissue Factors....... . . JJ Muscle Involvement 54
Classification JJ Pulmonary Disease 54
Anl<ylosing Spondylitis . J.) Skinlnvolvement... 55
PsoriaticArthritis . .. . . 36 Cardiac Disease 55
Enteropathic Arthritis. . Malignancy. 55
Reactive Arthritis. . . . . . 3B Types of Idiopathic Inflammatory Myositis . . . 55
Diagnosis 3B Dermatomyositis. . . 55
LaboratoryStudies .... 3B Polymyositis. 55
Imaging Studies. . . . . . . 39 Antisynthetase Syndrome 57
Management 4I Immune-Mediated Necrotizing Myopathy 57
General Considerations. 4t Inclusion Body Myositis 57
Ankylosing Spondylitis. 47 Overlap Syndromes. 57
PsoriaticArthritis ... . . 42 Diagnosis 57
Enteropathic Arrhritis. . 42 Classifi cation Criteria 57
Reactive Arthritis. . .. . . 42 Muscle-Related Enzymes 57
Autoantibodies .... 57
Systemic Lupus Erythematosus Imaging 58
Epidemiologr and Pathophysiologz 42 Electromyography. . . 58
Clinical Manifestations... . . . . . . .. 43 Histopathologz s8
Mucocutaneous Involvement. . 43 Management 59
Musculoskeletal Involvement. . 44 Prognosis 59
Kidney Involvement 44
Neuropsychiatric Involvement . 45 Systemic Sclerosis
Cardiovascular Involvement. . . 45 Epidemiologr and Pathophysiologz . 59
Pulmonary Involvement. . . . . . 45 Classification 59
Hematologic Involvement. . . . . 45 Clinical Manifestations and Diagnosis 60
Gastrointestinal Involvement . . 46 Cutaneous Involvement 60
Comorbidities .. _..... 46 Musculoskeletal Involvement. . ., 62

x
Vascularlnvolvement . . . . . . 62 LymeArthritis..... 75
Cardiac Involvement 63 My cobacterium tuberculosis Infection 7S
Gastrointestinal Involvement 63 Fungal Infections. 75
Kidney lnvolvement 63 Virallnfections ..... 76
Lunglnvolvement. ....... . 64 Prosthetic Joint Infections . . . . 77
Management 64 Management 77
Pregnancy 65
Systemic Uasculitis
Mixed Connectiye lissue Disease and Undifferentiated Overview ..........78
Connective fissue Disease Large-VesselVasculitis ......79
Overview ..........66 GiantCellArteritis .....79
EpidemiolograndPathophysiologz . . .......66 PolymyalgiaRheumatica .......80
ClinicalManifestations andDiagnosis . . . . . . . .......66 TakayasuArteritis ...... 8l
Management .......66 Medium-VesselVasculitis .... 81
Prognosis ..........66 PolyarteritisNodosa ....81
Primary Angiitis of the Central Nervous System . . . 82
CrystalArthropathies KawasakiDisease ......83
Gout .. 67 Small-VesselVasculitis ......83
Epidemiologr 67 ANCA-AssociatedVasculitis...... ...... 83
Pathophysiolory .... 67 Immune Complex-MediatedVasculitis. . . . . . . . . 85
Clinical Manifestations. . . . . . 67
Diagnosis 68 Other Rheumatologir Diseases
Management 69 Behqet Syndrome 87
Calcium Pyrophosphate Deposition. . . . ., 7l RelapsingPolychondritis . . ... ..... . 88

i
Epidemiologz and Pathophysiolory . , 7l AutoinflammatoryDiseases. . . . . . . . . 89
Clinical Manifestations and Diagnosis 7l Adult-Onset Still Disease 90
t Management 72 Sarcoidosis 90
\ Basic Calcium Phosphate Deposition . . . . 73 Igc4-Related Disease 92
Genetic Diseases of Connective Tissue 92
lnfectious Arthritis
Diagnosis 73 Bibliography 93
Clinical Manifestations. . . . . 73
Laboratory and Imaging Studies 73
Self-Assessment Test. 97
Causes. 74
Infection With Gram-Positive Organisms 74
Infection With Gram-Negative Organisms 74 lndex t75

xl
Rheumatology High Value Care
Recommendations
The American College of Physicians, in collaboration with o All tumor necrosis factor inhibitors provide similar bene-
multiple other organizations, is engaged in a worldwide fit in patients with rheumatoid arthritis (see Item 36).
initiative to promote the practice of High Value Care (HVC). . Laboratory testing is usually not necessary to diagnose
The goals of the HVC initiative are to improve health osteoarthritis but is helpful if other causes of arthritis are
care outcomes by providing care ofproven benefit and being considered or to help define the safety ofpotential
reducing costs by avoiding unnecessary and even harmful therapies.
interventions. The initiative comprises several programs . Topical NSAIDs are safe and effective for treatment of
that integrate the important concept ofhealth care value knee and hand osteoarthritis and should be considered
(balancing clinical benefit with costs and harms) for a before oral NSAIDs (see Item 24).
given intervention into a broad range of educational mate- . The use of chondroitin sulfate (with or without glucos-
rials to address the needs oftrainees, practicing physicians, amine), biologics, intra-articular hyaluronic acid, platelet-
and patients. rich plasma, stem cells, or botulinum toxin is not recom,
mended in the treatment of knee osteoarthritis
HVC content has been integrated into MKSAP 19 in several
(see Item 5o).
important ways. MKSAP 19 includes HVC-identified key
o Acetaminophen provides no benefit for hip or knee
points in the text, HVC-focused multiple-choice questions,
osteoarthritis; it may be considered as add-on therapy
and, in MKSAP Digital, an HVC custom quiz. From the text
for short-term and episodic use but not as initial therapy
and questions, we have generated the following list of HVC
(see Item 54).
recommendations that meet the definition below of high
o Tai chi is as beneficial as physical therapy for knee and hip
value care and bring us closer to our goal of improving
osteoarthritis pain (see Item 34).
patient outcomes while conserving finite resources.
. Exercise is beneficial in hip, knee, and hand osteoarthri-
HighValue Care Recommendation: A recommendation to tis; no one exercise program is superior (see Item A2).
choose diagnostic and management strategies for patients in o Arthroscopic surgery is not indicated in patients with
specific clinical situations that balance clinical benefit with osteoarthritis unless there is joint buckling, instability, or
cost and harms with the goal of improving patient outcomes. locking, or a concomitant and symptomatic mechanical
disorder.
Below are the High Value Care Recommendations for the
o Patients with fibromyalgia should not be treated with
Rheumatology section of MKSAP 19.
anti-inflammatory drugs, including NSAIDs and glu-
o An accurate history and a thorough musculoskeletal cocorticoids, and do not respond to opioids, with the
physical examination are essential to diagnose and differ- exception of tramadol (see Item 7).
entiate inflammatory and noninflammatory symptoms o Borrelia burgdorferi DNA can be detected by polymerase
and can help to avoid unnecessary testing. chain reaction in slnovial fluid, but this test offers no
o Antinuclear antibody testing should not be performed in advantage to serologic testing (see Item 12).
a patient with nonspecific symptoms and normal find- o A muscle biopsy is not necessary to diagnose dermato-
ings on clinical examination because it does not establish myositis in patients with characteristic clinical and labo-
the diagnosis of a connective tissue disease. ratory findings (see Item 38).
r Antinuclear antibody (ANA) subserologr testing should . A muscle biopsy can help to confirm the diagnosis of
not be performed routinely, even in the setting of a pos- inclusion body myositis but is not needed when clinical
itive ANA result, without strong clinical suspicion of an features are characteristic (see Item 45).
underlying connective tissue disease. o In the absence of additional suggestive symptoms, physi-
. Radiography is usually the first imaging test ordered in cal findings, and supporting laboratory data, oral dryness
the evaluation of rheumatologic diseases because it is should not be attributed to a rheumatologic condition
readily available, is inexpensive, exposes patients to only (see Item 47).
a low level of ionizing radiation, and is useful in monitor- o If a patient has many features of spondyloarthritis, a pos-
ing arthritis progression. itive HLA-B27 result adds little to the posttest probability
. Ultrasonography is an inexpensive means to assess (see Item 5).
soft-tissue abnormalities, assess disease activity, and o Antibiotics generally are not indicated in reactive arthritis
assist with tendon or joint injections. because they do not affect illness outcomes (see ltem 67).

xill
a
I
I
Rheumatology

Approach to the Patient t(rY P0lflT5


. Inflammatory symptoms include pain, erythema, swell-
i
With Rheumatologic ing, and warmth; noninflammatory conditions usually
Disease lack these features, except for pain.
o An accurate history and a thorough musculoskeletal HVC
lnflammatory Versus physical examination are essential to diagnose and dif-

L
Noninflammatory Pain ferentiate inflammatory and noninflammatory symp-
The differentiation between inflammatory and noninflamma toms and can help to avoid unnecessary testing.
tory signs and symptoms is central to the evaluation of
patients with musculoskeletal pain. Autoimmune conditions Arthritis
typically present with inflammation, whereas mechanical or Monoarthritis
degenerative disorders are characteristically noninflamma-
Monoarthritis involves a single joint and is classified as acute
tory. The cardinal signs of inflammation are pain, erythema,
or chronic.
swelling, and warmth; noninflammatory conditions usually
Acute monoarthritis can be noninflammatory (e.g.,
lack these features, except for pain. Patients may simultane-
caused by trauma, hemarthrosis, or internal derangement) or
ously experience more than one type of pain. Table 1 com
infl ammatory (e. g., crystal-induced or infectious). Evaluation
pares the features of inflammatory and noninflammatory
for infectious arthritis should be guided by the clinical presen-
arthritis.
tation and examination, but suspicion should always be high.
Joint aspiration is usually the most effective means of diagnos
ing the underlying cause.
The M usculoskeletal Examination Chronic inflammatory monoarthritis (>26 weeks) can be
An accurate history and a thorough musculoskeletal physical caused by chronic infection (e.g., mycobacterial, fungal, or
examination are essential to diagnose and differentiate inflam- Borrelia burgdorferi) or by autoimmune rheumatologic dis-
matory and noninflammatory symptoms and can help to ease. Synovial fluid cell count analysis can help determine the
avoid unnecessary testing. Musculoskeletal pain may be presence of inflammation but may be inadequate for diagnosis;
articular, periarticular, or referred. Pain with passive range assessment for systemic disease (complete history; examina-
of motion suggests an articular condition, whereas pain tion; and laboratory studies, including serologic testing) may be
only with active range of motion suggests a periarticular indicated. Rarely, synovial biopsy may be required to rule out
condition. chronic infection, deposition diseases, or malignancy.
See MKSAP 19 General Internal Medicine 1 for more Chronic noninflammatory monoarthritis is usually caused
information. by osteoarthritis.

TABLE I . Features of lnflammatory Versus Noninflammatory Arthritis


Feature lnflammatory Arthritis Noninf lammatory Arthritis
Morning stiffness >60 min; worsens with immobility <30 min
Constitutional sym ptoms Fever; fatigue; malaise Generally absent
Physical examination findings Erythema; warmth; soft-tissue swelling; joint Minimal or no warmth; no soft-tissue swelling;
effusions; reduced ROM is frequent bony enlargement and joint effusions may
occur in osteoarthritis; reduced ROM may occur
Synovialiluid Leukocyte count >2000/pL(2.0 x 10e /L), Leukocyte count of 200-2000/pL (0.2-2.0 x 1 0ell),
predominantly neutrophils in acute inflammation predominantly monocytes
and monocytes in chronic inflammation
Other laboratory findings Elevated inflammatory markers (ESR, CRP); lnflammatory markers usually normal or
anemia of inflammation minimally elevated

CRP = C reactive protein; ESR = erythrocyte sedimentation rate; ROI\l = range of motion
Approach to the Patient With Rheumatologic Disease

Oligoanhritis Extra-Articur M a n ifestatio ns


I a
Oligoarthritis involves two to four joints, typically in an asym
metric pattem.
of Rheumatologic Disease
Acute inflammatory oligoarthritis may be caused by Constitutional Symptoms
gonorrhea or rheumatic fever. Chronic inflammatory oligoar Fever, morning stiffness, and fatigue occur in numerous rheur-na
thritis can be caused by autoimmune conditions. such as tologic conditions. Fever is usually low grade but may be high and
seronegative spondyloarthritis (e.g., psoriatic arthritis. reac- spiking in some conditions (e.g., adult onset Still disease and
tive arthritis, arthritis related to inflammatory bowel disease, autoinflammatory diseases). Morning stiflhess lasting more than
or ankylosing spondylitis). 60 minutes is most commonly described in RA but also occurs
Chronic noninflammatory oligoarthritis is usually caused with other forms of inflammatory afthritis. Significant and even
by osteoarthritis. disabling fatigue is a prominent feature of fibromyalgia.

Skin lnvolvement
Polyarthritis
Skin involvement is common in rheumatologic conditions and
Polyarthritis involves five or more joints. In many cases, it
may go unnoticed by the patient (Table 2). It may also be an
involves the small joints of the hands and/or feet.
adverse eflect of n-redications used to treat rheumatologic con
Acute polyarthritis (<6 weeks in duration) can be caused
ditions. including skin inf'ections secondary to immunosup
by viral infections (e.g., with parvovirus 819, HIV hepatitis
pressive therapy.
viruses, rubella, or chikungunya virus) or may be an early
manifestation of chronic (>6 weeks in duration) inflammatory
polyarthritis, such as rheumatoid arthritis (RA), systemic Eye lnvolvement
lupus erythematosus (SLE), or psoriatic arthritis. Eye involvement in different rheumatologic diseases usually
follons fairly distinct patterns, and the location and type of
involvement can help narro\{ the differential diagnosis
Soft-Tissue Abnormal ities (Table 3).If not quickly recognized and treated. certain fbrms
ofeye involvement can have devastating consequences, includ
Common nonarticular sources of musculoskeletal symptoms
ing permanent loss of vision. See MKSAP 19 General Internal
are the soft tissues (tendons, ligaments. and bursae) around or 'I

Medicine 2 {br additional information on ocular rnanifesta


away from the joints. Isolated tendon andi'or ligament involve
tions of rheumatologic disease.
ment usually suggests noninflammatory disorders, such as
mechanical injury/irritation, overuse, or degeneration (e.9..
lnternal Organ lnvolvement
rotator cuff disorders or tennis elbow). Disorders of wide
Rheumatologic diseases fiequently aft'ect internal organs. with
spread musculoskeletal pain (e.g., fibromyalgia) also cause
different diseases tending to follow characteristic patterns
symptoms localizing to these structures.
(Table 4).
The enthesis is a complex structure at the site of the inser
tion of a tendon or ligament onto the bone. Severe persistent I(EY POIXI
inflammation of the enthesis (enthesitis) strongly suggests o Rheumatologic disease can cause constitutional symp-
spondyloarthritis, especially when it affects multiple sites. The toms and extra-articular manifestations affecting the
inflammation may extend along the associated tendon and skin, eyes, and internal organs.
Iocal ligaments; it results in dactylitis ("sausage digits"), which
is typically a feature ofspondyloarthritis, particularly psoriatic
arthritis.
See MKSAP 19 General Internal Medicine 1 for more
Laboratory Studies
information. Laboratory studies are useful for diagnosing rheumatologic dis
eases. identilying the extent/severity of involvement, evaluating
t(EY POlt{rs disease activity, and monitoring therapeutic responses. Because
HVC . Joint aspiration is usually the most effective means of of limited specificity. results of these tests should always be
diagnosing the underlying cause of acute monoarthritis. interpreted in the context of the clinical history and physical
o Chronic noninflammatory arthritis is usually caused by examination and should be applied with great caution, if at all,
osteoarthritis. in the setting of low pretest probability(see MKSAP 19 General

r Internal Medicine 1 for discussion of pretest probabilities).


Isolated tendon and/or ligament involvement usually
suggests noninflammatory disorders, such as mechani-
cal injury/irritation, overuse, or degeneration.
Tests That Measure lnflammation
o Ery.throcyte Sedimentation Rate
Persistent inflammation of the enthesis (enthesitis)
The erythrocyte sedimentation rate (ESR) measures the fall of
strongly suggests spondyloarthritis.
erythrocytes (in millimeters per hour) through anticoagulated

2
I
:
I

:
Approach to the Patient With Rheumatologic Disease
i

TABLE 2. Dermatologic Manifestations of Rheumatologic Disease


Rheumatologic Disease Dermatologic Manifestations
Systemic lupus erythematosus Acute cutaneous lupus erythematosus (commonly a malar rash); subacute cutaneous lupus
erythematosus; photosensitive rash; chronic cutaneous lupus erythematosus (most commonly
discoid lupus); oral ulcerations (on the tongue/hard palate; usually painless); alopecia; lupus
panniculitis (painful, indurated subcutaneous swelling with overlying erythema of the skin)
Dermatomyositis Gottron papules (erythematous plaques on extensor surfaces of MCP and PIP joints);
photodistributed poikiloderma, including shawl sig n (over the back and shoulders) and V sign
(overthe posterior neck/back or neck/upper chest); heliotrope rash (violaceous rash on the upper
eyelids); mechanic's hands (hyperkeratotic, fissured skin on the palmar and lateral aspects of fingers);
nailfold capillary abnormalities; holster sign (poikiloderma along lateralthigh); can occur in the
absence of myositis (a myopathic dermatomyositis)
Systemic sclerosis Skin tightening, thickening, and hardening; nailfold capillary changes; calcinosis; telangiectasias;
decreased oral aperture; bi- or tri-color Raynaud phenomenon
Vascu litis Palpable purpura; nodules; ulcers; necrosis; Raynaud phenomenon if cryoglobulins present
Behget syndrome Painful oral and genital ulcers; erythema nodosum; acne/folliculitis; pathergy (skin inflammation/
ulceration from minor trauma)
Sarcoidosis Erythema nodosum; infiltrated plaques; maculopapular and papular lesions; nodules; soft
infiltrates of the nose (lupus pernio); on blanching with a glass slide, sarcoid skin lesions reveal
"apple jelly" discoloration
Psoriatic arthritis Plaque psoriasis typically on extensor surfaces, umbilicus, glutealfold, scalp, and behind ears;
pustular psoriasis on palms and soles; nail pitting; onychodystrophy
Reactive arthritis Keratoderma blennorrhagicum (psoriasiform rash on soles, toes, palms); circinate balanitis
(psoriasiform rash on penis)
Adult-onset Still disease Evanescent, salmon-colored rash on trunk and proximal extremities
Rheumatic fever (secondary to Erythema marginatum (annular pink to red nonpruritic rash with central clearing)
streptococcal infection)
Lyme disease Erythema chronicum migrans (slowly expanding, often annual lesion with central clearing)

MCP = metacarpophalangea; PIP = proximal interphalangeal.

TABTE 3, Ocular Manifestations of Systemic lnflammatory Disease


Systemic lnflammatory Disease Ocular Manifestations
Ankylosing spondylitis, reactive arthritis, and inflammatory bowel Uveitis (inflammation of anterior and/or posterior chamber and/or
disease (anterior chamber); sarcoidosis and BehEet syndrome retina); anterior uveitis symptoms include pain, redness, visual
(anterior and/or posterior chamber); granulomatosis with change; posterior uveitis symptoms include painless visual change
polyangiitis (posterior chamber) or floaters
Rheu matoid arth ritis; spondyloarthritis; system ic vascu litis; rarely, Episcleritis; symptoms include redness, watering, and irritation but
SLE no vision loss
Rheumatoid arthritis; relapsing polychondritis; systemic vasculitis; Scleritis; symptoms include severe eye pain that is worse at night
inflammatory bowel disease; rarely, SLE and with eye movements; redness; photophobia
Systemic vasculitis; antiphospholipid syndrome; SLE Retinal ischemia; symptoms include painless vision loss or change
Sjogren syndrome Keratoconjunctivitis sicca; symptoms include dry eyes
Giant cell arteritis Anterior/posterior ischemic optic neuropathy; central retinal artery
occlusion; symptoms include loss of vision
Sarcoidosis; granulomatosis with polyangiitis; lgG4-related Proptosis/retrobulbar inf la m matory infiltrate
diseases
Reactive arthritis Conjunctivitis; symptoms include red eye

SLE = systemic lupus erythematosus.

plasma. Erythrocytes tend to be negatively charged on their many rheumatologic diseases, as well as in nonrheumatologic
surfaces, leading to repulsion and a prolonged ESR. Fibrinogen inflammatory conditions, such as chronic infections and
and other acute phase reactants, as well as hypergammaglobu malignancies. The normal ESR increases with age and is usu
linemia (polyclonal or monoclonal), neutralize the erythro ally higher in women. A well accepted rule of thumb is to
cytes' surface charges, promoting their ability to settle at a adjust the upper limit of normal as (age in years)/2 lor men
faster rate. Elevated fibrinogen levels and high ESRs are seen in and (age in years + l0) 2 for women.

3
Approach to the Patient With Rheumatologic Disease

TABLE 4. lnternal Organ lnvolvement in Rheumatologic Disease


Disease Type of lnvolvement
Heart
Kawasaki disease Coronary artery vasculitis
Systemic sclerosis Arrhythmia; myocardial fibrosis
SLE Pericarditis; valvular disease; myocarditis
RA Perica rditis; myocarditis

Rheumatic fever; antiphospholipid syndrome Valvular disease


Giant cell arteritis Aortic aneurysm/dissection; aortitis; large'vessel obstruction
Sarcoidosis Atrioventricular block; a mia; cardiom
Lung
RA Serositis; ILD; rheumatoid nodules
SLE; CTDs; myositis; Henoch-Schonlein purpura Serositis; pneumonitis; pulmonary hemorrhage from vasculitis
Pulmonary hemorrhage; cavitary nodules
Diffuse cutaneous systemic sclerosis ILD; pulmonary hypertension

Limited cutaneous systemic sclerosis Pulmonary hypertension


Antiphospholipid syndrome Pulmonary embolism; pulmonary hemorrhage in catastrophic antiphospholipid
syndrome
Sarcoidosis Hilar lymphadenopathy; ILD
Goodpasture syndrome Pulmonary hemorrhage
Kidney
SLE; CTDs; AAV systemic vasculitis (except PAN) G lomeru loneph ritis
PAN Renal artery vasculitis; pseudoaneurysms

Antiphospholipid syndrome Renal infarct; renal vein thrombosis

Sjogren syndrome Acute interstitial nephritis/renal tu bular acidosis


Goodpasture syndrome G lomeru lonephritis
Gastrointestinal System
PAN Mesenteric vasculitis
Henoch-Schonlein purpura lntestinal vasculitis and ulcerations
Diffuse and limited cutaneous systemic sclerosis Esophageal and small-bowel hypomotility
BehEet syndrome Mucosal ulcerations
Familial Mediterranean fever Peritonitis
Nervous System
SLE; CTDs; AAV; systemic vasculitis CNS involvement may include mental status changes, stroke, seizures;
peripheral involvement may include mononeuritis multiplex and peripheral
neuropathy
PACNS CNS vasculitis

AAV = ANCA associated vasculitis; CNS = central nervous system; CTD = con nective tissue disease; I LD = interstitial lung disease; PACNS = primar angiitis o{ the central nerous
system; PAN = polyarteritis nodosa; RA = rheumatoid afthritis; SLE = systemic lupus erythematosus.

In addition to inflammatory conditions, ESRs can be A markedly elevated ESR (>100 mm/h) should alert phy-
elevated in pregnancy, diabetes mellitus, obesiry and end- sicians to conditions such as giant cell arteritis, multiple mye-
stage kidney disease. Because ofrheostatic properties, anemia loma, metastatic cancer, or other overwhelming inflammatory
and macrocytosis are also associated with an increased ESR. states (infection or autoimmune disease).
ESR can be excessively low in low-fibrinogen states, such as
liver or heart failure, and in conditions promoting rouleaux C-Reactive Protein
formation (e.g., polycythemia vera). Sickle cell disease and C-reactive protein (CRP) is produced by the liver mainly in
microcytosis (including spherocytosis) may also decrease response to interleukin-6 generated by leukocytes during
ESR. the inflammatory state. CRP levels and ESR usually follow a

4
Approach to the Patient With Rheumatologic Disease

common pattern, but CRP is often more rapidly responsive to Antinuclear antibodies (ANAs) are directed against
changes in inflammation. In rheumatologic conditions, CRP is nuclear antigens and are traditionally associated with SLE. Up
typically elevated 2 to l0 times the normal level; a higher level to one third of the healthy population has a low titer (1:40) for
(especially >10 mg/dl [tOO mg/L]) should prompt considera, ANA, and up to 5'X, have a titer of 1:160 or more. ANA can also
tion of an alternative diagnosis, such as infection. CRP is be seen in other autoimmune conditions, infection, and
thought to be a better marker than ESR for measuring inflam malignancy and may be drug induced. ANA testing should not
mation in spondyloarthritis. In contrast, in some patients with be performed in a patient with nonspecific symptoms and
SLE, ESR is a better marker of disease activity and the CRP normal findings on clinical examination because it does not
level may remain normal despite active disease. An elevated establish the diagnosis of a connective tissue disease.
CRP level in a patient with SLE is often related to infection. A higher ANA titer is more often associated with an
CRP can be elevated in obesity and low with the use of certain underlying rheumatologic disease, although not always SLE.
antibiotics and interleukin 6 blockers. However, almost all patients with SLE (>95'/") are positive for
ANA. ANA titer does not correlate with SLE disease activi!/
Complement and should not be used fbr activity assessment.
The complement system is an essential part of the immune ANA specificity or subserologz testing (i.e., testing for
response, promoting vasodilation, attracting leukocy.tes, and antibodies to specific nuclear components, such as DNA or
assisting in the lysis of opsonized bacteria during humoral centromeres) should be reserved for patients positive fbr ANA
immunity. and a clinical syndrome suggesting an underlying connective
Complement components are acute phase reactants that tissue disease. ANA subserologr testing should not be rou
are synthesized in the liver and rise in many inflammatory tinely performed, even in the setting of a positive ANA result,
states. However, in response to diseases that lead to immune without a strong clinical suspicion of an underlying connec
complex formation (SLE; cryoglobulinemic and urticarial vas- tive tissue disease.
culitis) and other states, such as infections (subacute bacterial Table 5 provides details on these and other autoantibodies
endocarditis, sepsis, viremia) and glomerulonephritis, com and their associations with specific conditions.
plement cascades are activated, and complement levels fall
XEY POItrII
because of excessive consumption. Paradoxically, genetic defi
ciency of early complement components may increase the risk
. Erythrocyte sedimentation rate and C-reactive protein
(CRP) levels usually follow a common pattern, but CRP
tbr lupus like autoimmune diseases.
is often more rapidly responsive to changes in inflam-
C3 and C4 are the commonly measured complement
mation.
components. The CH50 assay should not be perlbrmed rou
tinely because of cost and limited utility. o Antinuclear antibody testing should not be perfbrmed HVC

in patient with nonspecific symptoms and normal


a
Autoantibody Tests findings on clinical examination because it does not
Rheumatologic diseases are commonly associated with establish the diagnosis of a connective tissue disease.
autoantibodies, but their presence does not equate with the . Antinuclear antibody (ANA) subserologr testing should HVC
diagnosis ol an underlying condition because they lack speci not be routinely performed, even in the setting of a pos
ficity and may be seen in patients with other conditions and in itive ANA result, without strong clinical suspicion of an
healthy persons. In commercial laboratories, autoantibody underlying connective tissue disease.
testing has been automated with enzyme-linked immuno-
sorbent assays in a sequential algorithm, which may simplify
physician assessment but tend to have reduced sensitivity and
specificity.
lmaging Studies
Rheumatoid lactor is an IgM antibody directed against the Radiography
Fc portion of IgG. Although characteristically associated with Radiography is essential in the evaluation of many rheuma
RA. rheumatoid factor is present in less than 70'X, of patients tologic diseases and can assess and differentiate inflamma
with RA and is common in several other diseases, such as bac- tory arthritis, osteoarthritis, and crystal arthropathies
terial endocarditis and hepatitis C vims infection' Anti cyclic (Table 6). Radiography has limitations because it gives a two

citrullinated peptide antibodies are more specific (95%) for RA dimensional picture of three dimensional structures, is lim-
but less sensitive (67'7,). The presence of both autoantibodies ited in its ability to visualize soft tissues, and may not detect
together increases the Iikelihood of RA. Patients with RA who early or small erosive changes. Despite these limitations,
have anti cyclic citrullinated peptide antibodies are more radiography is usually the first imaging test ordered in the
likely to experience rapid joint damage from erosive disease. evaluation of rheumatologic diseases because it is readily
Patients with hepatitis C virus infection alone are typically available, is inexpensive, exposes patients to only a low level
positive for rheumatoid factor but negative for anti-cyclic of ionizing radiation, and is useful in monitoring arthritis
citrullinated peptide antibodies. progression.

5
1
I

'l
1

Approach to the Patient With Rheumatologic Disease I


a

I
It
Autoantibody Rheumatologic Disease Sensitivity/Specificity Comments I

SLE: >95% sensitivity, Poor Does not correlate with disease activity
\I
ANA SLE; also SSc, Sjogren
syndrome, MCTD specificity; indirect IFA is the
most appropriate method i t

Anti-double-stranded SLE 507"- 60"/" se nsitivity, >9 5% Found in more severe disease, especially kidney
DNA specificity; Crithidia IFA or disease; antibody levels commonly follow disease I
Farr assays more specific
than ELISA
activity and are usefulto monitor l I

Most specific test for SLE; does not correlate


Anti-Smith SLE 30% sensitivity,99%
specificity with disease activity
I
Anti-U 1-RNP MCTD; SLE High sensitivity for MCTD High titer seen in MCTD (>1 :1 0,000); does not I
correlate with disease activity 1
I
Anti-Ro/SSA; Sjogren syndrome; SLE; Sjogren syndrome: 7095 Sicca symptoms; in SLE, associated with
anti-La/SSB RA; SSC sensitivity; SLE:20% photosensitive rash; offspring of mothers who
sensitivity are positive for anti-Ro/SSA or anti-La/SSB are at
)
increased risk for neonatal lupus erythematosus
t
(rash and congenital heart block) I

Antiribosomal P SLE 15% sensitivity Associated with CNS lupus and lupus hepatitis l
I
,,]

Anti-Scl-70 DcSSc 10%-30% sensitivity Seen more often in patients with DcSSc who have
(antitopoisomerase-1 ) ILD leading to pulmonary fibrosis ;
Anticentromere LcSSc (CREST) 10%-30% sensitivity Patients with LcSSc with this antibody are more
likely to develop pulmonary arterial hypertension

c-ANCA GPA 90% sensitivity when disease Correlation with disease activity is unclear
(antiproteinase-3) is active; high specificity in
classic presentations

p-ANCA MPA; EGPA MPA: 80% sensitivity; Atypical p-ANCA (antimyeloperoxidase


(antimyeloperoxidase) EGPA: 60% sensitivity; less negative) can be seen in inflammatory bowel I
specific than c-ANCA disease and with positivity for ANA; may be
seen in drug-induced vasculitis
Anti-Jo-1 Polymyositis 20%-30% sensitivity Associated with antisynthetase syndrome, which
may include mechanic! hands, Raynaud
phenomenon, and lung inflammation
Anti-SRP Polymyositis Found in 5% of patients with Associated with immune-mediated necrotizing
myositis myopathy with muscle fiber necrosis and
minimal inflammation on biopsy
Anti-Mi-2 Dermatomyositis Rare Acute onset of skin rash in shawl distribution,
usually responsive to treatment
Anti-TlF-1-1 Dermatomyositis Approximately 70% lncreased risk for malignancy; also seen in
sensitivity and 90% specificity juvenile dermatomyositis
Anti-MDA-5 Dermatomyositis Rare Associated with rapid lLD, cutaneous
ulcerations, Gottron papules; poorer prognosis
Rheumatoid factor RA; Sjogren syndrome; RA: 70% sensitivity; limited RF iscommon in multiple other diseases (e.9.,
cryoglobulinemia specificity, especially in hepatitis C virus infection, endocarditis, SLE);
patients without a classic 30% of patients with RA are RF negative but may
disease presentation become positive later in RA course
Anti-cyclic RA 70% sensitivity; Can be positive in RF-negative patients with RA;
citrullinated peptide 95% speciflcity often present before RF becomes positive;
associated with erosions; predicts disease
progression in undifferentiated arthritis
Antihistone DILE 95% sensitivity; poor Also seen in primary SLE
specificity
Cryoglobulins Vasculitis; hepatitis C Type ll or lll cryoglobulins May be present in connective tissue diseases in
virus infection; myeloma; seen in cryoglobulinemic the absence of vasculitis
SLE; RA vasculitis
ANA = antinuclear antibody; CNS = central nerous system; CREST = calcinosis, Raynaud phenomenon, esophageal dysmotility, sclerodactyly, and telangiectasia; DcSSc = difuse
cutaneous systemic sclerosis; DILE = drug induced lupus erythematosus; EGPA = eosinophilic granulomatosis with polyangiitis; ELISA = enzyme-linked immunosorbent assay;

tissue diseasei MDA = melanoma diferentiation-associated; MPA = microscopic polyangiitis; RA = rheumatoid arthritis; RF = rheumatoid factor; RNp = ribonucleoprotein; SLE =
systemic lupus erythematosus; SRP = signal recognition particle; SSc = systemic sclerosis; TIF = transcription intermediary factor.

6
Approach to the Patient With Rheumatologic Disease

TABLE 6. Standard Radiographic Findings of Common Rheumatologic Diseases


RheumatologicDisease RadiographicFindings
Rheumatoid arthritis Soft-tissue swelling and periarticular osteopenia early; bony marginal erosions and uniform joint-space
narrowing later
Swan neck deformity and ulnar deviation if unresponsive or untreated; MCB PIB wrist, and MTP involvement;
noncalcified soft-tissue nodules
Osteoarthritis Asymmetric joint space narrowing; osteophytes; subchondral sclerosis and cystic changes; degenerative
disk disease with collapse of disks; degenerative joint disease with facet joint osteophytes; spondylolisthesis
(anterior/posterior misalignment of the spine); kyphosis
Diffuse idiopathic skeletal Calcification (ossification)o{ anterior longitudinal ligament (bone spurs); bridging horizontal
hyperostosis syndesmophytes; usually seen in thoracic spine and more prominent on right side of spine
Ankylosing spondylitis Sacroiliitis best seen on modified anteroposterior Ferguson view of sacrum on radiograph, usually
bilateral; squaring o{ the vertebral bodies in early disease; bridging vertical syndesmophytes (ossification
o{ annulus fibrosus) in later disease; shiny corners; ankylosis does not skip veftebrae

Psoriatic arthritis Destructive arthritis with erosions and osteophytes; DIP involvement is common; pencil-in-cup deformity
on hand radiograph; arthritis mutilans; syndesmophytes
Gout Soft-tissue swelling; punched-out erosions with sclerotic borders and overhanging edges; periarticular
tophi appear as high-density radiopaque deposits
Calcium pyrophosphate Chondrocalcinosis, most commonly of knees, shoulders, wrists, pubic symphysis; osteoarthritis, including
deposition in locations atypical for primary osteoarthritis (MCPs, wrists, shoulders); hooked osteophytes of second and
third MCP joints
DIP = distal interphalangeal; MCP = metacarpophalangeal; MTP = metatarsophalangeal; PIP = proximal interphalangeal.

CT structures, and can provide real time data in the clinic with-
CT provides multiple views and orientations from a single out exposure to ionizing radiation. It can assess soft tissue
study but is more useful fbr bony abnormalities than for soft abnormalities, including synovitis, tendonitis, bursitis, crystal
tissue inflammation or fluid collections. CT is more sensitive deposition, and effusions; assess disease activity using Doppler
for detecting bone erosions than is radiography or MRI. technolory; and assist with tendon or joint injections.
However, CT is more expensive than radiography and exposes Howeveq it is operator dependent, and training and practice
the patient to more radiation. It is mainly used in acute trauma are needed to achieve competence.
or when a patient cannot undergo MRl. Dual ener$/ CT may t(tY P0t xTs
be used to detect tissue urate deposits but is not indicated in
routine testing.
r Radiography is usually the first imaging test ordered in HVC
the evaluation of rheumatologic diseases because it is
readily available, is inexpensive, exposes patients to
MRI
only a low level of ionizing radiation, and is useful in
MRI is the most sensitive routine radiologic technique fbr
moniloring arthril is progression.
detecting soft tissue abnormalities, inflammation, and fluid
collections but is Iess eflective than CT in demonstrating bony
o CT is more sensitive for detecting bony abnormalities or HVC

abnormalities or erosions. MRI is sensitive for detecting early erosions than is radiography or MRI, whereas MRI is the
spine and sacroiliac joint inflammation and may be indicated most sensitive routine radiologic technique for detect
for the evaluation of suspected spondyloarthritis ifradiographs ing soft tissue abnormalities, inflammation, and fluid
are negative. MRI is also indicated in the evaluation of osteo- collections.
necrosis if the findings on plain imaging are normal and sus, o Ultrasonography is an inexpensive means to assess soft HVC
picion is high. MRI does not expose patients to radiation but is tissue abnormalities, assess disease activity, and assist
associated with high cost, limited availability, and possible with tendon or joint injections, but it is operator
patient intolerance due to claustrophobia or body habitus. The dependent.
American College of Rheumatolos/ Choosing Wisely list rec
ommends against routine MRI of the peripheral joints to mon-
itor RA because of inadequate data supporting its use. Joint Aspiration
Joint aspiration and synovial fluid analysis are essential for
Ultrasonography discriminating between inflammatory and noninflammatory
The use of ultrasonography to evaluate patients with rheuma efTusions and for distinguishing between infectious arthritis
tologic diseases has expanded dramatically. Ultrasonography is and acute crystal arthropathies. ln the evaluation of any
relatively inexpensive, can scan across three-dimensional monoarthritis or when infection is being considered, joint

7
Principles of Therapeutics

TABTE 7. Synovial Fluid Analysis


Normal Noninflammatory lnflammatory Crystal-lnduced lnfectious Hemorrhagic
Appearance Clear/yellow/ Clear/yellow/ Yellow/white/ Yellow/white/ Yellow/white/ Red/opaque
transpa rent tra nspa rent translucenV tra nsl u ce nt/ opaq ue
opaque opaque
Leu kocyte <200/Stl(0.2x 200-2000/stL 2000-20,000/pL 1 0,000-50,000/pL >50,000/prL
cou nt 10e/L) (0.2 2.0 x 10e /L) (2.0-20 x 10e/l) (1050x1Oe/L) (50 x 1 Oell)
(may be higher) (may be higher) (may be lower)
Other studies Negative Gram Negative Gram Negative Gram Negative Gram Positive Gram Negative
stain; negative stain; negative stain; negative stain; positive stainb; positive Gram stain;
cu ltu re culture culture crystals" culture' negative
culture

blue when paralle to the axis and ye low when perpendicular.

stain sensitivity ior infection is approximately 30% to 50%.

all cultures are positive except for infection caused 6y Ndsseria gonorrhoeae, which may be posrtive in 507o or fewer cases.

aspiration should be performed to diagnose the underlying XEY POIl{TS


cause. Aspirated synovial fluid should be sent for leukocyte
. Synovial fluid leukocy.te counts greater than 50,000/pL
count. Gram stain, and cultures, as well as evaluation fbr crys-
(50 x 10e/L) with polymorphonuclear cell predomi
tals under polarized light. See Table 7 for more infbrmation.
nance have a high likelihood of infection; counts less
l'here is no absolute cutoff of synovial fluid leukocyte
than 2000/trrL (2.0 x 10'q/L) are usually associated with
counts for ruling out infectious arthritis; however. counts
noninflammatory causes.
greater than 50.000/gL (SO x 10e/L) with polymorphonuclear
cell predominance have a high tikelihood of inf'ection. Counts
. Tissue biopsy of involved organs can be helpful in diag-
less than 200019L (2.0 x 10e/t.) are usually associated with
nosing numerous rheumatologic conditions and in
assessing disease activity in some conditions.
noninflammatory causes. Notably, crystals can coexist with
inf'ection. and their presence does not rule out infection if
suspicion is high.

Pri nci ples of Therapeutics


Tissue Biopsy
When appropriate, tissue biopsy of involved organs can be
Overuiew
helpful in diagnosing numerous rheumatologic conditions. This section reviews the indications for use. mechanisms ot'
such as vasculitis (lung, kidney, or temporal artery biopsy) and action, major toxicities, and monitoring requirements of nredica
SLE or dermatomyositis (skin biopsy). Tissue biopsy may also tions used in rheumatologic disease. Dmg applications in specific
help assess disease activity (e.g., kidney biopsy in SLE). The disease states are elaboruted on in their respective sections.

benefits should be appropriately balanced with possible risks


of the procedure.
Anti-l nfla m matory Agents
Glucocorticoids
Mental Health Screening Clucocorticoids are eff'ective in many rheumatologic dise:rses,
Practitioners should be aware that patients with arthritis including rheumatoid arthritis (RA), acute crystal arthropathy,
experience mental distress and have :r history of depression at systemic vasculitis, polymyalgia rheumatica. systemic lupus
higher rates than the general population. Concurrent depres erythematosus, inflammatory myopathies, and autoinflam
sion in chronic conditions, such as arthritis, is associated with matory diseases. Advantages include rapid onset, ease of use,
reduced adherence to medical treatment recommendations. low cost, and universal availability; they are often disease
Among patients with rheumatoid arthritis, anxiety and modifzing and sometimes lifesaving.
depression are associated with reduced response to treatment Adverse eflects include osteoporosis, immunosuppression.
and poorer quality of life. Active screening for depression, skin fragility, glaucoma, cataracts, weight gain. diabetes melli
recommended lbr all adults, is therefore of still greater impor tus. hypertension, psychomotor agitation, osteonecrosis. and
tance in individuals with arthritis and may improve outcomes. suppression of the hypothalamic pituitary adrenal axis. These
Actively engaging adults with arthritis in evidence based self- effects are more likely with higher doses and longer treatment.
management programs is helpful not only tbr arthritis but also The immunosuppressive adverse etfects of glucocorticoids
for reducing depression and improving self efficacy. are apparent at moderate (prednisone at >20 mg/d) and high

8
Principles of Therapeutics

doses but can also occur at lower doses (e.g., z.s mg/d for more diclofenac may also be prelerred fbr patients at high risk for
than a few weeks). The American College of Rheumatologr toxicity from oral NSAIDs or those 75 years of age or older.
(ACR) recommends that patients who are anticipated to be
receiving long term glucocorticoid treatment (prednisone at Colchicine
>2.5 mg/d fbr >3 months) should have a baseline clinical risk Colchicine inhibits microtubules, impairs neutrophil func
assessment for osteoporosis within 3 to 6 months of initiation tion. and inhibits inflammasome-mediated interleukin 1
of therapy. Those with risk factors and those older than age activation. Inflammasome is a central signaling system that
40 years should also undergo periodic bone mineral density regulates the inf'lammatory response. It is most commonly
testing. Furthermore, those at moderate or high risk for osteo- used for gout and acute calcium pyrophosphate crystal arthri
porotic fractures who are prescribed long-term glucocorticoid tis (pseudogout). It is also a treatment for hypersensitivity
therapy should be treated prophylactically with an antiresorp- vasculitis and familial Mediterranean f'ever.
tive agent, preferably an oral bisphosphonate. Gastrointestinal adverse eff'ects (particularly diarrhea) are
common but reversible with dose adjustment or discontinua
NSAIDS tion. With overdose, severe (even fatal) myelosuppression can
NSAIDS prevent prostaglandin production by inhibiting the occur. Dosing must be adjusted fbr kidney disease. When
two isofbrms of cyclooxygenase (COX): COX-1 and COX-2. given over the long term, colchicine can rarely cause neuro
COX-2 is an inducible enzyme typically expressed in inflam muscular toxicity, particularly if coadministered with strong
matory milieus, whereas COX 1 is constitutively expressed and CYP3A4 inhibitors (e.9., clarithromycin) that reduce the
helps maintain organismal homeostasis. Nearly all available hepatic catabolism of colchicine. Such coadministration
COX inhibitors are nonselective (i.e., they inhibit both COX should be avoided.
isoforms), down regulating prostaglandin production in
rtY P0rilTs
inflammatory states, and interfering with functions of pros
tanoids (e.g., renal blood flow and gut mucosal integrity main
r Patients anticipated to be receiving long-term glucocorli
coid treatment (prednisone at >2.5 mg/d for >3 months)
tenance) (Table 8). Nonselective COX inhibitors also inhibit
should have a baseline clinical risk assessment for
thromboxane Ar, thereby inhibiting platelet function.
osteoporosis within 3 to 6 months of initiation of
Although they alleviate symptoms, COX inhibitors are not
therapy.
modiffing, with the possible exception of ankylosing
disease
spondylitis. Major concerns surrounding all COX inhibitors . Because of the increased risk for gastrointestinal bleed
include increased risk lbr gastrointestinal bleeding and adverse ing and adverse cardiovascular and renal events.
cardiovascular events; therefore, they should be prescribed at cyclooxygenase inhibitors should be prescribed at the
the lowest dose fbr the shortest time possible. COX inhibitors lowest dose for the shortest time possible.
should generally be avoided in patients receiving concomitant
anticoagulation.
NSAIDs vary with regard to kinetics, COX-112 selectivity,
Analgesics and Pain
and other features, and they carry somewhat different degrees
oi cardiovascular and other risks; having experience with sev
Pathway Modulators
eral different NSAIDs is beneficial in clinical practice. Pain is a central symptom for patients with inflammatory
A topical preparation of the NSAID diclofenac is available arthritis and osteoarthritis. It is important to use a patient
both by prescription and over the counter fbr arthritis. It poses centered approach while managing pain in these patients. ln
a lower risk for systemic adverse effects compared with oral addition to pharmacologic interventions, treatment should
NSAlDs. Topical NSAIDs are strongly recommended fbr include patient education, guidance on physical activity and
patients with knee osteoarthritis and conditionally recom exercise, orthotics, psychological and social interventions,
mended for patients with hand osteoarthritis. Topical sleep hygiene education, and, il'indicated, sleep interventions.

TABLE 8. Potential Toxicities of NSAID Use


Category Toxicity
Cardiovascular Myocardial infarction; exacerbation of heart failure
Hemostatic Platelet dysfunction
Gastrointestina I Dyspepsia; reflux; peptic ulcer disease; gastrointestinal bleeding
Obstetric/Gyn eco log ic Bleeding; delayed labor; premature ductus arteriosus closure
Pulmonary Asthma exacerbation
Renal Hypertension; decreased glomerular filtration; increased salt and water retention; increased renin produ
uncommonly, allergic interstitial nephritis or acute tubular necrosis

9
Principles of Therapeutics

Acetaminophen Disease-Modifying
The efficacy of acetaminophen for osteoarthritis and lower
back pain has been questioned. Controlled trials and meta
Antirheumatic Drugs
analyses have shown no benefit from the drug, even at high
Nonbiologic Disease-Modifying
doses. Hor.never, because of its favorable safety profile, it may
Antirheumatic Drugs
be empirically tried fbr short-term or add on therapy. The Table 9 summarizes the mechanisms of action, indications, and

daily dosage should not exceed 3000 mg/d. ACR guidelines common monitoring parameters of various nonbiologic disease
conditionally recommend acetaminophen for patients with modi$zing antirheumatic drugs (DMARDs). See Medications and
knee, hip, or hand osteoarthritis and limited pharmacologic Pregnancy tbr infbrmation on these drugs in pregnancy.
options.
Methotrexate
Methotrexate is a first-line medication for RA and other auto
Tramadol immune diseases. Once weekly dosing is generally 10 to
Tramadol is a mixed opioid analgesic and weak serotonin 25 mg; the drug can be given orally or subcutaneously. At doses
norepinephrine reuptake inhibitor (SNRI) with a lower poten- above 15 mg, parenteral administration is more reliable but
tial lor addiction than traditional opioids. It may be considered much more expensive.
in a limited number of patients for whom other methods of Potential adverse effects include headaches, latigue, and
analgesia were ineffective or not tolerated. Adverse efI'ects may
nausea (particularly around the time of weekly dosing).
include nausea, vomiting, constipation. lightheadedness, and Hepatotoxicity and cytopenia can occur (especially macro
sedation. Traditional opioids should generally be avoided in cytic anemia), and dose adjustment is required with kidney
rheumatologic treatment because of limited efficacy, high disease. Methotrexate should be avoided in patients lvith sig
toxicity, and high potential lbr dependence. See MKSAP 19 nificant hepatic or kidney disease and is absolutely contrain-
General Internal Medicine 1 fbr discussion of risk assessment
dicated around pregnanc),. Folic acid supplements minimize
and monitoring of long term opioid therapy.
toxicity while preserving efficacy. Limiting alcohol intake is
recommended.
Serotonin-Norepineph rine Reu pta ke I n hi bitors
Duloxetine is an SNRI approved by the FDA fbr the manage Hydroxychloroquine
ment of chronic musculoskeletat pain and fibromyalgia. Hydroxychloroquine is an immunomodulator widely used in
Duloxetine provides modest pain relief for knee osteoarthritis, systemic lupus erythematosus, in which it decreases mortality
chronic lower back pain. and fibromyalgia. Milnacipran is and the likelihood olnephritis. It is rarely sufficient as single
another SNRI approved for fibromyalgia. 'tb avoid withdrawal drug therapy fbr RA but is useful as an adjunctive therapy.
symptoms, patients must be slowly weaned off SNRIs when
the drug is discontinued. Sulfasalazine
Sulfasalazine is used to treat RA and nonaxial psoriatic arthri

Gabapentinoids tis. It is now most frequently used as part of combination


DMARD therapy fbr RA. Serious adverse effects include blood
Gabapentinoids (gabapentin and pregabalin) inhibit voltage
dyscrasias, hepatitis, and hypersensitivity reactions. Because
gated calcium channels. thereby reducing pain signaling from
ol the benefit of its salicylate moiety for inflammatory bowel
the periphery to the central nervous system. Pregabalin is FDA
disease, it may be a useful strategr lbr patients with inflamma
approved for fibromyalgia. Common adverse effects (dizziness,
tory bowel disease-associated arthritis.
disequilibrium, somnolence, weight gain. peripheral edema.
and cognitive difficulties) may limit its utility. The FDA has
Leflunomide
issued a safety alert stating that serious breathing difficulties
Leflunomide is FDA approved for RA and psoriatic arthritis,
may occur in patients using gabapentin or pregabalin who
with eflicacy similar to that ol methotrexate. Patients must be
have respiratory risk factors (e.g., older persons, patients with
monitored for hepatotoxicity and myelosuppression. Other
COPD, and patients receiving opioids and other drugs that
common adverse effects include nausea, headaches, rash. diar
depress the central nervous system).
rhea, and elevation o1'serum aminotransferase levels. Peripheral
I(EY POIilI5 neuropathy is an uncommon adverse effect that is usually self'
. The American College of Rheumatologr conditionally limited if the drug is discontinued. Leflunomide is highly tera
recommends acetaminophen for knee, hip, and hand togenic and absolutely cclntraindicated around pregnancy.
osteoarthritis.
o Traditional opioids should generally Azathioprine
be avoided in rheu-
matologic treatment because of limited efficacy, high Azathioprine is an immunosuppressant used in various inflam

toxicity, and high potential for dependence. matory diseases. Its primary toxicity is myelosuppression,
especially in individuals with a decreased or absent thiopurine

10
Principles of Therapeutics

TABLE 9. Nonbiologic Disease-ModifyingAntirheumatic Drugs


Agent Mechanisms of Action lndications Common Monitoring Parameters
Methotrexate Low dose: anti-inflammatory agent via RA; psoriasis; psoriatic Baseline: chest radiography, hepatitis
up-regulation of adenosine A2a signaling arthritis; IBD; SLE screening, CBC, LCTs, serum creatinine
(arthritis only); reactive
High dose: antimetabolite/{olate Thereafter: CBC, LCTs, serum creatinine
arthritis; DM; PM;
antagonist used in neoplastic disease after first month, then approximately
vasculitis
every 2-3 months"
Hydroxych loroquine Uncertain; appears to involve SLE; RA Baseline: CBC, LCTs, serum creatinine
stabilization of lysosomal vacuoles,
leading to inhibition of antigen Retinal examinations at baseline and
processing and/or inhibition of Toll-like annual examination after 5 years of
receptor activation therapy to evaluate for retinopathy

Sulfasalazine Unknown; the prodrug is broken down RA; SpA; IBD Baseline: CBC, LCTs, serum creatinine
into 5-amino salicylic acid (active
metabolite in gastrointestinal tract) and Thereafter: CBC, LCTs, serum creatinine
every 3-6 months
sulfapyridine (exerts systemic action)
Leflunomide lnhibits mitochondrial enzyme RA; psoriatic arth ritis Baseline: hepatitis screening, CBC, LCTs,
dihydroorotate dehyd rogenase to block serum creatinine
pyrimidine synthesis (decreasing
Thereafter: CBC, LCTs, serum creatinine
lymphocyte prod uction); antiproliferative
after 4 weeks, then every 3 months
Azathioprine Prodrug of 6-mercaptopurine; purine SLE; DM; PM; Baseline: CBC, LCTs, serum creatinine
analogue; inhibits DNA synthesis vasculitis; IBD
Thereafter: CBC, LCTs, serum creatinine
essential for proliferating T and B
every 3 monthsu
lymphocytes
Cyclophosphamide Alkylating agent; blocks DNA synthesis Severe and life- Close monitoring clinically and
and causes cell death, especially of threatening measuring CBC, chemistries, LCTs,
T cells complications in SLE, urinalysis every 4-8 weeks
DM, PM, and vasculitis;
may be used when
other agents fail
Mycophenolate Active metabolite (mycophenolic acid) SLE (especially lupus Baseline: CBC, LCTs, serum creatinine
mofetil inhibits purine synthesis; preferentially nephritis); vasculitis
(maintenance Thereafter: CBC, LCTs, serum creatinine
inhibits T and B lymphocytes
after 4 weeks and then every 3 monthsa
therapy); DM; PM; SSc
Cyclosporine, lnhibit calcineurin (transcription SLE; psoriasis; RA Baseline: CBC, LCTs, serum creatinine
voclosporin activating factor); preferentially target (cyclosporine)
Thereafter: CBC, LCTs, serum creatinine
T cells
Lupus nephritis every 2-3 months"
(voclosporin)
Tofacitinib, Janus kinase inhibitors RA (tofacitinib, Baseline: CBC, LCTs, serum creatinine,
baricitinib, baricitinib, lipid panel
upadacitinib upadacitinib)
Thereafter: CBC, LCTs, serum creatinine
Psoriatic arthritis every 8 weeks, lipids after 8 weeks and
(tofacitinib) then every 6 months
Apremilast Phosphodiesterase-4 in hibitor Psoriasis; psoriatic Baseline: weight
arthritis; oral ulcers
Thereafter: weight, neuropsychiatric
associated with
effects
Behget syndrome

CBC=complerebloodcount; DM=dermatomyositis; IBD=in{lammatorybowel disease; LCT=liverchemistrytest; PM=polymyositis; RA=rheumatoidarthritis; SLE=systemic


lupus erythematosus; SpA = spondyloarthritis; SSc = systemic sclerosis.

methyltransferase enzyme. Because azathioprine is metabo- Cyclophosphamide has largely been displaced by safer drugs
lized by xanthine oxidase, concomitant use with xanthine for first line treatment of ANCA-associated vasculitis and
oxidase inhibitors (allopurinol, febuxostat) is contraindicated. lupus nephritis (rituximab and mycophenolate mofetil,
respectively) but is still used in severe cases or when these
Cyclophosphamide agents fail. Serious potential adverse effects include severe
Cyclophosphamide has a rapid onset of action (days to weeks). immunosuppression, leukopenia, hemorrhagic cystitis, and
It is used to treat vasculitis, life threatening complications of ovarian failure, as well as long-term risk for bladder cancer,
systemic lupus erythematosus, and interstitial lung disease. leukemia, and lymphoma.

11
Principles of Therapeutics

Mycophenolate Mofetil Apremilast


Mycophenolate mofetil is the first-line agent for lupus nephri- Apremilast is modestly elfective for psoriasis and psoriatic
tis and may be effective for systemic sclerosis with associated arthritis. It does not cause immunosuppression or myelosup
interstitial lung disease. Gastrointestinal adverse effects, pression. However, apremilast is less efficacious than biologic
including diarrhea, are common. Myelosuppression may DMARDs and has a slow onset of action, and its effect on pro
occur. gression of erosive damage is unknown. Risks include gastro
intestinal adverse effects (mainly nausea and diarrhea) and
weight loss. It should be used with caution in patients with a
Calcineurin Inhibitors
Calcineurin inhibitors include cyclosporine, tacrolimus, and
history of depression. Apremilast is also FDA approved for
voclosporin. Cyclosporine is now rarely used for rheumato treatment of oral ulcers in Behget syndrome.
logic conditions because of renal adverse effects, hyperten TEY POIl{IT
sion, hyperuricemia (often presenting as gout), and need for . Methotrexate is a first-line medication for rheumatoid
frequent drug level monitoring. Tacrolimus may be considered arthritis and other autoimmune diseases.
as alternative therapy for some patients with lupus nephritis.
Voclosporin, a next generation calcineurin inhibitor with
. Hydroxychloroquine decreases mortality and the likeli-
hood of developing nephritis in patients with systemic
higher potency and improved side effect profile, is FDA
lupus erythematosus.
approved for lupus nephritis.
r Cyclophosphamide is used in severe cases of vasculitis
and lupus nephritis or when other agents fail.
Janus Kinase Inhibitors
Tofacitinib is an oral agent that inhibits Janus kinase (JAK) o The oral Janus kinase inhibitor tofacitinib is FDA
1/3 signaling. Tolacitinib is FDA appro'"ed for RA and psori approved for rheumatoid arthritis and psoriatic arthritis
atic arthritis, with efTicacy equal to that of biologic DMARDs. and has efficacy equal to that ofbiologic agents.
Risks include hyperlipidemia, hepatotoxicity, leukopenia,
reactivation of zoster (at a rate higher than seen with bio- Biologic Disease-Modifying Antirheumatic Drugs
logic therapies), and thrombotic events when used at higher Biologic DMARDs are highly specitic, parenterally adminis
doses. Newer JAK inhibitors approved for rheumatologic tered, protein based agents with extracellular targets (cytokines,
diseases include baricitinib (JAKI/2) and upadacitinib cytokine receptors, or cell surface molecules on immune cells)
(JAK1). (Flgure 1). The end of the generic name of a biologic agent

Anakinra
Macrophage

Dendritic cell
Neutrophil z'Ji-:',ii,"' tL-1

proteases

tL-6

k/'INF-a rilumab
T cell
Cartilage MMPs
Synovial Tocacitini f-Abatacept
Baricitinib
Upadacitinib
Bone
B cell
MMPs
TNF Rituximab
Osteoclast
T
lnf liximab RF,
\
anti-CCP
Etanercept anti bod ies
Adalimumab
Golimumab
Certilizumab
tIGURE l.Thein{lammatorycascadeinrheumatoidarthritis(RA).Dendriticcells,macrophages,andBcellspresentincitingantigenstoTcells.Macrophagessecrete

and other enzymes that contribute to the degradation of articular cartilage and activate neutrophils, which mediate joint damage through proteases and other enzymes.
Activated osteoclasts additionally secrete MMPs that contribute to marginal erosions of bone. lLs, TNF, T cells, and B cells may all be targeted for inhibition by the various
disease-modifying antirheumatic drugs useful in RA. CCP = cyclic citrullinated peptide; lL= interleukin; LTB4 = leukotriene Ba; MMP = matrix metalloproteinase; 02 = oxygen;
PG E2 = prostag la ndi n Ez; RF = rheu matoid factor; IN F = tu mor necrosis factor.

12
Principles of Therapeutics

TABLE 1O. Tumor Necrosis Factor lnhibitors"


Agent Agent Structure lndications
lnflixima Er Chimeric (mouse-human) monoclonal antibody RA; psoriatic arthritis; ankylosing spondylitis; IBD
Adalimumab Humanized monoclonal antibody RA; psoriatic arthritis; ankylosing spondylitis; IBD
Etanercept Fusion protein made of two p75 TNF receptors linked to RA; psoriatic arthritis; ankylosing spondylitis
lgG Fc segment
Certolizumab pegol Fab'segment of humanized monoclonal antibody RA; psoriatic arthritis; ankylosing spondylitis
attached to polyethylene glycol strands
Golimumab Humanized monoclonal antibody RA; psoriatic arthritis; ankylosing spondylitis

IBD = inf ammatory bowel d sease; RA = rheumatord arthr t s; TNF - tumor necrosis factor.

at baseline and every 3 to 6 months thereafter.

indicates what type of molecule it is: mcrb indicates a monoclo- term "biosimilar"); therefbre, they must undergo phase III
nal antibody, -kin indicates an interleukin |lpe substance. ro testing to prove equivalent efficacy to the parent biologic. In
is for a receptor antagonist, and .cept is for receptor based the United States, biosimilars are distinguished from their
molecule. A11 biologics require parenteral administration. originator molecules by the presence of a four character
Table l0 and Table 11 summarize the structures. targets. alphabetic suffix. Biosimilars hoid promise lbr decreasing cost
indications, and common monitoring parameters of various and improving access, but those benelits have rarely been real
biologic DN4ARDs. See Medications and Pregnancy fbr inlor ized because of regulatory and market factors.
mation on these drugs in pregnancy. Biologic DMARDs increase
r(tY P0 t llTs
the risk fbr infection to varying degrees. Targeted screening is
therefore necessary before initiation (see Vaccination and
. All biologic agents increase the risk for infection; there
fore, targeted screening is necessary before initiation.
Screening in Immunosuppression).
The cost of biologic agents is significant and may be a bar . Tumor necrosis factor inhibitors pose a particularly
rier to access. high risk for reactivation of tuberculosis, and patients
must be screened for latent infection befbre initiation of
Tumor Necrosis Factor Inhibitors therapy.
Tumor necrosis factor (TNF) inhibitors (see Table 10) are . Biologic agents that are not directed at tumor necrosis HVC
widely used for treating RA. psoriasis. psoriatic arthritis, and factor (TNF) are usually started after one or two TNF
ankylosing spondylitis, as well as several nonrheumatologic inhibitors have failed.
diseases.
TNF inhibitors are generally well tolerated; increased risk
for infection is the primary safety concern. Because TNF U rate-Loweri n g TheraPY
inhibitors pose a particul:rrly high risk for reactivation of
tuberculosis, all patients being considered for treatment must
Allopurinol
be screened for latent infection and, if needed, receive treat
Allopurinol is the recommended tjrst line urate lon'ering
agent. lt competitively inhibits the enzyme xanthine oxidase,
ment. Except for nonmelanoma skin cancer and possibly mel
blocking the conversion of hypoxanthine (a breakdown prod
anoma, l'NF inhibitors do not appear to increase the risk for
uct of purines) to uric acid. Allopurinol is metabolized kr oxy-
new cancers; the risk for malignant recurrence remains
purinol, which also inhibits xanthine oxidase. Allopurinol is
unclear. TNF inhibitors may exacerbate heart failure and rarely
FDA approved fbr dosages up to 800 mg/d. Allopurinol should
provoke a demyelinating condition. Over time, individual TNF
be initiated at 100 mg/d and titrated in 100 mg increments
inhibitors may lose efficacy owing to fbrmation of antidrug
until the therapeutic target is achieved. For patients with stage
antibodies.
chronic kidney disease, allopurinol should be initiated at
4 or 5
50 mg/d and titrated in 50 to 100 mg increments as needed.
Other Biologic Disease-Modiffing Antirheumatic Drugs
Multiple biologic DMARDs with non TNF extracellular and Allopurinol use is rarely associated with a hypersensitivity
cell surface targets have been approved by the FDA. Most of syndrome, most severely as DRESS (drug reaction with eosino-
these agents are started after one or two TNF inhibitors have
philia and systemic symptoms), a potentially latal reaction'
DRESS is also increasingly being reterred to as drug induced
tailed. See Table 11 for more information.
hypersensitiuity syndrome (DIHS) to emphasize the lact that
Biosimilars eosinophilia is not always present. Risk factors include chronic
Biosimilar agents are "copycat" versions of brand name bio kidney disease and diuretic use; allopurinol dosage titration
logic medications. The drugs are not exact replicas (hence the appears to substantially reduce the risk. Anclther DRESS risk

13
a

t
I

Principles of Therapeutics !
!

t
I

I
1

Agent Agent Structure Target lndications Comments I


I I

Abatacept Soluble CTLA4 receptor/lgG CD8O/CD86; RA Preferred for patients with history of
Fc segment chimera blocks T-cell severe i n{ection; relatively !
costimulation contraindicated in COPD 1

Rituximab Chimeric (mouse-human) CD20* B cells RA; ANCA-associated Given as intravenous infusion over l
i

monoclonal antibody vasculitis; occasionally several hours; has higher risk for infusion I

I
for SLE (off-label); reactions than other biologic DMARDs;
lgG4-related disease can cause hypogammaglobulinemia

Tocilizumab Humanized monoclonal lL-6 receptor RA; JIA; Castleman Can cause elevated aminotransferase I

antibody disease; GCA levels, hyperlipidemia, leukopenia,


thrombocytopenia; avoid in patients
with history of diverticulitis because of I
attendant risk for bowel perforation
Sarilumab Human monoclonal antibody lL-6 receptor RA Can cause elevated aminotransferase
levels, hyperlipidemia, leukopenia,
throm bocytopenia

Belimumab Human monoclonal antibody BLyS/BAFF SLE; lupus nephritis Phase llltrials showed small but
statistical ly significant im provement
versus standard therapy alone, with
glucocorticoid-sparing effect
Ustekinumab Humanmonoclonalantibody tL-12/lL-23 Psoriasis; psoriatic lnjectable; less robust effect than other
a rth ritis biologic DMARDs
Secukinumab Humanmonoclonalantibody lL-17 a Psoriatic arthritis; Phase lll trials suggest efficacy egual to
ankylosing spondylitis that of TNF inhibitors; can cause IBD
flares

lxekizumab Humanized monoclonal lL-17 a Psoriatic arthritis; Phase lll trials suggest efficacy equal to
antibody ankylosing spondylitis that of TNF inhibitors; can cause IBD
flares

Guselkumab Human monoclonal antibody lL-23 Psoriasis; psoriatic Selective lL-23 inhibitor
afthritis
Anakinra Recombinant receptor lL-1 B receptor RA; CAPSb; AOSD(off Rarely used in RA because efficary is
antagonist label); acute gouty inferior to that of other biologic DMARDs;
arthritis (off-label) reversible neutropenia can develop

Canakinumab Human monoclonal antibody rL-1 p CAPSb More expensive lL-1p inhibitor
Ri lonacept Dual lL-1B receptors chimerically tL-1 CAPSb; refractory More expensive lL-1 inhibitor
attached to lgG Fc segment gout
Mepolizumab Humanized monoclonal tL-5 EGPA; eosinophilic Only biologic approved fortreatment of
antibody asthma EGPA

arthritis; RA = rheumatoid arthritis; SLE = systemic lupus erythematosus; TNF = tumor necrosis factor.

'Before initiation of any biologic, tuberculosis screening must be performed. Complete blood counts should be performed every 3 to 6 months for all biologics, and aspartate
aminotransferase/alanine aminotransferase and a lipid panel should be checked every 2 to 3 months for tocilizumab.
blhe cropyrin-associated periodic syndromes (CAPS) include familial cold autoin{lammator syndrome, Muckle-Wells syndrome, and neonatal-onset multisystem inflammator
disease (chronic infantile neurologic, cutaneous, articular syndrome).

factor is the HLA-B'58:01 allele, which is more common in enzymes should be monitored. Concomitant use with purine
Black persons and persons of Han Chinese, Thai, and Korean analogues is contraindicated. Incidence of hypersensitivity is
descent and increases the risk for hypersensitivity by several rarer and usually less severe than with allopurinol. In a 2018
hundred-fold. Screening for HLA 8'58:01 in these populations safety study, febuxostat users had an increased risk for cardio
is conditionally recommended before initiation of allopurinol. vascular death and all-cause mortality compared with patients
Xanthine oxidase inhibitors should not be coadministered with receiving allopurinol. These data do not define febuxostat
purine analogues (such as azathioprine). See MKSAP 19 General as raising risk compared with no treatment, and other studies
Internal Medicine 2 for more information on DRESS. have failed to support this observation; however, the results
prompted the FDA to mandate a boxed warning for febuxostat
Febuxostat regarding increased risk compared with allopurinol. The FDA
Febuxostat is a noncompetitive xanthine oxidase inhibitor. has also limited the approved use of febuxostat to patients who
Elevated aminotransferase levels can rarely occur, and liver are unresponsive to or cannot tolerate allopurinol.

14
Principles of Therapeutics

Uricosuric Agents DMARDs and/or JAK inhibitors, such as tofacitinib, should not
Probenecid is an organic acid transport inhibitor that decreases receive live attenuated vaccines (e.g., for measles. mumps and
renal reuptake of uric acid. It is uncornmonly used because of rubella; varicella herpes zoster; influenza; and yellor,r, fever)
limited eflficacy, inconvenience, and limitations on use (e.g., because of risk fbr active inf'ection. Such patients can receive
drug interactions and adverse effects). the killed influenza and pneumococcal vaccines, as rtell as
recombinant herpes zoster vaccine as needed. Other non live
Pegloticase vaccines that are indicated should be administered as per
Unlike most other mammals, humans lack a functioning standard care (see MKSAP 19 General lnternal N4edicine 2 lbr
uricase to break down uric acid. Pegloticase is a recombinant, more information). Patients receiving traditional oral DMARDs
nonhuman, infusible pegzlated uricase that is highly efl'ective (e.g., hldroxychloroquine, methotrexate, and sulfasalazine)
at lowering serum urate. Pegloticase is reserved fbr severe may receive any vaccines as needed.
and/or refractory gout. Because of its extreme potency, mobi Before initiation ot immunosuppressive therapy, the fbl
lization flares of gout are common, and prophytaxis against lowing screening is recommended:
acute gouty attacks is required. Pegloticase is administered o Tuberculosis screening with tuberculin skin testing or
intravenously every 2 weeks; if the preinfusion serum urate interferon y release assay, particularly for patients initiat
increases to more than 6.0 mg/dl (0.35 nrnroll[-) on two occa ing biologic DMARDs
sions, antibodies have probably formed. and the drug should . Hepatitis B and C virus serologic testing (for patients initi
be discontinued to prevent infusion reactions. ating biologic DMARDs and drugs that can cause hepato
I(EY POIilIS toxicity)
. Allopurinol is the first line urate-lowering agent; the o HIV screening
biggest risk the drug poses is DRESS (drug reaction Patients with latent or active tuberculosis, active hepatitis
with eosinophilia and systemic symptoms). B virus infection, or untreated HIV infection require initiation
o Concomitant use of xanthine oxidase inhibitors (allo of appropriate therapy befbre initiating immunosuppression.
purinol or febuxostat) with purine analogues is contra- Patients with risk firctors fbr ongoing tuberculosis exposure
indicated. should have annual tuberculosis screening.

KtY POIlITS
o Whenever possible, vaccinations should be updated
Medications and Pregnancy before initiating biologic disease modifying antirheu-
Some rheumatologic medications can have adverse effects matic drug regimens.
on pregnancy. Table 12 lists these agents and tl.reir relative
o Screening (and therapy ifneeded) for tuberculosis,
risks.
hepatitis B and C virus, and HIV is appropriate before
I(EY PO IilIS initiation of immunosuppressive therapy.
o Methotrexate is highly teratogenic and abortifacient; it
must be discontinued at least 3 months before pregnancy.
. Hydroxychloroquine is relatively safe in pregnancy and Nonpharmacologic and
should not be discontinued ifit is needed. Nontraditional Management
r Leflunomide is extremely teratogenic and must not be Because rheumatologic diseases fiequently aflect the muscu
used before or during pregnancy; upon discontinuation, loskeletal system. nonpharmacologic measures are ofter.r
cholestyramine is required to remove the drug fiom the used to address pain not eliminirted by medications. 'fhese
body in all women of childbearing potential and specifi- measures include physical therapy, occupational therapy.
cally in those wishing to become pregnant. surgery, weight reductior.r, psychosocial support, and self
management programs. Many patients turn to complemen
tary and alternative medicine as adjuncts to traditional
Vaccination and Screening in medical interventions.
lmmunosuppression
Patients with autoimmune diseases should be assessed fbr Physical and Occupational Therapy
immunization status at diagnosis and/or initiation ol treat Physical therapists can help primary care physicians assess
ments. Whenever possible, patients should be updated with aerobic fitness and conditioning as well as ability to carry out
vaccinations at least 2 to 4 weeks before initiating biologic activities of daily livir.rg. Pain and lunctional limitation can be
DMARD regimens. Vaccine response may be diminished addressed through manual therapy, assistive devices, joint
during biologic immunosuppressive treatment (particularly protection techniques, irnd therntal treatments. A targeted
with B-cell depleting therapy). and patients receiving biologic exercise program can be initiated, and adapting the program

15
Principles of Therapeutics

TABLE 12. Rheumatologic Medications and Pregnancy


Medication/Class Comments
Anti-l nflammatory Agents
NSAIDs May impede implantation and be associated with small increased risk for miscarriage when used before
20 weeks' gestation. NSAID use after 30 weeks' gestation can lead to premature closure of ductus arteriosus.

G lucocorticoids Whentakeninfirsttrimester,canincreaseriskforfetalcleftpalateandraiseriskformaternalgestational
diabetes throughout the pregnancy.
Useful in managing active autoimmune disease in pregnancy. Nonfluorinated glucocorticoids (e.9.,
prednisone, prednisolone, methylprednisolone) have limited ability to cross placenta and are preferred,
except when treating the fetus (e.g., neonatal lupus erythematosus).
Colchicine Should be used only i{ I benefit justifies potential risk to fetus.
Analgesics and Pain Pathway Modulators
Acetaminophen Generally considered safe at standard dosing but does cross the placenta.
Opiates Some opiates/opioids cross placenta; may cause fetal opioid withdrawal at birth.
Tramadol Should be used only if potential benefit justifies potential risk to fetus; postmarketing reports suggest
possibility of neonatal seizures, withdrawal syndrome, and stillbirth.
Topical agents Topical use may limit serum levels; individual agents should be reviewed {or pregnancy impact before use
Nonbiologic DMARDs
Methotrexate Highly teratogenic and abortifacient; must be discontinued at least 3 months before pregnancy.
Hyd roxych loroquine Relatively safe in pregnancy and should not be discontinued if it is needed.

Sulfasalazine Relatively safe during pregnancy.

Leflunomide Extremely teratogenic; must not be used before/during pregnancy; upon discontinuation, cholestyramine
administration is required to remove drug from the body in all women of childbearing potential and
specifically in those wishing to become pregnanU should be followed up with measurement of le{lunomide
and its metabolite levels to ensure removal of drug.
Azathioprine Routine use in pregnancy is not recommended; however, azathioprine may be safer than some other
DMARDs and may be used if immunosuppressive agent is imperative.
Cyclophospha m ide Not used in pregnancy unless absolutely necessary.
Mycophenolate mofetil Teratogenic; should not be used in pregnancy; discontinue for 3 months before pregnancy is attempted.
Cyclosporine May be used in pregnancy only if benefits outweigh risks.
Tofaciti ni b; baricitinib; May be teratogenic at high doses.
U citin ib
Biologic DMARDs
TNF inhibitors Accumulating retrospective data suggest low risk in pregnancy, but evidence is limited; can be continued if
absolutely needed; different agents may have different considerations regarding crossing placenta.
Ustekinumab; anakinra; Should be used only if potential benefit justifies undefined risk to fetus.
secukinumab; sarilumab;
ixekizumab; guselkumab
Abatacept; belimumab; Should be used only if potential benefit justifies potential risk to fetus.
canakinumab; rilonacept;
rituxi mab; tocilizumab

Urate-Lowering Therapy (rarely needed in premenopausal women)


Allopurinol Should be used only if potential benefit justifies potential risk to fetus.
Fe bu xostat Should be used only if potential benefit justifies potential risk to fetus.
Probenecid No current evidence for adverse impact on pregnancy.
Peg loticase Should be used only if potential benefit justifies potential risk to fetus.

Dl\,4ARD = d sease mod fying antirheumatlc drug; TNF = tumor necrosis factor
t

for home use is critical. Physical therapy ref'erral is appropriate Occupational therapists assess upper extremity function
for tendinitis; bursitis: many forms of arthritis; and chronic ing, including the ability to perform self care and job related
soft tissue pain due to overuse, injury and chronic pain syn tasks. Braces and splints may be provided for painful or unsta
dromes (e.g.. fibromyalgia). ble joints. An ergonomic evaluation of the workstation may

16
Rheumatoid Arthritis

accompany instruction in improved body mechanics and most important is the class II HLA group, especially HLA D
avoidance of repetitir,,e trauma. alleles. These risk alleles code for the shared epitope, a flve
amino acid sequence that preferentially binds and presents
Complementary and Alternative Medicine citrullinated peptide antigens important in the pathophysiol-
Nontraditionai options for symptom management are used by ogr of RA. Citrullinated proteins are immunogenic, especially
about one third of patients overall and up to 90'l. of patients in people who have the shared epitope.
with chronic pain, including arthritis and rheumatologic dis Citrulline is not a native amino acid in humans; instead,
eases. Commonly used over the counter supplements include it is formed by the action of the enzyme peptidylarginine
fish oil. vitamins, glucosamine, and chondroitin. Providers deiminase (PADI), which deiminates arginine to form citrul-
should ask about supplement use because patients rarely vol- linated peptides. PADI expression is typically limited to sites of
unteer this information. Significant drug interactions may inflammation and may therefore provide a link between early
occur! for example, some herbal preparations can interact inflammation and subsequent autoimmunity. Many of the
with anticoagulants. See MKSAP 19 General Internal Medicine other genes associated with RA modiflz immune responses to
1 for discussion of cannabis for pain management. provide a milieu for the development of autoantibodies.
1\4ind body interuentions, such as tai chi, meditation, and
yoga, can improve psychological well being, strength, balance, Environmental Factors
and pain level. Chiropractic and osteopathic manipulation as Environmental factors are responsible for the other,l0% of the
well as massage remain popular. Randomized controlled trials risk for RA. One of the most provocative environmental factors
support the use of tai chi for arthritis; smaller trials suggest is smoking. Smoking can lead to lung inflammation, which
benefit from meditation techniques, yoga, massage, acupunc activates enzymes (including PADI), and may promote local
ture, and manipulative medicine for various musculoskeletal protein citrullination. Patients who smoke are at increased
problems. risk for RA, particularly those with a family history of RA, and
should be counseled about smoking cessation. Exposure to
Role of Surgery silica dust has also been associated with increased risk.
Surgical procedures, such as carpal tunnel release or rotator
cuff tendon repair, can address conditions that arise from lnfectious Agents
repetitive trauma, injury and degenerative changes in the soft One potential risk factor for RA is periodontal disease.
tissue. Synovectomy of inflammatory pannus is occasionally Porphyromonos gingiuolis, a bacterium associated with peri
used when a single or limited number ofjoints in patients with odontitis, expresses its own PADI enzyme and provides a
RA do not respond to medications. Total joint arthroplasty, potential link to formation of citrullinated peptide. Other
particularly of the knee or hip, can reduce or eliminate pain infectious agents implicated in RA include Mycoplasma spe-
and restore function in patients with an inadequate response cies, Epstein Barr virus, and parvovirus B19. However, a direct
to medication and physical or occupational therapy. infectious cause of RA has not been identified. There is also
rEY POITI interest in the role of the intestinal microbiome in RA. Gut
. dysbiosis has been postulated to promote early RA, possibly by
Nonpharmacologic measures used in rheumatologic
activating proinfl ammatory lymphocytes.
diseases include physical or occupational therapy,
surgery weight reduction, psychosocial support, and
Hormones
self management programs.
Women are tvvo to three times as likely as men to develop RA.
The role of estrogen and other sex-specific factors is incom-
pletely understood, and estrogen and other sex hormones have
Rheumatoid Arthritis both stimulatory and inhibitory effects on the immune sys
tem. Estrogen receptors are present on synovial fibroblasts and
Pathophysiology and Risk Factors may drive the production of carlilage-damaging metallopro-
Rheumatoid arthritis (RA) is a systemic autoimmune disease teinases. Stimulation of estrogen receptors on macrophages

characterized by chronic inflammatory polyarlhritis affecting


can increase production of tumor necrosis factor, a key RA
both large and small joints, with a characteristic predilection inflammatory cytokine.
for the joints of the hands and feet. RA has a prevalence of t( EY P0 t t{Ts
0.5u1, to 1'2, in the general population; some specific popula- o Potential risk factors for rheumatoid arthritis include
tions have rates as high as 77,. genetic and environmental factors, infectious agents,
and hormones; genes make up 60% of the risk.
Genetic Factors
. Smoking is an important and modifiable risk factor for
Genes are responsiblelbr 60'/. of the risk for RA. Among some
rheumatoid arthritis.
100 genetic loci recognized as associated with RA risk, the

17
;

I
Rheumatoid Arthritis

Another sign of early RA is the inability to make a fist, which


TABLE 13. 2010 American College of Rheumatology/ \
European League Against Rheumatism Classification Criteria is due to flexor tenosynovitis.
for Rheumatoid Arthritis The pattern of joint involvement is useful for diagnosing
Criteria 5core" RA. RA most characteristically affects the metacarpophalan .

geal joints, metatarsophalangeal joints, and proximal inter-


Joint lnvolvementb
phalangeal joints of the hands and feet but spares the distal
:

1 large joint (shoulders, elbows, hips, knees, ankles) U


interphalangeal joints of both the upper and lower extremities r
2-10 large joints 1
(Figure 2 and Figure 3). Wrists, elbows, shoulders, hips, knees, :
1-3 smalljoints (MCPs, PlPs, wrists, 2-5 MTPs) 2 and ankles also can be involved. RA tends to affect joints sym-
metrically (i.e., joints on both sides of the body are generally
!

4-10 smalljoints 3

>10 smalljoints 5 involved), but severity may be asymmetric. RA may occasion


ally present as persistent involvement in a single joint. RA
Serology
spares the thoracic and lumbar spine but affects the cervical :
Negativity for RF or anti-CCP antibodies 0
spine, especially the C1-C2 (atlantoaxial) articulation. See
Low levels of RF or anti-CCP antibodies (<3 times 2
Table 14 for more information.
the upper limit of normal)
High levels of RF or anti-CCP antibodies (>3 times 3
the upper limit of normal) :
Acute Phase Reactants
Normal CRP or ESR 0

Abnormal CRP or ESR 1


:

Duration" )
<6 weeks 0
:
>6 weeks 1

-\
CCP = cyclic citrullinated peptide; CRP = C-reactive protein; ESR = erythrocyte
sedimentation rate; N4CP = metacarpophalangeal; IVlTP = metatarsophalangeal;
PIP = proximal interphalangeal; RF = rheumatoid facton

"Six points needed {or classification as rheumatoid arthritis. :


bAt least one joint with definite clinical synovitis that is not better explained by
another disease. Joints excluded from this classification schema: first
carpometacarpal, distal interphalangeal, and {irst metatarsophalangeal. I

'A duration of 6 weeks or longer is less often seen with crystalline or viral causes.
l
From Aletaha D, Neogi T, Silman AJ, et al. 201 0 Rheumatoid arthritis classification
criteria: an American College of Rheumatology/European League Against t
Rheumatism collaborative initiative. Arthritis Rheum. 201 0 Sep;62(9):2569-81.
doi:1 0.1 002/art.27584. IPMID: 20872595] Copyright 20 1 0, American Col ege of
Rheumatology.Adapted with permission from John Wiley & Sons, lnc.

FIGURE 2. Earlyrheumatoidarthritisofthehands,withswellingmostprominently

Diagnosis in the third and fourth proximal interphalangeal joints.


)
The 2010 American College of Rheumatolos/ (ACR)/European
League Against Rheumatism (EULAR) classification criteria
for RA are sensitive and emphasize early diagnosis and treat-
ment to prevent permanent consequences of chronic inflam .

mation (Table 13). .,

Clinical Manifestations
RA is a chronic disorder, and onset of symptoms is usually
gradual. Patients with RA typically report joint pain and
inflammatory symptoms, including swelling and morning
stiffness lasting at least 30 to 45 minutes and commonly sev-
eral hours. Stiffness is also worse following rest and alleviated
by ongoing activity. Slmovitis is palpable on joint examination,
)
manifesting as softness or bogginess of the affected joint. l
Gently squeezing the joints may also elicit tenderness. As part \
of screening examination, the metacarpophalangeal and/or t IGU R E 3. Severe rheumatoid arthritis 0f the hands, with ulnar deviation and
the metatarsophalangeal joints can together be squeezed. subluxation at the metacarpophalangeal joints on both sides.

18
Rheumatoid Arthritis

TAgLE 14, Consequences of Persistent lnflammation on nor anti-CCP antibodies). Although categorized as having RA,
Joints and Supporting Structures these patients have somewhat different genetics and risk fac
Joint Area lmplications tors and generally have a better prognosis than those with
seropositive RA.
C1-C2 articulation Laxity of transverse ligament resuhs in
and transverse increased posterior motion of dens on Erythrocy.te sedimentation rate and C-reactive protein
ligament C2 because of atlantoaxial subluxation. level are elevated in 75% ofpatients with RA and can be used
Rheumatoid synovitis may lead to
to monitor treatment response. The systemic inflammation
erosion of dens; with neck flexion,
dens can affectthe midbrain and other inherent to RA is also commonly reflected by anemia of
vital neurologic structures. This must inflammation and modest thrombocytosis.
be evaluated before intubation or neck
manipulation in patients with long-
standing RA. lmaging Studies
Shoulder and rotator Restricted range of motion of Plain radiography of the hands and/or feet is the standard imag
cuff tendons glenohumeral joint and rotator cuff ing study for RA and can aid in diagnosis and assessing progres
tears sion. However, radiographs obtained early in the disease may be
Elbow joint Elbow contractures and difficulty with normal or reveal soft-tissue swelling only. Tlpical radiographic
hand pronation and supination
changes include periarticular osteopenia, marginal erosions,
Wrist carpaljoints Restricted range of motion of wrist; and joint-space narrowing (Figure 4 and Figure 5). Radiography
and finger tendons carpal tunnel syndrome; rupture of
of the cervical spine with flexion/extension views is appropriate
finger extensor tendons, especially
fourth and fifth ifCl-C2 subluxation is suspected.
MCPs and surrounding Ulnar deviation and subluxation of MRI and ultrasonography are more sensitive than plain
structures MCPs radiography for detecting abnormalities in joints and soft tis-
PlPs and surrounding Swan neck or boutonniere deformity sues and may have utility in monitoring disease, assessing risk
structu res due to inflammatory disruption of
periarticular su pport structures
Hip joint Axial migration of femoral head in
acetabulum (protrusio acetabuli)
Knee joint Tricompartmental joint-space narrowing
Ankle and mid-foot Restricted range of motion of ankle;
joints and tendons progressive pronated fl at foot deformity
MTPs and Fibular deviation of MTPs; cock-up
surrounding deformities; skin ulceration underneath
stru ctu res subluxed MTP heads
MCP = metacarpophalangeal; N4TP = metatarsophalangeal; PIP = proximal
interphalanqeal; RA = rheumatoid arthritis.

Laboratory Studies
The most useful laboratory studies to aid in the diagnosis of RA
are rheumatoid factor and anti-cyclic citrullinated peptide
(CCP) antibodies. Rheumatoid factor is an immunoglobulin,
usually IgM, directed against the Fc portion of IgG. Rheumatoid
factor occurs in approximately 70"/,, of patients with RA.
Because rheumatoid factor can be associated with other dis-
eases (e.g., viral hepatitis, endocarditis) and because its pres
ence increases with age, the specificity of rheumatoid factor
is limited at about 857,. Anti-CCP antibodies are also present
in 7ou/,, of patients with RA but have a specificity of 95'/u.
Nonetheless, a high pretest probability is required to make
both of these autoantibodies optimally useful for diagnosing
RA. Both of these antibodies may predate the onset of clinical F IGU R E 4. Radiograph showing advanced rheumatoid arthritis in the hand.

disease, but anti-CCP antibodies are more predictive than Shown are ulnar deviation at the metacarpophalangeal joints; marginal erosions
most prominently at the second through fifth metacarpophalangeal joints and the
rheumatoid factor for erosive disease in patients already diag
second and third proximal interphalangeal joints (arows); and joint-space nanowing
nosed with RA. at the wrist, metacarpophalangeal, and proximal interphalangeal joints and within
Approximately 70"/,' to 2O"/,, of patients diagnosed with RA the carpus, which also represents erosive disease. Note the loss of the ulnar styloid
are seronegative (i.e., positive for neither rheumatoid factor (anowhead), anolher common sign of bony erosion in rheumatoid arthritis.

19
Rheumatoid Arthritis

bone marrow edema, synovitis, and erosions but in clinical


practice is generally reserved for assessment of soft-tissue
derangements, such as tendon ruptures. MRI of the cervical
spine is specifically indicated if atlantoaxial involvement is
suspected.

TEY POIIITI
. Tlpical clinical manifestations of rheumatoid arthritis
include pain, swelling, and prolonged morning stiffness
in a symmetric pattern, particularly in the joints of the
hands and feet.
o The most useful laboratory studies to aid in the diagno- HVC
sis of rheumatoid arthritis (RA) are rheumatoid factor
and anti-cyclic citrullinated peptide (CCP) antibodies;
anti-CCP antibodies have a specificity of 95% for RA.
. Plain radiography ofthe hands and/or feet is the stand- HVC
ard imaging study for rheumatoid arthritis; radiographic
changes include periarticular osteopenia, marginal ero-
sions, and joint-space narrowing, although early radio
graphs may be normal.

Com pl ications a nd Bcra-Articu la r


Manifestations
Joints
RA joint damage is a consequence of synovitis within the joint.
The synovial lining, normally only a few cells thick, becomes
significantly expanded through proliferation of resident fibro
blast-like synoviocytes and immune system cells (T cells, B
cells, macrophages), as well as angiogenesis. Various cl.tokines
(tumor necrosis factor, interleukins), often signaling through
Janus kinases, and cartilage-damaging metalloproteinases are
produced. Receptor activator ofnuclear factor kappa B ligand
(RANKL), produced by inflammatory cells, activates osteoclasts
to erode bone. The metalloproteinases and activated osteoclasts
result in various irreversible changes to the joints (see Table 14).

Skin
The most common RA skin changes are rheumatoid nodules
(Figure 6), present in up to 30% of patients. Nodules typically
occur in the olecranon region and can be confused with gouty
tophi, but they can also occur over the hand and feet joints and
even in the lungs. Nodulosis can rarely be induced by certain
drugs (e.9., methotrexate).
Patients with RA are also at an increased risk for neutro
t I G U R E 5. Radiograph of rheumatoid arthritis in the foot showinq marginal
philic dermatoses, such as pyoderma gangrenosum or Sweet
erosions and joint,space nanowing at the third, fourth, and fifth metaiarsophalangeal syndrome (see MKSAP 19 General Internal Medicine 2).
joints (arows). Erosion at the fifth metatarsophalangeal joint is often the first Palpable purpura and skin ulcers may rarely result from small-
radiographic sign of rheumatoid arthritis foot involvement. vessel cutaneous vasculitis.

for progression, and determining response to therapy. Eyes


Ultrasonography is becoming a common tool for detecting The most common eye manifestation in RA is dry eye (kerato-
joint fluid (effusion), synovial tissue thickening, early ero- conjunctivitis sicca). It occurs in 10% to 15% ofpatients and can
sions, and increased vascularity. MRI can be used to assess be severe. Most of these patients also have dry mouth and are

20
Rheumatoid Arthritis

7- such as dyslipidemia and hypertension, should be addressed.

v" Clinically significant pericarditis is rare. Granulomatous myo


carditis, valvular disease (mainly mitral), conduction block,
and aortitis can occur but are very rare.

Hematologic
Anemia of inflammation is the most common RA hematologic
abnormality Felty syndrome is a rare condition consisting of
neutropenia and splenomegaly; it occurs in patients with long-
standing, severe, seropositive RA. Patients with Felty syndrome
are at risk for serious bacterial infections, Iower extremity ulcer-
ation, lymphoma, and vasculitis. Patients with RA can also have
large granular lymphocyte syndrome, which can progress to
large granular lymphoqte leukemia. Findings overlap with
Felty syndrome and include neutropenia, anemia, thrombocy
F I G U R E 5. Rheumatoid nodules appear as slowly developing, firm, painless, topenia, splenomegaly, and recurrent infections. Patients with
subcutaneous nodules (arows) located al pressure points or over the extensor RA are at increased risk for lymphomas (particularly large B-cell
surfaces of joints and tendons.
lymphomas), and risk is correlated with disease activity.

classified as having secondary Sj6gren syndrome (see Sjogren


Blood Vessels
Syndrome). Less common (1'l.) are episcleritis (inflammation of
Small-vessel cutaneous vasculitis occurs in a small percentage
superficial scleral vessels) and scleritis (inflammation of deep
of patients with RA, leading to palpable purpura or periungual
scleral vessels). RA is one of the most common diseases associ-
infarcts. A rare, larger-vessel vasculitis similar to po$arteritis
ated with scleritis, which can be vision threatening and lead to
nodosa can affect multiple organ systems; before current ther-
thinning of the sclera and perforation. Keratitis (comeal inflam-
apy, it had a s-year mortality rate of 30% to 50'2,.
mation) can occur; it is ulcerative and occurs at the periphery of
the comea. Severe keratitis is known as corneal melt. Both IEY POIilT'
scleritis and keratitis require immediate referral to an ophthal o Extra-articular manifestations and complications of
mologist (see MKSAP 1g General Internal Medicine 2). rheumatoid arthritis include rheumatoid nodules, dry
eye, interstitial lung disease, pleural effusions, and ane-
Lungs mia of inflammation.
Interstitial lung disease may develop in 507, of patients with . Rheumatoid arthritis conveys a significantly increased
RA; clinically significant disease is seen in 10'/. of patients and
risk for cardiovascular disease.
contributes to excess mortality. Bronchiectasis and bronchioli
tis occur, and the bronchiolitis can be obliterative and/or
constrictive. Pleural disease occurs in up to 5% of patients;
pleural effusions are exudative and can be large. RA pleural Management
effusions are characterized by low glucose and pH (mimicking See Principles ofTherapeutics for details on the uses, mecha
bacterial or tubercular infection and malignancy) and low nisms of action, major toxicities, and/or monitoring require-
complement levels, as well as elevated levels of total protein, ments of the medications used in RA, as well as recommended
rheumatoid factor, and lactate dehydrogenase. Infl ammatory vaccines.
cells in the RA effusion are characteristically mononuclear; a
neutrophil predominant effusion suggests infection. Upper General Considerations
airway involvement from cricoarytenoid arthritis occurs The 2015 ACR RA treatment guidelines advocate for early diag-
rarely; symptoms include hoarseness, sore throat, dysphagia, nosis and aggressive early therapy ofRA to prevent irreversible
and stridor. Cricoarytenoid arthritis can pose problems for cartilage and bone damage. A key treatment goal is to treat to
endotracheal intubation. target, with the target being achievement of remission or low
disease activity. Disease activity assessment involves making a
Heart measured determination using combinations of numbers of
Atherosclerotic heart disease remains the major cause of tender and swollen joints (typically by using 28 joints that
excess death in patients with RA, although some data suggest exclude the feet); patient and physician impressions of disease
that risk for cardiovascular disease may be decreasing toward activity; and, in some activity scoring systems, measurement
that of the general population. Patients with RA should be of the erythrocyte sedimentation rate or C reactive protein.
considered at high cardiovascular risk for purposes ofperiop- These parameters are combined into a composite score, thus
erative evaluation, and cardiovascular disease risk factors, assigning a disease activity ranging from remission to Iow

21
Rheumatoid Arthritis

to doses as high as 25 mg per week in partial responders; the


DMARD-
naiVe RA treating physician should generally maximize methotrexate
dosing before adding other agents. At doses greater than 15 mg
weekly, methotrexate oral absorption approaches its effective
DMARD Iimit; switching to subcutaneous administration allows for
monotherapy
(usually MTX) higher serum drug levels. Folic acid supplements minimize
toxicity without diminishing efficacy. Of patients with RA tak
ing methotrexate alone, 30% to 50'7, achieve remission or low
disease activity.
Moderate/high Low disease
disease activity activity Leflunomide may be useful in patients who cannot toler
ate methotrexate in both monotherapy and combination ther
apy. Hydroxychloroquine and sulfasalazine are less potent
Combination DMARD or Continue current agents and are rarely used as single agents since biologic
TNFi +/- MTX or therapy; if in disease modi$zing antirheumatic drugs (DMARDs) became
non-TNF biologic +/- MTX or remission, consider
available. However, both can be used in combination with
tofacitinib +/- MTX tapering medications"
methotrexate as triple therapy. Data suggest that triple therapy
is similar in efficacy to methotrexate combined with a tumor
Moderate/high Low disease necrosis factor (TNF) inhibitor, except in the area of radio-
disease activity activity graphic progression. Triple therapy, however, may be poorly
tolerated.
Tofacitinib, baricitinib, and upadacitinib constitute a class
Consider alternate
biologic/TNFi/small of FDA approved oral small molecule Janus kinase inhibitors
molecule/MTX therapy whose efficacy in RA is similar to that of the biologics.

FIGURE 7. Asimplifiedalgorithm presenting an initial approachtothe


treatment of both early and established rheumatoid arthritis (RA). All patients with Biologic Disease-Modiffing Antirheumatic Drugs
RA should receive a disease-modifying antirheumatic drug initially and be Biologic DMARDs can be used as monotherapy but are typi-
advanced to more aggressive and/or combination therapy as needed to control cally added to methotrexate when moderate to high disease
disease. Disease activity should be assessed, wherever possible, by using a Iormal,
activity persists. TNF inhibitors are the most frequently used
validated, and consistent disease activity index. Refer to the 201 5 American
College of Rheumatology RA treatment guidelines for more complex algorithms biologic DMARDs; they have a relatively rapid onset of action
accounting for differences between agents and patient-specific complexities. and demonstrate synerry with methotrexate. The combination
DMARD = disease-modifying antirheumatic drug; MTX = methotrexate; RA = of methotrexate and a TNF inhibitor slows the rate of radio-
rheumatoid arthritis; TNF = tumor necrosis factor; TNFi = tumor necrosis factor graphic damage to cartilage and bone, even in the setting of
in h ibitor.
continued clinical disease activity. This effect also has been
rDo nol disco0tinue all RA trealments.
shown with other biologics in combination with methotrex
ate. Other biologic DMARDs used in RA include abatacept (a
moderate, or high. The Clinical Disease Activity Index (CDAI)
selective T-cell costimulation modulator), tocilizumab (an
and Disease Activity Score 28 (DAS28) are two commonly used
anti-interleukin-6 receptor antibody), and rituximab (an
instruments to assess disease activity and response to treat
anti-B-cell antibody).
ment. Treating to target in RA results in less radiographic
Until better data are available to guide therapeutic deci
damage, reduced cardiovascular risk, and increased work
sions, choice of a biologic DMARD remains empiric and based
productivity compared with conventional care.
largely on patient characteristics, including what agents
The 2015 ACR RA treatment guidelines can assist in mak-
should be avoided because of patient comorbidity (e.g., avoid
ing initial treatment decisions in early RA. Treatment is typi-
ing abatacept in a patient with COPD).
cally advanced at 12 week intervals with the goal of reaching
remission or low disease activity as rapidly as possible. Patients
who remain in remission or a state of low disease activity for NSAIDS
6 months or longer may be able to reduce treatment intensity. NSAIDs are used as adjunctive therapy to control symptoms
Treatment decisions for established RA are more complex. An while the full effect of DMARDs is being awaited. These agents
initial approach to the treatment of both early and established are not disease modifizing and do not prevent joint damage.
RA is outlined in Figure 7. Use with glucocorticoids should be avoided and caution
should be exercised with concomitant methotrexate.
Disease-Modifying Antirheumatic Drugs
Nonbiologic Disease-Modiffing Antirheumatic Drugs Glucocorticoids
Methotrexate is the anchor drug in RA, used in both mono- Unlike NSAIDs, glucocorticoids can have a disease modi$zing
therapy and combination therapy. Methotrexate can be titrated effect, in addition to their efficacy in symptom management.

22
Osteoarthritis

Low dose prednisone (s-ts mg/d) can be used to rapidly xEY POtilts
improve RA symptoms until long term medications become . There is an increased risk for developing rheumatoid
effective, or they can be used short term for disease flares. arthritis in the first year after a first pregnancy.
Although glucocorticoid use is associated with a reduction in . Among women with established rheumatoid arthritis
radiographic progression, long term therapy with glucocorti-
at the time of conception, two thirds will go into
coitls has substantial potential adverse effects, including osteo-
remission or a state of low disease activity during
porosis, diabetes mellitus, and intbction.
pregnancy, and one third will not improve or will get
worse.
Surgery
Surgical therapy has become less common in RA because of
current treatment strategies. Nonetheless, some patients
may require synovectomy for a single persistently swollen
joint, carpal tunnel release, repair of a ruptured tendon, Osteoarthritis
total joint replacement (shoulder, metacarpophalangeal
joints, hip, knee), or joint fusion lbr a painful damaged Pathophysiology
joint (wrist or ankle). See MKSAP 19 General Internal Osteoarthritis (OA) is a chronic progressive multifactorial
Medicine 2 for a discussion of perioperative RA medication disorder of maladaptive cellular repair responses to joint
management. stress. Previously deemed a "wear and tear" disease and an
inevitable consequence ofaging, OA is now recognized as a
XEY POITIS
disorder driven by a complex interplay of genetics; biome-
. In rheumatoid arthritis, treating to a target of remis- chanics; cell stress; biochemical change in cartilage and
sion or low disease activity results in less radio- extracellular matrix components; and degradation and
graphic damage, reduced cardiovascular risk, and
inllammation, which is usually subclinical. It affects all tis
increased work productivity compared with conven-
sues of the joint and is characterized by cartilage and
tional care.
meniscal degradation, subchondral bone changes (bone
HVC o Methotrexate is the anchor drug in rheumatoid arthri- marrow lesions, subchondral sclerosis), and osteophyte
tis; it is used as both monotherapy and a component of formation. OA is accompanied by low-grade synovitis and
combination therapy. is driven by imbalanced anabolic and catabolic processes
. Tumor necrosis factor inhibitors are the most fre that represent aberrant joint responses and result in joint
quently used biologics to treat rheumatoid arthritis; tissue degeneration.
they have a relatively rapid onset of action and demon-
strate synerS/ with methotrexate.
HVC o In rheumatoid arthritis, NSAIDs are not disease Epidemiology and Risk Factors
modifying and do not prevent joint damage; they are OA is the most common form of arthritis worldwide and is a
used primarily to control symptoms while the full
leading cause of pain and disability, affecting approximately
effect of disease-modifying antirheumatic drugs is
30 million U.S. adults. OA is the leading cause of lower extrem-
awaited.
ity disability among older adults, with an estimated lifetime
. Low-dose prednisone can be used in rheumatoid risk for knee OA approaching 50%. Prevalence is projected to
arthritis to rapidly improve symptoms until long,term more than double by 2030, Iargely because of increasing obe-
medications become effective, but long-term glucocor- sity and the aging ofthe population. Incidence and prevalence
ticoid use carries significant comorbid risk. vary by joint and depend on whether a clinical or a radio
graphic definition is being applied.
Epidemiologic risk factors for OA include age older than
Pregnancy 55 years, female sex, obesity, genetics, and occupations that
There is an increased risk for developing RA in the first year involve repetitive motions or physical labor. Age and female
after a first pregnancy. Breastf'eeding may decrease this risk. sex are nonmodifiable risk factors for OA of different sites.
Among women with established RA at the time of concep Obesity is the most important modifiable risk factor, especially
tion, two thirds will go into remission or a state of low dis- for knee OA. Increased body mass is also a risk factor fbr hand
ease activity during pregnancy, and one third will not OA, perhaps underscoring the systemic nature of obesity and
improve or will get worse. Medication management is a a role for metabolic and/or inflammatory mechanisms in the
major issue, and pregnancy plans should be discussed with pathogenesis of hand OA. Single inherited conditions rarely
any woman of childbearing age who will be placed on ther predispose to OA; however, mutations in genes involved in
apy. RA medications in pregnancy are discussed in Principles bone or articular cartilage structure or metabolism are possi
of Therapeutics. ble risk factors.

23
Osteoarthritis

Risk factors involving the joint itself include abnormal


Classification
loading, injury malalignment, and intrinsic cartilage or
bone tissue defects. Injury may be acute (e.g., anterior cruci-
Primary Osteoarthritis
ate ligament tear) or accumulate over time (e.g., physical In most cases, no identifiable proximal cause of OA is recog
labor or overuse); regardless of cause, injuries have been nized. Although the term "primary OA" is commonly used for
linked with the future development of OA of various sites. this category a complex interplay of genetics, age, sex, biome-
Hip dislocation, congenital dysplasia, femoroacetabular chanics, biochemistry and inflammation is implicit. Primary
impingement, and knee malalignment are also associated OA may affect almost any joint, but clinically significant
with incident OA. presentations occur in the small joints of the hands (distal
Given the multifactorial causes, OA may develop as a con- interphalangeal [DIP], proximal interphalangeal [PIP], carpo-
sequence of one or more risk factors; how the interaction of metacarpal) and feet (first metatarsal phalangeal, midfoot) and
these risk factors culminates in OA is complex and not fully in the spine, hips, and knees. One or more of these areas might
understood. be affected and symptomatic in a given individual.

TEY POIIIIt
o Osteoarthritis is a chronic progressive disorder charac- Erosive Osteoarthritis
terized by cartilage and meniscal degradation, subchon- Erosive (inflammatory) OA is a subset of primary hand OA.
dral bone changes, osteophyte formation, and low-grade Erosive OA prominently affects the DIP and PIP joints, Ieads to

synovitis. more joint erythema and swelling than other forms of primary
hand OA, and is more common in women. Radiographs reveal
. Obesity is the most important modifiable risk factor for
diagnosis-defining central erosions with a "seagull" or "gull
osteoarthritis of the knee.
wing" appearance in the finger joints (Figure 8); joint

fir;t carpometacarpal joints (blue arows). Furthermore, metacarpophalangeal ioints are spared'
sclerosis aie present at both

24
Osteoarthritis

anMosis (bony fusion) may also occur. Whether erosive OA suggestedthat the aorta may serye as a mechanical barrier to
comprises a separate disease entity or is part ofthe continuum the production of bony hyperostosis on the left side of the
ofl OA remains controversial. spine.

TEY POITIS
Secondary Osteoarthritis
Secondary OA is historically defined in the presence ofa pre-
. In most cases, no identifiable proximal cause of osteoar-
disposing disorder; however, an increasing number of risk thritis (OA) is recognized; this form is categorized as
factors for primary OA are being identified, thus blurring the primaryOA.
line between the primary and secondary forms. Pathologic r Erosive osteoarthritis mostly affects the distal inter
changes, clinical presentations, symptoms, and management phalangeal and proximal interphalangeal joints, is
are indistinguishable between primary and secondary OA. characterized by erythema and swelling, and is more
When OA occurs in a joint not typically affected by primary common in women.
OA, or when it presents at a younger age, secondary causes . Commonly recognized causes of secondary osteoarthri-
should be entertained. Commonly recognized causes of sec- tis include a history of joint damage; congenitally
ondary OA include a history of joint damage, such as trauma, abnormal joints and/or malalignment; systemic meta-
infection, or surgical repair (e.g., anterior cruciate ligament bolic, endocrine, and neuropathic disorders; and
repair or meniscectomy); congenitally abnormal joints (e.g., underlying inflammatory arthritis with accompanying
hip dysplasia) and/or malalignment; systemic metabolic, damage.
endocrine, and neuropathic disorders, particularly those that o Diffuse idiopathic skeletal hyperostosis is a noninflam-
affect cartilage (e.g., hemochromatosis and chondrocalcino
matory condition characterized by calcification and
sis); and underlying inflammatory arthritis with accompany
ossiflcation of spinal ligaments and entheses; radio-
ing damage (e.g., rheumatoid or psoriatic arthritis) (Table 15).
graphs usually show confluent "flowing" ossification of
at least four contiguous vertebral levels, usually on the
Diffuse tdiopathic Skeletal Hyperostosis right side of the spine.
Diffuse idiopathic skeletal hyperostosis (DISH) is a noninflam-
matory condition characterized by calcification and ossifica
tion of spinal ligaments (especially the anterior longitudinal Diagnosis
ligament) and entheses (tendon and ligament attachments to
bone). DISH is more common in men. DISH usually presents
Clinical Manifestations
as back pain and stiffness, with the thoracic spine most often
OA diagnosis is based on history and physical examination;
involved. Although spinal ligamentous ossiflcation is also seen radiography is confirmatory but may be unnecessary. In early
OA, clinical findings may not be accompanied by radiographic
in ankylosing spondylitis, the spinal calcifications in DISH are
more "flowing," wider, and less vertically oriented than those changes; conversely, some patients with prominent radio
graphic changes may have minimal or no symptoms. Patients
seen with ankylosing spondylitis. DISH also lacks involvement
of the sacroiliac joints. with OA are typically older than 50 years of age; diagnosis at
Radiographic changes characteristic of DISH include con an earlier age should prompt inquiry into a history of joint
damage, endocrine or metabolic disorders, or a genetic pro
fluent ossification ofat least four contiguous vertebral levels,
usually on the right side of the spine (Figure 9). It has been clivity for early disease.
Joints most commonly affected are the hands (DIP, PIP,
and first carpometacarpal joints) (Figure 1O), feet (especially
TABIE 1 5. Secondary Causes of Osteoarthritis the first metatarsophalangeal and midfoot joints), knees, hips,
Secondary Cause Joint(s) Typically lnvolved and spine, but the distribution varies. When OA affects a single
joint, it more often results from injury or joint asymmetry and
Hemochromatosis Second/third MCP joints
often occurs in weight bearing joints (hip or knee). Generalized
Calcium pyrophosphate MCP joints; wrists; knees; hips;
OA, which affects multiple joint groups, often symmetrically,
deposition shoulders; atlanto-axial joint
may be the result of a combination of genetic and environ
Alkaptonuria/och ronosis Spine; hips; knees
mental factors.
Acromegaly Knees; shoulders; spine Patients with OA usually describe an insidious onset of
Hyperparathyroidism Wrists; MCP joints intermittent symptoms, which become more persistent and
Joint injury Knees severe over time. The most common symptom is joint pain
Sensory neuropathy Feet; ankles; knees exacerbated by activity and alleviated with rest. Patients also
(Charcot joints) describe morning stiffness usually lasting less than 30 min
lnflammatory arthritis Hands; wrists; feet; knees utes, in contrast to the prolonged moming stiffness of inflam-
matory arthritis. A single joint may initially be involved, with
MCP = metacarpophalangeal
eventual involvement of multiple loints.

2S
Osteoarthritis

B


?


?4 tl GU RE 9. Plain spinal radiographs showing diffuse idiopathic skeletal
hyperostosis. Note the calcification at sites of tendinous and ligamentous insertion
of the spine, taking the form of flowing ossification of multiple contiguous
€ vertebrae (yel/ow a rrows in A a nd 8). Also seen here is the typica I extensive
involvement of the right side of the spine and relative sparing of the left of the
thoracolumbar spine (/). (C) Calcification ol the anterior longitudinal ligament,
also common, is shown in the cervical spine (redarrows).Also shown is the thick/
wide osteophytes originating from the vertebral bodies, which ditfer from those
seen in ankylosing spondylitis (yel/ow arrowin C).

Pdn A r0urle,y o'50lprio' Gyhopoulos llD [./lSc

On joint examination, joint-line tenderness, crepitus, groin pain and decreased range of motion, especially internal
without synovi
decreased range of motion, bony enlargement rotation. Knee symptoms include pain on walking and clinlb-
tis. and sometimes eflusion (most commonly in the knees) ing stairs and difficulty transferring from a seated to a stand
may be present. Patients with long-standing hand OA may ing position. OA affecting the spine (spondylosis) can affect
have Heberden and Bouchard nodes (bony enlargement ofthe the vertebral bodies, facet joints, and neural foramina and may
DIP and PIP joints, respectively) and squaring ofthe first car Iead to spinal stenosis. See MKSAP 19 General Internal
pometacarpal joint. FIip involvement {ypically manifests as Medicine 1 for further discussion of spinal stenosis.

26
Osteoarthritis

FIGURE 10. Progressionof handosteoarthritis(0A).Thisimageillustratestheprogressionfromnormal handjoints(/eft),todistal interphalangeal (Dlp)jointbony


hypertrophy (Heberden nodes) of hand 0A (middle), to extensive hand 0A findings of Heberden nodes and proximal interphalangeal (plp) joint bony hypertrophy (Bouchard
nodes) (nght). Ihe hand on the right also displays a gouty nodule (tophus) overlying the Heberden node on the second Dlp joint, underlining the potential coexistence of these
two arthropathies. White arrowheads = Heberden nodes; black anows = Bouchard nodes; black asterisk = tophus.

Patients with OA generally do not have systemic features.


Pain and structural changes, however, ultimately result in
Differential Diagnosis
functional impairment, disability, psychosocial isolation, and Diagnosing OA can be challenging when OA is present in
reduced quality of life. Inability to exercise regular$ may have
atypical joints, an accurate history is difficult to obtain, or a
adverse consequences for general health.
concurrent inflammatory arthropathy may be present.
Calcium pyrophosphate deposition disease, gout, rheumatoid
arthritis, and psoriatic arthritis may coexist with OA or mimic
Laboratory and lmaging Studies
Laboratory testing is not necessary for diagnosis of OA but is
helpful ifother causes ofarthritis are being considered, such
as crystal arthropathy, rheumatoid arthritis, psoriatic arthritis,
or hemochromatosis (all of which can coexist with OA). Acute
phase reactant levels are not elevated in OA. Routine labora
tory tests (complete blood count, kidney and hepatic function)
are not necessary for diagnosing OA but may be important to
help define the safety of pharmacologic therapy, especially in
older persons and patients with comorbidities.
If a joint effusion is present, evaluation for concurrent
crystal arthropathy, infections, or other inflammatory causes
should be considered, ideally by synovial fluid analysis. OA
slmovial fluid is typically clear in appearance and noninflam-
matory with a leukoclte count of 2000/pL (2.0 x 10e/L) or less.
Although imaging is not necessary to diagnose OA, it can
be helpful to confirm the diagnosis, establish baseline severity
and/or monitor progressive severity, and exclude other diag
noses. Radiographic features of OA include asymmetric joint-
space narrowing, subchondral sclerosis, osteophytes, and
bone cysts (Figure 11); however, these changes may not be
present in early disease. Even in established OA, symptoms
may correlate poorly with imaging findings. Although MRI
and ultrasonography can detect subtle OA changes, they are ; I G U R E 1 1 . 0steoarthritis of the knee showing joint-space narrowing (JSN),
not needed for routine OA diagnosis. osteophytes (0P), subchondral sclerosis (SS), and bone cysts (BC).

27
Osteoarthritis

OA. Calcium pyrophosphate deposition disease may occur in receive individualized, often multidisciplinary treatment that
joints also fypically affected by OA (hands or knees) but is considers their expectations, functional and activity levels, occu-
associated with intermittent "flares," and radiographs show pational and vocational needs, joints affected, severity of disease,
cartilage calcification (chondrocalcinosis). Gout and OA com- and any coexisting medical problems. Instead of a single algo-
monly co-associate, particularly in the DIP joints. Rheumatoid rithm, the practitioner should consider the patient's clinical state
and psoriatic arthritis can be differentiated from OA by pro- and institute treatment, which may include single or multiple
longed morning stiffness, signs and symptoms of chronic and modalities that may be rycled in and out as needed (Figure ljl).
persistent inflammation, and elevated inflammatory markers.
Unlike OA, rheumatoid arthritis typically spares the DIP joints, Sttongly
reommended
and the presence of rheumatoid factor and/or anti-cyclic cit-
rullinated peptide antibodies favors the diagnosis of rheuma Conditionally
toid arthritis. Psoriatic arthritis can involve the DIP joints, but recommended

dactylitis and a history of psoriasis help differentiate it from HAND KNEE HIP
OA. In older patients with psoriasis and insidious-onset arthri Exarcker
vl
tis, OA may be a more likely diagnosis than psoriatic arthritis. u,l
I
U Self.Efficary and Self-Management Prognms
Synovial fluid analysis can also help distinguish between OA
and these inflammatory disorders.
o Weight los
d
o-
Other conditions to consider in evaluating a patient for o.
Tai Chi
OA include nonarticular sources of pain, such as bursitis and o Cane
tendinitis. Hip pain that is not in the anterior groin but instead o
lo
around the lateral hip and buttock may indicate trochanteric o lstCMC Orthosis TF Knee Bmcet
z
bursitis or lumbosacral radiculopathy. Similarly, knee pain
Heat, Th€rapeutic Cooling
may be secondary to pes anserine bursitis or iliotibial band =
o
z
syndrome rather than intra-articular pathologz, which may Cognitive Behavioral Therapy
J
present with medial or lateral knee pain, respectively. See s Acupuncture
U
MKSAP 19 General Internal Medicine 1 for further discussion o
]i Kinesiotaping
of nonarticular pain conditions. o
I
U
f,EY POIIIS vt Balance Training
o-
o The most common symptom of osteoarthritis is joint J
Other Hand Orthosesn PF Knee Brace**
pain exacerbated by activity and alleviated with rest; U
6 Pamffin Yoga
morning stiffness usually lasts less than 30 minutes. I
o-
HVC o Laboratory testing is usually not necessary to diagnose
RFA

osteoarthritis but is helpful if other causes of arthritis


are being considered or to help define the safety of u Oml NSAIDs
I
potential therapies. U Topical NSAIDs Topical NSAIDs
. o
cL
Radiographic features of osteoarthritis include asym-
& l-A Steroids l-A Steroids (lmaging Guidance for Hip)
metric joint-space narrowing, subchondral sclerosis,
osteophl'tes, and bone cysts; however, these changes
I|9 Acetaminophen
o
I
J
may not be present in early disease, and symptoms may Tramadol
correlate poorly with imaging findings.
Duloxetine
=
c,
I
A.
Management Chondroitin Topical Capsaicin

To date, the FDA has approved no agents to prevent, delay, or


remit FIGURE 1 2. Strongly and conditionally recommended therapies for
osteoarthritis (0A) of hand, knee, and/or hip. The figure implies no hierarchy within
the structural progression of OA. Instead, current treatment is
categories; the various options may be used (and reused) at various times
directed to the management of pain and disability Evidence throughout a patient's disease. lA= intra-articular; RFA= radiofrequency ablation.
based guidelines for OA management are available from the *Exercise
for knee and hip 0A rould include walting, strengthening, neuromuscular training, and aquatic
American College of Rheumatologr and Arthritis Foundation, the erercise, with no hierarchyof0neoveranother. Exer(ise isassociated wilh better0uttomes when superuised

Osteoarthritis Research Society International, the European **Knee


brace recommendations: tibiolemoral (TF) brace for TF 0A (strongly recommended), patellofemoral (PF)
brace f0r PF 0A (conditionally recommended).
League Against Rheumatism, and other organizations. There is
*'*Hand orthosis re(0mmendati0ns: first (arpometacarpal (CMC) joinl neoprene or rigid orthoses for first CMC
considerable consensus among these guidelines, particularly that joint 0A (slrongly recommended),0rthoses for joints of the hand olher than the Ii6t j0int (conditionally
CMC

optimal OA management requires a comprehensive plan that may recommended).

include educational, psychological, and phlsical management, trom Kolasinski SL, Neogi I Hochberg MC, et al. 201 9 American College ol Rheumatology/Arthritis Foundation
guideline for the management of osteoarthritis 0f the hand, hip, and knee. Arthritis Care Res (Hoboken). 2020;72:
along with pharmacologic interventions. Patients with OA should 149 162.IPlVlD:31908149] Reproduced with permission fromJohnWiley& 50ns, lnc.

28
Osteoarthritis

Nonpharmacologic Therapy other sites. However, topical NSAIDs may be limited in use for
The nonpharmacologic approach to OA starts with assessment hand OA because of practical circumstances. They also may be
of physical status, activities of daily living, health education, associated with more skin reactions and are more expensive
motivation, beliefs, and other biopsychosocial factors. An indi than oral NSAIDs. Other topical agents, such as capsaicin,
vidualized management plan includes education on OA and lidocaine, and methyl salicylate preparations, may also be used
joint protection, an exercise regimen, weight loss, proper foot- as adjunctive measures, especially when NSAIDs are ineffec
wear, and assistive devices as appropriate. Recent data suggest tive or are not tolerated.
that participation in self-efficacy and self-management pro Recent systematic reviews and meta analyses suggest that
grams that use a format based on multidisciplinary groups is acetaminophen provides no benefit for hip or knee OA but
associated with improvement in OA symptoms. Physical activ- may be considered as an add on therapy, as well as for short
ity includes graduated aerobic exercise and strength training, term and episodic use. ACR guidelines conditionally recom
with attention paid to strengthening periarticular structures mend acetaminophen for knee, hip, and hand OA in patients
and minimizing injury. Tai chi has also been shown to be as with limited pharmacologic options.
beneficial as physical therapy lbr knee OA pain and is strongly Duloxetine, a serotonin norepinephrine reuptake inhibi
recommended by the American College of Rheumatologr tor with central nervous system activity, has shown efficacy for
(ACR) for knee and hip OA. The combination of diet and exer pain from knee OA, suggesting a role for central sensitization
cise is more effective at decreasing OA related knee pain and in OA pain modulation.
dysfunction than diet or exercise alone; no evidence supports Opioids for OA should be avoided because evidence
following a specific diet to decrease OA symptoms. Weight loss shows only slight benefit and a high risk for toxicity and
provides meaningful improvement in symptoms that increases dependence. Opioid use should be considered only in limited
with greater loss of weight. The ACR also conditionally recom- circumstances when no other alternatives exist. Tramadol. a
mends cognitive behavioral therapy, acupuncture, and thermal partial opioid with fewer adverse effects and less addictive
interventions (locally applied heat or cold) for knee, hip, and potential than pure opioids, may be considered in some
hand OA. Balance exercises are conditionally recommended for patients in whom other analgesics have not relieved pain.
knee and hip OA, and yoga is recommended for knee OA. Nonetheless, concerns about the potential for adverse effbcts
of tramadol continue.
Patients with OA frequently use glucosamine, an oral
Pharmacologic Therapy
dietary supplement. Although glucosamine appears to be safb,
ln the absence of disease modiffing OA drugs, pharmacologic substantial evidence suggests that it lacks efficacy. ACR guide
treatment is considered when symptoms are bothersome to the lines strongly recommend against glucosamine in patients
patient and are inadequately or incompletely addressed by
with knee, hip, and/or hand OA. Chondroitin sulfate is strongly
nonpharmacologic interventions. Pharmacologic therapy lor recommended against in patients with knee and/or hip OA, as
OA includes oral, topical, and intra-articular medications. are combination products that include glucosamine and chon
Choice of treatment depends on individualized assessment, droitin sulfate, but chondroitin is conditionally recommended
with particular attention to comorbidities, concomitant medi for patients with hand OA.
cations, and, especially, adverse treatment effects. See Principles
of Therapeutics for details on the medications used in OA. Intra-Articular Injections
Patients with knee or hip OA who have inefficacy, intolerance,
Oral and Topical Agents or contraindication to oral and topical therapies may benefit
Oral agents for OA include NSAIDs, acetaminophen, duloxe from intra articular glucocorticoid injections. Despite the lack
tine, and tramadol. NSAIDs are efficacious in OA, regardless of of formal evidence for the utility of this therapy in joints other
anatomic location, and are the initial oral treatment of choice. than the knee and hip, there may be a possible benefit in other
However, their side effect profiles make sustained use prob joints, such as the first carpometacarpal joint. The ACR
lematic, especially in older persons and patients with comor strongly recommends using ultrasound guidance for hip-joint
bidities. A proton pump inhibitor may improve tolerance and glucocorticoid injections.
minimize risk; the proper choice of an NSAID should consider Long term harm from intra articular glucocorticoids has
drug pharmacokinetics and gastrointestinal and cardiovascu not been consistently demonstrated; however, benefit is usu
lar effects. For OA, NSAIDs should be used in the lowest pos ally short term and wanes within 3 months. Injections can be
sible dose and for the shortest possible time. Nonetheless, administered repeatedly but are not usually given more often
NSAIDs remain the mainstay of OA treatment. than every 3 months. At least one study has suggested that
Topical NSAIDs are saf'e, associated with the least sys knee joints undergoing repeated injections may be more likely
temic exposure, and effective for treatment of knee OA and to experience cartilage loss, but the effect size was small and
hand OA. For OA ofthe hands and knees, they are preferable the clinical significance remains controversial.
and should be considered before use oforal NSAIDs' They are Intra-articular hyaluronic acid injections are widely
not effective for hip OA and have limited efficacy in OA of used for knee OA in patients with inadequate response to

29
Fibromyalgia

intra articular glucocorticoids. The quality of trials assessing condition (prevalence, 2'X, 3'X,) is more predominant in women
efficacy ofintra articular hyaluronic acid is variable, and the than men, and age at onset is usually be[ween the third and
degree of benefit is low. Hyaluronic acid injections may be sixth decades.
considered in patients with knee OA in whom other treat- Fibromyalgia is not an inflammatory condition but rather
ments have failed and surgery is not feasible. appears to be a disorder ofpain perception and processing. lts
Intra articular stem cells, platelet rich plasma, biologics, pathophysiologr is not well understood. However, it is thought
and botulinum toxin are not ef'ficacious and are not to relate to central sensitization, characterized by amplifica
recommended. tion of pain signaling within the central nervous system.
Neuroimaging of patients with fibromyalgia shows abnor
Surgical Therapy malities within structures responsible for pain transmission
Surgery for OA is considered when nonpharmacologic and and interpretation, and accruing data demonstrate a genetic
pharmacologic approaches fail to control pain or improve predisposition.
functional limitation. Multiple high quality studies have
shown that arthroscopic surgery for knee OA provides no bet-
ter outcomes than conservative management unless there is
Diagnosis
joint buckling, instability, or locking, or a concomitant and The characteristic clinical features of fibromyalgia are wide
symptomatic mechanical disorder. spread chronic pain; fatigue; sleep disorders (both disrupted
In contrast, total joint replacement is a "curative" option and nonrestorative sleep); and cognitive symptoms, often
fbr patients in whom conservative therapies have failed; it termed "fibrofog." Patients flrequently have polysymptomatic
relieves pain and improves function. Overall, Iong-term out distress across multiple systems, including paresthesia,
comes are excellent, although hardware loosening and inf'ec chronic headaches, temporomandibular joint disorder, irri
tion may occur. table bowel syndrome, and chronic pelvic pain. Two instru
Recent studies suggest that fbr patients with morbid obe ments are available to aid in the diagnosis of fibromyalgia:
sity and knee OA, bariatric surgery may significantly improve one issued jointly by the American Pain Society (APS) and
pain and function even without further OA management. the FDA, called the Analgesic, Anesthetic, and Addiction
Clinical Trial Translations lnnovations Opportunities and
l(tY PotilTs Networks (ACTTION) APS Pain Taxonomy, and a 2016 revision
HVC o Nonpharmacologic therapy for osteoarthritis includes of the 2010 American College ol Rheumatologr Preliminary
education, an exercise regimen, weight loss, proper Diagnostic Criteria (Table f6 and Figure 13). A diagnosis of
footwear, and assistive devices as appropriate. fibromyalgia obtained by using these tools is valid irrespective
. Pharmacologic therapy for osteoarthritis includes oral, of other diagnoses and does not exclude other clinically
topical, and intra articular medications; choice of treat- important illnesses.
ment depends on individualized assessment, with par These diagnostic tools are based entirely on patient
ticular attention to comorbidities and concomitant reporting of symptoms, and they dispense with the assess
medications. ment of tender points that was incorporated into prior criteria
(see Table 16).
Although tender points may be characteristic of
HVC o NSAIDs and duloxetine are conditionally recommended
for hip and knee osteoarthritis; acetaminophen is sug- fibromyalgia, especially in female patients, the finding of spe-
gested as add on therapy. ciflc tender points on physical examination has multiple
shortcomings in practice, including lack of reproducibility
HVC o Arthroscopic surgery is not indicated in patients with (particularly in men) and greater association of tender points
osteoarthritis unless there is joint buckling, instability,
with anxiety than with the pain itself. Technically, tender
or locking, or a concomitant and symptomatic mechan-
point assessments require properly administered pressure,
ical disorder.
such as may be achieved with a dolorimeter, an instrument
o Totaljoint replacement relieves pain and improves not routinely available.
function in patients with osteoarthritis in whom con- Extensive laboratory studies (e.g., antinuclear antibody
servative therapies have failed. [ANA] testing) should also not be performed unless other
specific diagnoses are suspected. Positive ANA test results,
particularly at low titers, are nonspecific and have a low

Fibromyalgia positive predictive value. Positive findings on ANA screening


tests are highly prevalent in both the general population and
patients with fibromyalgia. In addition to ANA positivity
Epidemiology and
driving unnecessary additional testing, patients with fibro
Pathophysiology myalgia and positive ANA results are often incorrectly
Fibromyalgia is characterized by widespread chronic pain, diagnosed with a connective tissue disease and given inap
fatigue, disturbed sleep, and mental fbgginess. This common propriate and potentially harmfut therapy. For example,

30
Fibromyalgia

TABLE 16. Comparison of ACR Diagnostic Criteria and ACTTION-APS Pain Taxonomy Diagnostic Criteria for Fibromyalgia,
201 0 ACR Preliminary Diagnostic Criteria (2016 Revision) ACTTION-APS Pain Taxonomy Criteria (2019,
Widespread pain by Widespread Pain lndex (19-point scale) Musculoskeletal pain defined as >6 pain sites from total o{
assessing number of regions in which patient has painb,' 9 possible sites
Symptom severity by Symptom Severity Scale (1 2-point scale) Moderate to severe sleep problems orfatigue
assessing fatigue, nonrestorative sleep, and cognitive symptomsb'

Duration of symptoms >3 months Musculoskeletal pain plus fatigue or sleep problems must have
been present for >3 months
No other disorder explaining the pain

ACR=AmericanCollegeofRheumatology;ACTTION=Analgesic,Anesthetic,andAddictionClinica TrialTranslationslnnovationsOppodunitiesandNetworks;APS=American

"A diagnosis of fibromyalgia does not exclude the presence of other clinically important illnesses.
r'Fibromyalgia is diagnosed in the setting of Widespread Pain lndex 27 plus Symptom Severity Scale score >5, or Widespread Parn lndex >3 plus Symptom Sever ty Scale score )9.

Head

illl,*t,,,'n
ftight erm

i*f cn"'t
il j*ao,"n
i]l unoo, back and spine

ll io*u, back and spine. includangbuttocks

Lert leg

Right leg

back front
FIGURE l3.Numberof painfulbodysites.Patientsareaskedtochecktheareasinwhichtheyexperiencepainonthetwoviewmanikins(ignoringthepreshadedareas).
Alternatively, patients may use the checklist ol body sites. Ihe number of separate sites is summed from a maximum of nine body sites.

studies have shown that many patients with fibromyalgia erythematosus, rheumatoid arthritis) are also associated with
with a positive ANA result are unnecessarily treated with a higher risk for coexistent fibromyalgia than seen in the gen
high dose glucocorticoids. eral population. In patients with autoimmune disease and
Alternative diagnoses of treatable conditions that may comorbid fibromyalgia, it is crucial to recognize when comor
provoke or be associated with fibromyalgia include hypothy bid fibromyalgia is the primary driver of musculoskeletal
roidism, polymyalgia rheumatica, and depression. Multiple symptoms in order to avoid overtreatment with immunosul.l
rheumatologic conditions (Slogren syndrome, systemic lupus pressive agents.

31
5 pondyloa rthritis

serotonin reuptake inhibitor (e.g., sertraline) in the morning,


Management may mimic the aggregate effect of the SNRIs while helping
Optimal management of fibromyalgia requires a holistic normalize the sleep-wake pattern.
approach, including education, exercise, and psychosocial
Tramadol has a dual effect as both an agonist of p opioid
support. Pharmacotherapy is often warranted, although non
receptors and an SNRI. Although studies have shown tramadol
pharmacologic measures remain a cornerstone of treatment.
is helpful, it is now classified as an opioid, with associated
concerns of class side effects, including tolerance and addic-
Nonpharmacologic tion. These concerns are less extreme than with other opioids,
Patients should be educated regarding their condition, with which are contraindicated in fibromyalgia.
validation that their symptoms are real and reassurance that Evidence does not support NSAIDs for treating
their pain is not due to tissue injury and will not lead to per fibromyalgia.
manent disability. Aerobic exercise can improve well being The patient and physician should understand that fibro
and function as well as reduce pain. Because patients initially
myalgia is a chronic illness. The benefit of treatment will likely
experience postexercise pain and thus may be unwilling to be partial and usually temporary often requiring a rotating
continue, exercise must be introduced gradually and sup sequence of treatments. Nevertheless, treatment can help
ported encouragingly. Although evidence supporting their use patients manage and cope with their symptoms as well as
is modest, modalities that can be helpful include tai chi, medi
maintain function and autonomy.
tation, and acupuncture.
Patients with fibromyalgia should be assessed for psycho- xtY Poil{Ts
social stressors and psychiatric illness, including a history of e The characteristic features of flbromyalgia are widespread
trauma and abuse. In such cases, referral for psychological chronic pain, fatigue, sleep disorders, and cognitive
support is essential, although access to practitioners with symptoms.
experience treating chronic pain conditions is often a barrier. r The diagnosis of fibromyalgia is no longer based on HVC
Social work support may help reduce the stress burden. Even physician examination findings but rather on a careful
in patients without specific psychosocial stressors, psychologi- characterization of symptoms using a validated scoring
cal support for illness and pain management may be advisable. tool.
Cognitive behavioral therapy has shown modest benefit in
. Nonpharmacologictherapy(education,exercise,psycho- HVC
reducing pain, negative mood, and disability.
social support) remains a cornerstone of treatment for
fibromyalgia, and pharmacotherapy is often warranted.
Pharmacologic
Choice of pharmacologic therapy is based on symptom profile,
patient comorbidities, and medication adverse effects because
no trials have directly compared the efficacy of medications. Spondyloafthritis
The antiepileptic agents gabapentin and pregabalin (the
latter FDA approved for fibromyalgia) improve quality of Iife Overview
and decrease pain in the short term. In principle, these agents Spondyloarthritis comprises arthritic disorders classified
act by reducing pain signals from the dorsal root ganglia to the together by common genetic and environmental risk factors
thalamus. Adverse effects, including sedation, disequilibrium, and clinical manifestations. Spondyloarthritis includes anky
weight gain, and peripheral edema, are frequent. In older losing spondylitis, psoriatic arthritis, reactive arthritis, and
patients, gabapentinoid doses should initially be low and then enteropathic arthritis. Their common feature is enthesitis
be slowly titrated, as appropriate. Pregabalin is now classified (inflammation at the points where tendons, ligaments, and
as a controlled substance. joint capsules insert into bone). Clinically, enthesitis mani
Tricyclic antidepressants (such as amitriptyline) raise fests as inflammation, pain, and new bone formation in and
norepinephrine and serotonin levels and have mild benefit, around peripheral joints; dactylitis (diflfuse digital swelling,
although efficacy may wane with time. These agents induce also called sausage digit, including the joint capsule, tendons,
drowsiness, with potential benefit for disordered sleep, and and ligaments) (Figure 14); and, in many cases, ossiflcation at
should be taken just before bedtime. Anticholinergic and anti the spinal entheses. The term spondyloarthritis reflects the
histaminic adverse effects may be limiting; low doses are rec fact that most of these conditions also tend to involve the
ommended (e.g., fO 25 mg of amitriptyline). spine.
Two serotonin-norepinephrine reuptake inhibitors
(SNRIs), duloxetine and milnacipran, are FDA approved for
fibromyalgia and have modest effects on pain. They may be Pathophysiology
particularly appropriate in patients with concomitant depres- Genes and the environment interact to contribute to spondy-
sion. Some practitioners report that the administration of a Ioarthritis pathogenesis. The exact mechanisms of these pro
tricyclic antidepressant at bedtime, combined with a selective cesses remain incompletely understood.

32
Spondyloarthritis

spondylitis. Consistent with this observation, fibrocartilage


rich joint structures, such as the iiium and acetabulum, are
preferentially destroyed in ankylosing spondylitis; the hyaline
cartilage of the sacrum and femoral head are mostly spared.
Fibrillin 1, a protein commonly found in fibrocartilage, may be
a target molecule. The anterior eye, proximal aorta, and ace-
tabulum are all occasionally involved in spondyloarthritis and
are rich in flbrocartilage/fibrillin 1.

TEY POIIT'
r Spondyloarthritis comprises arthritic disorders classi-
fied by common genetic and environmental risk factors
and clinical manifestations; they include ankylosing
spondylitis, psoriatic arthritis, reactive arthritis, and
enteropathic arthritis.
tIGURE 14. Dactylitisor'tausagedigit"oftheleftseconddigitinpatientwith o Certain subtlpes of HLA-B27 are strongly associated
psoriatic arthritis, showing {usiform swelling (swelling extending throughout the with axial spondyloarthritis; however, HLA-827 alone is
digit and not limited to the joint) of the metacarpophalangeal and interphalangeal insufficient to cause axial spondyloarthritis.
joints and interarticular soft tissue as compared to the normal third digit.

Genetic Factors
Certain subtypes of HLA-B27, a class I major histocompatibility
Classification
complex (MHC) molecule that presents antigens to immune Four distinct but overlapping diseases comprise spondyloar
cells, are strongly associated with spondyloarthritis with axial thritis (Table 17):

involvement (most commonly ankylosing spondylitis). Up to l. Ankylosing spondylitis


90'2, of White patients with ankylosing spondylitis and 60% of 2. Psoriatic arthritis
Black patients with ankylosing spondylitis are HLA-B27 posi-
3. Enteropathic arthritis
tive. However, HLA 827 alone is insufficient to cause ankylos
ing spondylitis: Among White persons who carryr the HIA 827 4. Reactive arthritis
gene, only 6% will develop disease. Other genes are therefore Some taxonomies include a fifth, undifferentiated cate-
also important to spondyloarthritis risk, some of which have gory for patients with features of spondyloarthritis but insuf-
been identified (e.g., interleukin-23 receptor polymorphisms). ficient manifestations to permit classification into one of the
four categories. For example, a patient with episodic dactylitis
Environmental Factors and a history of uveitis would not flt neatly into one of the
The prevalence of spondyloarthritis is low in West Africa, even standard categories but would be classified as having periph
among HIA B27 positive people. However, when HIA B27 eral spondyloarthritis by criteria discussed below.
positive West Africans immigrate to the United States or Emphasizing the relatedness of all four categories, the
Europe, the prevalence of spondyloarthritis increases to the Assessment of SpondyloArthritis international Society devel
local population level. This suggests acquired risk related to oped classification criteria that eschew individual diseases and
environmental modifi cation. instead divide patients into those with axial involvement
Intestinal dysbiosis or a proinflammatory microbiome is (Figure 15) and those with involvement of peripheral joints
common in spondyloarthritis and may contribute to this only (Figure 16). Although these criteria were designed for
observation. Terminal ileitis, similar to that seen in Crohn study enrollment rather than clinical practice, they demon
disease, occurs in up to 60% of patients with ankylosing spon- strate the common clinical features that are part of the spon
dylitis, emphasizing the overlapping nature of the spondyloar dyloarthritis family.
thritis disorders. Intestinal inflammation is thought to lead to Axial spondylarthritis can be categorized as radiographic
a permeable gut, allowing exposure of dysbiotic bacteria (definite radiographic sacroiliitis) or nonradiographic (mini
(especially flagellated bacteria) to the intestinal immune sys mal or no radiographic sacroiliitis). Although patients with
tem. Dysbiosis may also play a role in psoriatic arthritis. nonradiographic spondyloarthritis often have milder disease
Similarly, reactive arthritis has a well-known association with and HLA-B27 is less common, these patients have lower rates
enteropathic as well as urogenital infection. of response to current treatments.

I Tissue Factors Ankylosing Spondylitis


: The enthesis organ has a high content of fibrocartilage, which Ankylosing spondylitis is a chronic inflammatory disease
seems to be a target in spondyloarthritis, especially ankylosing affecting the axial skeleton (including sacroiliac joints),

I 33
Spondyloa rth ritis

Ankylosing Spondylitis PsoriaticArthritis EnteropathicArthritis ReactiveArthritis


Musculoskeletal
Axial involvement Axial involvement is May occur at any level; May have Less common than in
hallmark; initially may start in the cervical asymptomatic or other forms of
symmetric involvement spine; may skip regions asymmetric Sl joint spondyloarthritis but
o{ Sl joints and lower disease or have typical may be asymmetric
spine, progressing ankylosing spondylitis;
cranially axial disease does not
parallel bowel activity
Peripheral Enthesitis; may have Clinical subtypes of PsA: Pauciarticular large and Enthesitis, tenosynovitis
involvement asymmetric large-joint medium joints; and asymmetric large-
1. Oligoarthritis peripheral arthritis can joint oligoarthritis;
oligoarthritis, including
2. Polyarthritis parallel IBD activity usually self-limited and
hips and shoulders; hip
3. DlP-predominant nonerosive; persistent
involvement can cause
4. Arthritis mutilans Polyarticular peripheral arthritis in up to 25olo
si gnifica nt functional
5. Axial spine involvement arthritis of small and and may develop MTP
limitation; dactylitis large joints does not
Enthesitis, dactylitis, and erosions
parallel bowel disease;
tenosynovitis are dactylitis; enthesitis
commonly seen
Dermatologic Psoriatic-like lesions Psoriasis typically hToderma Keratoderma
may rarely occur precedes joint gangrenosum; blennorrhagicum and
involvement; nail erythema nodosum circinate balanitis (rare)
pitting; onychodystrophy

Ophthalmologic Anterior uveitis Conjunctivitis more Anterior uveitis most Conjunctivitis more
(unilateral, recurrent) common than anterior common; conjunctivitis, common than anterior
uveitis keratitis, and episcleritis uveitis
are rare
G astrointestina I 60% with ileocolitis on Dysbiosis may play a Gl inflammation is Preceding Gl infection
colonoscopy role in inflammation hallmark common
Genitourinary Urethritis (rare) Prior GU infection in
some patients; sterile
urethritis; prostatitis;
cervicitis; salpingitis
Cardiovascu lar lncrease in CAD; aortic lncrease in CAD Thromboembolism Rare aortic valve disease
valve disease; aortitis; and conduction
conduction abnormalities abnormalities
Pulmonary Restrictive lung disease Rare cases of large
from costovertebral airway stenosis
rigidity; apical fibrosis
(rare)
Bone quality Falsely elevated bone lncreased risk for High risk for vitamin D Localized osteopenia
mineral density from fracture (multifactorial) deficiency, low bone
syndesmophytes; density, and fracture
increased risk for spine
fractu re

CAD = coronary artery disease; DIP = distal interphalangeal; Gl = gastrointestinal; GU = genitourinary; IBD = inflammatory bowel disease; MCP = metacarpophalangeal; MTP =
metatarsophalangeal; PIP = proximal interphalangeal; PsA = psoriatic arthritis; Sl = sacroiliac.

entheses, and peripheraljoints. Ankylosing spondylitis is also Articular Manifestations


known as a radiographic axial spondyloarthritis. Prevalence Ankylosing spondylitis initially presents with subacute onset
is 0.32% in North America but only O.O7ok in Africa, largely of inflammatory low back pain, including pain and stiffness of
following the population prevalence of HLA-B27. Ankylosing the low back and/or buttocks at night and in the morning that
spondylitis is more frequent in men than women (ratio, improves with activity. The synovial facet joints are also
2 3:1). The penetrance of ankylosing spondylitis is 12'lu lrom involved, and back extension may be painful. Over time, the
father to son and 5'2, from father to daughter, highlighting the thoracic and cervical spine can become involved. The disease
protective effect of female sex. The disorder usually begins in has three phases: inflammation; fibrosis; and ossification,
the third to fourth decades (typically before age 45 years). It which can drastically limit range of spinal motion. If untreated,
can occur as early as the teen years and as late as the sixties patients can assume a kyphotic stance and may only be able to
in rare cases. look at their shoes as they walk. This is due to the 40'l" flexion

34
Spondyloarthritis

Back pain >3 months


and
Age at onset <45 years

Sacroiliitis on plain radiography or MRI

Yes No

1 or more features of spondyloarthritis HLA-B27 positivity


meet criteria for radiographic plus
axial spondyloarthritis 2 or more features of spondyloarthritis
meet criteria for nonradiographic
axial spondyloarthritis

Features of spondyloarthritis
. lnflammatory back pain
. Arthritis
. Enthesitis
o Uveitis
. Dactylitis
. Psoriasis
. lnflammatory bowel disease
. Good response to NSAIDs
. Family history of spondyloarthritis
. HIA-B.27 positivity
. Elevated C-reactive protein level

FIGURE 1 5. AssessmentofSpondyloArthritis international Societyclassification criteria foraxial spondyloarthritis.

rule of swollen synovial joints: Patients are most comfortable


Arthritis or enthesitis or dactylitis
when the spinal facet joints are in the 40% flexed position plus
when swollen; when lying down, patients will prop their back 1 or more additional spondyloarthritis feature

with two or three pillows to be comfortable. Fibrosis and ossi-


fication of the spine then occur in this position. Ossification of
the spine occurs at the vertebral entheses and is visible radio-
graphically as syndesmophytes and ossification of the spinal Spondyloarthritis features:
o Uveitis
ligaments.
o Psoriasis
Enthesitis is most commonly appreciated in the feet at the o Preceding infection
plantar fascia and where the Achilles tendon inserts into the . lnflammatory bowel disease
heel (Figure 17). Enthesitis of the costovertebral joints can also . HLA-827 positivity
o Sacroiliitis on imag;ng
affect the chest wall, leading to pain at night and in the morn-
ing that improves with movement.
Peripheral joint involvement may include the hips
(Figrre 18), shoulders, knees, and, less commonly, other joints.
The synovial fluid is inflammatory with a predominance of neu- lf none of the above spondyloarthritis
features are present, patient must
trophils and cell counts ranging from 10,000/pL to 50,000/pL have 2 or more of the following:
(ro to so x loell). . Arthritis
o Enthesitis
. Dactylitis
Extra-Articular Manifestations . History of inflammatory back pain
Anterior uveitis occurs in up to 27% of patients with ankylos . Family history of spondyloarthritis
ing spondylitis. It is episodic, unilateral, and typically self-
limited. It generally does not cause permanent visual tIGURE l6.AssessmentofSpondyloArthritisinternationalSocietyclassification
damage. criteria for peripheral spondyloarthritis.

35
Spondyloarthritis

Psoriatic Arthritis
Psoriatic arthritis is an inflammatory joint disease associated
with psoriasis. Psoriasis affects approximately 3% of the U.S.
population, and psoriatic arthritis may develop in up to 30% of
patients with psoriasis. Hyperuricemia and gout are comor
bidities associated with psoriasis and psoriatic arthritis.
An estimated 15% of patients with psoriasis who are
monitored by dermatologists have unrecognized psoriatic
arthritis. On the other hand, some patients considered to
have psoriatic arthritis because of concurrent psoriasis and
joint pain actually have an unrelated joint condition, most
commonly osteoarthritis. In most cases, psoriatic arthritis
develops years after psoriasis, but 15% of patients may
develop psoriatic arthritis concurrently with or even before
psoriasis.
The male to-female ratio for psoriatic arthritis is 1:1
(other forms of spondyloarthritis generally have a higher
FIGU R E I 7. Marked thickening of the distal right Achilles tendon and Achilles
insertion on the calcaneus (right) as the result of chronic Achilles tendinitis in a prevalence in men). Risk factors include obesity, metabolic
patient with psoriatic arthritis (left image shows normal contralateral on Achilles syndrome, severe psoriasis, psoriasis involving the scalp or
i nsertion). genitals, or inverse psoriasis. Psoriatic nail changes are pre-
sent in more than 80% of patients with psoriatic arthritis.
Fibrosis affecting the upper lobes of the lung, visualized The genetics of psoriatic arthritis are complex. HLA-C06 is
by high-resolution CT, is reported in 7% of patients with early
associated with psoriasis but not psoriatic arthritis. HLA-
ankylosing spondylitis and 2l% of patients after 10 years of BO8, -B27, -B38, and -B39 are seen in psoriatic arthritis, and
disease. Involvement of the thoracic spine, costochondral the particular HLA type can influence clinical expression. In
junctions, and sternomanubrial joint can affect chest expan- part because of HLA expression, psoriatic arthritis is rare in
sion, contributing to restrictive lung disease. Asians and Africans.
Renal amyloidosis is a rare manifestation of chronic Five clinical subtypes ofpsoriatic arthritis are described:
inflammation from ankylosing spondylitis that occurs in
1. Asymmetric oligoarthritis involving four or fewer, typically
patients with long-standing, poorly controlled disease. AA
large, joints. Occasionally, a single joint may be involved.
amyloid deposition can lead to proteinuria and eventually
kidney failure and is associated with increased mortality. In 2. Polyarticular arthritis affecting multiple joints of the
patients with poorly controlled ankylosing spondylitis, AA hands, which may be symmetrical and resemble rheuma
amyloidosis is the most common cause of kidney disease. IgA toid arthritis.
nephropathy can also occur in anlcylosing spondylitis, pre- 3. Distal interphalangeal joint (DlP)-predominant variety,
senting as hematuria and proteinuria. which may occur with the preceding subtl,pes or in isolation
Cardiac involvement in ankylosing spondylitis includes
aortitis ofthe subaortic valve region (an area ofhigh fibrocar-
tilage content), leading to aortic dilatation and aortic valve
regwgitation. Atrioventricular block occurs more commonly
in patients with ankylosing spondylitis and those who are
HLA-827 positive without rheumatologic disease than among
the general population. Myocardial infarction occurs in
patients with antr<ylosing spondylitis at a rate two to three
times that in the general population; this increased rate is
thought to be due to chronic inflammation hastening the
development of atherosclerosis.
Vertebral fractures can occur in patients with ankylosing
spondylitis, often after only minor or minimal trauma. Fractures
can occur through the vertebral body or through a disk space
that has become fixed because of slmdesmophyte formation;
either can lead to nerve injury or myelopathy Another rare nzu-
rologic manifestation is cauda equine qmdrome, manifested by
F IG UR E 1 8. Radiograph showing hip involvement in ankylosing spondylitis.
saddle anesthesia and bowel and bladder incontinence. MRI of Note the fused sacroiliac joints bi lateral ly. Diffuse joi nt-space narrowi ng is present
the lumbar spine shows arachnoid diverticula in these patients. in both hip joints, especially in the left hip.

36
Spondyloarthritis

Enteropathic Arthritis
Ankylosing spondyloarthritis, isolated sacroiliitis, and periph-
eral arthritis occur in patients with Crohn disease, ulcerative
colitis, and unspecified inflammatory bowel disease (IBD).
Spondyloarthritis is the most common extra-articular mani-
festation of IBD.
The prevalence of ankylosing spondylitis is 47o in Crohn
disease and 2% in ulcerative colitis; the prwalence of isolated
sacroiliitis is 137" and 7olo, respectively. Of interest, HLA-827 is
found in 257o to 75% ofpatients with enteropathy-associated
ankylosing spondylitis but in only 7"/" to lSTo of those with
isolated sacroiliitis, suggesting a different pathophysiologr.
Isolated sacroiliitis is unilateral in 60% of cases and can be
associated with psoriasis, erythema nodosum, and peripheral
arthritis in patients with IBD. Dactylitis and enthesitis can also
occur with enteropathic spondyloarthritis.
There are two types of peripheral arthritis in IBD. Type I
is pauciarticular (fewer than five joints) and asymmetric;
usually involves medium to large joints, especially in the
lower extremities; is nonerosive; and parallels the activity of
the bowel disease. Type II is polyarticular (five or more
joints); is qmmetric; involves a wide range of joints, some-
t I G U n E t 9 . Distal interphalangeal joint swelling (best seen in the second and
times including the upper extremities; is nonerosive; and is
third digits) and nail pitting (best seen in the fourth digit nail) in a patient with
psoriatic arthritis.
persistent and independent of bowel disease activity. The
prevalence of peripheral arthritis is highest in younger
patients (up to ZS%) and much lower in patients older than
(Figure 19). Although inflammation in a DIP joint in a age 5o years (2%).
patient with psoriasis strongly suggests psoriatic arthritis, The risk for IBD in patients with established spondy-
DIP involvement is also common in osteoarthritis, is occa- loarthritis is as high as 147o and is highest in patients with
sionally seen in gout, and also occurs in a rare condition ankylosing spondylitis. As noted previously, 60% ofpatients
called multicentric reticulohistiocytosis. with ankylosing spondylitis have evidence of asymptomatic
4. Arthritis mutilans, a more aggressive form of psoriatic bowel inflammation, and most patients with ankylosing
arthdtis irvolving the hands. Telescoping of the digits from spondylitis who develop IBD are in this group. Patients
zubstantial joint destruction may ocu:r. Arthritis mutilans is with ankylosing spondylitis should be asked about bowel
probably a progressive form ofthe preceding thrce subtypes. symptoms on follow-up visits. Elevated serum and fecal
calprotectin levels can identi$r patients with ankylosing
5. Axial spine involvement with sacroiliitis and spine involve-
spondylitis who have greater disease activity and those with
ment. This may occur in isolation or in conjunction with
other subtypes. It occurs in up to 5Oo/. of patients with
psoriatic arthritis, more commonly among patients with
the HIA-827 gene. Axial psoriatic arthritis involvement
may be distinguished radiographicallyby asymmetry skip
lesions, and bulkier syndesmophytes.
Enthesitis is common in psoriatic arthritis and indicates
more severe disease. Dactylitis of the fingers or toes develops
in 4O% to 50% of patients with psoriatic arthritis (Flgur€ 2O);
enthesitis ofthe plantar fascia or Achilles tendon occurs in up
to 5O%.
Eye involvement also develops in psoriatic arthritis.
Conjunctivitis is more common than uveitis or iritis. Uveitis
occurs in up to 8% of patients with psoriatic arthritis.
Like ankylosing spondylitis, psoriatic arthritis is associ-
ated with an increased risk for cardiovascular disease, includ-
ing myocardial infarction, valvular disease, and conduction FTGURE 20. Dactylitisof thesecondtoesbilaterallyinapatientwithpsoriatic
abnormalities. arthritis.

37
Spondyloarthritis

more active bowel inflammation and may predict develop


ment of IBD.

Reactive Anhritis
Reactive arthritis occurs after specific bacterial infections. It is
classified as a spondyloarthropathy and represents only 2'7, of
patients in this category.
The most common organism responsible for reactive
arthritis is Chlomydio trachomatis; 4% to 8"/,' of infected
patients develop reactive arthritis. Gastrointestinal pathogens
include several species of Salmonella and Shigella,
Campylobacter jejuni, and Yersiniq species; reactive arthritis
occurs in approximately 1'X, of infected indMduals. Respiratory
pathogens, including Chlamydia pneumoniae and Mycoplosma tl GUR E 2 I . Keratoderma blennorrhagicum on the palms and circinate balanitis
on the glans penis are psoriasiform skin rashes seen in this patient with reactive
pneumoniae, are also known causes of reactive arthritis.
arthritis.
Clostridioides difficileis an uncommon but important cause of
reactive arthritis given its frequency in hospitalized patients. persistent disease. Complications of reactive arthritis can
Approximately 7.4% of patients with C. dfficile colitis will include erosive joint disease, especially at the metatar
develop reactive arthritis. An inciting organism is identifled in sophalangeal joints; aortitis with aortic valve insufficiency;
only about 50% of patients. Women develop reactive arthritis atrioventricular conduction block: and uveitis.
more commonly than men (relative risk, 1.5). H[A-B27 is found
in 50'/" to 80% of patients with reactive arthritis; the highest rEY PO I l{TS

frequency (90%) is seen in those who develop reactive arthritis . AnMosing spondylitis is a chronic inflammatory disease
from Yersin io infection. affecting the axial skeleton (including sacroiliac joints),
Reactive arthritis develops 2 to 3 weeks after genitouri entheses, and peripheral joints.
nary and gastrointestinal infections. Synovial fluid Ieukocyte . Psoriatic arthritis is an inJlammatory ioint disease asso-
counts can be elevated, often around 50,000/pL (50 x 10e/L) ciated with psoriasis, multiple possible joint patterns,
with a predominance olneutrophils. Synovial fluid in reactive enthesitis, tenosynovitis, and dactylitis.
arthritis can also demonstrate rice bodies: small pieces ol . Ankylosing spondyloarthritis, isolated sacroiliitis, and
hypertrophic synovial tissue that are sloughed and accumulate peripheral arthritis can occur in patients with inflam
in the joint and may require a large bore needle to aspirate.
matory bowel disease.
The arthritis is typically pauciarticular and involves
larger joints. Enthesitis of the Achilles tendon or plantar fas
. Reactive arthritis can occur 2 to 3 weeks following spe-
cific gastrointestinal and genitourinary infections; the
cia is commoni enthesitis may also occur in the knee, a joint
with multiple entheses. Dactylitis of fingers and toes is also arthritis is tlpically pauciarticular and involves larger
joints, and dactylitis and enthesitis can occur.
common in reactive arthritis. Sacroiliac joint and spine
inflammation and pain are less common than peripheral
manifestations.
Mucocutaneous manifestations include genitourinary dis
Diagnosis
ease (such as urethritis, cervicitis, cystitis, or prostatitis). Laboratory Studies
Patients can have painless oral ulcers, tlpically affecting the Except for incidental overlap with other conditions, patients
hard palate. Conjunctivitis is the most common eye finding, but with spondyloarthritis are negative fbr rheumatoid factor.
anterior uveitis and even keratitis can occur. Keratoderma blen anti cyclic citrullinated peptide, and antinuclear antibodies.
norrhagicum, a psoriasiform skin rash, occurs on the palms and Serologic testing should be performed only if an alternative
soles but occasionally can become generalized (Figure 21). The diagnosis is being considered.
lesions begin as discrete plaques and then enlarge and coalesce. For patients with inflammatory low back pain but normal
Circinate balanitis is a similar psoriasiform lesion on the glans or nondiagnostic plain radiographs ofthe spine and sacroiliac
penis in men. Erythema nodosum is rare. joints, a positive result on a test for HLA B27 antigen can be
Important differential diagnostic considerations include useful for deciding to pursue advanced imaging. HLA-B27
infectious arthritis, disseminated gonococcal infection, antigen testing is never diagnostic but can help determine risk
Whipple disease, IBD, or Behqet syndrome. for spondyloarthritis in uncertain situations. A patient with
The typical course ofa reactive arthritis flare is 6 weeks to inflammatory low back pain, uveitis, and radiographic evi
6 months. Manifestations can last I year. Episodes may recur dence of bilateral sacroiliitis does not need HLA B27 antigen
in some patients; about 25% of patients will have chronic testing because the diagnosis is clear.

38
Spondyloarthritis

Patients with spondyloarthritis may have anemia of Figure 23 with findings seen in Figure 24. Radiographs of the
inflammation as a result of chronic inflammation. Erythroclte spine can show squaring of the vertebrae anteriorly (due to
sedimentation rate and C reactive protein help confirm an erosion at the insertion of entheses); sclerosis at the anterior
inflammatory process and can be monitored during therapy. edges of the vertebrae ("shiny corner" sign [Figure 25]);
However, these values are not always elevated, particularly in
ankylosing spondylitis. All patients with chronic spondyloar-
thritis should be monitored for standard risk factors for car
diovascular disease.

lmaging Studies
Radiography ofthe sacroiliac joints is an initial test for patients
suspected of having ankylosing spondylitis. Anteroposterior
radiography ofthe pelvis or a Ferguson view ofthe pelvis (in
which the x ray beam is directed 20 degrees cephalad to show
all sacroiliac joints in one plane) can depict the changes of
sacroiliitis (Figure 22). The radiographic changes are initially
more intense on the iliac side of the joint, with sclerosis and
erosions. Erosive changes lead to a pseudo-widening appear-
ance of the sacroiliac joints; in advanced cases, ankylosis of
the sacroiliac joints can occur; compare normal findings in
FIGU R E 24. CIscan of the sacroiliac joints in a patientwith sacroiliitis (same
patient shown in Figure 22). Note the marked sclerosis/new bone formation,
especially on the iliac side of the joint and erosions of the sacroiliac joints.

t IGU R E 22. Radiograph of the sacroiliac joints (Ferguson view) in a patient


with sacroiliitis (same patient shown in Figure 24). Note the iliacside sclerosis and
! i nd istinctness of the sacroi liac joi nts on both sides of the joi nt, i nd icati ng erosions.

F IG UR E 2 5. Lateral lumbosacral spine radiograph in a patient with ankylosing


spondylitis. Note the squared appearance of the anterior aspect of the lumbar
vertebrae and the "shi ny corner" sig n on the anterosu perior aspect of the L3 (anow)
FIGU R E 2 3. Normal CT scan of the sacroiliac joints. Note the intact sacroiliac vertebrae. The "fl uffy" appea rance to the facet joints posteriorly is typical of the new
joints and lack of subchondral sclerosis. bone formation seen in ankylosing spondylitis.

39
Spondyloarthritis

Peripheral joint changes in spondyloarthritis include ero-


sions and new bone formation. In psoriatic arthritis in par-
ticular, radiographic findings classically take the form of
erosions of the distal head of the proximal joint and juxta-
articular new bone formation at the proximal head of the
distaljoint, leading to a 'pencil-in-cup" appearance across the
joint (Figure 27).

. Except for incidental overlap with other conditions, HVC


patients with spondyloarthritis are negative for rheu-
matoid factor, anti-cyclic citrullinated peptide, and
antinuclear antibodies; HI,A-B27 antigen tesung can
help determine risk for spondyloarthritis in uncertain
situations.
r Erythrocyte sedimentation rate and C-reactive protein
can help confirm an inflammatory process and can
be monitored during therapy; however, these values
are not always elevated, particularly in ankylosing
spondylitis.
o Radiography of the sacroiliac joints is an initial diagnos-
tic test for patients suspected ofhaving ankylosing
spondylitis; radiographic widence of sacroiliitis
includes pseudo-widening of the joints, erosions,
sclerosis, and ankylosis.
e MRI of the sacroiliac joints is more sensitive than CT HVC

F IGUR E 26 . More advanced disease of the lumbar spine in late ankylosing


and can identiff sacroiliac inflammation even in the
spondylitis. Note the syndesmophyte formation or bony connections between absence ofradiographic changes; however, 25"/, of
vertebrae across the disc space ("bamboo" appearance). healthy individuals can have similar MRI changes.
(Continued)

syndesmophytes (delicate bony bridging between vertebrae


that can result in a "bamboo" appearance of the spine in
advanced ankylosing spondylitis [Figure 26]); sclerosis at the
facet joints; and, in more advanced cases, ossification of the
anterior longitudinal ligament. Patients with psoriatic arthritis
or reactive arthritis with spondylitis can also have syndesmo-
phytes; these tend to be bulkier and more asymmetrically
placed than syndesmophytes in ankylosing spondylitis.
Whiskering, the result of new bone formation at the insertion
of the hamstrings into the pelvis, is also seen on radiography.
Bony proliferation can occur in other entheses, such as the
Achilles tendon or plantar fascia insertion into the heel.
Low dose CT and MRI of the sacroiliac joints can be done if
necessary Low dose CT is more speciflc and is usefirl to clari$r
ambigrous changes seen on plain radiographs, whereas MRI is
more sensitive and can ident$ sacroiliac inflammation even in
the absence ofradiographic changes. However, 25% ofhealthy
indMduals can have MRI changes that meet Assessment of
SpondyloArthritis international Society criteria for sacroiliitis,
and up to 41% of persons who regularly participate in impact-
loading athletics may have similar MRI changes. The extent of J
involvement and presence of erosions on MRI increase the
tI G U R E 2 7 . Radiograph showing "pencil-in-cup' deformity of the fifth
speciflcity of the result. The presence of MRI changes also aids metatarsal joint and ankylosis of the fou rth metatarsal joint in a patient with
in the diagnosis of nonradiographic axial spondyloarthritis. psoriatic arthritis.

40
Spondyloa rth ritis

f,EY P0lI|TS {ontinued) long-term doses of anti inflammatory agents (e.g., naproxen,
. In psoriatic arthritis radiographic findings take the form 500 mg twice daily; piroxicam, 20 mg daily) are required to
modi$r new bone formation and decrease inflammation. A dose
of erosions of the distal head of the proximal joint and
juxta-articular new bone formation at the proximal of a longer acting NSAID at dinnertime can make the patient
comfortable throughout the night and into the moming.
head of the distal joint, leading to a "pencil in cup"
According to the 2019 ACR guidelines, patients in whom
appearance across the joint.
NSAIDs fail can be treated with a tumor necrosis factor (TNF)
inhibitor. Numerous randomized controlled trials have shown
improvements in clinical, radiographic, and MRI outcomes in
Management patients with active ankylosing spondylitis taking TNF inhibi
General Considerations tors. Routine serial spine radiography for monitoring is not
Management of spondyloarthritis focuses on treatment of inflam recommended.
mation and autoimmunit5r to address pain and prwent structural When a TNF inhibitor fails, patients should receive an
damage. If damage has occurred, mechanical pain must be interleukin-l7 inhibitor in lieu of a second TNF agent. For
addressed as well. Patient education, exercises to maintain range patients with significant peripheral arthritis, sulfasalazine is
of motion and control weight, and modilication of diet and life- recommended over methotrexate and can be added to the
style habits to prevent cardiovascular disease are vital. biologic agent if needed. However, these agents have little or
no ellect on axial disease.
Ankylosing Spondylitis Uveitis is generally treated with topical glucocorticoids.
Figure 28 details recommendations for initial treatment of For severe or frequent recurrences, the TNF inhibitors adali-
active ankylosing spondylitis. NSAIDs are recommended as mumab or infliximab can be considered. Etanercept, an anti
initial medication for most patients with ankylosing spondylitis TNF receptor fusion protein, is effective for the spine but
because they relieve pain and improve stiffness. Given the probably not eye disease.
unique potential for disease modiflzing effects (oniy in ankylos- Physical therapy to maintain general range of motion is
ing spondylitis), the American College of Rheumatolo$/ (ACR) essential. Patients with damage to hip or shoulder joints may
conditionally recommends continuous NSAID use (rather than require total joint arthroplasty, which may dramatically
on demand use) for patients with active disease. Higher and improve function and quality of life.

Assess for additional lsolated sacroiliitis


Axial disease
disease manifestations or enthesitis

Continuous NSAIDs Peripheral-predominant NSAIDs


and active, land-based disease
physical therapy

Symptoms controlled?
NSAIDs
Symptoms controlled?
Yes No

Yes No
Symptoms controlled? Continuous Local glucocorticoid
NSAIDs injections (avoid
Continuous Add TNF Achilles tendon, patella,
NSAIDs inhibitor Yes No and quadriceps)

Continuous <2 joints


NSAIDs
Symptoms
controlled? Yes No

Yes No Local glucocorticoid Sulfasalazine


injections

Continue TNF Switch to


inhibitor secukinumab
or ixekizumab

FIGURE 28. Recommendationsfortheinitialtreatmentofactiveankylosingspondylitis.INF=tumornecrosisfactor.

41
Systemic Lupus Erythematosus

Psoriatic Arthritis For mild arthritic disease, NSAIDs or joint injections with
Thble 18 lists agents available to treat psoriatic arthritis. In gen- glucocorticoids may be sufficient until the disease self
eral, most ofthese agents are also effective for psoriasis. Because resolves. For more persistent or severe disease, sulfasalazine
agents such as TNF inhibitors are often given in higher doses (preferred) or methotrexate may be required. There are few
for skin disease than for joint disease, the predominant feature data from controlled studies for either agent, but case series
of the disease may drive the prescribing dose. Weight loss and and reports suggest efficacy. TNF inhibitors may also be needed
smoking cessation may positively affect disease activity. if these agents fail, but again data are scant. Given the mono-
For limited disease, such as a single digit with dactylitis or phasic nature of reactive arthritis in many patients, a trial off
a single swollen joint, an oral or topical NSAID or a local injec therapy is warranted 3 to 6 months after remission.
tion of glucocorticoids may suffice. High dose oral glucocorti Topical glucocorticoids can be used to help address the
coids should be avoided because eventual tapering of the skin manifestations, and topical ocular glucocorticoids are
dosage raises the risk for erythrodermic psoriasis. useful for uveitis.
For more active disease, the ACR suggests initiating a TNF KEY POITTS
inhibitor, methotrexate, or an interleukin 17 inhibitor. If one
. For ankylosing spondylitis, NSAIDs are recommended
agent is insufficient, another may be tried. Combination ther-
as the initial medication to relieve pain and improve
apy does not have the same effect in psoriatic arthritis as in
stiffness; physical therapy to maintain general range of
rheumatoid arthritis. Surgery may be indicated for significant
motion is essential.
damage to larger joints.
. Tumor necrosis factor inhibitors in patients with active HVC

Enteropathic Arthritis ankylosing spondylitis are associated with improvements


Enteropathic arthritis is best addressed by focusing on control of
in clinical, radiographic, and MRI outcomes.
bowel inflammation. For spondyloarthritis associated with o For psoriaticarthritis, limited disease can be treated with
bowel involvement, medications to address both peripheral atopical NSAID or local glucocorticoid injection; for
arthritis and bowel inflammation include sulfasalazine, azathio- more active disease, a tumor necrosis factor inhibitor,
prine, methotrexate, and glucocorticoids. The TNF inhibitors are methotrexate, or interleukin-l7 inhibitor can be used.
especially useful for bowel and joint symptoms, including . The peripheral arthritis tied to inflammatory bowel dis-
spine disease. Interleukin 17 inhibitors may cause IBD to flare ease is best addressed by focusing on control ofbowel
and are contraindicated in these patients. NSAIDs should be inflammation; for spondyloarthritis that is associated
avoided when possible because ofthe potential for IBD flare. with bowel involvement, medications to address both
peripheral arthritis and bowel inllammation include
Reactive Anhritis sulfasalazine, azathioprine, methotrexate, and gluco-
Because of a lack of benefit, antibiotic treatment of reactive corticoids.
arthritis due to enteric organisms is not usually warranted. o Because of a lack of benefit, antibiotic treatment of
However, patients with severe infections or immunosuppres-
reactive arthritis due to enteric organisms is not usually
sion or those with urogenital infection with C. trachomatis
warranted; for mild arthritic disease, NSAIDs or joint
may benefit from antibacterial therapy.
injections with glucocorticoids may be sufficient, and
for more persistent or severe disease, sulfasalazine or
TABLE 1 8. Medications to Treat Psoriatic Arthritis methotrexate may be required.
Category Medications
Anti-inflammatory agents NSAIDs; oral or intra-articular
glucocorticoids
Non biologic disease-modiiying Methotrexate, leflu nomide, Systemic Lupus
antirheumatic drugs sulfasalazine, cyclosporine,
apremilast Erythematosus
Tumor necrosis factor inhibitors Etanercep! adalimumab,
infliximab, certolizumab, Epidemiology and
golimumab Pathophysiology
lnterleukin-1 7 inhibitors Secukinumab, ixekizumab, Systemic lupus erythematosus (SLE) is a multisystem autoim-
brodalumab
mune disease characterized by a heterogeneous constellation
lnterleukin-1 2/interleukin-23 Ustekinumab
inhibitor oforgan involvement and the presence ofantinuclear antibod
ies (ANA) and other autoantibodies.
Cytotoxic T-lym phocyte Abatacept
antigen-4 immunoglobulin In SLE, a complex and varying interaction of genes, envi-
ronment, and random events leads to a breakdown of self-
Janus kinase inhibitor Tofacitinib
tolerance and autoimmunilr. Defects in cellular apoptosis

42
Systemic Lupus Erythematosus

result in inadequate clearance of intracellular proteins, espe-


cially nuclear antigens, promoting the generation of self
directed T and B cells and the initiation/propagation of
autoimmunity. Cytokine generation supports autoreactiviry
with type I interf'erons playing a major role. Autoantibodies
may directly induce tissue damage or promote the formation
of immune complexes that lead to complement activation and
tissue inflammation and damage. Inheritance of SLE risk is
polygenic, including major histocompatibility complex genes.
The risk for SLE is higher in those with a family history
Incidence increases at pubefi and peaks in the third decade.
The disease is more common and more severe in Black and
Hispanic populations. Approximately 90% of adult patients are
women; SLE is frequently more severe in men than women.
tIGURE 29. Thefacial eruption of acutecutaneous lupuserythematosus
f,lY POrilTt (malar or butterfly rash). This patient has fixed erythematous raised lesions over the
. Systemic lupus erythematosus is a multisystem autoim malar eminences, the bridge of the nose with sparing of the nasolabial folds, and
mune disease characterized by a heterogeneous constella- the chin.

tion oforgan involvement and the presence ofantinuclear


antibodies and other autoantibodies. desquamation. The facial eruption of ACLE (the classic malar
. Approximately 9O"/" of adult patients with systemic lupus or butterfly rash) is characterized by fixed, rather than tran-
erythematosus are women. sient, erythema/edema over the cheeks and bridge ofthe nose,
sparing the nasolabial folds (Figure 29), but the neck, upper
chest, and dorsum ofthe arms and hands can also be involved.
In some patients, a bullous eruption occurs. ACLE usually
Clin ica I Manifestations responds to therapy and heals without scarring or atrophy.
Mucocutaneous lnvolvement Subacute cutaneous lupus erythematosus (SCLE) is a
Skin disease occurs in most patients with SLE and is classified photosensitive rash occurring especially on the arms, neck,
as acute, subacute, or chronic. and upper trunk, usually sparing the central face (Figure 3o).
Acute cutaneous lupus erythematosus (ACLE) presents as It consists of erythematous annular/polycyclic or patchy papu-
an erythematous, macular, patchy eruption, sometimes with losquamous lesions, often with a fine scale, that may leave

fIGURE 30. Subacutecutaneouslupuserythematosusischaracterizedbyerythematous,macular,orpatchyskinlesionsthatarescalyandcanevolveas(/)annular/polycyclic


lesions or (B) papulosquamous plaques.

43
Systemic Lupus Erythematosus

unexplained pain and/or reduced range of motion. Long-


term and higher-dose (>20 mg/d) prednisone treatment,
severe/active SLE, and vasculitis are associated with
increased risk for osteonecrosis. MRI may be needed to iden-
tiff early disease. Although small lesions can improve with
out invasive treatment, larger areas of involvement can lead
to bony collapse. Treatment requires extended non-weight-
bearing and, some cases, core decompression of the
in
affected bone. In the wake of collapse, joint replacement
may be necessary.
Myalgia and subjective weakness are common, but true
myositis is rare. Glucocorticoids often cause muscle weakness,
and, rarely, antimalarial agents can affect muscle. Thus, medi-
cation effects must be differentiated from active SLE disease.
F tGUn E 3 I . Discoid lupus erythematosus.This patient has hyperpigmented,
raised patches with keratotic scaling and follicular plugging involving the malar
Fibromyalgia is a common comorbidity (30%); symptoms may
and perioral areas as well as the bridge of the nose. Areas of atrophic scarring are overlap those of active SLE disease.
also present.
Kidney lnvolvement
postinflammatory hypo- or hyperpigmentation. SCLE is asso- Kidney disease occurs frequently among patients with SLE and
ciated with anti-Ro/SSA autoantibodies (prevalence >75%) and remains an important source of morbidity. Lupus nephritis can
can occur in isolation or as a manifestation of underlying SLE. present with minimal laboratory abnormalities (non-nephrotic
Discoid lupus erythematosus (DLE) is the most common proteinuria, hematuria), nephritis (hypertension, edema,
manifestation of chronic cutaneous Iupus
erythematosus active urine sediment, and elevated serum creatinine), and/or
and can cause scarring, atrophy, and permanent alopecia nephrosis (nephrotic-range proteinuria, edema, hypoalbu-
(Figure 31 and Figure 32). DLE presents as hlpo- or hyperpig- minemia, hypercholesterolemia, and, in some cases, thrombo-
mented patches or plaques, with erythema during active dis- sis). Untreated active disease may progress to kidney failure, in
ease, which may be variably atrophic or hyperkeratotic. Like severe cases requiring dialysis or transplant.
SCLE, DLE can occur as an isolated flnding in the absence ofSLE
and in such cases is typically limited to the neck, face, and scalp.
Painless oral or nasopharyngeal ulcerations occur in
5'1, of patients with SLE. Involvement of the hard palate is
characteristic. Rarely, DLE is associated with painful ulcers.
Nonscarring alopecia is a common feature of active SLE,
with hair regrowtha sign of disease control.
Raynaud phenomenon occurs frequently, reflecting arte-
rial vasospasm of digital arteries. Other vascular changes,
including livedo reticularis or periungual erythema, may be
present as well, although these findings are nonspecific. Some
patients with SLE may develop cutaneous vasculitis, most
often in the distal extremities.

M usculoskeletal I nvolvement
Joints are affected in 90% of patients with SLE. Many patients
have arthralgia, and a much smaller group exhibits arthritis.
Many patients with SLE may have evidence of synovitis on
imaging even with minimal symptoms or swelling on exami-
nation. Typical distribution includes small peripheral joints,
often resembling rheumatoid arthritis, but large joints are also
affected. SLE arthritis is nonerosive, but reducible subluxation
of the digits, swan neck deformities, and ulnar deviation
(Jaccoud arthropathy) can occur, a result of damage to the
tendons and ligaments.
Osteonecrosis is a serious complication of SLE that most
commonly affects the hips but can also involve other large FIGU R E 3 2. Active discoid lupus erythematosus ol the scalp with scaning hair
joints. It should be suspected in patients with otherwise loss.

44
Systemic Lupus Erythematosus

All patients with SLE should be regularly evaluated for Pulmonary lnvolvement
kidney involvement through assessment of serum creatinine Pulmonary involvement is common in SLE, with most patients
and urine for protein and microscopic evaluation. Active kid- presenting with pleuritis (45"1, 6O'X,). Pleural effusions occur
ney disease should be suspected when there is active urine in approximately half of these patients and are typically exu
sediment or proteinuria greater than 500 mgl24 h (or a spot dative; fluid analysis may reveal lymphocytic pleocytosis and
urine protein to creatine ratio >500 mg/g). Elevated or rising mildly depressed glucose levels.
anti-double stranded DNA antibody titers or complement Parenchymal lung involvement occurs in less than 10'1, of
consumption are commonly associated with active kidney patients with SLE. A nonspecific interstitial pneumonia pat-
disease. tern is most common, and evaluation centers on assessing SLE
Kidney biopsy defines both the histologic subtype and the activity and excluding other causes of diffuse parenchymal
activity/chronicity of disease, which are important for thera lung disease. Two rare but potentially life threatening compli
peutic decisions. Indications fbr kidney biopsy include an cations of SLE lung disease are acute lupus pneumonitis (pre-
increase in serum creatinine level, unexplained decrease in senting as fever, cough, dyspnea, hypoxemia, pleuritic chest
glomerular filtration rate, or proteinuria greater than pain, and infiltrates) and diffuse alveolar hemorrhage (pre-
5OO mgl24 h (or a spot urine protein-to-creatinine ratio senting with dyspnea, hypoxemia, diffuse alveolar infiltrates,
>500 mg/g), especially in the presence of hematuria or an a dropping hematocrit, and a high Dr.co). Both carry a high
active urine sediment. mortality rate (>50'X,). Early recognition, rapid evaluation (CT
See MKSAP 19 Nephrologz fbr information on the classes and/or bronchoscopy with bronchoalveolar lavage or biopsy),
and treatment of lupus nephritis. and aggressive respiratory support combined with high dose
glucocorticoids and immunosuppression are required. With
Neuropsychiatric lnvolvement new pulmonary infiltrates, differentiation between these dis
Neuropsychiatric systemic lupus erythematosus (NPSLE) may orders and infection can be difficult. and antibiotics and
involve the central and/or peripheral nervous systems. NPSLE immunosuppressive therapy are often administered simulta
prevalence is high (75%), but most of the common manifesta neously until the diagnosis is clear.
tions (headache, mild cognitive dysfunction, and mood disor Shrinking lung syndrome is a rare but characteristic syn
der) are nonspecific. Peripheral neuropathy occurs in 10"/n to drome consisting of pleuritic chest pain and dyspnea, with
14% ofpatients. Severe acute presentations, including seizures progressive decrease in lung volumes. The cause is uncertain,
and psychosis, are uncommon (<5'1,) but require aggressive but pleuropulmonary disease and/or diaphragmatic dysfunc
symptomatic as well as disease speciflc treatment. tion may contribute. Immunosuppression may reverse the
Patients suspected of having serious central NPSLE, process in some patients.
such as meningitis, stroke, and psychosis, should undergo
central nervous system imaging (CT, MRI, or PET) and cere-
brospinal fluid analysis as appropriate. ln some patients Hematologic lnvolvement
with severe disease, measurement of cerebrospinal fluid for In patients with SLE, normocytic, normochromic anemia of
NPSLE associated autoantibodies (antineuronal, anti inflammation is common; autoimmune hemolytic anemia
N methyl r) aspartate receptor, antiribosomal P, and others) occurs in approximately 10u1, of cases and correlates with SLE

may be useful. For patients suspected of having peripheral activity. Lymphopenia/leukopenia is also common but usually
neuropathies, electromyography and nerve conduction mild. Thrombocytopenia occurs in 30'1, to 50% of cases, and
studies should be performed. Neuropsychologic testing may approximately 10'2, of patients develop severe thrombocytope
help define and distinguish organic versus functional cogni nia (<50,000/pL [50 x 10e/L]) in isolation or in conjunction
tive changes. with hemolytic anemia.
Cytopenia in SLE may be caused by immune and non-
Cardiovascular lnvolvement immune destructive mechanisms (including microangiopa-
Asymptomatic pericarditis is the most frequent cardiac mani- thy), medications, and kidney and liver disease. Moderate and
festation of acute SLE (40'1,). When symptomatic, features severe or rapidly progressive cytopenia requires prompt evalu-
include chest pain, exudative effusion, and, rarely, tamponade ation with serologic studies and/or bone marrow biopsy. An
or chronic constriction. Myocarditis occurs in 5'1, to 10% of exact cause of cytopenia may be diflicult to ascertain, and a
patients with SLE and usually presents as insidious heart fail trial of medication adjustment in concert with evaluation for
ure but can be acute. other causes is often necessary.
Valvular abnormalities occurring in SLE include those Antiphospholipid antibodies and lupus anticoagulant are
associated with antiphospholipid syndrome (nonspecific present in about 40'2, of patients with SLE and may be associ
thickening of the mitral and aortic valve leaflets, vegetations, ated with a false positive result on a rapid plasma reagin test
regurgitation, and stenosis). Libman Sacks endocarditis (non fbr syphilis. Most patients are asymptomatic. Thrombotic
infectious verrucous vegetations) preferentially affects the events occur in about 30%, of patients; these include
mitral valve and can cause embolic complications. venous and arterial thrombosis, miscarriage, stillbirth, livedo

45
Systemic Lupus Erythematosus

reticularis, and cardiac valve thickening/vegetations. The risk of death among older patients with SLE, even those whose
for thrombosis is highest in the presence of triple positivity fbr lupus has become quiescent. A history of high SLE disease
lupus anticoagulant, anti pr-glycoprotein, and anticardi activity (especially nephritis) and prednisone dosages greater
olipin antibodies. Patients with SLE are at increased risk for than 10 mg/d are independent risk factors for coronary artery
thrombotic events even in the absence of antiphospholipid disease. Patients with SLE are also at increased risk for
antibodies. See MKSAP 19 Hematologz for more information. ischemic stroke.

XEY POIT{TS
Gastrointestinal lnvolvement o Patients with systemic lupus erythematosus are at
Gastrointestinal involvement is a common (+oy,) and under
increased risk for malignancy; immunosuppressive use
recognized SLE manifestation. Serositis presents as abdominal
contributes to this increased risk.
pain, is usually associated with active disease, and improves
with treatment. Mesenteric vasculitis, inflammation of the . Patients with systemic lupus erythematosus have a 2- to
small and large bowel, pancreatitis, protein losing enteropa- 10 fold increased prevalence of coronary artery disease;
thy, and diffuse peritonitis are uncommon but may be severe high SLE disease activity and prednisone dosages
and associated with cutaneous vasculitis. greater than 10 mg/d are independent risk factors for
Noninfectious hepatitis can occur and is associated with coronary artery disease.
the presence of antiribosomal P antibodies. Patients with
SLE who have Raynaud phenomenon and anti U1
ribonucleoprotein antibodies are at increased risk fbr esopha Diagnosis
geal disease and reflux.
General Considerations
Medications used to treat SLE (NSAIDs, prednisone,
The diagnosis of SLE should be considered in patients, espe
mycophenolate, azathioprine) also frequently affect the gas-
cially young women, with any individual manifestation of SLE
trointestinal system and may cause esophagitis, gastritis, pan
or with symptoms affecting multiple organ systems. The most
creatitis, and other manifestations.
common presenting clinical features that differentiate patients
XEY PO Il{IS with SLE from those with other mimicking conditions include
. The facial eruption of acute cutaneous lupus erythema- malar rash, photosensitivity, inflammatory arthritis, weight
tosus (malar or butterfly rash) is characterized by ery loss, and fever, along with such laboratory features as positiv
thema/edema over the chin, cheeks, and bridge of the ity lor ANA, low complement levels, and presence of lupus
nose, sparing the nasolabial folds. specific antibodies. Patients with subjective reports of fatigue,
r myalgia, and/or arthralgia but lacking objective flndings most
Many patients with systemic lupus erythematosus have
likely have an alternative diagnosis and should not be evalu
arthralgia, and a much smaller group exhibits arthritis.
ated for SLE.
o A11 patients with systemic lupus erythematosus should
Several classification criteria for SLE have been used;
be regular$ evaluated for kidney involvement through
although intended for recruitment of homogenous SLE popu-
assessment of serum creatinine and urine for protein
lations for research sfudies, they can also be used to suggest a
and microscopic evaluation.
clinical diagnosis ofSLE. A 2019 update ofthese criteria from
. The most common manifestations of neuropsychiatric the European League Against Rheumatism/American College
systemic lupus erythematosus are headache, mild cog- of Rheumatologr includes the requirement for a positive ANA
nitive dysfunction, and mood disorder. result at least once, with the addition of clinical and/or immu-
nologic criteria totaling at least 10 points on a weighted scale
Comorbidities (Figure 33). These criteria compare favorably to prior pub
Patients with SLE have a higher overall risk for malignancies Iished criteria, with a sensitivity of about 96% and specificity
(particularly hematologic); the risk for non Hodgkin lym ofabout 93%.
phoma is at least two to three times higher than in the general
population. Malignancy risk in SLE is tied to the use of immu Laboratory Studies
nosuppressive agents. Higher cumulative cyclophosphamide Initial evaluation for SLE includes routine laboratory test-
doses are associated with increased risk for solid organ tumors, ing to establish organ specific involvement, including com
and azathioprine use is associated with an increased risk for plete blood count, chemistry panel, and urinalysis with
myeloproliferative syndromes. Cervical cancer is also increased microscopy.
in patients with SLE, especially those receiving immunosup ANA should be obtained to screen for nuclear directed
pressive therapies. Hydroxychloroquine use does not appear to autoantibodies. The most appropriate method for testing
increase malignancy risk and may be protective. ANA is the indirect immunofluorescence assay, which is
Patients with SLE have a 2- to 10 fold increased preva highly sensitive (>9s'l.) for SLE but not specific. ANA tests
lence of coronary artery disease. It is the most common cause should be interpreted in the context of the probability of

46
Systemic Lupus Erythematosus

Emry Grttetton

Antinuclear antibody at a titer >1:80 on


Hep-2 cells or equivalent positive test result

lf absent do not classify as SLE

lf present apply additive criteria

Addldvr Ct{trria
Do not count a criterion if there is a mora likely explanation than SLE, Occurrenca of a criterion on at least one occasion is
sufficient SLE classification requires at least one clinical criterion and >10 points. Criteria need not occur simultaneously.
Wrthin each domain, only the highest weighted criterion is counted toward the total score.

(llnlcd Domrlnr end Crlterlr Wclglrt lmmunologlc Domdnr rnd Critorle Wdgh0

Gonrtltrtlonrl
Fever (temparature >38.3 'C) 2 Anticardiolipin antibodi€s or
Anti-pr-glycoprotein I antibodies or
Lupus anticoagulant 2

H.m.tologlc Gmphmrntprctdm
teukopenia (<4000/yL [4.0 x 1 0'lL]) 3 Low C3 or low C4 3
Thrombocytopenia (<1 00,000/pL [1 00 x 1 0'lL]) 4 Low Crl and low C4 4
Autoimmune hemolysis (with positive 4
direct antiglobulin test result)

Nouroprydrlrtrlc SlErpeclfic endbodlor


Delirium 2 Anti-dsDNA antibody or
Psychosis (delusions and hallucinations in Anti6mith antibody 6
absence of delirium) 3
Seizure (primary generalized or focal seizure) 5

tlucocutrneout (obcorvcd by dlnlcien)


Nonscarring alopecia 2
Oral ulcers 2
Subacute cutaneous or discoid lupus 4
Acute cutaneous lupus 6

!hrcrel
Pleural or pericandial effusion 5
Acute pericarditis 6

Murculorkclctrl
Joint involvement (synovitis in >2 joints or
tendemess of >2 joints with >30 minutes of
moming stiffness) 6

Ronel
Proteinuria >0,5 g/24 h (or equivalent rpot urine
protein-to-creatinine ratio) 4
Renal biopsy showing class ll or V lupus nephritis 8
Renal biopsy showing class lll or lV lupus nephritis 10

is fulfilled'
Classifu as systemic lupus erythematosus with a score of 10 or more if entry criterion

DNA; Hep-2 = human epithelial 2; 5LE = systemic lupus erythematosus'


f IGUR E 33 . Classification criteria for systemic luPus erythematosus' dsDNA = dou ble*tranded

2019;78;I15i-1159. PMID: 3l383717 doi:10.1136/annrheumdis 20l8 214819

47
Systemic Lupus Erythematosus

disease because ANA may be present in patients with other and objective abnormalities that cannot be categorized or
autoimmune diseases as well as in healthy individuals. Low diagnosed as a specific connective tissue disease (see Mixed
titer ANA (<1,80) is considered negative; even ANA at a more Connective Tissue Disease and Undifferentiated Connective
positive titer in the absence ol specific features of SLE may be Tissue Disease).
noninformative. Certain medications can cause drug induced lupus ery
lf the ANA result is positive and the clinical context sup thematosus (DILE), which mimics SLE (Table 2o). The syn
portive, SlE-specific autoantibodies (anti-double stranded drome is usually milder; malaise, fever, arthritis, and rash are
DNA, anti Smith, anti Ul ribonucleoprotein, anti Ro/SSA, associated with transient positivity for ANA and antihistone
and anti LalSSB), as well as tests for other autoimmune antibodies. Symptoms usually resolve after discontinuation of
diseases under consideration. should be obtained to further the offending agent. Kidney and central nervous system dis
characterize the disease (Table 19). ease are uncommon. Patients with SLE are at no more risk for
Disease activity markers, including complements C3 DILE than the general population, and medications associated
and C4, should be assessed initially and regularly thereaf with DILE are not contraindicated in patients with SLE.
ter. Complement levels are typically reduced during SLE
f,EY POIilTt
activity, reflecting immune complex formation and com
plement consumption. Rising titers of anti-double-
. Patients with systemic lupus ery.thematosus typically

stranded DNA antibody levels may be concordant with SLE initially present with skin and joint manifestations;
kidney disease activity. Other SLE autoantibodies, includ- many also have fever or weight loss.
ing ANA, do not reflect disease activity and need not be . Initial evaluation for systemic lupus erythematosus
repeated. Erythrocyte sedimentation rate and C reactive includes antinuclear antibody testing as well as routine
protein are variably associated with disease activity; laboratory testing to establish organ-specific involvement,
some patients with SLE have little to no elevation in including complete blood count, chemistry panel, and
C reactive protein during SLE flares, which may help dis urinalysis with microscopy.
tinguish flares from infection once the individual patient's o Ifresults ofantinuclear antibody testing are positive
pattern of responsiveness is established. and the clinical context is supportive, autoantibodies
specific to systemic lupus erythematosus (anti-double-
Differential Diagnosis stranded DNA, anti-Smith, anti-Ul-ribonucleoprotein,
The differential diagnosis of SLE includes multisystem dis- anti-Ro/SSA, and anti-LaiSSB) should be obtained to
eases, acute and chronic infections, medication effect, malig- further characterize the disease.
nancies (particularly hematologic), and neurologic diseases
(e.g., multiple sclerosis). Multisystem autoimmune diseases
(ANCA associated vasculitis, rheumatoid arthritis, adult-onset
Still disease, dermatomyositis, Sjogren syndrome, and mixed Management
connective tissue disease) have overlapping features but may SLE most often follows a relapsing remitting pattem (ZO'7,)
be distinguished through a careful assessment oftheir unique with periods of inactive disease, although some patients may
manifestations. have a monophasic or persistently active pattern. The most
SLE should also be distinguished from undifferentiated commonly used instrument for monitoring disease activity is
connective tissue disease, which presents with milder slmptoms the SLE Disease Activity Index (SLEDAI), which incorporates

TABLE 1*. Common Autoantibodies in Systemic Lupus Erythematosus


Autoantibody Frequenry in SLE Comments
Antinuclear >95"/o Useful as an initial screening test; assesses multiple antigens simultaneously
Anti-double-stranded DNA 50"/"-60"/" Found in more severe disease, especially kidney disease; antibody levels
commonly {ollow disease activity and are useful to monitor
Anti-Ro/SSA 30% Associated with photosensitive rashes, discoid lupus erythematosus, and neonatal
lupus erythematosus; also common when secondary Sjogren syndrome is present
Anti-U I -ribonucleoprotein 35% as.sgTrled with Raynaud phenomenon and esophageal dysmotility; also seen
in MCTD
: Anti-Smith 30% Specific for SLE; often associated with more severe disease
Anti LalSSB 20% Common in Sjogren syndrome; less common in SLE and neonatal lupus
erythematosus
Antiribosomal P 150/" Associated with CNS lupus and lupus hepatitis
CNS = central nervous system; MCTD = mixed connective tissue disease;
SLE = systemic lupus e rlthematosus.

48
Systemic Lupus Erythematosus

TABLE 20. Medications Commonly Associated With Drug-lnduced Lupus Erythematosus


Medication Antibodies Detected Comments
Procainamide ANA(75%); antihistone 207o develop DILE; fever; arthritis; serositis
Hyd ralazine ANA(20%); antihistone 5%-B% develop DILE; fever; arthritis; rare vasculitis and kidney
disease
Minocycline ANA; ANCA; anti-dsDNA rare Arthritis; vasculitis; autoimmune hepatitis
Antithyroid drugs ANA; ANCA; antihistone Vasculitic rash; rare pulmonary and kidney disease
Statins ANA; antihistone; anti-dsDNA SLE, SCLE, dermatomyositis, and polymyositis all reported
Calcium channel blockers ANA; anti-Ro/SSA; antihistone rare SCLE

Thiazide diuretics ANA; anti-Ro/SSA; antihistone rare SCLE


ACE inhibitors ANA; anti-Ro/SSA; antihistone rare SCLE

TNF inhibitors ANA (23%-57%); chromatin and anti DILE most common with in{liximab, uncommon for
dsDNA common; antihistone rare etanercept; SLE, SCLE, DLE all reported
ANA = antinuclear antibodies; DILE = drug induced lupus erghematosus; DLE = discoid lupus erythematosus; dsDNA = double stranded DNA; SCLE = subacute cutaneous lupus
erythematosus; SLE = systemic lupus erythematosus; TNF = tumor necrosis factor.

both clinical and laboratory measures. Clinical remission or (especially skin and joints) and in combination with other
low disease activity are typical goals of therapy. agents in severe disease.
Pharmacologic therapy is almost always required; the Glucocorticoids are also used in most patients with SLE,
choice of agents should reflect disease activity and specific particularly in acute disease. The glucocorticoid dose should be
organ involvement. Management requires frequent disease determined by the level of disease activity and organ systems
assessment and adjustment of treatment to reflect changing threatened. For severe disease actMty (including profound cyto-
conditions (Table 21). penia, class III/IV nephritis, and NPSLE), high-dose glucocorti
Hydroxychloroquine is a mainstay of treatment in SLE coids are recommended. For life- or organ-threatening disease
because it reduces disease-associated damage, prevents dis (such as rapidly progressive glomerulonephritis or seizures),
ease flares, and improves kidney and overall survival. In addi high dose intravenous glucocorticoids are given, typically fol
tion, hydroxychloroquine may reduce the risk for thrombosis, lowed by daily oral glucocorticoids. After disease stability is
liver disease, and myocardial infarction; improve lipid pro- achieved, glucocorticoids are tapered to the lowest effective dos
files; and improve outcomes in high-risk pregnancies. Almost age, preferably to no more than 7.5 mg/d within 4 to 6 months.
all patients with SLE without contraindications should receive Literature supports limiting exposure to glucocorticoids, espe
hydroxychloroquine. With very rare exceptions the dosage cially at high doses, because ofassociated risk for organ damage,
should be limited to 5 mg/kg/d or less, and patients should infection, and premature mortality Glucocorticoids should be
receive annual monitoring by an ophthalmologist after 5 years discontinued entirely when possible.
of treatment to reduce the risk for retinal toxicity. For moderate or severe disease, immunosuppressive ther-
Hydroxychloroquine can be used alone for mild disease apy should be initiated concurrently with glucocorticoids to

TABLE 21 . Medications Commonly Used to Treat Systemic Lupus Erythematosus


Medication Common Uses in SLE lmportant Adverse Effects
NSAIDs Arthritis; pain; fever Hypertension; Gl bleeding; AKI
Prednisone Used for all manifestations in varying doses Hypertension; glucose intolerance; weight
gain; i nfection; osteonecrosis
Hydroxychloroquine Used in almost all patients without contraindications; Gl intolerance; rash; blurry vision; retinopathy;
especially useful{or skin involvement and to prevent vacuolar myopathy
disease flares
Mycophenolate mofetil Moderate to severe disease; as effective as Bone marrow suppression; elevation of liver
cyclophosphamide for remission induction for nephritis enzymes; infection
Azathioprine Moderate to severe disease Bone marrow suppression; elevation of liver
enzymes; hematologic malignancy
Cyclophosphamide Severe organ- or life-threatening disease Bone marrow suppression; hemorrhagic
cystitis; i nfection ; ma g na ncy; i nferti ity
Ii I

Belimumab Add-on therapy for moderate to severe disease lnfusion reactions; infections

AKI = acute kidney injury; Gl = gastrointestinal; SLE = systemic lupus erythematosus.

49
Systemic Lupus Erythematosus

achieve and maintain disease control and to allow tapering of Pregnancy and Childbirth lssues
glucocorticoids. The choice of medication should be deter-
SLE is associated with a five to eightfold increase in miscar-
mined by the organs involved and disease severity. Intravenous
riage, stillbirth, premature delivery and intrauterine growth
cyclophosphamide is used as induction therapy for severe or
retardation. Outcomes are worse in patients with active dis
refractory disease (e.g., severe active nephritis, acute central
ease, nephritis, or anti-Ro/SSA and/or antiphospholipid anti-
nervous system lupus, diffuse alveolar hemorrhage, or myo-
bodies. The best time to consider pregnancy is when SLE is
carditis), followed by maintenance therapy with mycopheno
quiescent, and conception should be planned after at least
late mofetil or azathioprine. Mycophenolate mofetil is the
6 months ofadequate disease control.
preferred oral agent for lupus nephritis and is as effective as
Proteinuria may increase during pregnancy in patients
cyclophosphamide for induction therapy. The biologic agent
with SLE, making the distinction betlveen SLE and pre-
belimumab is FDA approved for patients with incomplete
eclampsia a challenge. Increases in anti-double stranded
response to conventional treatments and is useful for skin/
joint involvement and moderate/severe disease. Experience DNA antibody titers, decreasing complement levels, or the
development of active urine sediment suggests SLE as the
suggests a possible benefit as add on therapy for more severe
cause. In contrast. serum urate levels are increased in
organ involvement.
preeclampsia but not during SLE flares.
Less commonly used medications include quinacrine,
Fetuses of women who have anti-Ro/SSA or anti LalSSB
methotrexate, leflunomide, calcineurin inhibitors, and
antibodies are at risk for neonatal lupus ery.thematosus, which
rituximab. Other agents under investigation include pro
is characterized by rash and congenital heart block. Although
teasome inhibitors, Janus kinase inhibitors, interleukin 12
the risk for congenital heart block in the offspring ofan anti
and interleukin-23 inhibitors, and other B cell-acting
Ro/SSA positive woman is only 2'7,, this condition is associ
agents.
ated with significant fetal and neonatal morbidity and
Adjunctive agents may be used for specific clinical fea
mortality. After a woman bears a child with neonatal lupus
tures. NSAIDs can be used to treat arthritis and pleuropericar
erythematosus, the risk for congenital heart block is
ditis but are not disease modi[zing and may adversely affect
substantially increased (20%) in subsequent pregnancies.
kidney function and blood pressure. Statins and antihyper-
Hydroxychloroquine may reduce the overall risk. Women with
tensive agents are often used to reduce cardiovascular risks.
SLE who have concurrent antiphospholipid syndrome are at
ACE inhibitors should be considered in patients with pro-
increased risk lor miscarriage.
teinuria. Adequate vitamin D intake is important in patients
Management of medications during SLE pregnancy is
with SLE. Bone health should be monitored, with treatment
complicated. Hydroxychloroquine and low dose glucocorti-
for appropriate patients, especially those receiving glucocor
coids can be started during or continued throughout the preg
ticoids. Sun avoidance and regular use of sunscreen that
nancy. Higher dose glucocorticoids can be used to treat
blocks both UV-A and UV B should be recommended for all
patients with SLE.
flare ups or end-organ involvement. The preferred immuno
suppressive agent for SLE during pregnancy is azathioprine,
See Principles of Therapeutics for information on SLE
but this drug should be used only if necessary. Belimumab,
medication toxicities, monitoring parameters, and more. See
methotrexate, mycophenolate mofetil, and cyclophosphamide
MKSAP 19 Nephrologr for details on the treatment of lupus
should be avoided because of risks for teratogenic effects.
nephritis.
Cyclophosphamide is associated with age and dose dependent
XTY POIf,TS infertility. See Principles of Therapeutics for information on
HVC o Almost all patients with systemic lupus erythematosus medications and pregnancy.
without contraindications should receive hydroxychlo-
XEY POIilIS
roquine because it can reduce disease-associated dam-
age, prevent disease flares, and improve kidney and . Systemic lupus erythematosus is associated with a five-
overall survival. to eightfold increase in miscarriage, stillbirth, premature
o Glucocorticoids delivery and intrauterine growth retardation.
in most patients with systemic
are used
lupus erythematosus, particularly in acute disease; after o The best time to consider pregnancy is when systemic
disease stability is achieved, glucocorticoids should be lupus erythematosus is quiescent, and conception
tapered to the lowest effective dose, ideally to discontin- should be planned only after at least 6 months of ade-
uation. quate disease control.
. For moderate or severe systemic lupus erythematosus, o Fetuses of women who have anti-Ro/SSA or anti-La/SSB
immunosuppressive therapy should be initiated antibodies are at risk for neonatal lupus erlrthematosus
(rash and congenital heart block). 'l
concurrently with glucocorticoids to achieve and
maintain disease control and to allow tapering of . Hydroxychloroquine and low-dose glucocorticoids can
glucocorticoids. during or continued throughout preg-nancy.
be started

50
Sjiigren Syndrome

Prognosis TABLE 22. Extraglandular Clinical Manifestations of


Sjcigren Syndrome
The prognosis in SLE has improved significantly. The 5 year
survival rate is 90u1,, although mortality remains high com Site/Organ Manifestation/Frequency
pared to that in age-matched controls. Early mortali[z is usu General Fatigue (70%), fever (6%)
ally related to SLE disease and infections, and late mortality is S kin Dry skin (xerosis), cutaneous vasculitis:
related to cardiovascular disease. Factors adversely affecting 10o/"-1 67"
;
survival include myocarditis, nephritis, low socioeconomic Joint Arthralgia/arthritis: 36%
status, male sex, and age older than 50 years at diagnosis. Lung lnterstitial pneumonitis: 5%-9%
I(EY POIilI Kidney lnterstitial nephritis, type 1 (hypokalemic
distal) renal tubular acidosis,
. Factors adversely affecting survival in systemic lupus glomeru loneph rilis: 57o- 60/"
erythematosus include myocarditis, nephritis, low soci-
Neurologic CNS: demyelinating disease, myelopathy,
oeconomic status, male sex, and age older than 50 years cranial nerve neuropathy
at diagnosis. Peripheral nervous system: small-fiber
neuropathy, mononeuritis multiplex,
peripheral neuropathy
8o/"-27"/ofor CNS and peripheral nervous

Sjtigren Syndrome Gastrointestina I


system

Autoimmune hepatitis, primary biliary


cirrhosis: 3"/o-207"
Epidemiology and
Hematologic Lym p homa, cytope ni a : 27o
Pathophysiology
Other Systemic vasculitis (7%), cryoglobulinemia
Sjogren syndrome is a systemic autoimmune exocrinopathy 1 2o/"), Rayna ud phe nomenon ( 1 6%),
(4o/"-
primarily affecting salivary and lacrimal glands. The condition thyroid d isease (1 0"/"- 1 5"/")
is female predominant, with a female-to-male ratio between 6 CNS = central nervous system-
and 9 to 1. Sjogren syndrome most commonly presents in the
fifth and sixth decades of life, but patients of any age may be
affected; a pediatric variant is also recognized. Primary SjOgren include fatigue and Raynaud phenomenon. Both peripheral
syndrome occurs in isolation; secondary Sjogren syndrome and central nervous system involvement may be seen, although
develops with other rheumatologic diseases, most commonly the former is more common. The typical peripheral nervous
rheumatoid arthritis and systemic Iupus erythematosus. system manifestation is axonal polyneuropathy. Central nerv
Whereas the primary form is uncommon (ranging from 0.5 ous system involvement often presents as demyelinating dis
to 5 patients/l0O0), secondary Sjdgren syndrome is often ease, such as optic neuritis, transverse myelitis, or a multiple
observed (10%-30%) among populations with predisposing sclerosis-like presentation. Cutaneous, pulmonary and kid
rheumatologic conditions. Sjdgren syndrome is thought to ney disease are relatively rare. Nonspecific laboratory findings
result from activation of mucosal epithelial cells by unknown include hypocomplementemia, hypergammaglobulinemia,
stimuli. This, in turn, leads to tonic activation of the innate and leukopenia, and anemia of inflammation.
adaptive immune systems, including extensive influx of lym Patients with Sjogren syndrome are at increased risk for
I phocytes into exocrine glands. The resulting epithelial and non Hodgkin lymphoma compared with the general popula
obliterative damage impairs the ability to produce or deliver tion. Estimates suggest as much as a 44 fold increased risk fbr
secretions. mucosal associated lymphoid tissue lymphoma; this increase
presumably results from an elevated risk for lymphomatous
transformation due to chronic activation of tissue lympho-
Cli nica I Manifestations cytes. Clinical features conferring a higher risk include lym
The most common presentation of Sjdgren syndrome is sicca, phadenopathy, recurrent parotid gland swelling, monoclonal
I
consisting of ocular and oral dryness. Patients report gritty gammopathy, depressed C4 complement, and decreased rheu
matoid factor if titer is elevated at baseline. About 5'7, of
.

eyes or a foreign body sensation. Oral dryness can cause diffi


i
culty eating unmoistened food and increases the risk for den patients with Sjdgren syndrome develop lymphoma over time,
tal caries. Symmetric parotid and lacrimal swelling can occur. usually within the first decade after diagnosis.
Other exocrine glands can be involved, leading to skin and
vaginal dryness. Pancreatic involvement has been reported.
Diagnosis
I

Extraglandular manilestations may occur in some


patients (Table 22). Arthralgia and fibromyalgia are common, An evaluation for Sjogren syndrome is typically triggered
but nonerosive arthritis in the absence of rheumatoid arthritis when the patient reports symptoms of ocular or oral dryness.
is uncommon. Other common extraglandular manifestations It requires objective confirmation of exocrinopathy along with

51
Sjtigren Syndrome

demonstration of autoimmunity (Table 23 shows validated enlargement. Sarcoidosis, IgG4 related disease, granulomato
classification criteria). Ophthalmic dryness can be assessed sis with polyangiitis, lymphoma, and infection (such as

by using the Schirmer test, in which a strip of filter paper is mumps, hepatitis C virus infection, and HIV infection) may be
placed under the lower eyelid and wetting is measured; less associated with similar findings. Ocular and oral dryness may
than 5 mm of wetting in 5 minutes indicates dryness. More also occur with aging, in other disease states (such as rheuma-
formally, dryness and corneal damage are ascertained with toid arthritis), or as an adverse effect of medications or treat
slit lamp examination by an ophthalmologist. Topical stain- ment (such as antidepressants and antihistamines or radiation
ing may be used to identiff injury and fissures in the cornea. of the head and neck). Unilateral or single enlargement of a
Dry mouth is assessed by direct examination, with dental gland should raise concern for obstruction or malignancy.
caries and a lack of saliva suggesting the diagnosis.
Measurement of salivary flow, or sialometry should ideally be
performed to determine whether true dryness is present (and
Management
to show ductal architecture). However, this is rarely done Management of sicca centers on preserving moisture and
because it requires special equipment that is not available in relieving symptoms. Eye dryness is treated with over the
most settings. Other forms of imaging (e.g., CT, gallium scan counter artificial tears, gels, and ointmentsi lor refractory or
ning) are not generally helpful but may rarely be useful in more severe symptoms, topical cyclosporine or a small
defining organ involvement. molecule integrin antagonist may be added. Plugging the lac-
Multiple autoantibodies may be found in Sjdgren syn- rimal ducts to promote tear retention is widely performed, but
drome, but none are independently diagnostic. Antinuclear clinical trials have not shown its effectiveness. For oral
antibodies and/or rheumatoid factor are often present at high dryness, first-line treatment in patients with less severe
titers; their coexpression is more suggestive of Sjogren s1m- symptoms should be nonpharmacologic salivary stimulation,
drome than is positivity of only one. The presence of anti-Ro/ both with gustatory stimulants (e.g., sugar-free candy) and
SSA autoantibodies is specific in primary Sjdgren syndrome mechanical stimulants (e.g., sugar-free gum).
but less so in secondary Sjdgren syndrome because these anti Pharmacologic therapies should be reserved for moder
bodies are also common in systemic lupus erythematosus. ate or severe oral dryness. These include the muscarinic
Anti LalSSB autoantibodies are characteristic ofSjogren syn- agonists pilocarpine and cevimeline, which may not be well
drome but are not included in formal classification criteria. tolerated; they are also contraindicated in narrow-angle
When initial evaluation is uninformative, a lip (minor salivary glaucoma. Artificial saliva may be used for severe dysfunc
gland) biopsy should be considered. Characteristic histologic tion, but its effects are so transient that most patients do not
features include aggregate foci (>50 cells/aggregate) of CD4 T find it useful. Meticulous dental care is essential to prevent
cells, B cells, and plasma cells surrounding secretory ducts. gingivitis and dental decay, regardless of symptom severity.
The differential diagnosis of SjOgren syndrome tlpically Vaginal symptoms may be treated with topical lubricants or
relates to the presence of sicca and/or parotid and lacrimal estrogen.
Active extraglandular autoimmune disease should be
TABLE 23.
American College of Rheumatology/European treated with systemic therapies. In most cases, glucocorticoids
League Against Rheumatism Classification Criteria for may be considered as a first line option, but only at the
Primary Sjogren Syndrome" minimum dose and length of time necessary to control the
Presence of subjective finding of Sjtigren syndrome active disease. Immunomodulatory/immunosuppressive (e.g.,
(e.9., sicca) plus any combination of the following
hydroxychloroquine, methotrexate, cyclophosphamide) and
resulting in a score of >4:
biologic (e.9., anti-B cell therapy with rituximab or beli-
Item Score mumab) agents should be considered for second or third line
Salivary gland biopsy specimen showing )1 foci of 2 therapy, or for glucocorticoid sparing. To the extent possible,
lymphocytic infiltrate/4 mm2 decisions should be tailored to severity and the organ or organs
Anti- Ro/SSA autoa ntibodies 3 affected (e.9., pneumonitis, central nervous system disease).
Ocular staining score >5 in at least one eye Musculoskeletal pain is common in Sjogren syndrome but
Schirmertest<5 mm/5 min in at least one eye should not be treated with immunomodulatory or biologic
Unstimulated whole saliva flow rate <0.1 mUmin
therapy unless there is objective evidence of active inflamma-
tory arthritis. Myalgia and arthralgia, along with chronic non
nExclusion criteria include prior head/neck radiation and
history of active hepatitis
C virus infection, AIDS, sarcoidosis, amyloidosis, graft-versus host disease, and specific pain, should be treated symptomatically.
lgG4 related d sease.

From Shiboski CH, Shib,oski SC, Seror R, et al; lnternational Sjogren's Syndrome
Criteria Working Group. 2016 American College of Rheumatology/European
League Against Rheumatism Classification Criteria for Primary Sjogren's
Prognosis
Syndrome: a consensus and data-driven methodology involving three
international patient cohorts. Arthritis Rheumatol. 201 7 Jan;69(1 ):35-45. The prognosis for primary Sj6gren syndrome is favorable, with
doi:1 0.1 002/art.39859. IPMID: 27785888]. Copyright 201 6, American College of no overall increase in mortality. Management of sicca reduces
Rheumatology. Adapted with permission from John Wiley & Sons, lnc.
the risk for corneal damage or tooth loss. In rare instances,

52
ldiopathic lnflammatory Myopathies

end-organ involvement can be life threatening. Secondary


Sj6gren syndrome may be associated with significant morbid-
Evaluation of the Patient With
ity from the primary condition. Muscle Pain or Weakness
The differential diagnosis of muscle disease is broad and
rEY POITII
includes hereditary environmental, infectious, and acquired
o Sjdgren syndrome is an autoimmune exocrinopathy
conditions (Table 2a). The IIMs commonly present with sym
affecting salivary and lacrimal glands; the most com metric proximal muscle weakness without substantial muscle
mon presentation is sicca (dryness of the eyes and/or pain or tenderness. Serum creatine kinase levels are almost
mouth). universally elevated, and this elevation is a marker of muscle
o Patients with Sjdgren syndrome are at increased risk for damage. The presence of pain or tenderness should prompt
non-Hodgkin lymphoma. consideration of alternative causes, such as infectious, endo-
. A diagnosis of Sjogren syndrome is lzpically triggered crine (thyroid), or drug induced myopathies. Medications and
by the patient's report ofocular and oral dryness and drugs are common causes of muscle pain and weakness and
requires objective confirmation of exocrinopathy, along should also be considered in the differential diagnosis ofthe
with demonstration of autoimmunity inflammatory myopathies (Table 2S).
Muscle weakness must be differentiated from general
. In patients with Sjogren syndrome, management of
ized weakness, which is common and caused by conditions
sicca centers on preserving moisture and relieving
such as cardiopulmonary disease, malignancy, depression,
symptoms; extraglandular involvement is treated with
and deconditioning. Primary neurologic disorders (e.g.,
immunosuppression.
spinal muscle atrophies, amyotrophic lateral sclerosis, and
myasthenia gravis) may cause muscle weakness but are dis
tinguished by involvement of nonproximal muscle groups
and nerve involvement on electromyography; an elevated
ldiopathic lnflammatory serum creatine kinase level is uncommon. Exercise-induced
Myopathies weakness may point to a metabolic or mitochondrial cause
rather than an IIM.
Overview
Idiopathic infl ammatory myopathies (llMs) are characterized
by muscle inflammation and proximal muscle weakness.
Epidemiology and
Subcategories in adult patients include polymyositis, dermato Pathophysiology
myositis, immune mediated necrotizing myopathy, and inclu IIMs are uncommon and heterogeneous. The annual incidence
sion body myositis. is between 2 and 10 cases per million. Two peaks of onset

TABLE 24. Differential Diagnosis of Myopathy


Myopathy Common Examples Comments
ldiopathic inflammatory Dermatomyositis; polymyositis; immune-mediated Dermatomyositis and polymyositis: acute-subacute
myopathies necrotizing myopathy; inclusion body myositis onset, symmetric, proximal
Dermatomyositis: pathognomonic rash
lmmune-mediated necrotizing myopathy: acute
onset, symmetric, proximal
lnclusion body myositis: insidious onset, proximal
and distal
Systemic rheumatologic Systemic lupus erythematosus; systemic sclerosis; Prominent extramuscular features typical of
disease mixed connective tissue disease underlying disorder
Muscular dystrophies Duchenne muscular dystrophy; Becker muscular Childhood onset; cardiomyopathy possible
dystrophy
Metabolic myopathies Glycogen storage diseases; carnitine Exercise intolerance
pal mitoyltra nsferase deficiency

Mitochondrial myopathies Kearns-Sayre syndrome; Leigh syndrome Variable age of presentation and features; isolated
myopathy or with neurologic, multisystem disease
Endocrine disorders Hypothyroidism; hyperthyroidism; adrena I Prominent myalgia
insufficiency
Infection-ind uced Bacterial: pyomyositis; viral: influenza; parasitic Prominent myalgia; sometimes accompanied by
trichinosis fever

53
ldiopathic lnflammatory Myopathies

TABLE 25. Drug-lnduced MyoPathies


Drug Time Course Clinical Presentation
Alcohol lncreased with long-term use Asymptomatic elevations of serum creatine kinase;
chionic muscle atrophy; acute, severe rhabdomyolysis
with kidney failure
Antimalarials Can occur after prolonged use lnfrequent elevation of serum creatine kinase (30%);
muscle weakness (50%); myopathic electromyog ram
findings (100%); cardiomyopathy can occur
Cocaine Can occur after single use Asymptomatic elevations of serum creatine kinase; acute,
severe rhabdomyolysis with kidney failure

Colchicine Usually months to years; increased risk with Proximal muscle weakness with elevations of serum
coad ministration of cytochrome P450 inhibitors creatine kinase; mild sensory symptoms; reduced reflexes

Glucocorticoids lncreased with long-term use Proximal muscle weakness in the absence of elevations
of serum creatine kinase
Statins Usually weeks to months but can occur at any time; Elevations of serum creatine kinase with myalgia and
increased risk with preexisting neuromuscular disease, weakness
hypothyroi d ism, kid n ey fa i I u re, a nd/o r coad m i n istrati o n
of cytochrome P450 inhibitors; may also trigger immune-
mediated necrotizing myopathy
Zidovudine Variable; may be more common after long-term use Elevations of serum creatine kinase with myalgia and
weakness

occur: the first in childhood and the second in middle age. A f,EY POtrtt
female adult predominance is approximately 2:1, with the . Inflammatory myopathies commonly present with
exception of inclusion body myositis, which preferentially
symmetric proximal muscle weakness without substan-
affects men.
tial muscle pain or tenderness; elevation ofserum cre-
Like many other autoimmune diseases, IIMs probably
atine kinase level is almost universal and is a marker of
result from interactions between genetic and environmental
muscle damage.
factors. The clearest genetic association is with HLA alleles,
o Muscle pain or tenderness should prompt consideration
including HLA-DRBI.03:01 with polymyositis and HLA
DRB'08:01 with dermatomyositis. Distinct HLA alleles have of infectious, endocrine (thyroid), or drug-induced
also been associated with different IIM subgroups, as myopathies.
defined by myositis-specific antibodies. However, because
HLA associations in IIM vary by ethnic groups around the
world, their utility in diagnosis and categorization is Clinical Features
limited. Muscle lnvolvement
Environmental factors may activate or modify gene Most patients with IIM present with symmetric weakness
expression and likely influence disease progression and sever affecting the proximal muscles. Neck flexors and shoulder and
ity in addition to onset. Associations with infections, medica- hip girdle muscles are commonly involved. Patients may expe-
tions, occupational exposures, ultraviolet light, and other rience difficulty rising from a chair or climbing stairs.
factors have been reported. The clearest associations have been Symptoms usually progress over months, except in inclusion
with medications, including statins, cytokine-targeted thera body myositis, which progresses over years. Respiratory mus-
pies (interferons and tumor necrosis factor inhibitors), and cles can be affected, and some patients experience difficulty
checkpoint inhibitors. swallowing (dysphagia or dysmotility) related to involvement
Immune mechanisms that contribute to IIM vary of the upper gastrointestinal tract. Cardiac muscle can be
between subtypes and include the presence of T- and involved in some subsets of IIM.
B-lymphocytic infiltrates, as well as increased cytokine and
chemokine levels in muscle tissue. Autoantibodies are Pulmonary Disease
expressed in many but not all patients; specific autoantibod Interstitial lung disease, most commonly nonspecific intersti
ies and differing histologic patterns are associated with dis tial pneumonitis, occurs in both polymyositis and dermato-
tinct sets ofclinical characteristics, suggesting that pathogenic myositis and is an important cause of morbidity and mortaliry
mechanisms vary across subsets. Non-immune-mediated particularly in patients the antisynthetase syndrome (see
mechanisms are also important, especially for the progres below). Muscle weakness of the chest wall can also cause
sion to atrophy, fibrosis, and structural damage to muscle breathing difficulties. Pulmonary involvement is not typically
tissue. seen in inclusion body myositis.

54 i
ldiopathic lnflammatory Myopathies

Skin lnvolvement body myositis. Although clinically useful, these categories do


Pathognomonic skin findings are associated with dermatomy- not fully account for the varying features within each group,
ositis but not polymyositis. Mechanic's hands can occur in and alternative ways of subcategorizing patients are under
patients with the antisynthetase syndrome associated with development. Some patients with dermatomyositis or polymy
either polymyositis or dermatomyositis. ositis may further demonstrate the antisynthetase syndrome.

Cardiac Disease Dermatomyositis


In addition to cardiac muscle involvement, arrhythmias, atrio In addition to muscle involvement, patients with dermatomy-
ventricular block, and pericarditis have been reported. Patients includ-
ositis usually have characteristic skin findings (Table 27),
with myositis are at higher risk for heart failure and myocar ing heliotrope rash (Figure 34), Gottron papules (Figure 35), and
dial infarction. Gottron sign. Other skin findings include periungual erythema,
mechanic's hands (a dermatologic manifestation characterized
Malignancy by rough, cracked, scaly skin along the lateral aspects of the
There is a clear association between malignancy and IIM; digits and palms with horizontal lines resembling the weath
patients with dermatomyositis have a 3 to l2-fold higher risk ered hands of a laborer; Figure 36), facial erythema or photo
for malignancy than age matched populations. The associa- sensitive poikiloderma (shawl and V signs) (Figure 37), and
tion between malignancy and polymyositis is smaller. The risk calcinosis cutis. Although both skin and muscle biopsies can be
for malignancy peaks in the 3 years after the diagnosis of der- helpful for diagnosis and prognosis, some cases of dermatomy
matomyositis or polymyositis. Commonly associated cancers ositis can be diagnosed on the basis of characteristic skin find-
include ovarian, lung, pancreas, stomach, and colon cancers ings without a muscle biopsy.
and lymphoma. Malignancy risk varies by antibody associa- A small subset of patients with dermatomyositis never
tion (Table 26). Age and sex-appropriate cancer screening is develop muscle involvement (amyopathic dermatomyositis) ;
indicated at the time of diagnosis of dermatomyositis or poly- these patients can be diagnosed with skin biopsy in the setting
myositis and regularly for several years thereafter. Many of at least 6 months without muscle involvement.
experts advocate CT of the chest, abdomen, and pelvis, par-
ticularly in patients with dermatomyositis. Polymyositis
Patients with polymyositis have proximal muscle weakness
without the skin involvement of dermatomyositis. Because
Types of ldiopathic patients with polymyositis lack pathognomonic clinical fea-
lnflammatory Myositis tures, a muscle biopsy is recommended to confirm the diagno
IIMs can be subcategorized into dermatomyositis, polymyosi- sis and differentiate this subgroup from other forms of myosi
tis, immune-mediated necrotizing myopathy, and inclusion tis. In general, patients with polymyositis have a lower risk for

TABLE 26. ldiopathic lnflammatory Myopathy-SpecificAntibodies


AntibodyTarget Type of llM Skin Features Muscle Features Other Clinical Features
TIF-1-1(formerly DM Classic skin rashes Typical lncreased cancer risk
p1 55/1 40)

Mi-2 DM Classic skin rashes Typical Low risk for cancer and ILD
NXP-2 DM Severe, ulcerative; high lncreased Low risk for ILD and high
risk for calcinosis risk for cancer

SAE DM, amyopathic DM Classic skin rashes Mild or none


MDA5 DM, amyopathic DM Mechanic's hands Mild High risk for ILD
Jo- , EJ, OJ, PL-7
1 , Antisynthetase Mechanic's hands Typical Very high risk for ILD; often
PL-12,KS,Ha,Zo syndrome includes arthritis and
Raynaud phenomenon

SRP IMNM None Severe/necrotizing


HMG-CoA IMNM None Severe/necrotizi ng May be associated with
statin use
NT5c1A IBM (poor sensitivity) None May include hand/
forearm weakness,
may be asymmetric

DM=dermatomyositis;HMG-CoA=hydroxy-methylglutarylcoenzymeA;lBM=inclusionbodymyositis;lM=idiopathicinflammatorymyopathy;lLD=ioterstitiallungdisease;
IMNM = immune-mediated necrotizing myopathy; MDA5 = melanoma differentiation-associated gene 5; NXP 2 = nuclear matrix protein 2; SAE = small ubiquitin like modifier
activating enzyme; SRP = signal recognition particle; TIF- 1 -1= transcription intermediary factor 1 1.

55
Idiopathic lnflammatory Myopathies

TABLE ??. Cutaneous Manifestations of Dermatomyositis


Cutaneous Manifestation Location Clinical Appearance
Gottron rash (Gottron Metacarpophalangeal and proximal Erythematous to violaceous papules; occasional scale; can
papules and Gottron sign) interphalangeal joints (Gottron papules); ulcerate; atrophic scars may occur
occasionally on distal interphalangeal joints,
elbows, knees (Gottron sign)
Heliotrope rash Eyelids Subtle pink to deep purple or brown discoloration; may
be associated with edema of eyelids or periorbital edema
V sign V of neck Erythematous to violaceous papules to patches
Shawl sign Base of posterior neck to upper back Erythematous to violaceous papules to patches
Fixed erythema Malar area (may cross nasolabial folds); Erythematous to violaceous papules to patches
flanks of trunk
Nail area changes Periungual area; cuticles Periungual erythema; capillary dilatation and dropout;
cuticular hypertrophy; cuticular infarcts
Mechanic's hands Lateral aspects of digits and palms Hyperkeratotic fissuring of the palmar and lateral surfaces
of the fingers resembling the hands of a laborer

F IG UR E 3 4. The heliotrope rash of dermatomyositis is a distinctive purple or FtG U RE 36. MechaniCs hands with hyperkeratotic, fissured skin on the palmar
lilac, symmetric erythema of the eyelids that may be accompanied by slight edema, and lateral aspects of fingers seen in a patient with antisynthetase syndrome.
generally focused around the orbits.

f I G U R E 3 5 . Gottron papules are violaceous, slightly scaly plaques over the bony FIGURE 37. The shawl sign demonstrating a photodistributed erythematous
prominences on the hands. Ihe Gottron sign consists of similar findings over the patchy rash on the upper back in a patient with dermatomyositis.
extensor surfaces of joints other than the hands, usually the elbows, knees, and ankles.

56
I

i ldiopathic lnflammatory Myopathies


l

pulmonary or other extramuscular disease features than those clinical features and different antibody associations. See Mixed
t with dermatomyositis. Connective Tissue Disease and Undifferentiated Connective
i
Tissue Disease for further discussion.
I
Antisynthetase Syndrome .xIY P:01ilT5
t The antiqmthetase syndrome occurs in approximately 30% of
'.
I

patients with dermatomyositis or polymyositis. The syndrome is


. Pulmonary and cardiac involvement are important
sources of morbidity and mortality in idiopathic
associated with antibodies to aminoaryl-transfer RNA (IRNA)
qmthetases (most commonly anti-Jo 1, directed at histidyl- infl ammatory myopathies.
IRNA rynthetase) and is characterized by a high risk for and o Age- and sex-appropriate cancer screening is indicated
i greater severity ofinterstitial lung disease. Other features include in dermatomyositis and polymyositis because of an
arthritis, Raynaud phenomenon, and mechanic's hands. increased risk for malignancy.

i
r Gottron rashes and heliotrope rashes are characteristic
lmmune-Mediated Necrotizing Myopathy of dermatomyositis.
Some patients previously included in the poll,rnyositis group o Immune-mediated necrotizing myopathy is character-
are now recognized as belonging to a distinct subset known as ized by severe, rapidly progressive proximal muscle
immune mediated necrotizing myopathy. This condition is weakness; high serum creatine kinase levels; and few
characterized by rapidly progressive, severe muscle weakness; extramuscular manifestations.
increased likelihood of muscle pain; high serum creatine o Inclusion body myositis is an insidious condition in
kinase levels; and muscle biopsy specimens demonstrating
older adults that affects both the proximal and distal
prominent myonecrosis with only limited inflammation. These
flexor muscles.
patients tlpically lack extramuscular features. An immune-
mediated mechanism is suggested by the presence of antibod
ies to signal recognition particle or 3-hydroxy 3 methylglutaryt Diagnosis
coenzyme A (HMG-CoA) reductase in about two thirds of IIM should be considered in any patient who has progressive
patients (see Table 26). HMG CoA is the clinical target of action proximal muscle weakness without another explanation.
of statins in lipid lowering, and statin exposure may result in
antibodies to HMG-CoA reductase, which can sometimes lead Classification Criteria
to immune-mediated necrotizing myopathy. However, statins Classification criteria are desigrred to allow researchers and cli-
more commonly induce muscle disease through direct rather
nicians to better distinguish IIM from other conditions affecting
than autoimmune myotoxicity. In addition, many patients with
muscles and to better differentiate among IIM subgroups.
anti HMG CoA reductase antibodies have never been exposed Although intended for research studies, classification criteria
to a pharmacologic statin. In patients with statin-triggered can help support a clinical diagnosis. Current criteria may allow
immune mediated necrotizing myopathy, weakness may per
classification without a muscle biopsy. These criteria first deflne
sist or progress even after statins are discontinued.
the probability of presence or absence of IIM and then apply the
classification tree for subgroups of IIMs (Figure 38). Limitations
lnclusion Body Myositis include the fact that there are too few patients with immune-
Inclusion body myositis is an insidious condition affecting mediated necrotizing myopathy or antisynthetase syndrome to
older adults. It is distinguishable from other forms of myositis characterize these subsets, along with lack of inclusion of MRI
by its slow progression and different pattern of muscle involve- fi ndings or most myositis-specific antibodies.
ment. Patients report a history of weakness for an average of
5 years before seeking care. Both proximal and distal muscle Muscle-Related Enzymes
involvement occur; finger and forearm flexor involvement is Elevated serum muscle en-4lrnes are useful in dffierentiating
nearly pathognomonic. Muscle distribution is usually but not intrinsic muscle disease from neurologic causes of weakness.
always symmetric. Up to half of patients with inclusion body Elerated serum creatine kinase levels are a sensitive indicator of
myositis have cricopharyngeal muscle involvement, leading to muscle pathologz, and lwels uzually parallel the severity of involve-
dysphagia and increased risk for aspiration. ment. Aldolase level may be ele'"ated when serum creatine kinase
Serum creatine kinase levels are elevated to a lesser extent levels are normal. Serum aminotransferase and serum lactate
than seen in other forms of IIM. Autoantibodies are less often dehydrogenase levels may also be elevated but are nonspeciflc.
present, but the anti NT5c1A autoantibody is found in up to Troponins can be used to screen for cardiac muscle involvement.
t
i half of patients and rarely in controls (see Table 26).
! Autoantibodies
Overlap Syndromes Only about half of patients with IIM have identifiable autoan-
I Other autoimmune connective tissue diseases may include tibodies. Some autoantibodies, such as antinuclear antibodies,
myositis as a clinical feature but typically have other prominent lack specificity. The myositis specific antibodies have more

I
57
I
tdiopathic lnf lammatory Myopathies

Patient meets the EULAR/ACR classification criteria for llM

Age at onset of first


symptom <1 8 years
No Yes

Heliotrope rash or
Heliotrope rash or
Gottron papules or
Gottron papules or
Gottron sign Gottron sign

No Yes

Objective symmetric weakness, usually


progressive, of the proximal upper extremities or
Clinical features* Objective symmetric weakness, usually
or progressive, of the proximal lower extremities or
Muscle biopsy
feature** Neck flexors are relatively weaker than neck
extensors or in the legs, proximal muscles are
relatively weaker than distal muscles
No Yes

No Yes

PM Juvenile myositis
(rMNM) other than JDM ***
IBM ADM DM JDM

tIGURE 38.2017 classificationcriteriaforadultandjuvenileidiopathicinflammatorymyopathies(llM)andtheirmajorsubgroups,fromtheEuropeanLeague


Against Rheumatism (EULAR)/American College of Rheumatology (ACR). ADM = amyopathic dermatomyositis; DM = dermatomyositis; IBM = inclusion body myositis;
IMNM = immune-mediated necrotizing myopathy; Jltyl = juvenile dermatomyositis; pM = polymyositis.
*Finger flexor weakness and response to treatment: not improved.

**Muscle biopsy: rimmed vacuoles required for diagnosls.

***Juvenilemyositis0lherthanJDl\,4wasdeve0pedbasedonexpertopini0f l[./lN[,4andhyp0myopalhicD]\.,lwereloofewtoallowsubclassification.

IPMID:290795901d0i:10.1136/annrheumdis 2017 211468. Reproduced with permissi0n 01 BMJ Publishing Gr0up Ltd.

diagnostic and prognostic value and can be useful for identify- muscle fiber necrosis, degeneration, and regeneration, but
ing risk for specific features or subsets (see Table 26). differ in the types and locations of the cell infiltrates. Muscle
biopsy findings in inclusion body myositis include mononu-
lmaging clear cell infiltrates, "rimmed" vacuoles, and a lack of muscle
Although not required to diagnose IIMs, MRI is useful in iden cell necrosis. Electron microscopy can show characteristic
ti[zing regions of active muscle inflammation and in guiding inclusion bodies.
muscle biopsy. MRI can also assess the extent and severity of Skin biopsy can be useful in the diagnosis of dermatomy-
muscle involvement and therapeutic efficacy. ositis, with typical findings including a perivascular lympho
High-resolution CT should be used to assess for intersti- cytic infiltrate, vacuolar changes at the dermal-epidermal
tial lung disease in high risk patients. junction, and increased dermal mucin.

xtY P0tilTs
Electromyography o Elevated semm creatine kinase levels are a sensitive
Although no single linding is pathognomonic, electromyogra indicator of muscle pathologz, and levels usually parallel
phy may reveal findings characteristic of active myositis. This the severity of involvement.
test can be uncomfortable, and results may be falsely normal . Myositis specific autoantibodies can be used to diag-
in the presence ofactive disease because ofpatchy distribution
nose and subcategorize some idiopathic inflammatory
of muscle inflammation (sampling error).
myopathies but are not present in all patients.
r In patients with idiopathic inflammatory myopathies,
Histopathology MRI can help locate a site for muscle biopsy and assess
Muscle biopsy is valuable for diagnosing and distinguishing therapeutic efficacy.
IIMs and for excluding metabolic myopathies, mitochondrial
o Muscle biopsy is valuable for diagnosing idiopathic
disease, and infection.
inflammatory myopathies and for excluding metabolic
Polymyositis and dermatomyositis share common histo
myopathies, mitochondrial disease, and infection.
pathologic features, including inflammatory cell infiltrates,

58
Systemic Sclerosis

l
Management XEY POITII
Glucocorticoids are the cornerstone of therapy for most forms o Initial treatment of most idiopathic inflammatory myo
of tlMs. Starting dose varies by severity and organ involvement pathies consists of glucocorticoids, with additional
but is fypically high (up to 1 mg/kg of prednisone) and should immunomodulatory agents (most commonly metho
be tapered within 6 months. Prr-rlonged exposure to high-dose trexate and azathioprine) often required to control
glucocorticoids, particularly in older patients, can lead to inflammation or serve as glucocorticoid-sparing agents.
glucocorticoid induced myopathy, resulting in proximal mus . Prolonged symptoms before treatment, greater weak-
cle weakness and diagnostic conlusion in a patient who had ness at presentation, extensive multisystem disease, co-
previously been improving. occurrence of malignancy, and some autoantibodies
Immunomodulatory agents are used to better manage and histologic patterns confer a poorer prognosis in
intlammation or as glucocorticoid sparing agents; methotrex patients with idiopathic infl ammatory myopathies.
ate and azathioprine are the prefbrred initial agents. Intravenous r Inclusion body myositis usually does not respond to
immune globulin (IVIG) can be used in addition to or as an immunosuppression and typically results in wheelchair
alternative to traditional agents; evidence for IVIG is strongest
dependence within 10 to 15 years.
fbr dermatomyositis and immune mediated necrotizing myo
pathy. Mycophenolate, tacrolimus, cyclosporine, rituximab, or
cyclophosphamide can be considered in patients in whom
initial treatment has failed or who could not tolerate initial
Systemic Sclerosis
treatment. Immune mediated necrotizing myopathy is often Epidemiology and
acute and severe and should be treated aggressively with com-
binations of glucocorticoids and immunosuppressive agents; it
Pathophysiology
may respond particularly well to IVIG and rituximab. Statins Systemic sclerosis (SSc) is a multiorgan disease characterized by

should be strictly avoided. fibrosis and vasculopathy. The skin is the most apparent target,
Inclusion body myositis usually does not respond to but intemal organs are also affected. SSc is relatively rare, with a
prevalence of 275 cases per million and an annual incidence of
immunosuppression, and any therapeutic trials should be
19 cases per million. The peak incidence occurs in patients age 20 to
discontinued if benefit is not appreciated. An initial response
50 years. It is more common among women and Black persons.
to empiric glucocorticoids should encourage the physician to
consider an alternative diagnosis. Reports suggest that IVIG Vascular injury vascular and visceral fibrosis, and innate
may occasionally be helpful in managing dysphagia. and adaptive immune activation with autoantibody production
play interactive roles in SSc. Genome wide association studies
Physical therapy can help maintain muscle lunction and
have identified up to 28 risk loci for SSc and underlined the
should be universally recommended.
importance of natural killer cell activity in SSc. One of the earli
est SSc manifestations is Raynaud phenomenon, suggesting that
vascular injury precedes the fibrotic reaction. With vascular
Prognosis
damage, the endothelial cells release endothelin-l, a potent
The prognosis in IIM varies fiom resolution to severe,
vasoconstrictor that also induces vascular smooth muscle prolif-
treatment-resistant disease with functional impairment and
eration and fibroblast activation. Platelet activation occurs con
early mortality. The degree of serum creatine kinase elevation
currentlywith subsequent release of growth factors that promote
at diagnosis does not predict outcome. Prolonged disease further vasoconstriction, fibroblast activation, and collagen pro
activity before treatment, greater weakness at presentation,
duction. B cells are also activated and produce interleukin 6,
extensive multisystem disease, and the co occurrence of
which further stimulates fibroblasts and leads to autoantibody
malignancy confers a poorer prognosis, as do certain myositis production. These and other processes cause increased collagen
specific autoantibodies or histologic features. production and deposition by activated scleroderma fibroblasts,
Patients with anti Jo 1 antibodies have worse outcomes
along with progressive obliterative vasculopathy
(due to interstitial lung disease), whereas those with anti
Mi 2 antibodies often respond completely, despite a rapidly
progressive course of dermatomyositis. Patients with Classification
immune-mediated necrotizing myopathy may respond well The 2013 American College of Rheumatology/European
to statin discontinuation (if appropriate) and aggressive League Against Rheumatism classification criteria for SSc pro
immunosuppression. vide a rigorous definition of SSc that is not formally intended
Giren the lack of effective treatment, patients with inclu Ibr diagnosis and may be too restrictive in clinical practice.
sion body myositis typically demonstrate a slow and gradual However, they can provide useful guidance when a patient
loss of function, with progression to wheelchair dependence presents with features suggesting SSc (Table 28).
within 10 to 15 years of symptom development. However, life SSc is divided into three subtypes based on the extent
expectancy does not appear to be affected. of skin involvement: Iimited cutaneous systemic sclerosis

59
Systemic Sclerosis

TABLE 28. American College of Rheumatology/European League Against Rheumatism Classification Criteria for Systemic Sclerosis
Manifestation Add:tional Manifestations WeighVScore"
Skin thickening of fingers of both hands extending I
proximalto the MCPs
Skin thickening of the fingers (count higher score Puffy fingers 2
only)
Sclerodactyly of fingers distal to the MCPs but proximal to PlPs 4
Fingertip lesions Digitaltip ulcers 2

Fingertip pitting scars 3

Tela ngiectasia 2

Abnormal nailfold capillaries 2

Pu lmonary hypertension and/or interstitial lung Pulmonary hypertension 2


disease (maximum score is 2)
lnterstitial lung disease 2

Raynaud phenomenon 3

SSc-related autoantibodies (maximum score is 3) Anticentromere )


Anti-Scl-70 (antitopoisomerase-1 ) 3

Anti-RNA polymerase lll 3

MCP = metacarpophalangeal; PIP = proximal interpha angeal; SSc = systemic sclerosis.

uThe total score is determined by adding the maximum weight (score) in each category- A score of 9 or more equates to definite systemic sclerosrs.

From van den Hoogen F, Khanna D, Fransen J, Johnson SR, Baron M, Tyndall A, et al. 201 3 classification criteria for systemic sclerosis: an American College of Rheumatology/
European League Against Rheumatism Collaborative lnitiative. Ann Rheum Dis 201 3;72: T 747-55. doi:1 0.1 1 36/ann rheumdis-20 1 3-204424. IP MID: 240926821 Copyright 201 3,
American College of Rheumatology. Adapted with permission from BMJ Publishing Group Ltd.

(LcSSc), diffuse cutaneous systemic sclerosis (DcSSc), and


nifestations
Cl inica I Ma
systemic sclerosis sine scleroderma (internal organ involve-
ment only). Specific long term complications may be more and Diagnosis
likely in one subtype than another; however, overlap is com The diagnosis of SSc depends on the presence of specific clini
mon. LcSSc, formerly known as the CREST syndrome, is less cal findings and autoantibodies, which are present in 90'l, to
commonly accompanied by fibrosis of internal organs than is 95"1, of patients (Table 30).
DcSSc but is more commonly associated with pulmonary arte-
rial hypertension. Patients with LcSSc have manifestations Cutaneous lnvolvement
that can include calcinosis, Raynaud phenomenon, esopha The skin is the organ most commonly involved in SSc, with the
geal dysmotility, sclerodactyly, and telangiectasia (CREST); all hands being universally affected. In LcSSc, the skin over the
five conditions may not be present. fingers/hands, face, and neck is typically affected. In DcSSc,
Various disorders may present with skin thickening and skin involvement is more extensive and additionallv includes
other manifestations that overlap with SSc findings; these the upper arms, trunk, and lower extremities.
conditions should be considered in the differential diagnosis of The earliest skin manifestations in DcSSc are often dif
SSc (Table 29). These conditions are not typically associated fusely swollen fingers/hands. This will later give way to thick
with Raynaud phenomenon, and the absence of Raynaud phe- ened, bound down, atrophic skin. Skin thickening in LcSSc
nomenon in a patient with skin thickening makes SSc unlikely. may be more subtle than in DcSSc, and the inability to tent the
skin over the fingers (sclerodactyly) may be an important clue.
t(EY port{Ts The skin changes can lead to joint contractures (Figure 39).
. Systemic sclerosis is a multiorgan disease characterized Small mat-like telangiectasias are common in LcSSc and occa
by fibrosis and vasculopathy; the skin is the principal sionally occur in DcSSc. They are found on the hands and face
target, and Raynaud phenomenon is one ofthe earliest and can also be seen on the oral mucosal side of the lips. They
manifestations. blanch with light pressure. Poikiloderma can occur in areas of
. Systemic sclerosis is divided into three subtypes based fibrosis and consists of areas of hyperpigmentation mixed
on the extent of skin involvement: limited cutaneous with hypopigmentation that give the skin a salt-and pepper
systemic sclerosis, diffuse cutaneous systemic sclerosis, appearance. Facial skin involvement can lead to limitation of
and systemic sclerosis sine scleroderma. the oral aperture and difficulty eating. In addition, the face is
o typically devoid of wrinkles.
The absence of Raynaud phenomenon in a patient with
Calcinosis occurs in approximately 25% of patients with
skin thickening makes systemic sclerosis unlikely.
SSc, typically in the hands, forearms, elbows, gluteal region,

50
l
L

Systemic Sclerosis
I

; TABLE 29. Common Manifestations/Features of Scleroderma Spectrum Disorders


Disorder Ma nifestation/Feature Comments
Systemic Sclerosis
t Diffuse cutaneous Distal and proximal skin thickening (face, chest, Skin involvement is extensive and is commonly
systemic sclerosis abdomen; proximalto elbows and knees); commonly accompanied by internal organ fibrosis and ILD
l (DcSSc) has visceral organ involvement
Limited cutaneous Distal (face, neck, hands, feet), but not proximal, skin More likely to develop PAH and Raynaud
systemic sclerosis thickening; typically not accompanied by internal phenomenon early in the disease and frequently
(LcSSc) organ fibrosis develop calcinosis cutis, telangiectasia, and
esophageal dysmotility
i, Systemic sclerosis Fibrosing organ involvement without skin thickening Difficult to diagnose; prognosis may be similar to
sine scleroderma LcSSc
t

Localized Scleroderma
Morphea Focal plaques of skin thickening, generally on the Systemic manifestations or Raynaud phenomenon is
trunk extremely rare
Linear scleroderma Streaks/lines of thickened skin Same as morphea

Scleroderma-like Conditions"
Eosinophilic fasciitis Orange peel induration (peau d'orange) of proximal Full-thickness skin biopsy demonstrates lymphocytes,
extremities with sparing o{ hands and face; peripheral plasma cells, and eosinophils infiltrating the deep
eosinophilia; skin retraction overthe superficial veins fascia; glucocorticoids are mainstay of treatment
may be more apparent with elevation of an affected
limb
Nephrogenic Secondary to gadolinium in patients with kidney Skeletal muscle fibrosis with contractures and/or
systemic fibrosis disease; brawny, wood-like induration of extremities, cardiac muscle involvement can occur, with
sparing the digits cardiomyopathy and increased mortality; changes in
use and formulation of gadolinium have reduced
incidence
Scleredema lndurated plaques/patches on back, shoulder girdle, Typically seen in long-standing diabetes mellitus
and neck
Scleromyxedema Waxy, yellow-red papules overthickened skin of face, Associated with paraproteinemia (lgG)") and may
upper trunk, neck, and arms; deposition of mucin with therefore occur in the setting of multiple myeloma or
large numbers of stellate fibroblasts in the dermis AL amyloidosis; more frequent in men

Chronic graft-versus- Lichen planus-like skin lesions, or localized or Occurs most commonly after hematopoietic stem cell
host disease generalized skin thickening transplantation; may occasionally be seen after blood
transfusion in immunocompromised host
Drug and toxin Can produce scleroderma-like tissue changes Examples: bleomycin, docetaxel, pentazocine,
exposure L-tryptophan, organic solvents

ILD = interstitial lung disease; PAH = pulmonary arterial hypertension.


oScieroderma-like skin changes may also be a manifestation of systemic endocrine, kidney, or infiltrative disorders.

TABLE 30. Autoantibodies and Their Associations in Systemic Sclerosis


Autoantibody MainClinicalAssociations Comments
Antinuclear antibodies DcSSc; LcSSc Overall prevalence in SSc, 70oZ; not associated with specific
manifestations
Anticentromere ( kinetochore proteins) LcSSc with or without PAH Overall prevalence in SSc, <30%; highly associated (>90%)
with LcSSc
Anti-Scl-70 (DNA topoisomerase-1 ) DcSSc; ILD Overall prevalence in SSc, <30%; highly associated with
DcSSc

Anti-RNA polymerase lll DcSSc; scleroderma renal crisis Useful in DcSSc with negativity for anti-Scl-70

Anti-U3-RNP (fibrillarin) DcSSc; PAH; myositis Associated with poor outcome; more common in Black men

Anti-PM-Scl Myositis Associated with overlap syndrome and polymyositis

Anti-Ku Myositis Rare

Anti-Th/To LcSSc; PAH Ra re

DcSSc = diffuse cutaneous systemic sclerosis; ILD - interstitial lung disease; LcSSc = limited cutaneous systemic sclerosis; PAH = pulmonary arterial hypertension;
RNP = ribonucleoprotein; SSc = systemic sclerosis.

61
Systemic Sclerosis

l'2, and 5'X, of patients have a rheumatoid arthritis-SSc over


lap. with positivity for anti cyclic citrullinated antibodies and
classic rheumatoid arthritis manifestations along with those
of SSc.
Patients with SSc may develop acro osteolysis. or
resorption of the terminal bony tuft of the fingers and,
less commonly. the toes; the prevalence is as high as 20'7,
to 4O"/, in more severe disease (Figure 41). This manifesta-
tion is typically seen in patients with significant distal
vascular involvement, including ulceration. ischemia, and
calcinosis.
Tendon rubs can be felt or heard with a stethoscope
because fibrosis affects tendons or tendon sheaths. These
occur in about 10'7, ol patients with DcSSc.
SSc-associated myositis occurs in 10'1, to 15% of patients.
t IGUR E 3 9. Hands of a 35-year-old woman with diffuse cutaneous systemic Myalgia and proximal muscle weakness are common symp
sclerosis. Note the shortening of fingers and tight atrophic appearance of the skin toms. Serum creatine kinase and/or serum aldolase levels are
This patient is unable to make a full fist.
elevated, and electromyogram demonstrates myopathic
changes. There is a strong association between SSc-associated
and iliac crest (Figure 4O). These deposits can be seen or felt
myositis and myocardial involvement. The presence of anti
and are easily detected on radiographs.
PM Scl antibodies identifies patients with an overlap between
polymyositis and SSc. These patients usually have less skin and
Musculoskeletal lnvolvement gastrointestinal involvement but have more calcinosis and
Joint involvement occurs in 12'2, to 65'7, of patients with SSc; it lung disease.
is mainly polyarticular and can be erosive, with hand/wrist
predominance. SSc is one of the few forms of inflammatory
Vascular lnvolvement
arthritis that affects the distal interphalangealjoints. Between
Raynaud phenomenon occurs in approximately 95'2, of
patients with SSc; it is the most common early manifestation
of SSc, typically occurring years before recognizable SSc.
Raynaud phenomenon is initially episodic, but with'uascular
fibrosis, the blood supply becomes permanently restricted.
SSc will develop in approximately 80'7, of patients with the
combination of Raynaud phenomenon, an SSc-associated
autoantibody, and nailfold capillary changes in an SSc pattern
(Figure 42).

FI G U R E 4 0. Calcinosis seen in limited cutaneous systemic sclerosis.Ihis F IGUR E 4 1 . Posteroanterior radiograph of the hands in a patient with systemic
patient has deposits of calcium in the subcutaneous tissues around the elbow. sclerosis and acro-osteolysis. Note the destruction of the distal phalanges, particularly
Calcinosis often occurs in the hands and forearms but can also affect other locations, of the first and second digits bilaterally, which will eventually result in clinical
such as the trunk or lower extremities.
shortening of the affeoed digits.

62
Systemic Sclerosis

Approximately B0% of patients with SSc have involve


ment of the lower two thirds (smooth muscle portion) of the
esophagus, manifesting primarily as symptoms of dysmotility
and/or gastroesophageal reflux disease. Esophageal involve-
ment is associated with an increased risk for Barrett esopha
gus and adenocarcinoma. A dilated esophagus on chest
imaging is a clue to esophageal involvement.
Gastric symptoms also result mainly from dysmotility.
Patients describe early satiety and may report nausea and
vomiting as a result of delayed gastric emptying. Gastric antral
vascular ectasia is most common in patients with DcSSc and
those with anti RNA polymerase III antibodies. Bleeding ecta
sias can cause significant blood loss.
Small intestinal bacterial overgrowth is common and
can cause malabsorption and diarrhea. The diarrhea is
described as explosive and t1pically follows a meal. Diagnosis
is established by a hydrogen breath test, although an empiric
trial of rotating antibiotics is often used. Small intestine dys
motility can result in pseudo-obstruction. Large intestine
dysmotility may lead to constipation, and vascular ectasia
can occur in the large intestine. Some patients may develop
fecal incontinence.

Kidney lnvolvement
Scleroderma renal crisis affects 5% of patients with DcSSc and
2% of patients with LcSSc. Scleroderma renal crisis was previ-
ously the chiefcause ofdeath in SSc. Risk factors for sclero
derma renal crisis include DcSSc, use of glucocorticoids
(>z.s mg/d), andthe presence of anti-RNA polymerase III anti-
bodies. The more recent lower incidence of scleroderma renal
crisis is ascribed to the avoidance ofglucocorticoids in SSc and
the use ofcalcium channel blockers, which are renal protec-
FIGU R E42. Capil lary loops i n systemic sclerosis. Iop, Early cha nges with loss
of capillary loop density (or drop out) with marked dilatation of the remaining
tive, to treat Raynaud phenomenon.
capillary loops. Bottom, Late changes with extensive loss of capillary loops.

Digital ulcers occur in about 15% of patients with SSc,


typically on the extensor surfaces and tips of the fingers
(Figure 43) and toes but also on the dorsum of hands and feet
and even on the lower extremities. Vascular changes in the
extremities may indicate similar involvement of internal organs.

Cardiac lnvolvement
Myocardial fibrosis is a consequence of microvascular disease
and a cause of early death. It results from repetitive ischemia-
reperfusion injury of the myocardium. Myocardial involve-
ment may lead to heart failure, intraventricular conduction
block, and arrhlthmias. Diastolic dysfunction is more com
mon than systolic dysfunction, and a right bundle branch
block is a predictor of early mortality in SSc.

Gastrointestina I I nvolvement
Symptoms of esophageal dysmotility and reflux are common
in all forms of
SSc and may offer an early clue to the diagnosis t IGUR E4 3. Digital pitting (soft'tissue defects and scarring in the pulp space
when associated with other symptoms. 0f the di$al phalanges) in a patient with systemic sclerosis.

63
Systemic Sclerosis

Scleroderma renal crisis occurs within 4 years of SSc f,tY PotxTs


onset in 75'2, of cases. Patients characteristically present with . Raynaud phenomenon is the most common early mani-
manifestations of hypertensive emergency, including head-
festation of systemic sclerosis.
ache, encephalopathy, seizures, and hypertensive retinopathy.
Rarely, a normotensive form of scleroderma renal crisis can
. The earliest skin manifestations of systemic sclerosis are
often diffusely swollen fingers/hands; with time, the skin
occur. Risk factors for a poor prognosis include male sex, older
becomes thickened, atrophic, and immobile.
age, and heart failure. Changes in blood pressure from ambu-
latory baseline measurements may alert physicians to an . Symptoms of esophageal dysmotility and reflux are
impending renal crisis. common in all forms of systemic sclerosis and may offer
Laboratory fi ndings include microangiopathic hemoly'tic an early clue to the diagnosis when associated with
anemia, thrombocytopenia, and proteinuria. The serum cre- other symptoms.
atinine level is typically elevated and may remain so for some . Patients with scleroderrna renal crisis characteristically
time aflter blood pressure is controlled. Abnormalities in the present with manifestations of h)?ertensive emergency.
renin angiotensin-aldosterone system and in endothelin 1 o Lung involvement in systemic sclerosis includes pul
are thought to contribute to the pathophysiologz. Although
monary arterial hypertension and interstitial lung dis-
ACE inhibitors have dramatically improved the outcome of
ease (lLD); ILD is the main cause of disease-associated
patients with scleroderma renal crisis, prophylactic use of
mortality.
ACE inhibitors may actually worsen scleroderma renal crisis
outcomes and is not recommended even in high-risk patients.

Lung lnvolvement Management


Lung involvement in SSc includes pleuritis and pleural effu Treatment options for systemic sclerosis are mainly sympto
sions, interstitial lung disease (lLD), pulmonary arterial matic and organ system specific. Table 31 outlines treatment
hypertension (PAH), bronchiolitis, pulmonary veno occlusive options for manifestations of SSc.
disease, respiratory muscle weakness, and skin involvement of ACE inhibitors (typically captopril) can be lifesaving in
the trunk that restricts chest wall movement. Patients are also patients with scleroderma renal crisis and should be titrated to
at increased risk for lung cancer. control blood pressure. Dialysis may be needed temporarily for
Significant ILD occurs in 50'7, of patients with DcSSc (857, scleroderma renal crisis. ACE inhibitor therapy should be con
of patients with anti-Scl-70 antibodies) and 35% of patients tinued in scleroderma renal crisis even in the presence of a
with LcSSc and is the main cause of disease associated death. rising serum creatinine level and the need for dialysis because
In patients with SSc, men are more likely than women to late improvement may occur. The addition of plasma exchange
develop ILD, and Black persons tend to have more severe to ACE inhibitors in patients with scleroderma renal crisis and
disease than other ethnic groups. The most common high microangiopathic features has been suggested; angiotensin
resolution CT pattern is nonspecific interstitial pneumonitis receptor blockers should be considered only when ACE inhibi-
followed by usual interstitial pneumonitis. An abnormal FVC tors are not tolerated.
early in the disease course (within the first 5 years) is highly The Scleroderma Lung Study ll compared cyclophospha-
predictive of the development of more severe disease, as is mide for 1 year versus mycophenolate mofetil over 2 years for
fibrosis of more than 20% of lung volume on baseline high- ILD in SSc. With both agents, FVC improved in approximately
resolution CT. There is a strong association between ILD and 75"/,, to BO"/,, and FVC worsened in 2O"1, to 25% of patients. In
gastroesophageal reflux disease, but whether the latter contrib 7o"/,, of patients in each group (especially those with DcSSc).
utes to ILD (through aspiration) is uncertain. All patients should skin scores improved significantly. Mycophenolate was better
undergo pulmonary function testing and high resolution CT at tolerated and has become a mainstay of therapy in these
the time of initial SSc diagnosis, and pulmonary function test patients.
ing with Dlco should be repeated every 6 to 12 months for Nintedanib (a tyrosine kinase inhibitor) significantly
5 years. A 10% decline in FVC or l5'1, decline in DLCo within slowed the decline in lung function among patients with
12 months should raise concern for progression. ILD associated with SSc but did not affect other disease
PAH has a prevalence of 10'7, in SSc. Patients with PAH manifestations. The combination of nintedanib plus
present with exertional dyspnea. With more advanced disease, mycophenolate resulted in the slowest rate of decline in FVC
they may have chest pain and lower extremity edema. Patients per year.
with SSc should undergo echocardiography annually. and for Rituximab is a promising agent in SSc. In a controlled trial
new or concerning symptoms. Pulmonary function testing comparing rituximab versus cyclophosphamide for ILD asso
can provide a clue to underlying PAH; an FVC/Dr-co ratio of 1.6 ciated with SSc, rituximab improved FVC and skin score to a
or greater suggests the diagnosis. See MKSAP 19 Pulmonary greater degree than did cyclophosphamide. Autologous hemat-
and Critical Care Medicine for further discussion of diagnosis opoietic stem cell transplantation has been studied for several
and treatment. years in patients with rapidly progressive, severe SSc; one third

64
Systemic Sclerosis

TABLE 31. Treatment Options for Common Manifestations of Systemic Sclerosis


Manifestations Treatment
Cutaneous manifestations Methotrexate, mycophenolate, cyclophosphamide, and rituximab have been shown to improve skin

For pruritus, antihistamines can be tried; low-dose prednisone can also help but doses >7.5 mg/d
should be used with caution because of risk for scleroderma renal crisis.
Laser therapy can be used for telangiectasias.
Musculoskeletal NSAlDs, intra-articular or low-dose prednisone, hydroxychloroquine, or methotrexate can be used {or
ma nifestations arth ra g iala rth ritis.
I

Methotrexate, azathioprine, or mycophenolate can be used for significant myositis.


Raynaud phenomenon Cold avoidance; gloves; central warmth
Calcium channel blockers; sildenafil; losartan; prazosin
Topical nitrates; low-dose aspirin
Digital sympathectomy for refractory cases
Gastroesophageal refl ux Avoid eating close to bedtime; head-of-bed elevation
disease
H2 blockers; proton pump inhibitors
Surgical fundoplication
Gastrointesti na I dysmoti ity
I Metoclopramide (avoid long-term use); domperidone (investigational limited-access program),
erythromycin; cisapride (investigational limited-access program)
Gastric antral vascular ectasia Ablation laser therapy
Small intestine bacterial Rotating antibiotics; if fat malabsorption, cholestyramine
overgrowth
Pseud o-obstruction Promotility agent such as domperidone can be tried; if resistant, octreotide may be useful.
Constipation Fiber; stool softeners; polyethylene glycol
Scleroderma renal crisis ACE inhibitors (typically captopril), plasma exchange
Dialysis

lnterstitial lung disease Mycophenolate mofetil; cyclophosphamide; nintedanib; rituximab; stem cell transplant in rapidly
progressive cases
Lung transplantation
Pulmonary arterial Supplemental oxygen; treat right-sided heart failure
hypertension
Calcium channel blockers (if positive response to vasoreactivity testing); prostanoids; endothelin
receptor antagonists; phosphodiesterase-5 inhibitors; soluble guanylate cyclase stimulator
Combination therapy (ambrisentan and tadalafil); lung transplantation

to one half of patients had significant improvement in lung weight. A meta-analysis of pregnancies in patients with SSc
function, skin thickening, and antibody status. However, suggested that although disease manifestations, including
cost and potential toxicity limit the utility of stem cell digital ulcers, may improve, 14'X, of patients may develop new
transplantation. manifestations or worsening of current disease during preg-
nancy; in another 10'/., disease will worsen in the first
r( Ev P0 t 1{Ts
6 months postpartum. Contraindications to pregnancy in
. Treatment options for systemic sclerosis are mainly patients with SSc include PAH and severe restrictive lung
symptomatic and organ system specific. disease (FVC <1 L).
r ACE inhibitors can be lifesaving in patients with sclero-
derma renal crisis and should be titrated to control t(EY P0ilaTS
blood pressure even in the presence of rising serum
. Pregnant patients with systemic sclerosis may experience
creatinine.
hypertensive disease, preeclampsia, preterm delivery
and low birth weight.

Pregnancy o Contraindications to pregnancy in patients with systemic


sclerosis include pulmonary arterial hypertension and
Pregnant patients with SSc may experience hypertensive
severe restrictive lung disease.
disease, preeclampsia, preterm delivery and low birth

55
\

Mixed Connective Tissue Disease and Undifferentiated Connective Tissue Disease

Mixed Connestive nical Manifestations


Cli
Tissue Disease and and Diagnosis
See Table 32 for details on the clinical manifestations and
Undifferentiated diagnosis of MCTD. Raynaud phenomenon and pulmonary
involvement are common features; pufff hands and inflam
Connective Tissue Disease matory arthritis are also seen. Pulmonary arterial hyperten-
Overview sion is the major cause of death; severe kidney and neurologic
diseases are uncommon. Although antinuclear antibodies,
Most patients with rheumatologic disease have well estab
rheumatoid factor, and anti-cyclic citrullinated peptide anti
lished and well classified diagnoses. However, given the com
bodies are often present, the presence of anti double
mon underlying processes of autoimmunity and inflamma-
stranded DNA, anti-Smith, or anti-Ro/SSA antibodies makes
tion, it is not surprising that some patients have features of
primary SLE a more likely diagnosis.
more than one rheumatologic disease simultaneously or
sequentially (overlap syndromes). Other patients present with
signs and symptoms that are autoimmune in nature (often
including Raynaud phenomenon) but do not rise to the level of
Management
a specific rheumatologic condition; such patients have undif- Treatment for MCTD is directed at the specific organ system
ferentiated connective tissue disease (UCTD). Mixed connec involved in the individual patient and is guided by the man
tive tissue disease (MCTD) is a specific variant of overlap syn- agement of these manifestations as would occur in the "par-
drome that includes clinical manifestations meeting a dual or ent" condition (see Table 32).
triple diagnosis of systemic lupus erythematosus (SLE), auto-
immune myositis, and/or systemic sclerosis. MCTD is associ-
ated with positivity for anti Ul-ribonucleoprotein (RNP) Prognosis
antibodies. Prognosis of MCTD depends on the spectrum and severity of
See Table 32 for a comparison of MCTD and UCTD. organ involvement. Patients with initially mild disease may
develop nelv and more severe manifestations over decades,
including interstitial lung disease and pulmonary arterial
Epidemiology and hypertension. As in patients with other systemic inflamma-
Pathophysiology tory disorders, patients with MCTD are at increased risk for
MCTD is rare, and B0% of patients are women. B-cell abnor atherosclerotic cardiovascular disease.
malities and an association with anti-U1-RNP antibody have UCTD usually remains mild and stable but in a minority
been described. The antibody binds to the 70 kD epitope of of patients may evolve into a more specific autoimmune
the RNP complex, but its actual role in pathogenesis is not disorder, most commonly SLE. Most patients with UCTD
understood. Genetic risk alleles include HLA DRBI-04:01 who develop a more specific disease do so within the first 2
and HLA-B-O8, underlining the autoimmune nature of the to 5 years after presentation. A definite disease is more likely
condition. to develop in patients with cytopenias; antibodies to

TAALE 32. Comparison of Mixed Connective Tissue Disease and Undifferentiated Connective Tissue Disease
Condition Typical Clinical Features Diagnosis Treatment
Mixed Raynaud phenomenon (bi- or Positivity for anti- Treatment determined by the management of
connective tricolor); puffy fingers; inflammatory U1-RNP antibodies, the specific disease diagnoses and organs
tissue disease a rthritis; sclerodactyly; serositis; 7O-kD protein involved; may include anti-inflammatory
esophageal dysmotility; myositis; immunomodulatory and biologic agents
Fulfills diagnostic
interstitial lung disease; PAH
criteria for at least two Vasodilators for PAH and Raynaud phenomenon
of the following: SSc,
PPI for esophageal disease
autoimmune myositis,
SLE

Undifferentiated Variable; most common include Does not meet Same as for mixed connective tissue disease
connective Raynaud phenomenon (bi- or diagnostic criteria for
tissue disease tricolor), particularly with abnormal any specific connective
nailfold capillaroscopy, persistent tissue disease; usually
inf la m matory a rth ritis/a rth ra g iaI positivity for ANA, with
unexplained by other diagnoses, titer >1 :80
skin rash, cytopenias, and serositis

PAH = pulmonary arterial hypertension; PPI = proton pump inhibitor; RNP = ribonucleoprotein; SLE = systemic lupus erythematosus; SSc = systemic sclerosis.

65
Crystal Arthropathies

extractable nuclear antigens; anticentromere, anti-Ro/SSA, Pathophysiology


or antiphospholipid antibodies; and/or abnormal nailfold Uric acid is the end product of purine metabolism, and about
capillaries.
two thirds ofurate production derives from cellular turnover of
rTY POIT{15 nucleic acids; the remainder is derived from dietary intake. A
o ]\rtixed connective tissue small proportion ofpatients overproduce urate on an inherited
disease includes clinical
manifestations of systemic lupus erythematosus, metabolic basis. Unlike most mammals, humans do not express

autoimmune myositis, and/or systemic sclerosis; uricase, an enzyme that converts urate into highly soluble
anti-Ul-ribonucleoprotein antibodies are gzpically allantoin. Urate therefore accumulates and can precipitate as
present. crystals in joints and other tissues if serum concentrations
exceed the saturation point. At normal body temperature and
o Treatment for mixed connective tissue disease is pH, the saturation point of urate is 6.8 mg/dl (0.40 mmol/L).
directed at the specific disease states and organ
At lower temperatures and more acidic environments (e.g., in
systems involved.
the distal extremities), the saturation concentration is lower.
r Undifferentiated connective tissue disease does not Almost all serum urate is filtered at the glomerulus, but
meet criteria for a specific rheumatologic condition; approximately 90% is reabsorbed in the proximal tubule; in
most cases are mild, but a minority of patients develop most patients with gout, hereditary underexcretion of uric acid
a more specific autoimmune disorder. is the primary cause of hyperuricemia (Table 33). Not all
patients with hyperuricemia develop gout. The factors that lead
to the development of gout in hyperuricemia are unclear, but

Crystal Arthropathies degree ofelevation in serum urate level is predictive. A serum


urate Ievel greater than 10 mg/dl (0.59 mmol/L) is associated
Gout with a 30'/o likelihood of an initial gout flare within 5 years.
A gout flare (acute gouty arthritis) is triggered when resi
Epidemiology
dent synovial macrophages ingest monosodium urate crystals,
Gout is characterized by intermittent painful flares of inflam
Ieading to activation of the NLRP3 inflammasome and genera-
matory arthritis in response to crystals formed in joints
tion of interleukin (lL)-18. IL 1p causes local vasodilation,
caused by excessive serrrm levels of urate. Gout is increas
triggers production of other inflammatory cytokines, and
ingly common, with a U.S. prevalence of approximately 4'l..
recruits and activates neutrophils. Complement activation
Factors contributing to this increase include dietary changes,
on the surface of urate crystals also promotes neutrophil
increasing obesity, and an aging population. Nearly all
recruitment.
patients with gout experience comorbidities, including
hypertension, coronary artery disease, hyperlipidemia, dia
betes mellitus, and chronic kidney disease (CKD). Some Clinical Manifestations
comorbidities (e.g., CKD) contribute to hyperuricemia; all Three requirements generally must be f'ulfilled for onset of
comorbidities complicate treatment by posing relative con clinically apparent gout: (1) hyperuricemia; (2) monosodium
traindications for appropriate therapeutics (e.g., NSAIDs in urate deposition in joints and/or soft tissues; and (3) a reaction
patients with CKD). to phagocytosed crystals that leads to an acute inflammatory
Men reach a steady state serum urate level after puberty, response.
whereas premenopausal women are generally protected from
hyperuricemia by estrogenic effects. Men can have their flrst Gout Flares
gout flare in the third to fifth decade; women are more likely Ear$ acute gout flares are typically monoarticular, and at least
to experience the first flare after menopause. 50% of first flares in men involve the first metatarsophalangeal

TA,8LE 33. Causos of Hyperuricemia


Primary renal uric acid underexcretion (hereditary, renaltubular basis)
Chronic kidney disease of any cause (secondary uric acid underexcretion)
Uric acid overproduction due to primary defea in purine metabolism: PRPP synthetase overactivity; HPRT deficiency
Conditions of increased cellturnover leading to purine/urate generation: leukemia/lymphoma/other myeloproliferative disorders;
psoriasis; hemolytic a nemia; polycythemia vera
Drug-induced hyperuricemia (agents reducing renal glomerularfiltration and/ortubular urate excretion): thiazide and loop diuretics;
cyclosporine; tacrolimus; low-dose salicylates; ethambutol; pyrazinamide; lead ingestion/toxicity; alcohol; nicotinic acid
Diet-induced hyperuricemia (agents high in purines or inducing purine/urate biosynthesis): alcohol; shellfish; red meat; high-fructose
corn syrup-sweetened beverages and foods; impact on serum urate generally is limited in the absence of other causes

HPRT = hypoxanthine-guanine phosphoribosyltransferase; PRPP = phosphoribosylpyrophosphate

67
Crystal Arthropathies

Chronic Aspects of Gout


If gout is not treated relatively early in its course, gout flares
tend to become increasingly frequent and severe and are often
polyarticular. These flares may eventually evolve into persis
tent inflammatory arthritis (chronic gouty arthritis), with
intermittent flares superimposed on the baseline persistent
disease.
Some patients develop tophi. Tophi are solid chalky white
masses of uric acid, surrounded by inflammatory cells and a

rind of flbrous tissue. They are located around joints and in


soft tissues, with a predilection for the extensor surfaces ofthe
elbows, distal Achilles tendon, fingers (usually the proximal
and distal interphalangeal joints) (Figure 45), and helices of
the ears. Tophi are deforming, can interfere with function, and
directly erode bone. Ulceration of overlying skin can occur,
and accompanying infection can be difficult to treat because
tophi are avascular.

Diagnosis
A diagnosis ofgout (the chronic disease state) should be con
sidered in any patient who has had or is experiencing one or
more episodes of acute monoarticular or oligoarticular inflam
matory arthritis that may represent a gout flare. Although
designed primarily for research, gout classification criteria are
available, including as an online calculator (hftp://goutclassifi-
cationcalculator.auckland.ac.nz), and are 92'ln sensitive and
89'X, specific.
tIGURE 44. Goutof theleftbigtoe.
The differential diagnosis ofa gout flare is listed in Table 34.
The gold standard for diagnosing a gout flare is identification
(MTP) joint of the great toe (podagra) (Figure aa). Inflammatory of negatively birefringent, needle-like monosodium urate
states, such as infection, surgery and myocardial infarction, crystals, along with neutrophils, in synovial fluid. Crystals
can provoke gout flares. The hallmarks ofa flare are pain, ten- within neutrophils are usually also seen. "Negatively birefrin
derness, swelling, redness, and warmth. Flares typically begin gent" means that under polarized light, the crystals appear
at night and peak within 12 to 24 hours. Untreated, most gout yellow when parallel to and blue when perpendicular to the
flares self resolve within days to a few weeks, although with polarizing axis of an optical filter. Arthrocentesis is not always
long standing disease, flares can persist for months. In men performed when gout flare is suspected, but when infection is
with established disease, flares eventually affect the proximal
feet, ankles, and knees, and later almost any joint, including
the spine. Postmenopausal women may present differently,
with initial flares often involving the knees, or finger joints
affected by osteoarthritis. For patients with more severe or
long-standing disease, polyarticular flares may occur. Soft tis
sues can also be involved, manifesting as acute bursitis (olecra
non and prepatellar, most commonly), periarthritis, and gouty
panniculitis and cellulitis, the last of which can be misdiag-
nosed as a refractory bacterial soft-tissue infection.

Intercritical Gout
lntercritical gout is the period between gout flares. Early in the
disease course, the intercritical period can be years. As the
disease progresses, the intercritical period can progressively
shorten. Even in the absence of symptoms, urate crystals still t IGUR E4 5. Fingers of a patient with numerous bulky tophaceous deposits of
reside within joints and soft tissues, and low-grade systemic monosodium uric acid crystals, a consequence of years of gout with uncontrolled
inflammation persists. hyperu ricem ia.

68
Crystal Arthropathies

TABLE 34. Differential Diagnosis of Acute Gout Flare


Condition Comments
lnfeclious arthritis Presentation may be identicalto gout. lnfectious arthritis is usually monoarticular but can be
polyarticular. Onset may be less acute than gout. Gout and infectious arthritis can coexist.
Acute calcium pyrophosphate crystal Pseudogout is less likely to present in the great toe, but acute presentations may otherwise
arthritis (pseudogout) be identicalto gout. Synovial fluid analysis can distinguish these entities. Gout and
pseudogout can coexist.
Basic calcium phosphate deposition Basic calcium phosphate deposition occurs in articular cartilage and periarticular tissues.
Because of their small size, basic calcium phosphate crystals are unlikely to be seen in
synovialfluid on light microscopy except as aggregates stained with alizarin red.
Trauma Trauma can lead to local pain and swelling, with or without a fracture, and can also trigger
a gout flare.

Other forms o{ inflammatory arthritis (e.g., Clinical context, affeaed joints, and pattern of arthritis help distinguish these entities.
reactive arthritis, rheumatoid arthritis, Synovial fluid analysis is particularly important when there is diagnostic uncertainty.
psoriatic arthritis, acute rheumatic fever)

a concern, joint aspiration for Gram stain and culture is Management


mandatory. See Principles of Therapeutics for details on the medications
In any acute arthritis in an adult, gout flare should be described in this section.
considered. Findings that support a gout flare diagnosis when
arthrocentesis is not feasible include onset of severe pain and Treatment of Gout Flares
swelling over several hours, involvement of the first MTP joint Treatment of acute gout flares focuses on anti-inflammatory
or midfoot/ankle, presence of erythema, and previous similar therapy; colchicine, NSAIDs, and glucocorticoids are all rea-
acute arthritis episodes. Male sex and associated cardiovascu- sonable options. Treatment choice should be determined by
lar disease are also supportive. The involved joints are warm, potential drug interactions and patient comorbidities. The
red, swollen, and very tender. Low grade fever may be pre- simplest is colchicine, 1.2 mg at the first symptoms of a gout
sent, particularly during polyarticular flares. Serum urate flare, followed t hour later by a 0.6-mg dose. Colchicine is
measured at time of flare may not be helpful because levels most effective when used less than 24 hours after symptom
can drop during acute systemic inflammation. Therefore,
although an elevated serum urate level supports the possibil-
ity of gout, hyperuricemia alone does not establish the diag-
nosis, and lack of hyperuricemia at time of an acute arthritis
does not definitively rule out gout. A serum urate level
2 weeks after gout flare resolution more accurately measures
baseline level. C reactive protein and erythrocyte sedimenta-
tion rate are elevated during gout flare but are nonspecific.
Joint fluid has an inflammatory leukoclte count (>2000ipl
[2.0 x 10e/L] and occasionally >100,000/pL [100 x 10ei L], with
neutrophil predominance). The presence of intracellular crys-
tals and leukocytes in synovial fluid does not rule out a con-
comitant infection.
Imaging is recommended when arthrocentesis is not pos-
sible and a clinical diagnosis of gout is uncertain. The charac-
teristic radiographic changes of established disease may help
confirm a history ofpreviously unrecognized gout and support
a diagnosis ofcurrent gout flare (Figure 46). Musculoskeletal
ultrasonography and dual energr CT can demonstrate mono-
sodium urate deposition in joints and are being assessed for
their role in diagnosis. Ultrasonography can show unsus-
pected tophi or a "double-contour" sign at cartilage surfaces
(Figure 47), which is highly specific for urate deposits in joints.
The double contour sign, identification of monosodium urate
F I G U R E 4 6, Periarticular bony erosions, appearing as punched'out bone
crystal deposition on dual-energr CT, and gout-related joint lesions with disruption of bone cortex (overhanging edges), represent areas of
damage on radiography are included in gout classification tophaceous monosodium uric acid deposition and accompanying inflammation in
criteria. a patient with severe, long-standing, inadequately treated gout.

69
!

Crystal Arthropathies
I
:
't
more subcutaneous tophi; evidence of radiographic damage I
I
Soft tissue (any modality) attributable to gout; or frequent gout flares, .\

with "frequent" being defined as two or more annually. For


patients who are experiencing their first flare and have CKD
Cartilage Double-contour sign
(urate on cartilage) stage 3 or greater, a serum urate level exceeding 9.0 mg/dl
(0.53 mmol/L), or urolithiasis, the ACR conditionally recom-
mends initiating urate-lowering therapy. These guidelines
strongly recommend that patients taking urate-lowering ther-
apy continue it to achieve and maintain a serum urate target
less than 6.0 mg/dl (0.3s mmol/L).
Current evidence supports initiating urate-lowering ther-
Bone-cartilage apy during an acute flare, but only if adequate anti-inflamma-
inter{ace tory therapy is concurrently started; doing so may improve
Subchondral bone
Iong-term adherence. Administering urate lowering therapy
during an acute flare, in the absence of anti-inflammatory
FIGU R E4 7. Ultrasound of the tibial side of the knee showing monosodium therapy, may extend the duration ofthe attack. Ifthe patient is
urate deposition on the cartilage surface of a patient with gout. Ihe light already receiving urate-lowering therapy, it should not be
heterogeneous area at the top represents soft tissue; the dark area at the bottom is
stopped during an acute flare.
su bchond ral bone. The bottom brig ht wh it e line (green anow) represents the bone/
Three classes of urate-lowering therapy are available:
cartilage interface, and the clear zone aboueit(yellow arrow) is the cartilage. Above
the cartilage, and between the ca(ilage and the soft tissue, a second bright white
xanthine oxidase inhibitors (which reduce urate production),
line may be seen (red arrow\,creatinga double-contour sign, which indicates the uricosuric agents (which decrease renal urate resorption), and
presence of urale and is found in many patients with gout. pegloticase (a uricase).

onset. High-dose NSAIDs for 5 to 7 days are effective, as are Xanthine Oxidose I nhibitors
glucocorticoids in any form: intra articular injection, intra Allopurinol is the recommended flrst-line therapy for most
muscular injection (e.g., depo-methylprednisolone, 40 80 mg), patients and is FDA approved for dosages up to 800 mg/d.
or an oral "burst" of prednisone (e.g., 0.5 mg/kg/d for According to the ACR, allopurinol should be initiated at 100 mg/d
5 days). Topical ice helps reduce duration of the flare and may and the dosage titrated in 100-mg increments every several weeks
be considered as adjunctive therapy. until a serum urate level less than 6.0 mg/dl (0.35 mmol/L) is
Most gout flares respond to therapy lasting a week or less, achieved (or lower as needed to control flares or resolve tophi).
although more severe flares may require longer treatment. For For patients with stage 4 or 5 CKD, allopurinol should be initiated
patients with severe and refractory flares, or with contraindi- at 50 mg/d and the dosage titrated in 50 to 100 mg increments
cations to other treatments, off label use of IL-lp inhibitors until serum urate target or dosage of 800 mg/d is achieved.
(anakinra or canakinumab) can be considered. Gouty cellulitis An uncommon but serious complication of allopurinol is
should be treated as any other acute gout flare. a hypersensitivity reaction heralded by a rash that may pro
gress to DRESS (drug reaction with eosinophilia and systemic
Urate-Lowering Therapy symptoms) syndrome (DRESS is also increasingly being
Guidelines conflict as to whether patients with gout should referred to as drug inducedhypersensitiuiru sundrome [DIHS]
receive dietary counseling to reduce serum urate levels. Ifrec- to emphasize the fact that eosinophilia is not always present).
ommended, patients may limit intake of shellfish, oily fish, red The drug should therelore be discontinued in patients who
meat, high-fructose foods, and alcohol. However, overly strict develop a rash. Patients with the HLA-B.58:01 allele have a
diets that are unlikely to be adhered to are not helpful. Weight much higher incidence of this syndrome. The 2020 ACR gout
loss (when appropriate) and increasing dairy intake may also guidelines therefore conditionally recommend genetic testing
lower serum urate levels. In patients with hypertension, agents for HLA-B.58:01 before initiation of allopurinol therapy in all
other than diuretics (which block renal urate excretion) should patients of southeast Asian (Han Chinese, Thai, and Korean)
be considered; losartan may be a good alternative because it and African descent, given the high prevalence (+% to'2,) of
has uricosuric effects. the allele in those populations. Patients who are HLA-B'58:01
The 2020 American College of Rheumatolory (ACR) and positive should usually receive alternative treatment.
2016 European League Against Rheumatism (EULAR) recom Febuxostat is as efficacious as or more efficacious than
mendations support a "treat-to-target" approach, reducing the allopurinol. It is less likely to cause hypersensitivity reactions
serum urate level to less than 6.0 mg/dl (0.35 mmol/L) and, in than allopurinol and does not require dose adjustment in mild
the EULAR guidelines, to less than 5.0 mg/dl (0.30 mmol/L) to moderate CKD. In February 2019, the FDA mandated a
in patients with tophi. boxed warning regarding cardiovascular risk with febuxostat
The ACR strongly recommends initiating urate-lowering compared with allopurinol and has limited the approved use
therapy for patients with gout and any of the following: one or offebuxostat for patients who are unresponsive to or cannot

70
Crystal Arthropathies

tolerate allopurinol. For patients taking febuxostat who have a l(tY P0 I tlTS (contlnucd)
history of cardiovascular disease or a new cardiovascular o Colchicine, NSAIDs, and glucocorticoids are options for
event, ACR guidelines conditionally recommend switching to
treatment of acute gout.
alternative urate lowering therapy if available.
o The American College of Rheumatolory strongly recom
mends that patients taking urate lowering therapy con
Uricosuric Agents
tinue doing so to achieve and maintain a serum urate
The uricosuric agent probenecid is infrequently used as a sin
target less than 6.0 mg/dl (0.35 mmol/L).
gle agent because it is less effective than xanthine oxidase
inhibitors; it should be avoided in patients with CKD (esti o Patients starting urate-lowering therapy should receive
mated glomerular filtration rate <50 mLlmirrl7.73 m2) or anti inflammatory prophylaxis to prevent flares.
nephrolithiasis. Combination therapy with a xanthine oxidase
inhibitor and probenecid (or the incidentally uricosuric agent
losartan) may be more effective than a xanthine oxidase inhib- Calcium Pyrophosphate
itor alone. A more effective uricosuric agent, lesinurad, is
available in Europe. Reversible kidney insufficiency may
Deposition
develop in some palienls. Deposition of calcium pyrophosphate (CPP) crystals in cafti
lage can provoke an acute inflammatory arlhritis that clini
cally resembles a gout flare. CPP deposition (CPPD) disease is
Pegloticase
less well characterized than gout, but four subgroups are
Pegloticase is an option for patients with severe recurrent and/
described: (1) asymptomatic CPPD, (2) acute CPP crystal
or tophaceous gout who are intolerant ofor resistant to stand-
arthritis, (3) chronic CPP crystal inflammatory arthritis, and
ard therapies. Infused every 2 weeks, pegloticase lowers semm
(4) osteoarthritis with CPPD.
urate to nearly zero. However, 30% to 50% ofpatients develop
antibodies to the drug within weeks, rendering it ineffective
and increasing the likelihood of infusion reactions, including Epidemiology and Pathophysiology
anaphylaxis. Pegloticase should be discontinued in any patient CPPD primarily affects older patients, and prior joint trauma
for whom it ceases to work because this suggests drug anti is a significant risk factor. The risk for cartilage calcification
body formation. Serum urate should be checked before each (chondrocalcinosis) doubles for every decade past 60 years,
infusion, and pegloticase should be discontinued if the serum and nearly halfofpatients in their late BOs have CPP visible on
urate level exceeds 6.0 mg/dl (0.35 mmol/L) on two consecu plain radiographs. For younger patients with CPPD, laboratory
tive assessments. evaluation for contributory metabolic disease (hyperparathy
roidism, hemochromatosis, hypophosphatasia, hypomagne
Prophylaxis semia) is warranted.
Whenever urate lowering therapy is initiated for gout, mobili- The pathophysiologz of CPPD is incompletely under
zation of monosodium urate crystals from joints and soft stood. Pyrophosphate produced by chondrocytes likely pre
tissues can provoke flares. Accordingly, patients starting urate cipitates with calcium to form CPP crystals, which then
lowering therapy during intercritical periods should be pre- deposit and can activate inflammatory pathways, resulting
scribed anti inflammatory prophyl&xis to prevent flares. The in an acute arthritic flare. CPPD may also drive osteoarthritis
ACR strongly recommends continuing prophylaxis for 3 to by causing mechanical damage and inducing proinflamma
6 months, with ongoing evaluation and continued prophylaxis tory activity in chondrocytes and synovial fibroblasts.
as needed ifflares continue. I\4ost patients tolerate colchicine, Epidemiologic evidence suggests that cartilage calcification
0.6 mg once or twice daily; for those who do not, low-dose might play a role in osteoarthritis of the knees and wrists, but
NSAIDs or glucocorticoids are appropriate substitutes. Most not at the hip.
patients can use the same drug used for the acute flare, at a
lower dose lor prophylaxis.
Clinical Manifestations and Diagnosis
KEY POIilTS Asymptomatic Calcium Pyrophosphate Deposition
. The gold standard for diagnosis of gout flare is identifi- Asymptomatic CPPD occurs with radiographic changes in
cation of negatively birefringent, needle-like intracellular the absence of clinical symptoms. Cartilage calcification
and extracellular monosodium urate crystals within appears as a Iinear opacity below the surface of articular
synovial fluid. cartilage (Figure 48). It most commonly occurs in the knees,
. wrists (triangular fibrocartilage), pelvis (symphysis pubis),
The diagnosis ofgout flare, even when confirmed by
and metacarpophalangeal (MCP) joints, in descending order.
intracellular crystals in synovial fluid, does not defini-
Asymptomatic CPPD is common in older patients and in
tively rule out concomitant infection.
osteoarthritic joints, and it may be a precursor of sympto
(Continued)
matic osteoarthritis with CPPD.

71
Crystal Arthropathies

Patients present with acute neck pain, fever, and headache


mimicking meningitis.

Chronic Calcium Pyrophosphate Arthropathy


Chronic calcium pyrophosphate arthropathy may be present
in one of two patterns: (t) chronic CPP crystal inflammatory
arthritis and (2) osteoarthritis with CPPD. Chronic CPP crystal
inflammatory arthritis is a polyarthritis involving the wrists
and MCP joints ("pseudo rheumatoid arthritis"); it is rare and
difficult to treat. Osteoarthritis with CPPD manifests as typical
osteoarthritic findings involving joints not commonly associ
ated with osteoarthritis (such as shoulders or MCP joints);
radiographic CPPD often precedes osteoarthritis onset, sug-
gesting a causal role for CPP
FIGU R E 4 8. Cartilage calcification (chondrocalcinosis) of the knee. This radiograph
shows linearly arranged calcific deposits in the articular cartilage (arow).
Management
In contrast to gout, there is no known mechanism for dissolv
Acute Calcium Pyrophosphate Crystal Arthritis ing articular CPP crystals or preventing their formation.
Acute CPP crystal arthritis (pseudogout) typically presents as a lnstead, treatment aims at abrogating the inflammatory mani
monoarticular or oligoarticular inflammatory arthritis, char- festations of the disease (Table 35). Associated metabolic con-
acterized by sudden onset of swelling, pain, loss of function, ditions should be treated, but this treatment rarely results in
tenderness, and warmth of the alfected joint (usually a knee or remission of the CPPD disease.
wrist). Like acute gout, CPP flares may be provoked by sys
XEY POIIITS
temic insults, such as major surgery or acute illness. Flares are
usually milder than those of gout and less commonly affect the
. Asymptomatic calcium pyrophosphate deposition is
MTP joint, but if untreated they can persist for weeks or characterized by radiographic changes, including carti-
months. Definitive diagnosis requires identification of CPP lage calcification (chondrocalcinosis).

crystals (along with leukocyes) in synovial fluid; in contrast to . The presentation of acute calcium pyrophosphate crys-
urate crystals, CPP crystals are rhomboid-shaped and posi- tal arthritis (pseudogout) resembles gout; however,
tively birefringent (blue when parallel to and yellow when crystals are rhomboid shaped and positively birefrin-
perpendicular to the polarizing axis ofan optical filter). As in gent under polarized light.
gout flares, joint infection may coexist with CPP arthritis, and o Chronic calcium pyrophosphate crystal inflammatory
synovial fluid Gram stain and culture are usually warranted. arthritis often involves the wrists and metacar-
Radiographic evidence of cartilage calcification in older pophalangeal joints ("pseudo-rheumatoid arthritis").
patients with acute monoarticular arthritis suggests, but is not
o Osteoarthritis with calcium pyrophosphate deposition
diagnostic of, acute CPP crystal arthritis.
causes typical osteoarthritic findings involving joints
The crowned dens syndrome is a rare presentation of CPP
not commonly associated with osteoarthritis (shoulders,
crystal arthritis involving the C2 vertebra. Radiographically it
wrists, metacarpophalangeal joints).
may present as calcification of the cartilage over the dens.

TABLE 35. Management of Calcium Pyrophosphate Deposition


Clinical Presentation TreatmenVComments
Cartilage calcification (chondrocalcinosis) No specific treatment
Acute calcium pyrophosphate crystal Local treatment: joint aspiration, followed by intra-articular glucocorticoid injection, joint
arthritis ( pseudogout) immobilization; ice packs
Systemic treatment: anti-inflammatory doses of NSAIDs; colchicine; glucocorticoids (oral,
intramuscular, or parenteral); if there are contraindications to the preceding drugs,
anakinra may be considered off-label
Prophylaxis if recurrent attacks (three or more annual attacks): low-dose colchicine or daily
NSAI Ds (with gastrointestina I protection)

Chronic calcium pyrophosphate crystal Low-dose colchicine or daily low-dose NSAIDs (with gastrointestinal protection);
inflammatory arthritis low-dose glucocorticoids
Osteoarthritis with calcium pyrophosphate Same treatment as for osteoarthritis without calcium pyrophosphate deposition (e.9.,
deposition physical therapy, pain control, local glucocorticoids)

72
Infectious Arthritis

Basic Calcium Phosphate fluid leukocyte counts are typically greater than 50,000/pL
(50 x 10e/L) and often exceed 100,000/pL (100 x 10e/L), with
Deposition a polymorphonuclear cell predominance. Such high leuko-
Basic calcium phosphate (BCP) deposition occurs in older cyte counts may occur in other conditions (e.g., crystal
persons and targets both articular cartilage and periarticu- arthropathies) but should be presumed infectious until
lar tendons and ligaments. BCP deposition is rarely associ- proven otherwise. Notably, the presence of crystals does not
ated with inflammatory arthritis and periarthritis, most exclude the possibility of concomitant infection. Synovial
classically manifesting in older women as "Milwaukee fluid leukocyte counts may be lower in patients with gono
shoulder," an inflammatory state that causes progressive coccal, mycobacterial, or fungal infections; patients who
destruction of the rotator cuff and glenohumeral joint. inject drugs; or those who are immunocompromised. A Gram
Diagnosis is usually clinical; radiographs show calcium stain positive for bacteria should be considered definitive, but
deposition in the affected tissues. Visualization of BCP crys- the sensitivity of the test is inadequate for a negative result to
tals requires special stains and/or electron microscopy and rule out infection when suspicion is high. Synovial fluid cul-
is rarely performed. tures are usually positive in bacterial infections unless antibi-
TEY ?OITT otics were administered before arthrocentesis. Peripheral
. blood leukocyte count, erythrocyte sedimentation rate, and
Basic calcium phosphate deposition can rarely cause
C-reactive protein level are often elevated; however, normal
"Milwaukee shoulder," a destructive inflammatory
levels do not exclude the diagnosis, and elevated levels may
arthritis and periarthritis ofthe shoulder; diagnosis is
usually clinical.
occur in noninfectious inflammatory arthritis and other
conditions.
Blood cultures should always be drawn before antibiotic
administration. When gonococcal arthritis is suspected, uro-

lnfestious Arthritis genital, rectal, and pharyngeal specimens should also be


obtained for nucleic acid amplification testing.
Diagnosis Plain radiographs are usually normal early in the course
of the infection, but baseline fllms are helpful to identi$/ other
Clinical Manifestations diseases or contiguous osteomyelitis. Radiographs obtained
Infectious arthritis typically presents with pain, swelling, later (from weeks to months) often show nonspecific changes.
warmth, and erythema of the affected joint, accompanied by In advanced or particularly virulent infection, periosteal reac-
fever and constitutional symptoms. Knee joint infection is tion, marginal or central erosions, and subchondral bone
common, but any joint may be affected. Previously damaged destruction may be seen (Figure 49). Bony ankylosis is a late
joints are at increased risk for involvement; acute monoarthri-
tis or new inflammation of a single joint in a patientwithwell-
controlled inflammatory arthritis should prompt evaluation
for infection.
Physical examination commonly shows loss of both active
and passive range of motion in addition to signs of inflamma-
tion. Careful skin examination can reveal signs suggesting
gonococcal infection (pustular skin lesions) or potential
portals of entry for pathogens (scratches, bites, or foreign
bodies).

Laboratory and lmaging Studies


In patients suspected of having infectious arthritis, the most
important diagnostic procedure is expeditious arthrocentesis.
The slmovial fluid should be sent for Gram stain, culture (bac-
terial and, when indicated, mycobacterial and/or fungal),
leukocyte count, and crystal analysis. Synovial fluid levels of
glucose, lactate dehydrogenase, and total protein do not aid in
diagnosis and should not be ordered. Depending on the
clinical picture, the fluid may be sent for polymerase chain
reaction assay to detect mycobacteria or Borrelia burgdorferi
DNA.
Bacteria-infected synovial fluid is cloudy and less viscous FIGURE 49. Radiographshowing infectiousarthritis-related bonedestruction
than that seen in noninflammatory arthritis. Elevated synovial of the hip.

73
lnfectious Arthritis

sequela. In joints that are difficult to evaluate clinically or have TABLE 36. Risk Factors for lnfectious Arthritis in Adults
complex anatomic structures, ultrasonography, Cl and MRI
General
can delineate the extent of an effusion and identi$r early bony
Age >80 years
changes.
Alcoholism
TEY POIXIS
Cutaneous ulcers or skin infections
o Infectious arthritis typically presents with pain, swell
Diabetes mellitus
ing, warmth, and erythema of the affected joint, accom-
End-stage kidney disease
panied by fever and constitutional symptoms.
lnjection drug use
o Acute monoarthritis or new inllammation of a single
joint in a patient with well-controlled inflammatory Malignancy

arthritis should prompt evaluation for infection. Hereditary immunodeficiency states or therapy with
immunosuppressive agents
. Diagnosis of infectious arthritis is confirmed by arthro-
Preexisting arthritis or joint damage (e.g., rheumatoid arthritis
centesis and evaluation of the qmovial fluid for Gram or osteoarthritis)
stain, culture, leukocyte count, and crystal analysis.
Prosthetic joint or recent joint surgery
Sickle cell disease

Gonococcal Arthritis
Causes
Sexually active patients
See MKSAP 19 Infectious Disease for details on the specific
Terminal complement deficiency
infections and diseases discussed in this section.
Lyme Arthritis
lnfection With Gram-Positive Organisms Travel to or residence in an endemic area
Bacterial arthritis is a medical emergency requiring immedi Documented tick bite or erythema ns
ate diagnosis and treatment. Risk factors include an immuno Mycobacterial or Fungal Arthritis
compromised state (including diabetes mellitus), injection
HIV infection or other immunosuppression
drug use, a history of joint surgery and having a prosthetic
joint. A recent break in the skin may be a contributing factor. Therapy with tumor necrosis factor inhibitors

lnfections occur rarely after invasive procedures, such as Travel to or residence in an endemic area
arthrocentesis or joint injection (Table 36). Most joint infec- Viral Arthritis
tions are monoarticular and derive from hematogenous seed- Chronic viral infection (HlV, hepatitis B virus, hepatitis C virus)
ing to the synovium. Polyarticular infectious arthritis may
Exposure to infectious agent or vaccine (e.g., rubella)
occur in persons who use injection drugs, those with systemic
Travel to or residence in an endemic area (e.9., chikungunya,
inflammatory disorders (such as rheumatoid arthritis), dengue, Zika)
immunosuppressed patients, and patients experiencing over-
Exposure to affected children (parvovirus B19, rubella)
whelming sepsis.
Approximately 75"/,, of cases of nongonococcal infectious
arthritis in adults are caused by gram-positive cocci. Staphylcxrtccus this syndrome from other forms of infectious arthritis. Risk
oureus is the most frequent microorganism in both native and factors for dissemination include HIV infection, pregnancy,
prosthetic joints. Stophylococcus epidermidis infections occur recent menstruation, and deficiencies in immunoglobulin
more commonly in prosthetic than in native joints. and terminal complement. The second presentation is a
purulent arthritis, usually without associated skin lesions or
lnfection With Gram-Negative Organisms fever. Patients present with acute onset of mono or oligo-
Disseminated Gonococcal Infection arthritis; the knees, wrists, and ankles are most commonly
Neisserio gonorrhoeoe most typically occurs in younger, sexu- involved.
ally active individuals. Disseminated infection occurs in 1% to Patients with the arthritis dermatitis syndrome are more
3'7, of patients infected with N. gonorrhoeae, with arthritis a likely to have positive blood cultures, whereas those with
common feature. purulent arthritis are more likely to have positive synovial
Disseminated gonococcal infection may present in one fluid cultures. Nucleic acid amplification testing should also
of two ways, although there may be overlap. The first is be obtained on samples from genital, rectal. and pharyngeal
the arthritis-dermatitis syndrome, a triad of tenosynovitis, sites.
dermatitis (usually painless pustular or vesiculopustular
lesions) (Figure 5O), and polyarthralgia without frank arthri Nongonococcal Gram-Negative Infections
tis. Feveq chills, and malaise are common. Inf'lammation of Aerobic gram-negative bacilli are the predominant cause of
tendons of the wrists, fingers, ankles, and toes distinguishes nongonococcal gram-negative joint infections. Predisposing

74
lnfectious Arthritis

positive results on enzyme-linked immunosorbent assay and


Western biot confirmatory serologiesl this is the primary
means of diagnosing Lyme arthritis and is preferred to poly
merase chain testing for B. burgdorleri DNA or culture of
blood or synovial fluid.

Mycobaaerium tuberculosis lnfection


The most common musculoskeletal manifestation of tuber
culosis is vertebral osteomyelitis (Pott disease), usually
resulting from the hematogenous spread of Mycobacterium
tuberculosis into the cancellous bone ofthe vertebral bodies.
The onset of symptoms is commonly insidious and is not
accompanied by systemic symptoms. Disease progression is
slow sometimes resulting in delayed diagnosis. Predisposing
F I G U R E 5 0. Disseminated gonococcal infertion can present as a febrile
factors for skeletal tuberculosis include previous tuberculosis
arthritis dermatitis syndrome with migratory polyarthralgia that may evolve to
infectious arthritis, tenosynovitis, and painless skin rash that may involve the infection, malnutrition, alcoholism, diabetes mellitus, and
palms and soles. Skin lesions can vary from maculopapular to pustular, often with a HIV infection.
hemorrhagic component. Peripheral arthritis may also occur and typically presents
with chronic pain in a single weight-bearing joint, such as the
Iactors include injection drug use, advanced age, and an knee or hip. As in vertebral osteomyelitis, concurrent pulmo-
immunocompromised state. Pseudomones oerugirloso can nary tuberculosis is present in only a minority of patients
be seen in infection related to injection drug use. Patients with peripheral arthritis. Laboratory indicators of inflamma
with sickle cell anemia may become infected with tion may be only slightly abnormal, synovial fluid findings
Salmonella species. Gram negative anaerobes account fbr can be nonspecific, and radiographic abnormalities can be
only 5'7, to 7'7, of bacterial arthritis cases, most commonly in delayed. Synovial biopsy is usually necessary for diagnosis
prosthetic joint infections and/or immunocompromised because the yield of synovial fluid stain and culture is less
hosts. than 50'1,.
Atypical mycobacteria can occasionally infect joints.
Lyme Arthritis Mycobacterium marinum is acquired through skin breaks
Although arthralgia and myalgia often occur at the earlier exposed to fresh or salt water (seen in fishermen and
stages of Lyme disease, Lyme afthritis is a late-stage manifesta aquarium hobbyists), and usually causes skin nodules and
tion. It is typically monoarticular, most commonly in the knee arthritis.
(Figure 5l). It should be suspected in patients who may have
had untreated or incompletely treated Lyme disease, although Fungal tnfections
not all patients will have a history compatible with prior Lyme Fungi are uncommon but important causes of bone and joint
disease. All patients with Lyme arthritis, however, shriuld have infections. Risk factors include immunosuppression and

FTGURE 51, Unilateral kneeeffusion in Lyme


arthritis.

75
lnfectious Arthritis

exposure to the fungi in endemic areas. Fungal joint infection oligoarthritis. Chronic HCV infection is often associated with
may be indolent and more difficult to diagnose than other circulating immune complexes, which may produce the clini
bone and joint infections. Fungal causes of osteomyelitis and cal syndrome of mixed cryoglobulinemia (purpura, weakness,
arthritis include coccidioidomycosis, blastomycosis, crypto- and arthralgia).
coccosis, candidiasis, and sporotrichosis. Fungal arthritis is Both HBV and HCV infections may be accompanied by
usually a result of hematogenous dissemination but can occur the presence of rheumatoid factor, which can cause diagnostic
after direct inoculation of the joint. Some fungal inlections confusion.
(e.g., blastomycosis and sporotrichosis) can be diagnosed by
synovial fluid examination and culture, whereas serologic Parvovirus B19
testing and synovial biopsy may be needed for other organ- Up to 60% of adults with parvovirus B19 experience arthritis.
isms (such as Coccidioides species). Synovial fluid leukocy.te It often presents acutely, is symmetric and polyarticular, and
counts vary. typically involves the proximal small joints of the hands.
Parvovirus B19 arthritis should be suspected when appropri
Viral tnfections ate clinical features are present in someone who has exposure
HTV to children, such as teachers and caregivers. (In contrast to
HIV infection is associated with several musculoskeletal adults, children frequently experience parvovirus B19 infec-
disorders, including painful articular syndrome, HIV- tion as a fever accompanied by a facial rash [slapped cheek
associated arthritis, reactive arthritis, infectious arthritis, appearancel, so-called fi flth disease.)
Acute parvovirus B19 infection is diagnosed by detecting
and diffuse infiltrative lymphocytosis syndrome. Painful
articular syndrome consists of bone and joint pain, espe- anti-parvovirus IgM antibodies in the serum. Anti parvovirus
cially in the lower extremities in an asymmetric pattern, IgG antibodies are highly prevalent in the general population
lasting less than 24 hours. HIV associated arthritis is a non- and indicate prior infection. Joint symptoms may persist for
destructive arthritis that involves joints of the lower extrem- weeks to months, and treatment is symptomatic (including
NSAIDs).
ity in an oligoarticular pattern and usually lasts less than
6 weeks. Reactive arthritis is the primary form of spondy
loarthritis seen in patients with HIV infection and is more Rubella
likely due to a response to other sexually transmitted or Arthritis is uncommon in childhood rubella infection, but up
enteric infections than to the HIV infection itself. Reactive to 60% of adults with rubella develop joint symptoms, mainly
arthritis in HIV may take a chronic relapsing course and may arthralgia, with the onset of rash. Joint involvement is usually
be accompanied by enthesopathy and mucocutaneous man- symmetric and migratory and most symptoms resolve within
ifestations. Psoriatic arthritis may be more common and is 2 weeks. The small joints of the hands, wrists, elbows, ankles,

often more severe in patients with HIV infection, with disa- and knees are most commonly affected. The pathogenesis of
bling enthesitis and joint erosion as well as axial involve- rubella arthritis is thought to be due to immune complexes
ment (including sacroiliitis). Diffuse infiltrative lymphocy and/or persistence of the virus in synovial cells or joint mac-
tosis syndrome is a rare condition that resembles Sjogren rophages. Diagnosis is usually made by detection of IgM
syndrome, but with CDB rather than CD4 cell infiltration of antirubella antibodies; the virus may also be cultured from
the exocrine glands. the nasopharynx or joint tissues. Joint fluid findings are
inflammatory.

Hepatitis Viruses
Hepatitis B virus (HBV) infection causes a self-limited arthri Mosquito-Borne Viruses
tis in up to 25% of infected patients during a prodromal stage Zika, dengue, and chikungunya viruses are transmitted by
before the onset ofjaundice. Joint involvement can be sudden Aedes mosquitos in tropical areas. Zika also can be vertically
and severe, with a pattern that is usually symmetric but can and sexually transmitted, and localized transmission in the
be migratory or additive. Hand and knee joints are most often southern United States has been reported. Zika infection is
affected; wrists, ankles, elbows, shoulders, and other large symptomatic in 2O"1, of infected individuals and causes
joints may be involved as well. Morning stiffness is common. arthralgia along with fever, rash, and conjunctivitis.
Fusiform swelling of the small joints of the hand may be Symptoms tend to be mild and last a week or less. Dengue
found on physical examination. Patients with chronic HBV fever causes arthralgia, fever, and rash but may be associated
infection may have recurrent but self limited polyarthralgia with severe muscle and bone pain, headache, and hemor-
or polyarthritis; it is not known to progress or cause joint rhagic complications. In chikungunya infection, arthritis is a
damage. predominant feature; patients may experience synovial
Acute hepatitis C virus (HCV) infection can cause acute thickening and tenosynovitis, often involving the hands and
onset polyarthritis, including the small hand joints, wrists, feet, in addition to fever and rash. The musculoskeletal symp
shoulders, knees, and hips. One third of patients have toms of Zika and dengue infections tend to subside within

75
lnfectious Arthritis

days to weeks, whereas those ofchikungunya infection usu


ally last longer, resembling rheumatoid arthritis (without
Management
rheumatoid factor or anti-cyclic citrullinated peptide anti ln patients suspected of having infectious arthritis, blood and
bodies) in some cases. Diagnosis of these mosquito-borne synovial cultures (and any other cultures appropriate to the
diseases is based on clinical features and serologic testing. clinical scenario) must be obtained before treatment. but
Polymerase chain reaction testing on blood samples is also empiric antibiotic therapy should be started pending culture
available. results (Table 37). In suspected bacterial arthritis, the initial
antimicrobial coverage should be broad and account for host
factors, such as immunosuppression, as well as likely causa
Prosthetic Joint lnfections tive microorganisms and regional antibiotic sensitivity data.
Bacterial infection complicates I"l, to 3"/,, of total knee and hip Antibiotics should initially be given parenterally. Given the
replacements; rot€S are higher in immunocompromised high prevalence of methicillin-resistant S. oureus, vancomycin
patients and those with rheumatoid arthritis. lnfection with is often included initially. The duration of treatment depends
gram positive organisms, such as S. oureus, is most common. on the causative organism and patient response and comor
Prosthetic joint infections are divided into early onset bidities, but bacterial arthritis usually requires 3 to 6 weeks of
(<3 months after placement), delayed (: to z+ months after treatment.
surgery), and late onset (>24 months after placement). These An infected joint must also be adequately drained. Needle
definitions are controversial, with some sources defining aspiration is an acceptable approach and should be performed
delayed infection as occurring 3 to 12 months after surgery. regularly (usually daily) as long as there is an effusion.
Early and delayed infections are usually related to surgical Ultrasound guidance may improve the ability to fully drain the
contamination at the time of the implantation, whereas late joint. Surgical drainage is an equally good alternative and is
infections result from hematogenous seeding of the joint. more immediately definitive. Additionally, surgical drainage is
Early and late prosthetic joint infections typically present with required for joints that are not easily accessible for needle
pain, warmth, effusion, and fever. Peripheral leukocyte count aspiration (e.g., sternoclavicular, sternomanubrial, shoulder,
and inflammatory markers are usually elevated. Delayed and hip joints), if there is evidence of soft-tissue extension of
infections, however, may be more dif'ficult to recognize infection, or if the clinical response to antimicrobial therapy is
because symptoms are less severe, and they are often caused inadequate. The goal of surgery is to remove all purulent mate
by less virulent microorganisms, such as coagulase negative rial and nonviable tissue and, in some cases, to perform syno-
staphylococci and other skin organisms. vial biopsy or synovectomy.
rEY POI]IIS Antibiotic treatment is recommended fbr all patients
o Approximately with Lyme arthritis. Approximately 90"/,, of patients will
75'/" of cases of nongonococcal infectious
respond to a28 day course of oral doxycycline, amoxicillin, or
arthritis in adults are caused by gram positive cocci,
cefuroxime axetil. Patients with incomplete responses may
wlth Staphylococcus qureus the most frequent micro
need treatment with a second course or a more aggressive
organism in both native and prosthetic joints.
drug regimen, usually intravenous ceftriaxone. Treatment
o Disseminated gonococcal infection may present as the beyond 1 month of ceftriaxone offers no benefit and should
arthritis dermatitis syndrome (tenosynovitis, dermati not be used. Antibiotic refractory Lyme arthritis occurs in 107,
tis, and polyarthralgia) or as purulent arthritis that pre- ofpatients or less, probably represents a progression to sterile
sents with mono- or oligoarthritis but usually without autoimmune arthritis, and responds to synovectomy or treat
skin lesions or fever. ment with disease modifying antirheumatic drugs (such as
. Lyme arthritis is a late-stage manifestation of the dis- hydroxychloroquine or methotrexate).
ease and istypically monoarticular (most commonly in Mycobacterial joint infections require at least 6 to
the knee) and inflammatory; diagnosis should be sus- 9 months of multidrug therapy. Treatment of arthritis associ
pected in patients who may have had untreated or ated with viral infections is largely supportive, although spe
incompletely treated Lyme disease. cific viral therapy is appropriate for HIV and HCV infections;
. MAcobacterium tuberculosis arthritis typically presents immunosuppressive therapy with glucocorticoids and rituxi
as chronic pain in a single weight-bearing joint, such as mab may be needed in refractory HCV related disease or severe
the knee; concurrent pulmonary tuberculosis is present mixed cryoglobulinemia.
in only a minority of patients. Treatment of prosthetic joint infections is challenging and
requires early surgical consultation. Orthopedic implants
. Early and late prosthetic joint infections usually present
serve as a nidus for microorganisms, and the avascularity of
with pain, warmth, effusion of the joint, and fever;
the infected hardware limits antibiotic penetrance. Many
delayed infections may be more difficult to recognize
patients require removal ofthe orthopedic device as part ofa
because they are often caused by less virulent micro-
two-stage procedure, with reimplantation of a new device
organisms, and symptoms may be less severe.
after an appropriate course of intravenous antibiotics.

77
Systemic Vasculitis

TABLf 3?. lnfectiousArthritisTreatment Based on Suspected Pathogen


Likely or ldentified Pathogen First-Line Therapy Second-Line Therapy Comments
Gram-Positive Cocci
MRSA Vancomycin Clindamycin; Narrow treatmentto MSSA coverage
daptomycin; linezolid asappropriate based on sensitivity data
MSSA Nafcillin; cefazolin
Gram-Negative Bacilli
Enteric gram-negative bacilli Third-generation Fluoroquinolones
cephalosporin (e.9.,
ceftriaxone or ce{otaxime)
Pse u d o m o n a s a e ru g i n osa Ceftazidime; cefepime; Carbapenems; aareonam;
pi peracillin-tazobactam fluoroq uinolones
Gram-Negative Cocci
Neisseria gonorrhoeae lV ceftriaxone for >7 days Fluoroquinolones (only if ln absence of specific culture
culture sensitivities sensitivity data, "stepping down" to
Oral doxycycline, 100 mg confirm susceptibility) oral therapy is no longer
orally twice daily for 7 days if recommended because of increasing
Chlamydia infection not resistance of N. gonorrhoeaeto
excluded commonly used oral agents
Suspected Bacterial lnfection, No Gram Stain
Always initiate therapy with Vancomycin; vancomycin + Appropriate to start with broad
coverage for MRSA; consider third-generation antibiotic coverage; narrow coverage
patient risk factors and cephalosporin; or if culture data become available
community patterns of infection; vancomycin +
consider coverage for gram- antipseudomonal antibiotic
negative organism if patient is
immunocompromised and at
risk for gonococcal infection
Confirmed Diagnosis
Bo r re I i a b u rg d o rf e ri (Ly me Oral doxycycline, amoxicillin, lf inadequate response, lV
arthritis) or cefuroxime axetil x 28 days ceftriaxone x 2-4 wk
M ycob a cte ri u m tubercu/osis 3- or 4-drug treatment (e.9., Duration mayvaryfrom 6 months or
isoniazid, pyrazinamide, longer depending on drug regimen
ri{ampin, ethambutol, (shortertreatment if rifampin is used)
streptomycin)
Fungal infections Amphotericin B, Prolonged treatment courses of
echinocandin, or azoles several months may be needed;
(f luconazole, itraconazole, maintenance therapy may be
voriconazole, posaconazole) required in high-risk patients
depending on suspected
organism or culture data

lV = intravenous; MRSA = methicillin-resistant Staphy/oco ccus aureus; MSSA = methicillin sensitive Staphyloco ccus aureus.

TEY ?OIilIS Systemic Vasculitis


. For patients suspected of having infectious arthritis,
blood and synovial cultures must be obtained before Overview
treatment, but empiric antibiotic therapy should be Vasculitis is inflammation of blood vessels, including capillar
started pending culture results. ies, arteries, and/or veins. Clinical manifestations result from
o In addition to antibiotic therapy, an infected joint must tissue ischemia in the affected areas. Vasculitis can be primary
also be adequately drained by needle aspiration or sur- secondary to another underlying autoimmune disease, or
gical drainage. triggered by other causes (Table 38). Vasculitis mimics must
be considered in the differential diagnosis (Table 39). Primary
. Many patients with prosthetic joint infections require
removal ofthe orthopedic device as part ofa two-stage
autoimmune vasculitis disorders are discussed in this
section.
procedure, with reimplantation of a new device after an
Vasculitis is not one disease, but many. These conditions
appropriate course of intravenous antibiotics.
are typically grouped into distinct categories according to

78
Systemic Vasculitis

TABLE 38. Causes of Secondary Vasculitis


Medications
Common causes: antimicrobial agents (e.g., minocycline, sulfadiazine); antithyroid agents (propylthiouracil 180y"-907o1, methimazole,
carbimazole, benzylthiouracil [10o/o-20%; not available in the United States]); cardiovascular drugs (hydralazine); tumor necrosis factor
inhibitors
Rare causes: vaccines; antiepileptic agents; antiarrhythmic agents; diuretics; anticoagulants; antineoplastic agents; hematopoietic
growth factors; NSAIDs; psychotropic drugs; sympathomimetic agents; allopurinol; interferon alfa; levamisole (associated with cocaine)
lnfections
Hepatitis A, B, and C viruses; HIV; bacterial endocarditis; parvovirus 819
Neoplasms
Hairy cell leukemia (associated with polyarteritis nodosa); other hematologic and solid malignancies
Autoimmune Diseases
Systemic lupus erythematosus; rheumatoid arthritis; Sjogren syndrome; inflammatory myopathies; systemic sclerosis; relapsing
polychondritis; inflammatory bowel disease; primary biliary cirrhosis

clinical presentation and pathophysiolo$r A well-established including the external carotid, subclavian, axilIary temporal,
classification scheme focuses on size ofinvolved vessels. Vessel ophthalmic, ciliary occipital, and vertebral arteries. The vessel
size can provide insight into processes and presentations, but involved dictates the clinical symptoms.
conditions and vessel sizes may overlap.
Clinical Manifestations and Diagnosis
Common GCA symptoms include headache (typically tempo
I

t,
Large-Vessel Vascu Iitis ral but occasionally more diffuse), constitutional symptoms,
L
Giant Cell Arteritis concomitant polymyalgia rheumatica (PMR), and scalp ten
I Epidemiolory and Pathophysiolory derness over the temporal artery. Symptoms can be unilateral
Giant cell arteritis (GCA; also called temporal arteritis) usually or bilateral. Jaw aching and fatigue with cherning indicate
affects patients older than 50 years (peak incidence, 70 B0 years). ischemia (claudication) of the muscles of mastication. The
GCA is more common in White persons (incidence in Europe, most feared complication of GCA is ischemic optic neuropathy
10-20/100,000). The female to male ratio is about 3:1. due to occlusion of the posterior ciliary or occasionally the
Histologically, GCA is characterized by granulomatous ophthalmic artery which can cause irreversible blindness.
inflammation of large and medium-sized arteries, with infil- Early recognition and treatment of any visual change (e.g.,
tration of CD4 positive lymphocytes, macrophages, and mult reversible diplopia or visual defect) are critical. Intracranial
inucleated giant cells. Inflammatory disruption of the internal ischemic strokes are rare.
elastic lamina is common. Involved vessels include the aortar Extracranial disease involving great vessels in the chest
its major branches olfthe arch; and secondary branch vessels, and arms occurs in up to 50'/" of patients but is usually

TABLE 39. Differential Diagnosis (lncluding Mimics) of Vasculitis


Disease Comments
lnfection (sepsis, endocarditis, hepatitis) Rash and/or musculoskeletal symptoms can occur.
Dru g toxicity/poisoni ng Cocaine, amphetamines, ephedra alkaloids, and phenylpropanolamine may produce
vasospasm, resulting in ischemia.
Coagulopathy Thrombotic diseases (disseminated intravascular coagulation; antiphospholipid
syndrome; thrombotic thrombocytopenic purpura)can produce ischemic symptoms.
Malignancy Paraneoplastic vasculitis is rare. Any organ system may be affected, but the skin and
nervous system are the most common. Vasculitic symptoms may precede, occur
simultaneously with, or follow diagnosis of cancer. Lymphoma occasionally may
involve the blood vessels and mimic vasculitis. Consider malignancy in patients with
incomplete or no response to therapy for idiopathic vasculitis.
Atrial myxoma Classic triad of symptoms is embolism, pulmonary congestion or heart failure, and
constitutional symptoms (fatigue; weight loss; fever). Skin lesions can be identical to i

I
those seen in leukocytoclastic vasculitis. i
I

Cholesterol emboli Typically seen in patients with severe atherosclerosis. Embolization may occur after I

abdominaltrauma, aortic surgery, or angiography. May also occur after heparin, l

l
warfarin, orthrombolytictherapy. Patients may have livedo reticularis, petechiae and
purpuric lesions, and localized skin necrosis.

79
Systemic Vasculitis

subclinical; it only sometimes results in upper extremity clau dose. Mild flares can be managed with modest increases of
dication. Severe but uncommon complications include aortic prednisone and slower tapering. The addition of low-dose
aneurysm and dissection. Up to 50% of patients with GCA methotrexate may facilitate glucocorticoid tapering and may
have PMR, which may occur before, concurrent with, or after be considered in patients with significant glucocorticoid
diagnosis of GCA (see Polymyalgia Rheumatica). contraindications, although data are limited. At treatment
Physical examination may reveal scalp or temporal artery initiation, all patients should undergo bone densitometry
tenderness and induration, reduced peripheral pulses, and to stratify risk for glucocorticoid-induced osteoporosis and
bruits. Laboratory studies almost always show elevated ery.th receive prophylaxis.
rocyte sedimentation rate (ESR) and/or C reactive protein GCA is self-limited (2-6 years) in some patients but may
(CRP). Rarely, however, the inflammatory markers may be be chronic in others. It also may recur. Appropriate and timely
normal. Nonspecific evidence of inflammation may include treatment can reduce risk for blindness.
anemia, elevated serum ferritin level, and thrombocytosis.
Serologic testing reveals no specific autoantibodies (although Polymyalgia Rheumatica
low-level antinuclear antibody levels and rheumatoid factor Epidemiologr and Pathophysiolory
are common in older patients). Although not a vasculitis, PMR is an inflammatory disorder
GCA is suspected on the basis of clinical presentation that frequently (up to 50'7, of cases) accompanies GCA. PMR
and should be confirmed, when possible, by temporal and GCA likely reflect the clinical spectrum of a single
artery biopsy and/or imaging of the great vessels. New or disease process, although PMR occurs 3 to 10 times more
atypical headache, jaw claudication, or visual changes in a frequently. Patients presenting with PMR should be asked
patient older than 50 years, especially with concurrent about arteritis symptoms because 107, to 20% have con
PMR, should raise suspicion. Temporal artery biopsy is comitant GCA.
diagnostic, but false-negative results are common because
of skip lesions; adequate biopsy sampling (>1-cm length of Clinical Manifestations and Diagnosis
temporal artery) is crucial; although bilateral temporal The classic picture of PMR is pain and stiffness of the shoul
artery biopsy can slightly increase the yield, unilateral der girdle and hip girdle. Symptoms may affect some or all of
biopsy is recommended. Temporal artery biopsy findings these areas; symmetric presentation is classic. Pain and stiff
remain abnormal for up to 4 weeks after initiation of glu ness are most pronounced in the morning. Peripheral arthri
cocorticoids. Given the risk for blindness. treatment should tis, generally mild and affecting the knees and wrists, may
therefore be initiated first and biopsy performed within occur. Synovitis in the hands and feet is uncommon.
2 weeks. If the pretest likelihood of disease is high, treat Constitutional symptoms and laboratory findings resemble
ment may be continued even in the presence of a negative those of GCA, although normal inflammatory markers are
biopsy result. more common in PMR than GCA. PMR is diagnosed clini
cally. Temporal artery biopsy should be performed only if
Management GCA is suspected.
Suspected GCA should be treated immediately to prevent
visual loss. Beginning high-dose prednisone (1 mg/kg/d, up Management
to 80 mg) is recommended. Intravenous pulse methylpre- Unlike GCA, PMR responds dramatically to moderate dose
dnisolone for 3 days is recommended for acute visual loss; prednisone (12.5 20 mg/d); lack of a rapid response should
however, data on this approach are limited, and estab- prompt consideration of alternate diagnoses. Prednisone
lished blindness is usually irreversible. Prednisone plus the taper is initiated 4 weeks after symptoms resolve and requires
interleukin 6 inhibitor tocilizumab is conditionally recom- months to years. Monitoring of recurrence is managed simi-
mended as initial therapy because the combination is superior larly to the approach used for GCA. For relapses, guidelines
to prednisone alone and typically permits faster glucocorti for the management of PMR (developed by the American
coid taper. Symptoms and inflammatory markers usually College of Rheumatologz and the European League Against
respond rapidly to treatment; lack of response should prompt Rheumatism) recommend increasing the prednisone dose to
reconsideration of the diagnosis. High dose prednisone is the last pre relapse dose at which the patient was doing well,
maintained for 2 to 4 weeks. Once symptoms resolve and followed by a gradual reduction within 4 to 8 weeks to the
inflammatory markers normalize, prednisone is tapered as relapse dose. Methotrexate can be used in select cases as a
tolerated, with a goal of reaching 20 mg/d within 2 to weak glucocorticoid-sparing agent.
3 months, at which point tapering is slowed. Glucocorticoids Prognosis is good, although periodic recurrences are
should be discontinued, when possible, within 6 months, but common. If PMR treatment is initiated properly, patients
longer treatment may be warranted as dictated by disease whose PMR is not accompanied by GCA rarely progress to
activity. Patients should be monitored for symptom recur GCA. Several studies suggest the benefit of tocilizumab for
rence. ESR and/or CRP should be monitored monthly but treating PMR in the absence of GCA, but the agent is not
should not be the sole indication for adjusting glucocorticoid approved for this purpose.

80
Systemic Vasculitis

Takayasu Arteritis

t
Epidemiolory and Pathophysiolory
Takayasu arteritis (TA) causes inflammation of the large ves-
sels, most commonly the aorta, followed by the subclavian,
common carotid, and renal arteries; the pulmonary arteries
may also be involved. TA is rare, with a prevalence of 40 per
million in Japan and up to 8 per million elsewhere. Although #
vascular involvement and clinical features overlap some- E #
what with GCA, TA predominantly affects younger women.
Arterial lesions are often stenotic ("pulseless disease"), and
one third are aneurysmal. Histopathology is similar to that
ol GCA.

Clinical Manifestations and Diagnosis


TA manifestations include new onset hypertension, Iimb clau-
dication, postprandial abdominal pain due to mesenteric
ischemia, reduced pulses, arterial bruits, and blood pressure
discrepancies between the arms. Heart failure related to aortic
insufficiency or coronary artery disease may occur. Neurologic
manifestations include transient ischemic attack, stroke, and
acute visual changes. As with GCA, laboratory studies are non
specific and reveal anemia as well as elevated ESR and CRP.
Angiography may demonstrate arterial stenosis or aneurysm
(Figure 52).

Management
Primary treatment of TA is high dose oral glucocorticoids
(1 mg/kg/d, up to 80 mg/d) with a slow taper. Guidelines con

ditionally recommend adjunctive treatment with nonbiologic


immunosuppressive medications (e.g., methotrexate, Iefluno-
mide, mycophenolate mofetil, azathioprine, and cyclophos-
phamide) to mitigate glucocorticoid exposure. For refractory t IGU R E 5 2. Aortic angiogram from a patient with Takayasu arteritis. Note the
high-grade stenosis of the proximal right subclavian artery (white arrcw) as well as
disease, adding a tumor necrosis factor inhibitor is preferred su bclavian artery just below the origin of the left v erlebral arhery (black anow\.
the left
over adding tocilizumab. Endovascular or reconstructive vas- lncidentally noted is an anatomic variation with a common origin of the right
cular procedures are sometimes required; if possible, these brachiocephalic artery and the left common carotid artery.

should be performed during periods of inactive disease. The


leading cause ofdeath is heart failure; stroke and cardiovascu
lar disease also contribute to morbidity. The 1O-year survival
Mediu m-Vessel Vascu litis
rate is 90'l". Polyarteritis Nodosa
Epidemiolory and Pathophysiolory
I(EY POITTS
Polyarteritis nodosa (PAN) is a rare systemic necrotizing vascu-
. Giant cell arteritis should be suspected in a patient litis that affects medium-sized and sometimes small-sized
older than 50 years with new or atypical headache, jaw arteries. Prevalence is approximately 31 per million but declin
claudication, or visual changes. ing. Average age at onset is 50 years.
HVC . Suspected giant cell arteritis should be treated immedi- Historically, PAN has been strongly associated with hep-
ately with prednisone to prevent visual loss; diagnosis atitis B virus (HBV). Because HBV prevalence has declined
can still be confirmed with temporal artery biopsy with the advent of the HBV vaccine and antiviral treatment,
within 2 weeks after initiation of prednisone. the proportion of patients with HBV-associated PAN has
HVC r Polymyalgia rheumatica is associated with pain and declined from 36% to less than 5'/. of all PAN cases; most
stiffness ofthe neck, shoulder, and pelvic girdles; it contemporary cases are autoimmune. Pathophysiologr is not
responds dramatically to moderate-dose prednisone. well understood.
. Takayasu arteritis occurs in younger women and causes
Clinical Manifestations and Diagnosis
inflammation of the aorta and other major noncranial
PAN most commonly affects the skin and peripheral nerv
vessels.
ous system, as well as the gastrointestinal tract and kidneys.

81
Systemic Vasculitis

ir: :r:, ,-li,' Clinical Features of Polyarteritis Nodosa


Organ System Symptoms Frequency Comments
Constitutional Fever; malaise; weight loss 65%
Musculoskeletal Arthralgia; myalgia 55%
Skin Purpura; nodules; necrotic ulcers 50%-600a
Neurologic Mononeuritis multiplex; peripheral neuropathy 79"/" Wrist drop; foot drop
Kidney Hypertension; hematuria; proteinuria 40% Renal artery microaneurysms with tissue
infarcVhematoma; no glomerulonephritis
Gastrointestina I Mesenteric ischemia; intestinal perforation; 38% One third of gastrointestinal cases manifest
pancreatitis; cholecystitis; appendicitis; as acute abdomen
gastrointestinal bleeding
Testicular Orchitis 17% Usually unilateral, due to testicular artery
involvement
Other Sensorineural hearing loss Case reports Bilateral; symmetric; sudden onset; rapidly
progressive
Laboratory Elevated erythrocyte sedimentation rate
in 82%; elevated C-readive protein;
leukocytosis; anemia; thromboqytosis;
increased liver chemistrytest result in 33%

Table 4O lists the clinical and laboratory findings of PAN. Management


The disease does not involve the lungs. When present, kid- Intravenous pulse glucocorticoids and ryclophosphamide are
ney involvement is renovascular rather than glomerular indicated for severe organ-threatening disease; glucocorti
and often leads to hypertension. Cutaneous PAN is a vari- coids and disease-modirying antirheumatic drugs are used for
ant confined to the skin. Cutaneous features of medium- milder disease. HBV-associated PAN is treated with short-term
vessel vasculitis include livedo reticularis (Figure 53) and glucocorticoids, antiviral medication, and plasmapheresis if
painful deep ulcers and skin necrosis due ischemic infarc- necessary. The S-year survival rate for treated PAN is 80%, and
tion (Figure 54). Because of the intense inflammation of the relapse rate is 10% to 2O%.
medium-sized vessels, subcutaneous nodules may be
palpable. PrimaryAngiitis of the Central Nervous System
Diagnosis is often confirmed via angiography of the mes- Epidemiolory and Pathophysiolory
enteric and/or renal vasculature, which demonstrates saccular Primary angiitis of the central nervous system (PACNS) is a
microaneurysms and areas of narrowing in medium-sized rare small- and medium-vessel vasculitis of the central nerv-
vessels. The gold standard for diagnosing PAN is histologic ous system; it affects the brain parenchyma, spinal cord, and
identification of focal segmental panmural necrotizing inflam- leptomeninges. Incidence is 2.4 per 100,000. Median age at
mation of a medium-sized vessel, obtained on biopsy of
involved, easily accessible tissue (e.g., skin or a superficial
nerve).

F IGUR E 54 . Retiform purpura on the lower legs due to vasculitis characterized


FIGU R E 5 3 . Livedo reticularis on the feet, recognized as a red-blue, reticulated by dark red or purple net-like, branching, or stellate configuration. Skin necrosis and
vascular network. ulceration may occur.

82
Systemic Vasculitis

onset is 50 years. The three histologic presentations have a


Sma I l-Vessel Vascu I itis
patchy distribution: granulomatous (most common), lympho
ANCA-Associated Vascu litis
cytic, and necrotizing.
ANCA-associated vasculitis includes three diseases character-
Clinical Manifestations and Diagnosis ized by the presence of ANCA (Table 4f), granulomatosis with
Patients with PACNS usually present with gradual and progres
polyangiitis, microscopic polyangiitis, and eosinophilic granu
sive symptoms of headache, cognitive impairment, neurologic
lomatosis with polyangiitis. (ANCA associated glomerulone
deficits, transient ischemic attacks, and strokes. Laboratory phritis is discussed in MKSAP 19 Nephrologr.) The presence of
findings are normal. Cerebrospinal fluid is abnormal but non- ANCA can be detected on serologic testing and helps to define
specific in 80'2, to 90% of patients, with elevated protein, lym the diseases.
phocltic pleocy.tosis, and occasional oligoclonal bands. MRI The two types of vasculitis associated ANCA are p ANCA
(perinuclear, directed against the neutrophil enzyme myelo
shows nonspecific white and gray mafter changes and infarcts.
Magnetic resonance angiography and CT angiography have peroxidase) and c ANCA (cl,toplasmic, directed against the
limited usefulness because of poor resolution. Cerebral angiog neutrophil proteinase 3). Perinuclear and cytoplasmic refer to
raphy may demonstrate vessel "beading" (alternating dilations patterns of immunofluorescent staining; enzyme linked
and stenoses) but has limited sensitivity and specificity. Brain immunosorbent assays against myeloperoxidase and protein-
biopsy should be performed in any patient in whom the diag- ase 3 are used to confirm antibody positivity and in some cases

nosis is seriously considered; however, the patchy distribution to replace immunolluorescence testing.
of findings results in a 50'/n false-negative rate. ANCA may play a direct role in disease propagation by
Evaluation also focuses on ruling out other conditions, activating primed endothelial cells and neutrophils, leading
including infection, malignancy (e.g., intravascular CNS lym- to vessel damage. The granulomatous inflammation in
phoma), and reversible cerebral vasoconstriction syndrome. some forms of ANCA associated vasculitis suggests a role for
cell-mediated immunity. Although most cases of ANCA-
Management associated vasculitis are idiopathic. a few are drug induced;
PACNS is treated with high dose glucocorticoids and cyclo several drugs can cause this rare reaction, including pro-
phosphamide. Patients often have permanent disability from pylthiouracil and levamisole (an adulterant often found in
neurologic damage, and the recurrence rate is 27"/,,. cocaine).
See MKSAP 19 Nephrologr for details on kidney involve

Kawasaki Disease ment in ANCA associated vasculitis.


Kawasaki disease is a medium vessel vasculitis and the most
common type of vasculitis in children. It presents as fever, Granulomatosis With Polyangiitis
rash, cervical lymphadenopathy, conjunctival congestion, and EpidemiologA and Pathophy siology
mucositis. Coronary vessel vasculitis is the most feared compli- Granulomatosis with polyangiitis (GPA) is the most common
cation and may leave permanent vasculopathy if not treated ANCA-associated vasculitis. with an annual incidence of 7 to
promptly. Although Kawasaki disease almost never develops in 12 per million.

adulthood, adults who had Kawasaki disease as a child may have


residual coronary aneurysms that require monitoring and man- Clinical Manifestations and Diognosis
agement. Recently a Kawasaki like syndrome (multisystem GPA affects the small vessels of the upper and lower airways
inflammatory syndrome in children) has been identified in (sinuses, lungs), kidneys, eyes (scleritis), skin, and peripheral
patients (rarely, adults) who have been infected with COVID-l9. nerves. At least 50'X, of,patients have constitutional symptoms.
More than 95'7, of patients are ANCA positive, with antibodies
r(EY POll{I5 overwhelmingly (>SO'2, of cases) directed against proteinase 3
r Polyarteritis nodosa most commonly affects the skin (anti PR3 antibodies; c ANCA).
and peripheral nervous system, as well as the gastroin- GPA presents as systemic or localized disease. The sys
testinal tract and kidneys; the gold standard for diagno- temic form is more common, involves major organs, and is
sis is focal segmental panmural necrotizing inflamma- associated with positivity for anti PR3 antibodies. Patients
tion of a medium-sized vessel on biopsy. with localized disease are more likely to be younger and
. Patients with primary angiitis of the central nervous female; have mainly ear, nose, and throat involvement; and are
system usually present with gradual and progressive less likely to be PR 3 positive. See Table 41 for clinical features
symptoms of headache, cognitive impairment, neuro- ofGPA.
logic deficits, transient ischemic attacks, and strokes. Cutaneous leatures of GPA include petechiae or palpable
r purpura (both of which are nonblanching lesions; Figure 55),
Adults who had Kawasaki disease as a child may have
hemorrhagic bullae, and superficial ulcers. However, any
residual coronary aneurysms that require monitoring
and management.
of the small vessel vasculitides may present with these
findings.

83
Systemic Vasculitis

TABLE 4'l . Clinical Features of ANCA-Associated Vasculitis Diseases


Granulomatosis With Microscopic Polyangiitis Eosinophili< Granulomatosis
Polyangiitis With Polyangiitis
ANCA c-ANCA (antiprotei nase-3 p-ANCA (anti myeloperoxidase p-ANCA (anti myeloperoxidase
antibodies) (>957o) antibodies) (507"-7 5"/") antibodies) (-50%)
Vascular histology Pauci-immune necrotizing Pauci-immune nongranulomatous Pauci-i m mu ne necrotizing
granulomatous vasculitis necrotizing vasculitis granulomatous vasculitis with
eosinophilic infiltration of vessel
walls and tissues; extravascular
granu lomas
Cardiac Pericarditis; myoca rd itis; Pericarditis; endomyocarditis;
conduction disorder (<1 0%) conduction disorder; heart failure
(27"/o-47"/")

Ea rslnose/th roat Crusting; rhinorrhea; sinusitis; Sinusitis; sensorineural hearing Nasal polyps; rhinitis; sinusitis
otitis media; chond ritis of ears and loss (9%-30%) (prodromal)
nose with saddle nose deformity;
septal perforation (7 0"/"-1 00"/")
Gastrointestinal Ulceration; perforation (5"/"-1 1%l Abdominal pain; bleeding Abdominal pain; bleeding
(30%-s8%)
Kidney Pauci-immune necrotizing Pauci-immune necrotizing Pauci-immune necrotizing
glomeru lonephritis (40%- 1 00%) glomerulonephritis (80%- 1 00%) loneph ritis (25o/"\
g lome ru

Lung Alveolar hemorrhage; nodules; Alveolar hemorrhage; pulmonary Asthma (prodromal, >90%);
trachea l/su bg lottic stenosis infiltrates; pulmonary fibrosis nodular opacities; infiltrates
(500/"-907o) (25%-55"/") (257o-86o/"\

Ocular Scleritis; episcleritis; retinal


vasculitis; retro-orbital
pseudotumor; dacryoadenitis
(14"/"-60%)

Skin Petechiae/purpura; painful skin Petechiae/purpura; painful skin Petechiae/purpura; painful skin
lesions; maculopapular rash; lesions; maculopapular rash; lesions; maculopapular rash;
livedo reticularis; ulcers livedo reticularis; ulcers livedo reticularis; ulcers
Neurologic Mononeuritis multiplex, Distal symmetric polyneuropathy; Sensori motor peripheral
sensorimotor peripheral m o n o ne u riti s m ulti pl ex (37 "/"-7 2"/"1 neuropathy, mononeu ritis
neuropathy (33%) multiplex (70%)
Central nervous system
Centra I nervous system involvement pachymeningitis; cerebral
(pachymeningitis) (<5%) hemorrhage; infaras (<20%)

In the setting ofa classic clinical presentation and positiv- GPA is common (historically, >50'X, after initial remission).
ity for c-ANCA/anti-PR3 antibodies, diagnosis of GPA is Glucocorticoids alone are insufficient to control GPA. Patients
straightforward. However, because of significant risks of treat- with limited presentations of GPA (such as arthropathy or
ment, biopsy of involved tissue is usually recommended if upper airway disease) without organ threatening disease can
possible. Histopathologz of most tissues demonstrates pauci- sometimes be treated with glucocorticoids plus methotrexate;
immune necrotizing granulomatous vasculitis; pauci-immune such patients should be carefully monitored for treatment
necrotizing glomerulonephritis without granulomas is seen failure or development of kidney or other organ threatening
on kidney biopsy. disease, necessitating a more aggressive regimen. With use of
these approaches, GPA mortality has declined from 90'ln to
Management around 10%. Kidney failure and infection remain the main
To induce remission in severe organ-threatening or life- causes ofdeath.
threatening disease, treatment of GPA consists of high-dose
oral glucocorticoids or intravenous glucocorticoids plus Microscopic Polyangiitis
rituximab (preferred) or cyclophosphamide; selected patients Ep idemiologA ond P ot ho p hg s iology
may benefit from plasma exchange. Once remission is The annual incidence of microscopic polyangiitis (MPA) is
achieved, patients should receive maintenance therapy con- estimated al 2.7 per million in Europe and lower elsewhere.
sisting of rituximab, methotrexate, or azathioprine; rituxi- Average age at onset is 50 to 60 years, with a predilection
mab appears to be most effective for preventing relapse and for men over women (1.8:1). Compared with GPA, ANCAs are
is preferred. It remains unclear whether suppressive mainte- Iess prevalent (507, 75'X,) and usually directed against
nance therapy can be fully withdrawn because relapse in myeloperoxidase.

84
Systemic Vasculitis

Clinical Manifestations ond Diognosis


The typical patient with EGPA has a prodromal history of
difficult-to-treat asthma (96% 7oo'k), rhinitis, and/or atopy. A
phase with increased peripheral and tissue eosinophilia
follows, with migratory pulmonary infiltrates and, Iess com-
monly, endomyocardial infiltration and gastrointestinal disease.
The subsequent acute vasculitic phase includes mononeuritis
multiplex or peripheral sensorimotor neuropathy (70'X,), kidney
involvement (2s'2,), and skin involvement (60'l,,). The vasculitis
phase is often associated with improvement of asthma. See
Table 41 for the clinical features of EGPA.
Laboratory findings show peripheral eosinophiiia of more
than 10'1,, or more than 1500/pL (1.5 x 10e/L). Only about 407,
to 60'l. of patients have a positive ANCA result, mostly directed
against myeloperoxidase.
Diagnosis is based on typical clinical findings, eosino
philia, and biopsy specimens demonstrating fibrinoid necrosis
and eosinophilic infiltration of vessel walls, as well as extravas
cular granuloma formation.

Management
In for mild
EGPA, glucocorticoids alone may be sufficient
disease without major organ involvement. With kidney,
gastrointestinal, cardiac, or neurologic involvement, cyclophos
phamide had been indicated in the past. However, mepoli
zumab, a monoclonal antibody that binds interleukin 5, has
been FDA approved for EGPA and has a more favorable safety
FIGU R E 5 5. Small-vessel vasculitis manifesting as palpable purpura and
profi le than cyclophosphamide.
petech iae.
I\4ortality for EGPA is the lowest among all forms of
ANCA associated vasculitis. The S-year survival is 977,, and
Clinical Manifestations ond Dicgnosis the relapse rate is 2B%. If not treated early, neuropathy may
Like GPA, MPA characteristically affects the lungs and kid- become permanent.
neys, along with other organ systems. In contrast to GPA,
I( EY PO I ilTS
MPA tends to spare the upper airways, and lung involve
ment is not granulomatous on biopsy. See Table 41 for the . Granulomatosis with po$angiitis affects the upper and
clinical f'eatures of MPA. Diagnosis is suspected on the basis Iower airways, kidneys, eyes, skin, and peripheral nerves;
of typical clinical findings and positivity for myeloperoxi induction of remission in severe organ threatening or
dase ANCA, although negativity for ANCA does not rule out life threatening disease consists ofhigh dose glucocor
the diagnosis. The diagnostic gold standard is a biopsy ticoids rituximab (pref'erred) or cyclophosphamide.
specimen demonstrating necrotizing pauci-immune vas r l\4icroscopic polyangiitis characteristically affects the
culitis of small vessels in any afl'ected tissue or pauci- lungs and kidneys; treatment is the same as that for
immune necrotizing crescentic glomerulonephritis in the granulomatosis with polyangiitis.
kidney. Absence of granulomas distinguishes MPA from . Eosinophilic granulomatosis with polyangiitis is associ-
GPA.
ated with asthma, rhinitis, sinusitis, atopy, peripheral
and tissue eosinophilia, migratory pulmonary infil
Management trates, and mononeuritis multiplex or peripheral senso
Treatment of MPA is similar to that of GPA. rimotor neuropathy; treatment consists of glucocorti-
coids for mild disease, with mepolizumab or
Eosinophilic Granulomatosis With Polyangiitis cyclophosphamide added for more severe disease.
Ep id e mi olo gU ond P atho phy s io lo gy
Eosinophilic granulomatosis with polyangiitis (EGPA) is
the rarest ANCA associated vascuiitis. with an annual lmmune Complex-Mediated Vasculitis
incidence of 0.11 to 2.66 per million. Eosinophil infiltra Immune complexes develop from cross linking of multiple
tion, activation, and degranulation participate in disease antigens and antibodies. Immune complexes can deposit
pathogenesis. in small vessels, Ieading to complement and neutrophil

85
Systemic Vasculitis

activation, with consequent inflammation and tissue Clinical Monifestations and Diognosis
damage. Although any tissue or organ may be affected, the Cutaneous symptoms (palpable purpura, digital ischemia,
classic finding is invariably in the skin. Inflammation and ulcers, necrosis, and livedo reticularis) are seen in about 90%
erythrocyte extravasation from damaged vessels result in of patients. Other common manifestations include peripheral
nonblanching palpable purpura, usually in dependent areas neuropathy, arthralgia without arthritis, and glomerulone-
(Figure s6). The various immune complex mediated vascu- phritis (usually membranoproliferative). Pulmonary and/or
litides are distinguished by the antigen that drives them, the gastrointestinal involvement is rare. Testing for cryoglobulins
class of antibody generated, and differences in clinical requires that a sample of blood be maintained at 37.0 "C
presentations. (98.6 'F) until clotting is complete to avoid incorporation of
the cryoglobulins into the clot. In addition to detection of
Cryoglobulinemic Vasculitis cryoglobulins, laboratory abnormalities frequently include
Cryoglobulins cause an immune complex mediated small depressed C4 complement and low CH50 levels. Patients
vessel vasculitis primarily affecting the skin, peripheral nerv- with type II and type III cryoglobulinemia have positivity for
ous system, and kidneys. CNS involvement may rarely occur. rheumatoid factor. A false-negative cryoglobulin result is not
There are three types of cryoglobulins. Type I occurs as a unusual given the complexity of cryoglobulin testing.
consequence of malignant monoclonal gammopathies and
B-cell lymphoma. Types II and III are polyclonal and "mixed" Management
in nature. Each is formed from the interaction of antigen Treatment of cryoglobulinemia requires first addressing the
targeted antibodies with rheumatoid factor; tlpe II cryoglobu- cause of the disease, such as instituting antiviral therapy for
lins include a monoclonal IgM rheumatoid factor, whereas in hepatitis C virus related cryoglobulinemia. When the conse
type III cryoglobulins the rheumatoid lactor is a polyclonal IgM. quences of the cryoglobulinemia are severe (e.g., glomerulo
The ability olrheumatoid factor to directly bind other antibod nephritis, ulcerating cutaneous lesions, digital gangrene), the
ies facilitates the formation of immune complexes even in the vasculitis itself must also be treated. Glucocorticoids and
absence of persistent antigen. In addition to their ability to rituximab in combination are the standard first line therapy.
form immune complexes, all cryoglobulins demonstrate the
unique feature of precipitating in the cold. See MKSAP 19 IgA Vasculitis
Hematologz for details on type I cryoglobulins and the differ- See MKSAP 19 Nephrologr for information on IgA nephropa
entiation of cryoglobulinemia from cold agglutinin disease. thy and on kidney involvement in IgA vasculitis.

Epi demiolo gA a nd P athophy s iol ogy Ep ide miologA and Potho phy s iology
Mixed cryoglobulinemia, which accounts for 75% to 90'l" of all IgA vasculitis (Henoch-Schonlein purpura) is a common vas
cases of cryoglobulinemic vasculitis, is rare, with an estimated culitis of childhood that rarely occurs in adults. Estimated
prevalence of 1in 100,000. About two thirds of mixed cryoglo annual incidence in adults is 14 per million. Onset is usually
bulinemia (type II and type III) cases are related to hepatitis C preceded by upper respiratory tract infection, often with
virus infection. Autoimmune diseases, such as systemic lupus streptococcal bacteria.
erythematosus and Sjdgren syndrome, more typically cause
type I I I cryoglobulinemia. Clinical Manifestations o nd Diognosis
Patients with IgA vasculitis typically present with palpable
purpura in dependent areas. Other common findings include
abdominal pain or gastrointestinal bleeding (50'1,), arthritis
and arthralgia (oo%), and glomerulonephritis. Although pro-
gressive renal damage may occur, 90% of adults with kidney
involvement fully recover.
There are no specific laboratory tests for diagnosis.
Diagnosis is confirmed by demonstrating IgA tissue deposi
tion, generally in the skin, by immunofluorescent microscopy.
Skin biopsy demonstrates leukocy.toclastic vasculitis with
healy deposits of IgA and complement on immunofluorescent
staining. Although rarely necessary renal histolory shows IgA
deposition within the mesangium.

F IG UR E 5 6. Palpable purpura is the classic rash of any small-vessel, immune


Management
complex-mediated vasculitis. The Iesions are nonblanching and represent
extravasations of blood from damaged vessels. Purpuric lesions are typically more
Although IgA vasculitis in children tends to be self-limited,
prominent 0n the lower exlremities, a consequence of the superimposed eflect of adults are more likely to develop persistent nephropathy and
gravity on hydrostatic pressure. may require glucocorticoids. Despite lack of evidence, some

86
Other Rheumatologic Diseases

patients may require additional immunosuppression (cyclo- Mediterranean. Prevalence is highest in Turkey (20-600/
phosphamide or rituximab). 100,000). Outside of endemic areas, Behqet syndrome
tends to be less severe. The main genetic risk factor is the
Hypersensitivity Vasculitis major histocompatibility complex I antigen HLA-B51, sup
Epidemiology and Pathophg siology porting Behqet syndrome as an autoimmune disease.
Hypersensitivity vasculitis is a small vessel vasculitis medi- However, many experts believe that the condition repre
ated by immune complex deposition confined to the skin. It sents mixed mechanisms involving both autoimmunity
may be triggered by an antigen, such as a drug or infection; in and autoinflammation. Most patients with Behqet syn-
50'/u of cases, the antigen is unknown. Men and women and drome present with painful oral and/or genital ulcers
people of all ages are aflected. (Figure 57). Genital ulcers typically occur on the scrotum
and vulva and tend to scar.
Clinical Manifestations ond Diognosis Gastrointestinal involvement may be present, making the
The most common presentation of hypersensitivity vasculi distinction between Behqet syndrome and Crohn disease dif
tis is palpable purpura in dependent regions, often develop ficult. Eye disease includes panuveitis, optic neuritis, and
ing 7 to 10 days after exposure to a triggering antigen; Iesions retinal vasculitis. Arthritis and enthesitis may be present.
appear in "crops" and resolve over a few weeks after the Cutaneous manifestations include pustular lesions, skin
antigen is removed. Internal organs are unaffected. Skin ulcers, and erythema nodosum. Vascular involvement is seen
biopsy with immunofluorescence demonstrates leukocyto in both the arterial and venous systems. In the arterial bed,
clastic vasculitis without heavy IgA deposits. Evaluation both aneurysms and thrombosis may be seen. The pulmonary
should be guided by clinical signs and symptoms and may bed may be affected, and Budd Chiari syndrome can occur.
require only a complete blood count, basic chemistries, and The most common form of venous disease is thrombosis.
urinalysis. which occurs in the lower extremities but may also occur in
the cerebral and cardiac circulations. Central nervous system
Management disease may include parenchymal brain involvement, espe
Removal of the antigen (if identified) and supportive care are cially in the brainstem. Pyramidal, motor, and behavioral
usually sufficient. Resolution within a month is the rule. If changes may occur.
symptoms persist or recur, anti-inflammatory agents, topical Diagnosis of Behqet syndrome is based on the presence of
or low dose systemic glucocorticoids, colchicine, or dapsone the characteristic signs and symptoms (Table 42). The pres-
may be helpful. ence of patherry-accumulation of a pustular neutrophilic
infiltrate after disruption of the skin with a sterile needle
XEY POIXTS
strongly suggests Behget syndrome (Figure 58). However,
o Mixed cryoglobulinemia is associated with cutaneous
results of pathergz tests are often negative, particular\ in
symptoms, peripheral neuropathy, arthralgia without
nonendemic areas. lmaging may define typical, but nonpath-
arthritis, and glomerulonephritis, with two thirds of
ognomonic, patterns of vascular involvement.
cases related to hepatitis C virus infection.
Management involves selecting agents appropriate to the
o IgA vasculitis (Henoch-Sch6nlein purpura) is character- type and severity of clinical involvement. For oral ulcers, both
ized by palpable purpura and abdominal pain or gastro topical and systemic glucocorticoids may be used. Apremilast,
intestinal bleeding and occurs mainly in children. a phosphodiesterase 4 inhibitor, is FDA approved for oral
I
o The most common presentation of hypersensitivity vas-
culitis is palpable purpura, developing 7 to 10 days after
exposure to a triggering antigen.

L Other Rheumatologic
Diseases
Behget Syndrome
i Behqet syndrome, a systemic vasculitis, consists of a constel-
lation of characteristic signs and symptoms, including oral
and genital ulcers and inflammatory eye disease. Less com-
mon but more ominous features include vascular and central FIGURE 57. Recurrent,painful genital mucosal ulcerationsareacharacteristic
nervous system disease. Epidemiologr varies widely; endemic clinical leature of BehEet syndrome. ln women, they typically appear on the vulva,
areas are along the historic Silk Route from East Asia to the as shown, and may heal with disfiguring scarring.

87
Other Rheumatologic Diseases

TABLE 42. lnternational Criteria for Behqet Disease Rela psing Polychond ritis
Manifestations Points" Relapsing polychondritis is a rare condition (annual incidence,
Recurrent oral ulcers 2 3.5 cases/l million) characterized by inflammation and
Recu rrent genital ulcers 2 destruction of cartilage. The most apparent involvement of
relapsing polychondritis is in the helix and antihelix of the
Pathergy
external eari the initial presentation there is redness and
Skin lesions (pseudofolliculitis; skin ulcers;
erythema nodosum) swelling, but it may progress to structural collapse and a floppy
appearance (Figure 59). Middle ear involvement may cause
Eye lesions (anterior uveitis; posterior uveitis; 2
retinalvasculitis) hearing loss. Other cartilaginous structures that can be
affected include the bridge of the nose and articular cartilage,
Central nervous system lesions
Ieading to inflammatory arthritis. Involvement of the tracheal
Vascular lesions (arterial thrombosis; large-vein
thrombosis; phlebitis) rings can cause airway obstruction, requiring surgical inter
vention. Other connective tissue structures involving the eyes,
'Four or more points are needed for the diagnosis of Behget syndrome.
tendons, heart valves, and skin may be affected.
Pathogenesis of relapsing polychondritis is poorly under-
stood. An association with HLA-DR4 supports an autoimmune
cause, and antibodies to various cartilage components are
sometimes present. On biopsy specimens, cartilage shows a
polymorphic inflammatory cell infiltrate.
Diagnosis is made clinicallyi biopsies are rarely necessary.
Tracheobronchial involvement mandates an aggressive evalu-
ation, including imaging (typically CT), pulmonary function
testing, and, in some cases, tracheobronchoscopy. In 30'7, of
cases, relapsing polychondritis may be associated with other
autoimmune diseases, including ANCA-associated vasculitis.
The MAGIC (mouth and genital ulcerations with inflamed
cartilage) syndrome appears to represent a confluence of
F IGUR E 5 8. Pathergy associated with Behget syndrome, characterized by relapsing polychondritis and Behqet syndrome.
pustular-appearing skin lesions occuning 48 hours after skin pricking with a sterile Management depends on severity. For mild symptoms,
need le. NSAIDs and dapsone may be considered. For acute and/or
severe involvement, high-dose glucocorticoids are the initial
ulcers in Behget syndrome. Thalidomide and colchicine are management. Oral immunosuppressants (e.9., methotrexate)
both accepted treatments; colchicine may be better for may be added. The tumor necrosis factor inhibitors infliximab
arthritis than for oral ulcers. Topical and/or oral 5-aminosali and adalimumab have been used to good effect. The anti
cylic acid derivatives may be used for gastrointestinal interleukin-6 agent tocilizumab may also be effective.
involvement. For resistant severe ulcers, or for vascular, oph
thalmologic, or central nervous system involvement, oral
disease modifying antirheumatic drugs, such as azathio
prine and cyclosporine, may be considered; tumor necrosis
factor inhibitor therapy, particularly infliximab and adali-
mumab, is frequently effective. In severe cases, cyclophos
phamide may be an option. Acute thrombosis should be
treated with immunosuppressives rather than anticoagulants
because the cause of the thrombosis is inflammatory rather
than thrombotic.

TEY POITIS
. Behqet syndrome is characterized by recurrent painful
oral and genital mucosal ulcerations, inflammatory eye
disease, and pathergr.
. Vascular, ophthalmologic, and central nervous system
involvement requires aggressive treatment with immu-
nosuppressive andior tumor necrosis factor inhibitor F IG UR E 5 9. Floppy ear in a patient with relapsing polychondritis. Chronic
inflammation of the upper section of the ear eventually results in loss of cartilage
therapy.
and structural collapse.The ear lobe, which does not contain cartilage, is unaffected.

88
Other Rheumatologic Diseases

I(EY POII{I arthritis, rash, and sterile peritonitis. Renal amyloidosis may
. ensue. The cryopyrin-associated periodic fever syndromes
Relapsing polychondritis is characterized by inflamma-
result from activation of the NLRP3 inflammasome. In the
tion and damage of cartilaginous tissues; tissues most
pediatric version, neonatal onset multisystem inflammatory
commonly affected include the cartilaginous portions
disease, inflammation leads to permanent joint deformity,
of the external and middle ear, nose, and tracheal rings.
encephalitis, and, ifnot treated, early death. Adult versions of
cryopyrin-associated periodic fever syndromes are milder and
may be triggered by cold. In TRAPS (tumor necrosis factor
Autoi nflam matory Diseases receptor-associated periodic fever syndrome), mutations in
Autoinflammatory diseases xre monogenic conditions in the tumor necrosis factor receptor lead it to be persistently
which innate immune pathways are activated independently activated on inflammatory cells. Patients have fever, chills, and
of any specific known antigen. The result is periodic or chronic severe muscle pain lasting more than 1 week. Other fbrms of
inflammation, f'ever, and multiorgan involvement. The best mutations can also lead to periodic fevers, such as through the
studied of these conditions result from mutations that cause activation of the signaling molecule nuclear factor rB or
enhanced function of inflammasomes. multimolecular com, through the unstimulated production of interferons.
plexes that raise the levels ofactivated interleukin 1p. To the extent possible, treatments address the signaling
In familial Mediterranean fever, mutations of pyrin, a problem involved. Table 43 lists the features and treatment of
protein, lead to inflammasome activation, causing fever, select autoinfl ammatory diseases.

TABLE 43. Features and Treatment of Select Autoinflammatory Diseases


FMFA TRAPS FCA5U MWSb NOMID/ Schnitzler Syndrome
ctNcAb
lnheritance AR AD AD AD AD Acquired
Gene MEFV TNFRSFlA NtRP3 NtRP3 NLRP3 Unknown
Protein furin TNF receptor Cryopyrin Cryopyrin Cryopyrin MGUS (hallmark but
type 1 not proven causal)
Age at onset 65% <1 0 years 50% <1 0 years <1 year of age Childhood Neonata I Adult(51 +10years)
o{ age;907"<20 of age; up to period
years of age fifth decade
Ethnicity Mediterranean Ail European European All White/European
Clinical
Ma nifestations

Attack duration 12-72 hours Days to weeks 12-24 hours 1-2 days Continuous Daily
Abdominal Pain; serositis Pain; serositis Nausea Pain

Pleuritis Common Common Rare Rare

M usculoskeleta I Monoarthritis Large joints Arthralgia Arthralgia; Epiphyseal Arthralgia; arth ritis;
of lower oligoarthritis overgrowth; bone pain
extremities co ntractu res

Rash Erysipeloid on Migratory with Cold- Urticaria-like U rticaria-like Urticarial rash


lower legs underlying induced;
myalgia urticaria-like
Oth er High risk for Conjunctivitis; Conjunctivitis; Sensorineural Sensorineural Leukocytosis;
amyloidosis periorbital headache; deafness; deafness; lymphadenopathy;
edema amyloidosis; conjunctivitis; aseptic hepatosplenomega ly;
sensorineural amyloidosis meningitis; MGUS (usually lgM r);
deafness mental may later develop
retardation; Waldenstrom
amyloidosis macrog lobu linem ia
Treatment Colchicine Glucocorticoids;
TNF inhibitors
lL-1 0
inh ibition
rL 1p
inhibition
lL-1 B
inh ibition
lL-1Ra or lL-1B
inhibition
AD=autosoma dominant;AR=autosomalrecessive;CINCA=chronicinfantileneurologic,cutaneous,articularsyndrome;FCAS=familialcoldautoinflammatorysyndrome;

syndrome;NOMID=neonatalonsetmultisysteminflammatorydisease;TNF=tumornecrosisfactor;TRAPS=tumornecrosisfactorreceptor associatedperiodicsyndrome.

i "FMF is the most common autoin{lammatory disease.

I disease (chronic infantiie neurologic, cutaneous, afticuiar syndrome).

89
Other Rheumatologic Diseases

rtY Potxl multiple organs and tissues. Although sarcoidosis most com
o Autoinflammatory monly affects the lungs (see MKSAP 19 Pulmonary and
diseases are characterized by epi-
Critical Care Medicine), approximately 50% of patients have
sodic and/or persistent inflammation and involve
extrapulmonary involvement, arrd 2"/,, have only extrapulmo
inherited or spontaneous mutation ofgenes that encode
nary involvement. The cause of sarcoidosis is unknown; the
for inflammatory responses.
primacy of lung involvement suggests an initial respiratory
exposure. ln the United States, Black persons are affected
more commonly and more severely than White persons.
Adult-Onset Still Disease Rheumatologic manifestations of sarcoidosis include
Adult onset Still disease is a rare systemic condition (preva arthritis involving multiple joints that may develop erosive
lence, 1 10 cases/l million) characterized by fever with daily damage, deformity, and dactylitis. Myopathy may also occur.
evening spikes, evanescent salmon colored macular or macu Bone involvement, reported in up to 15'X, of patients. is associ
lopapular rashes that peak with the fever, arthritis or arthral ated r,r,ith a chronic course and poorer prognosis (Figure 60).
gia, and elevated leukocyte counts. Other manifestations
include sore throat; lymphadenopathy; splenomegaly: and
inflammation of the lungs, heart, and liver.
The Yamaguchi criteria fbr adult-onset Still disease con-
sist of tbur major and four minor criteria. Diagnosis requires
the presence of at least five criteria, including at least two
major criteria. Major criteria are (1) daily evening spiking
fever to temperature of 39.0 'C (102.2 "F), (2) arthralgia/
arthritis fbr more than 2 weeks, (3) nonpruritic salmon
colored macular/maculopapular rash on trunk or extremities,
and (4) leukocyte count greater than 10.000/pL (10 x 10e/L),
with more than B0'2, neutrophils. Minor criteria are (t) lym
phadenopathy and/or splenomegaly, (2) elevated aspartate
aminotransferase, alanine aminotransferase, or lactate dehy
drogenase levels, and (3) negativig for antinuclear antibodies
or rheumatoid factor.
Erythrocyte sedimentation rate and C reactive protein
levels are elevatedl I'erritin levels are often extremely high.
Adult onset Still disease is thought to represent an overex
pression ofinterleukin-lB and other cytokines, and as such it
is sometimes conceptualized as a polygenic autoinflammatory
syndrome. The differential diagnosis is broad, including infec-
tious. malignant, and systemic autoimmune diseases. Adult-
onset Still disease may present as a single episode; be
recurrent; or, most commonly, be chronic and progressive
(zs"t, ol"t, of aflected patients). Past treatment options,
including glucocorticoids and methotrexate, were frequently
inadequate. The use of interleukin-B directed biologic thera
pies, including anakinra and canakinumab, has been success
ful. Anti-interleukin 6 therapies have shown promise in
clinical reports.
I(EY POItrI
. Adult-onset Still disease is characterized by fever with
daily evening spikes, evanescent salmon-colored rash,
arthritis, and leukocytosis.

Sarcoidosis
Sarcoidosis is an inflammatory disease of unknown cause F IGUR E 6 0. Hand radiograph showing the typical cystic lesions (red anow) and
characterized by fbrmation of noncaseating granulomas in a lacy pattern (green arrow\ characteristic of sarcoidosis.

90
Other Rheumatologic Diseases

Liver involvement is common but frequently asymptomatic:


more serious involvement includes central nervous system,
eye, and cardiac disease. Central nervous system sarcoidosis
most commonly presents as brainstem involvement with cra
nial nerve deficits, but any part of the brain may be involved.
Silent cardiac involvement occurs in at least 25',/, of cases:
cardiac sarcoidosis may also cause symptomatic arrhythmias
or heart failure.
Cutaneous sarcoidosis is scen in approximately 30'1, of
patients. The spectrum of skin involvement is broad, includ
ing asymptomatic or pruritic plaques or papules, often viola-
ceous, an),r,vhere on the body. Lr-rpus pernio is a specific lbrm
of sarcoidosis that manif'ests as violaceous infiltrative subcu
taneous plaques or nodules on the nasal tip and central face
(Figure 61).
f IGU RE 62. Erythema nodosum mani{ests as painful, red-brown nodules on the
The diagnosis of sarcoidosis can be confirmed through
anterior shins.
tissue biopsy specimens showing noncaseating granulomas.
All patients with sarcoidosis should undergo ECG; furtl.rer
evaluation of systemic sarcoidosis should be defined by Serum angiotensin converting enzyme levels are frequently
apparent signs and symptoms. Laboratory studies should elevated and may support the diagnosis, but they are
include inflammatory markers. calcium, and vitamin D.
nonspecific. Cardiac assessment may involve echocardiogra
ph),: advanced cardiac imaging, and/or electrophysiologic
monitoring.
The treatment of symptomatic extrapulmonary sar
coidosis almost always requires high-dose glucocorticoids
(e.g., prednisone, 40 60 mg/d). If glucocorticoids are inef
fective or contraindicated or resist tapering, patients may
be treated with thalidomide, methotrexate, or azathio
prine. Cyclophosphamide or tumor necrosis factor inhibi
tors may be warranted in severe cases or when essential
organs are at risk. Asymptomatic extrapulmonary sar
coidosis does not always require treatment. Sarcoidosis
limited to skin may respond to topical or intralesional
agents or hydroxychloroquine or may require more aggres
sive systemic treatment.
L6fgren syndrome is a form ol systemic sarcoidosis
characterized by bilateral hilar lymphadenopathy, erythema
nodosum (Figure 62), and migratory polyarthralgia, usually
accompanied by fever. Ankle arthritis may also occur. It is
more common in younger patients, women, and Europeans
ol Scandinavian descent. When alt three parts of the triad
are present, there is a 95'1, diagnostic specificity fbr sar
coidosis, obviating the need for biopsy. Because L0fgren
syndrome is usually sell limited, treatment focuses on
symptom management, often with an NSAID, until the con-
dition resolves.

l(EY P0tt{Ts
. Rheumatologic manifestations of sarcoidosis include
arthritis involving multiple joints that may develop ero
sive damage, deformity, and dactylitis; bone disease;
and myopathy.
t IGU R E6 I . Lupus pernio manifests as violaceous infiltrative subcutaneous (Conttnued)
plaques or nodules.

91
Other Rheumatologic Diseases

XEY P0lilTS (onttnud) I(EY POIXI


o The treatment of slmptomatic extrapulmonary sarcoido- . The conditions comprising IgG4-related disease are
sis almost always requires high-dose glucocorticoids. characterized by tumefactive infiltration of IgG4-bearing
. L0fgren syndrome is a form of systemic sarcoidosis that plasmablasts and other lymphocl'tes, along with stori-
consists of bilateral hilar lymphadenopathy, erythema form fibrosis, into one or multiple organs, with resultant
nodosum, and migratory polyarthralgia, usually accom- organ enlargement, fibrosis, and dysfunction.
panied by fever.
Genetic Diseases of
lgG4-Related Disease Connective Tissue
Mutations in genes encoding for collagen, fibrillin, and other
IgG4 related disease (lgGa-RD) is characterized by tumefac
components of connective tissue can lead to poorly functional
tive infiltration of IgG4 bearing plasmablasts and other lym-
skin and integument. Common clinical findings include skin
phocytes, along with storiform fibrosis, into one or multiple
and joint laxity, skeletal fractures, and cardiovascular complica
organs, with resultant organ enlargement, fibrosis, and dys-
tions related to involvement of the aorta and cardiac valves. The
function. Although initial reports of IgG4-RD concerned
three most common syndromes are Ehlers-Danlos syndrome,
involvement of the pancreas (see MKSAP 19 Gastroenterologr
Marfan syndrome, and osteogenesis imperfecta. Table 45 sum
and Hepatolory), a range of organs can be involved (Table aa).
marizes clinical findings and management of these syndromes.
In 80'1, ofcases, serum levels oflgG4 antibodies are elevated.
Vascular monitoring is necessary in Marfan syndrome and
The signs and symptoms ol IgG4 RD depend on the organ or
several types of Ehlers-Danlos syndrome. Patients with Marfan
organs affected. Pathogenesis remains inconclusive, with
syndrome should undergo echocardiography to assess the aor
uncertainty about whether plasmablasts are a cause or a
tic root and ascending aorta at the time of diagnosis and
consequence of the disease. An American College of
6 months later to ensure stability. Follow-up studies should be
Rheumatologr/European League Against Rheumatism clas
completed at least annually; MRI and CT are often used to fur-
sification process for identi$zing IgG4-RD involves the follow
ther delineate risk. Patients with Marfan syndrome and Ehlers
ing: (1) identifying characteristic clinical or radiologic organ
Danlos syndrome should undergo expert consultation. See
involvement and/or a suspicious biopsy specimen; (2) exclud
MKSAP 19 Cardiovascular Medicine for more information.
ing alternative diseases; and (3) confirming the diagnosis on
the basis of characteristic pathologic findings, usually includ IEY POITTS
ing IgG4 expression on biopsy tissue or in serum. Initial . Common clinical findings of genetic diseases of connec-
management of IgG4-RD includes moderate to high-dose tive tissue include skin and joint laxity, skeletal fractures,
glucocorticoids for 4 weeks, followed by a slow taper over 1 or and cardiovascular complications related to involvement
more years. Azathioprine is preferentially used as a glucocor of the aorta and cardiac valves.
ticoid-sparing agent, and rituximab has been reported to have . Regular vascular monitoring is required for patients
significant efficacy. In some cases, surgical intervention may
with Marfan syndrome and several types of Ehlers-Danlos
be needed (e.g., to address a mass jeopardizing a particular
syndrome.
organ).

TABLE 44. Select Conditions Associated With lgG4-Related Disease


Condition Organ lnvolved
Hypertrophic pachymeningitis Dura mater
Lymphocytic hypophysitis Pituitary gland
ldiopathic orbital inflammatory disease Periorbital often with involvement of the ocular adnexal tissues
Mikulicz disease Parotid glands
Dacryoadenitis Lacrimal glands
Kiittner tumor Submandibular glands
Riedel thyroiditis Thyroid
lnflammatory aortitis/vasculitis Aorta/arteries, often associated with retroperitoneal fibrosis
Autoimmune pancreatitis Pancreas
Ormond disease (retroperitoneal fibrosis) Periaortic mass
Tubulointerstitial nephropathy Kidneys
Sclerosing cholangitis Biliary tract

92
Bibliography
I

Ehlers-Danlos Syndrome
Hypermobility Classic Kyphoscoliotic Vascular Marfan Osteogenesis
Syndrome Imperfecta
Genetic Testing Gene is COLSA|;COLSA2 PLOD| caL3A1 FBNl CAL
Available unknown
ln heritance AD AD AR AD AD AD orAR
Clinical Findings
Musculoskeletal Joint laxity; joint Hypermobility; Hypermobility; Hypermobility; Hypermobility; Bone fractures;
instability; joint dislocations; progressive joint tall stature; short stature
musculoskeletal pes planus scoliosis; dislocations d o lich oste no m e lia u;
pain; early marfanoid arachnodactyly;
osteoa rth ritis habitus pectus excavatum/
carinatum; scoliosis
Skin Easy bruising Smooth, velvety; Easy bruising; Thin; Hyperextensibility
mild laxity easy bruising; hyperextensibility translucent;
hyperextensibility; easy bruising
striae atrophicae
(widened stretch
marks)
Cardiovascular No risk for organ Mitral/tricuspid Medium-size Arterial Aortic aneurysm/
or
rupture valve prolapse; arterial rupture; ru ptu re; dissection; mitral
dissection aortic root dilation mitra l/tricuspid aneurysm; valve prolapse
(rare); arterial valve prolapse dissection
rupture (rare)
Other Muscular Muscular Organ rupture Myopia (most Dentinogenesis
hypotonia; hypotonia; scleral (uterus, bowel, common feature imperfectab;
delayed motor {ragility with risk rarely spleen, [60%]); ectopia hearing loss;
i
development for globe liver); lentis; high arched blue sclerae
rupture; pneumothorax; palate;
restrictive lung gingival pneumothorax;
disease; recession blue sclerae
recurrent
pneumonia;
heart failure
Managementc Joint protection; Joint protection; Joint protection; Joint Annual ophthalmic Bisphosphonates;
supportive care echo/vascular echo/vascular protection; examination; audiology
monitoring; monitoring; echo/vascu lar periodic aortic assessments;
preconception preconception monitoring; arch imaging; joint and bone
counseling counseling; preconception B-blockersd; joint protection; dental
physical therapy; counseling protection; evaluations
bracing pregnancy
counseling/
monitoring
AD = autosomal dominanU AR = autosomal recessive.

"Dolichostenomelia describes when the extremities are disproportionately long for the size oI the trunk.
bDentinogenesis imperfecta is de{ined as discolored (blue/gray or yellow/brown), translucent, weak teeth prone to breakage.

'Genetic consultation is appropriate for all listed conditions.


dMay slow the rate of aortopathy.

Bibliography Maher L, Perugino C. Diagnostic pitfalls in immunolory testing. Clin Lab Med.
2Ol9 ;39:567 578. [PMID: 3166827O) doi:10.1016/j.cII.2019.07.005
Approach to the Patient with Rheumatologic Disease
Manson lJ, Wing C. The rheumatological history Medicine. 2018;46:156 160.
Al-Shakarchi l, Coakley G. Synovial fluid tests. Medicine. 2018:46:166-169. doi.orgi 10.1016/i.mpmed.2017.12.008
https: /,'doi.org/ 10.101 6/j.mpmed.2017.12.001
I
Mulhearn B, Tansley SL, McHugh NJ. Autoantibodies in connective tissue dis
Batten R, Coady D. The rheumatological examination. Medicine. 2O18;46r161- ease. Best Pract Res Clin Rheumatol. 2019:101462. [PMID: 318480551
165. doi.org/10.1016/j.mpmed.2017.12.002 doi: 1 0. 1016/j.berh .2019.101462
Burge AJ, Nwawka OK, Berkowitz JL, et al. Imaging of inflammatory arthritis Price JD, Barbour KE, Liu Y et al. State specific prevalence and characteristics
in adults: status and perspectives on the use of radiographs, ultrasound, of frequent mental distress and history of depression diagnosis among
and MRl. Rheum Dis Clin North Am. 2016;42:561-585. [PMID: 27742015) adults with arthritis - United States, 2017. MMWR Morb Mortal Wkly Rep.
doi:1O.1016/j.rdc.2016.07.001 2O2O;68:1173 1178. IPMID: 31895918] doi:10.15585/mmwr.mm685l52al

93
I
Bibliography
)

Singh N, Vogelgesang SA. Monoarticular arthritis. Med Clin North Am. Stoffer MA, Schoels MM, Smolen JS, et al. Evidence for treating rheumatoid
2Ol7 ;lOl$O7 - 613. IPMID : 2a3727161 doi:10.1016/j.mcna.2016.12.0O4 arthritis to target: results of a systematic literature search update. Ann
Warburton L, Hider SL, Mallen CD, et al. Suspected very early inflammatory Rheum Dis. 2016:75:16 22. [PMID: 259902901 doi:10.1136'annrheumdis-
rheumatic diseases in primary care. Best Pract Res Clin Rheumatol. 2015 207526
2O19;33:101419. IPMID: 31810550] doi:1O.1O16/j.berh.2019.06.001
Osteoarthritis
Principles of Therapeutics Bannuru RR. Osani MC, Vaysbrot EE, et al. OARSI guidelines for the non-
Buckley L, Guyatt C, Fink HA, et al. 2017 American College of Rheumatolog/ surgical management of knee, hip, and polyarticular osteoarthritis. :

guideline for the prevention and treatment of glucocorticoid induced OsteoarthritisCartilage.20lg;27:1578 1589.IPMID:31278997]doi:10.1016/j.
osteoporosis. Arthritis Care Res (Hoboken). 2017:69:1095-1110. [PMID: joca.2019.06.0u
285854101 doi:10. 1OO2 I acr.23279 Busse JW. Wang L. Kamaleldin M, et al. Opioids for chronic noncancer pain: a t
da Costa BR, Reichenbach S, Keller N. et al. Effectiveness ofnon-steroidal anti systematic review and meta analysis. JAMA. 2018;320:2448-2460. IPMID:
inflammatory drugs tbr the treatment of pain in knee and hip osteoarthritis: 305614811 doi:10.1001 /jama.2018.18472
I
a network meta-analysis. Lancet. 2017;390:e21 e33. [PMID: 28699595] da Costa BR. Reichenbach S, Keller N, et al. Effectiveness ofnon steroidal anti-
doi:10.1016/S0140 6736(17)3t744-O inllammatory drugs for the treatment of pain in knee and hip osteoarthri
I-'itzcerald JD, Dalbeth N, Mikuls T. et al. 2020 American College of tis: a netlvork meta analysis. Lancet. 2017:390:e2l e33. [PMID: 28699595]
Rheumatologr guideline for the management of gout. Arthritis Care Res doi:10.101 6/50140 6736(17)31744 0
(Hoboken). 2O2O:72:744-760. IPMID: 32391934] doi:10.1002/acr.24180 Ftrnandes L, Hagen KB, Bijlsma Iw, et ali Europ€an League Against Rheumatism
Furer V, Rondaan C, Heijstek MW. et al. 2019 update ofEULAR recommenda- (EULAR). EULAR recommendations for the non-pharmacological core
tions for vaccination in adult patients with autoimmune inflammatory management of hip and knee osteoarthritis. Ann Rheum Dis. 2Ol3:72:ll2S
rheumatic diseases. Ann Rheum Dis. 2020:79:39-52. [PMID: 31413005] 35. IPMID: 23595142] doi:10.11361annrheumdis 2ol2-2o2745
doi:10.1136/annrheumdis 2019-215882 Coh SL. Persson MSM, Stocks J, et al. Efficacy and potential determinants of
Geenen R, Overman CL, Christensen R, et al. EULAR recommendations for the exercise therapy in knee and hip osteoarthritis: a systematic review and
health professional's approach to pain management in inflammatory meta-analysis. Ann Phys Rehabil Med. 2019162:356 365. [PMID: 31121333]
arthritis and osteoarthritis. Ann Rheum Dis. 2078:77:797-807. [PMID: doi:10.1016/j.rehab.2019.04.006
297 247 26) doi:lO.1136/annrheumdis-2o17 -212662 Kloppenburg M, Kroon FP Blanco FJ, et al. 2018 update of the EUIAR recom
Kim H, Alten R, Avedano L, et al. The future of biosimilars: maximizing mendations for the management of hand osteoarthritis. Ann Rheum Dis. i
benefits across immune-mediated inflammatory diseases. Drugs. 2019:78:16 24. [PMID: 30154087] doi:l0.ll36rannrheumdis 2o18 213826
2020:80;99 113. IPMID: 32002851] doi:10.1007's40265 O20-O1256 5 Kolasinski SL, Neogi T, Hochberg MC. et al. 2019 American College of
Kolasinski SL, Neogi T, Hochberg MC, et al. 2019 American College of Rheumatolory/Arthritis Foundation guideline for the management of oste
Rheumatolos//Arthritis Foundation Cuideline for the Management of oarthritis ofthe hand, hip, and knee. Arthritis Rheumatol. 2O2Q:72:22O
Osteoarthritis of the Hand, Hip, and Knee. Arthritis Rheumatol. 233. IPMID: 31908163] doi: 10. 1OO2 t art.41142
2020 ;72:220 233. IPMID: 31908163] doi:lO.1O02/art.41142 Leopoldino AO, Machado CC, Ferreira PH. et al. Paracetamol versus placebo :

Mielenz TJ, Xiao C, Callahan LF. Self-management of arthritis symptoms by for knee and hip osteoarthritis. Cochrane Database Syst Rev. 2019;2:
complementary and altemative medicine movement therapies. J Altern CD013273. [PMID: 3O8Oll33] doi:1O.1002/1.1651858.CD013273
Complement Med. 2016122:404 7. IPMID: 27058260) doi:10.1089/acm.
2015.0222
Siemieniuk RAC, Harris IA, Agoritsas T. et al. Arthroscopic surgery for degen
erative knee arthritis and meniscal tears: a clinical practice guideline. BMJ.
i
Singh JA, Saag KG, Bridges SL Jr, et al. 2015 American College ofRheumatolos/ 2017 :357 :j79a2. IPMID: 28490431] doi:10.1136rbmj.j1982
guideline for the treatment of rheumatoid arthritis. Adhritis Rheumatol. Solomon DH, Husni ME, Libby PA, et al. The risk ol'major NSAID toxicity with
2076;681-26. IPMID: 26545940] doi:10.1002/art.39480 celecoxib, ibuprofen. or naproxen: a secondary analysis ofthe PRECISION
Smolen lS, l:ndewd RBM, Bijlsma JWJ, et al. EULAR recommendations for the trial. Am J Med. 2017;130:t415-t422.e4. IPMID: 287562671 doi:10.1016rj.
management of rheumatoid arthritis with synthetic and biological disease amjmed.2017.06.028
modiffing antirheumatic drugs: 2019 update. Ann Rheum Dis. 202o; rr
Wang C, Schmid CH, Iversen MD, et al. Comparative effecti!'eness of tai chi
79 :685- 699. IPM ID: 31969328] versus physical therapy for knee osteoarthritis: a randomized trial. Ann
White WB, Saag KG, Becker MA, et al; CARES Investigators. Cardiovascular Intern Med. 2016:165:77-86. IPMID: 27183035] doirl0.7326'M15 2143
I
safety of febuxostat or allopurinol in patients with gout. N Engl I Med. Zeng C, Dubreuil M, LaRochelle MR, et al. Association of tramadol with
2O18;378: 1200-1210. IPMID: 29527974) doi:10.1056/NUMoal7l0895 all-cause mortality among patients with osteoarthritis. JAMA. 2019;321:
ofoppor
Yang SC, Lai YY, Huang MC, et al. Corticosteroid dose and the risk 969 982. [PMID: 308605591 doi:10.1001 /jam a.2019.1347
tunistic infection in a national systemic lupus erythematosus cohort.
Lupus. 2O18i27:1819-1827. [PMID: 3010346] doi:10.1177 t 09612O331a792352 Fibromyalgia
Arnold LM, Bennett RM. Crofford U, et al. AAPT diagnostic criteria for fibro-
Rheumatoid Arthritis myalgia. I Pain. 2019120:611 628. [PMID: 30453109] doi:10.1o16ij.jpain.
Aletaha D, Neogi T, Silman AJ, Funovits J, et al. 2010 Rheumatoid arthritis clas- 2018.10.008
sification criteria: an American College of Rheumatolos//European League DuffieldSJ, Miller N. Zhao S, et al. Concomitant fibromlalgia complicating
Against Rheumatism collaborative initiative. Arthritis Rheum. 201O;62: chronic inflammatory arthritis: a systematic review and meta analysis.
2569 81. IPMID: 20872595] doi:10.1002/art.27584 Rheumatologr (Oxford). 2018;57:1453 i460. IPM ID: 29788461] doi:10.1093/
Blum A, Adawi M. Rheumatoid arthritis (RA) and cardiovascular disease. rheumatolory/key112 i
Autoimmun Rev 2019;18:679 690. IPMID: 31059840] doi:10.1016/j. Macfarlane GJ, Kronisch C, Dean LE, et al. EULAR revised recommendations I
autrev2019.05.005
for the management of fibromyalgia. Ann Rheum Dis. 2017:76:318 328.
Scher JU. Littman DR. Abramson SB. Microbiome in inllammatory anhritis IPMID: 27377815] doi:l0.ll36iannrheumdis 2016 20972.1
I
and human rheumatic diseases. Arthritis Rheumatol. 2016:68:35 ,15. Wang C, Schmid CH, Fielding RA. et al. Effect of tai chi versus aerobic exercise
IPMID: 263315791 doi:10.1002/art.39259
Singh JA, Saag KG, Bridges SL Jr, Akl EA, Bannuru RR, Sullivan MC, et al. 2015
fbr fi bromyalgia: comparative eflectiveness randomized controlled trial.
BMl. 2018;360:k851. IPMID: 295631O0] doi:10.1136/bmj.k851
)
!
American College of Rheumatologr guideline for the treatment of rheuma
Wolfe F, Clauw DJ, Fitzcharles MA, et al. 2016 Revisions to the 2010i 2011 fibro
toid arthritis. Arthritis Rheumatol. 2016;68:1 26. IPMID: 26545940] myalgia diagnostic criteria. Semin Arthritis Rheum. 2O16:,16:319-329.
doi:10.1002/art.39480
IPMID : 279762781 doi:10. lO16 ij.semarthrit.20l6.08.012
Smolen lS, Landewe RBM, Bijlsma JWJ, et al. EUIAR recommendations for the
management of rheumatoid arthritis with synthetic and biological disease Spondyloerthritis
t
modi$iing antirheumatic drugs: 2019 update. Ann Rheum Dis. 2020;79:685- Fragoulis GE, Liava C, Daoussis D, et al. Inflammatory bo\ €l diseases and I
699. [PMID: 31969328] doi:10.1136tannrheumdis 2Ol9 -21665s spondyloarthropathies: pathogenesis to treatment. World J
from
Spagnolo Il
Lee JS. Sverzellati N, et al. The lung in rheumatoid arthritis: fbcus Castroenterol. 2O19;25:2162 2176. [PMID: 31143068] doi:10.3748/wig.v25. q
on interstitial lung disease. Arthritis Rheumatol. 2018;7O:1544 1554. i18.2162 i
IPMID: 298060921 doi:10.1002/art.40574 Ghosh N, Ruderman EM. Nonradiographic axial spondyloarthritis: clinical and 1

Sparks JA. Rheumatoid arthritis. Ann Intern Med. 2019117O:lTC1 ITC16. therapeutic relevance. Arthritis Res Ther. 2Ol7:19:286. [PMID: 29273055]
IPMID: 3o5968791 doi:10.7326lAITC20l901olo doi:10.U86/s13075 017 1493-8

94 ;

'l
I

t
Bibliography

Jhaj G, Kopplin U. Ocular features of the H t-A B27 positive seronegative spon Ramos Casals M, Brito-Zerdn P, Bombardieri S, et al; EULAR Sjogren
dyloarthropathies. Curr Opin Ophthalmol. 2018;29:552-557. IPMID: Syndrome Task Force Group. EULAR recommendations for the manage
\ 301487 241 doi:10.1097/1CU.0000000000000525 ment of Sjogren's syndrome with topical and systemic therapies. Ann
l.iew JW Ramiro S, Gensler LS. Cardiovascular morbidity and mortality in :
Rheum Dis. 2O2O:79 :3-18. IPMID 316727751 doi:10.1136/annrheum-
I ankylosing spondylitis and psoriatic arthritis. Best Pract Res Clin dis-2019-216114
1
Rheumatol. 2018;32:369 389. IPMID: 31171309] doi:10.1016/j.berh. Sandhya B Kurien BT, Danda D, et al. Update on pathogenesis of Sjogren's
2019.01.002 syndrome. Curr Rheumatol Rev 201 7; I 3 :5 22. [PMID : 27 4726021 doi:lo.217 4 I
\ Maksymowych WP The role of imaging in the diagnosis and management of' I 57 3397 [26 6 67 6 07 1 476 4t49
L axial spondyloarthritis. Nat Rev Rheumatol. 2019;15:657 672. [PMID: Shiboski CH, Shiboski SC, Seror R, et al; International Sjogren's Syndrome
t
315915001 doi:10.1038/s41584-019 0309 4 Criteria Working Group. 2016 American College of Rheumatolos//
L
McGonagle D, Tan AL, Watad A, et a1. Pathophysiolos/, assessment and treat European League Against Rheumatism classification criteria fbr primary
ment of psoriatic dactylitis. Nat Rev Rheumatol. 2019;15:113 122. [PMID: Sjogren's syndrome: a consensus and data driven methodolory involving
L 306102191 doi:10.1038/s41584-018 0147 9 three international patient cohorts. Ann Rheum Dis.2O17;76:9 16. [PMID:
27789466) doi:7O.1136/annrheumdis-2016 210571
Pedersen Sl, Weber U, Said Nahal R, et al. Structural progression rate decreases
over time on serial radiogaphy and magnetic resonance imaging ol sacro- Idiopathic Infl ammatory Myopathies
iliac joints and spine in a five-year fbllow-up study of patients with ankry-
losing spondylitis treated with tumour necrosis factor inhibitor. Scand J Amlani A, Choi MY, Tarnopolsky M, et al. Anti NT5C1A autoantibodies as
Rheumatol. 2019;48:185 197. IPMID: 3042278] doi:10.1080/03009742.201 biomarkers in inclusion body myositis. Front Immunol. 2019:10:745.
i IPMID: 31024569] doi:10.3389/f]mmu.2019.00745
t 8.1506822
Cassius C, Le Buanec H, Bouaziz JD, et al. Biomarkers in adult dermatomyosi
I
Ritchtin CT, Colbert RA, Gladman DD. Psoriatic arthritis. N Engl I Med.
tis: tools to help the diagnosis and predict the clinical outcome. J lmmunol
I 2017 :376:957 970. [PMID: 282730191 doi:10.1056/NEJMra1505557
Res. 2019;2019:9141420. [PMID: 3O7668921 doi:10.1155/2019/9141420
Schmitt SK. Reactive arthritis. lnfect Dis Clin North Am.2O17;31:265 277.
IPMID, 282925401 doi:10.1016/j.idc.2017.01.002
Day JA, Limaye V Immune mediated necrotising myopathy: A critical review
of current concepts. Semin Arthritis Rheum. 2019;49:420 429. [PMID:
Singh JA, Guyatt G, Ogdie A, et al. Special article: 2018 American College of 311096391 doi:1o.1016/i.semarthrit.2019.04.oo2
Rheumatolos//National Psoriasis Foundation guideline for the treatment
of psoriatic arthritis. Arthritis Rheumatol. 2019r71:5 32. [PMID: 30499246] Lundberg IE, de Visser M, Werth VP. Classification of myositis. Nat Rev
a

doi: 10.1002/art.40726 Rheumatol. 2018:14:269 278. [PMID: 296511211 doi:10.1038/nrrheum.


2018.41
Ward MM. Deodhar A, Gensler LS, et al. 2019 update of the American College
t of Rheumatolory/Spondylitis Association of America/Spondyloarthritis Lundberg IE, Tjimlund A, Bottai M, et ali International Myositis Classification
I Research and Treatment Network Recommendations tbr the treatment of Criteria Project consortium, The Eurom],'ositis register and The luvenile
i ankylosing spondylitis and nonradiographic axial spondyloarthritis. Dermatomyositis Cohort Biomarker Study and Repository (JDRG) (UK and
I
! Arthritis Rheumatol. 2O1971:1599 1613. [PMID: 31436036] doi:10.1002/ Ireland). 2017 European League Against Rheumatism/American College of
art.41042 Rheumatolory classification criteria fbr adult and juvenile idiopathic
i
I inflammatory myopathies and their major subgroups. Ann Rheum Dis.
Systemic Lupus Erythematosus 2Ol7 :7 6:19 55 1 964. [PMI D: 2907 95901 doi: 10.1 136/annrheumdis 2017
211468
t Aringer M, Costenbader K, Daikh D, et al. 2019 European League Against
Rheumatism/American College of Rheumatolos/ classification criteria for Miller FW Lamb JA, Schmidt J, et al. Risk factors and disease mechanisms in
systemic lupus erythematosus. Arthritis Rheumatol. 2Ol9 :71:14OO 1412. myositis. Nat Rev Rheumatol. 2018;14:255 268. [PMID: 296746131
IPMID: 313854621 doi:10.1002/art.40930 doi:10.1038/nrrheum.2018.48
Chottawornsak N, Rodsaward [] Suwannachote S, et al. Skin signs in juvenile Oddis CV Aggarwal R. Treatment in myositis. Nat Rev Rheumatol. 2018;14:279
and adult-onset systemic lupus erythematosus: clues to different systemic 289. IPMID: 29593343] doi:10.1038/nrrheum.2018.42
involvement. Lupus. 201 8;27:2 069 2O7 5. [PMID: 30336755] doi:10.1177l Qiang lK, Kim WB, Baibergenova A, Alhusayen R. Risk of malignancy in der-
0961203318805851 matom),ositis and polymyositis. J Cutan Med Surg. 2017;21:131 136. [PMID:
Durcan [., O'Dwyer Petri M. Management strategies and future directions for
T, 27 53 47 7 9) doi:Lo.1177 I 1203 47 5 41 6665601

I systemic lupus erythematosus in adults. Lancet. 2019;3932332 2343.


I PM I D: 31180030] doi:10. 1016 /5014 O 67 36(19) 30237 5 Systemic Sclerosis
I
l,anouriakis A, Bertsias C. Changing paradigms in the treatment of systemic Blagoievic J, AlOdhaibi KA, Aly AM, et al. Pregnancy in systemic sclerosis:
I lupus erythematosus. Lupus Sci Med. 2019;6:e000310. [PMID: 311683981 results ofa systematic review and metaanalysis. J Rheumatol. 2020;47881
doi:10.1136/lupus 2018 000310 887. [PMID: 37474594] doi:1O.3899/jrheum.l 81460

i Fanouriakis A, Kostopoulou M, Alunno A, et a|.2019 update ofthe EULAR Distler O, Highland KB, Gahlemann M, et al; SENSCIS Trial Investigators.
i recommendations for the management of systemic lupus erythematosus. Nintedanib for systemic sclerosis associated interstitial lung disease. N
Ann Rheum Dis. 2o19;78:736-745. IPMID: 30926722] doi:10.1136/ Engl J Med. 2019;380:2518 2528. IPMID: 3111n79] doi:10.1056/NEJMoa
t annrheumdis 2019 215089 i903076
Lazzaroni MG, Dall'Ara E, Fredi M, et al. A comprehensive review ol the clinical Hung C, Mercurio V Hsu S, et al. Progress in understanding, diagnosing, and
I
approach to pregnancy and systemic lupus erythematosus. J Autoimmun. managing cardiac complications ofsystemic sclerosis. Curr Rheumatol Rep.
2016:74:1O6 117. IPMID: 273774531 doi:10.1016/j.jaut.2016.06.016 2019:21:68. [PMID: 31813082] doirl0.l007/sI1926 O19 0867-0

Mosca M, Costenbader KH, Johnson SR, et al. Brief report: how do patients McMahan ZH. Gastrointestinal involvement in systemic sclerosis: an update.
with newly diagnosed systemic lupus erythematosus present? A multi Curr Opin Rheumatol. 2019;31:561 568. IPMID: 31389815] doi:10.1o97l
{
center cohort ofearly systemic lupus erythematosus to inform the develop 8OR.00000000000006,1s
t ment of new classification criteria. Arthritis Rheumatol. 2019;71:91 98. Namas R, Tashkin DP Furst DE, et ali Participants in the Scleroderma Lung
I IPMID: 300353651 doi:10.1002/art.40674 Study I and members of the Scleroderma Lung Study ll Research Croup.
I Stoian G, Petri M. Epidemiolog/ of systemic lupus erythematosus: an update. Efficacy of mycophenolate mofetil and oral cyclophosphamide on skin
Curr Opin Rheumatol. 2018;30:144-150. [PMID: 29251660] doi:1o.1097/ thickness: post hoc analyses trom two randomized placebo controlled tri-
I
BOR.0000000000000480 als. Arthritis Care Res (Hoboken). 2018;70:439 444. [PMID: 28544580]
doi:10.1002/acr.23282
Tselios K, Gladman DD, Touma Z, et al. Disease course patterns in systemic
!
lupus erythematosus. Lupus. 2019 :28:114 -122. [PMID: 30526328] doi:10. Sandler RD, Matucci Cerinic M, tlughes M. Musculoskeletal hand involvement
117 7 / 09 6720 33t88t7 732 in systemic sclerosis. Semin Arthritis Rheum. 2020;50:329-334. [PMID:
318123531 doi:10.1016/j.semarthrit.20l9.11.003
Zayat AS, Mahmoud K, Md Yusof MY, et al. Defining inflammatory musculo
t skeletal manifestations in systemic lupus erythematosus. Rheumatologr Sircar G, Goswami RB Sircar D, et al. Intravenous cyclophosphamide vs
(Oxford). 2019;58:304-312. IPMID: 30265343] doi:10.1093/rheumatolo$// rituximab for the treatment ofearly diffuse scleroderma lung disease: open
key277 label, randomized, controlled trial. Rheumatologl (Oxford). 2018;57:2106
t 2113. IPMID: 3OO53212] doi:10.1093/rheumatolos//key213
Sjtigren Syndrome Tashkin DR Roth MD, Clements PJ, et al; Scleroderma Lung Study Il
: Mariette X, Criswell tA. Primary Sjogren's syndrome. N Engl J Med. 2018;378: lnvestigators. Mycophenolate mofetil versus oral cyckrphosphamide
931 939. [PM ID: 29514034] doirlO.l056/N EJMcp1702514 in scleroderma related interstitial lung disease (SLS II): a randomised
1

95
I
Bibliography

controlled, double-blind, parallel group trial. l-ancet Respir Med. 2016; Systemic Vasculitis
4t7o8-719. IPMID: 27469583] doi:10.1016/52213 2600(16)30152-7 Cacoub P Comarmond C, Domont E, et al. Cryoglobulinemia vasculitis. Am J

Valenzuela A, Song P, Chung L. Calcinosis in scleroderma. Curr Opin Med. 2015;128:950 5. IPMID: 25837517] doi:10.1016/j.amjmed.2015.02.017
Rheumatol. 2018;30:554 561. [PMID; 30124603] doi:10.1097/BOR.00000 Chung SA, Langford CA, Maz M, et al. 2021 American College of Rheumatolos/1
00000000539 Vasculitis Foundation guideline for the management of antineutrophil
Zanatta E, Polito P Favaro M, et al. Therapy ofscleroderma renal crisis: state of cytoplasmic antibody-associated vasculitis. Arthritis Rheumatol. 2021;73:
the art. Autoimmun Rev. 2018;17:882-889. [PMID: 300058601 doi:10.1016/j. 1366 1383. [PMID, 3423s894]
autrev.2018.03.012 Chung SA, Gorelik M, Langford CA, et al. 2021 American College of
Mixed Connective Tissue Disease and Undifferentiated Rheumatolory/Vasculitis Foundation guideline for the management of
polyarteritis nodosa. Arthritis Care Res (Hoboken). 2021;173:1061-1070.
C-onnective Tissue Disease
IPMID:3a2358891
Garcia Gonzdlez M, Rodriguez-Lozano B, Bustabad S, et al. Undifferentiated of
Dejaco C, Ramiro S, Duftner C, et al. EULAR recommendations for the use
connective tissue disease: predictors ofevolution into definite disease. Clin
imaging in large vessel vasculitis in clinical practice. Ann RheumDis.
Exp Rheumatol. 2017 Sep Oct;35,739-745. IPMID:287707A41
2Ol8:77 :636-643. IPMID; 29358285] doi:l0.1136iannrheumdis-2017-212649
Gunnarsson R, Hetlevik SO, Lilleby V et al. Mixed connective tissue disease.
Demirkesen C. Approach to cutaneous vasculitides with special emphasis on
Best Pract Res Clin Rheumatol. 2016;30:95 111. [PMID: 274212191
small vessel vasculitis: histopathologr and direct immunofl uorescence.
doi:10.1016/j.berh.2016.03.002
Curr Opin Rheumatol.2OlT;29:39 44. [PMID: 2778n371
Crystal Arthropathies Hellmich B, Agueda A, Monti S, et al. 2018 Update ofthe EULAR recommenda-
tions for the management of large vessel vasculitis. Ann Rheum Dis.
Abhishek A, Doherty M. Update on calcium pyrophosphate deposition. Clin
2O2O:79 :19 30. [PMID, 31270110] doi:10.1136/annrheumdis-2O19 -215672
Exp Rheumatol. 2016 Jul-Aug;34:32-8. IPMID: 27586801]
Jones RB, Hiemstra TF, Ballarin l, et al; European Vasculitis Study Group
Dalbeth N, Merriman TR, Stamp LK. Gout. Lancet. 2016;388:2039-2052. (EUVAS). Mycophenolate mofetil versus cyclophosphamide for remission
IPMID: 271120941 doi:10.1016/S0140 - 6736(16)00346-9
induction in ANCA associated vasculitis: a randomised, non inferiority
Fields TR. The challenges ofapproaching and managing gout. Rheum Dis Clin trial. Ann Rheum Dis. 2019:78:399-4O5. [PMID: 30612116] doi:10.1136/
North Am. 2019;45:145 157. [PMID: 3O4477431 doi:7O.1016/i.rdc.2018.09.009 annrheumdis 2078 -214245
FitzGeraid lD, Dalbeth N, Mikuls T, et al. 2020 American College of Mandal l, Chung SA. Primary angiitis of the central nervous system. Rheum
Rheumatologl guideline for the management of gout. Arthritis C-are Res Dis Clin North Am. 2017;43:503-518. [PMID: 29061238] doi:10.1016/j.
(Hoboken). 2o2o;72:744 760. IPMID: 32391934] doi:1o.1002/acr.24180 rdc.2017.06.001
RichetteP, Doherty M, Pascual E, et al. 2016 updated EUTAR evidence based Maz M, Chung SA, Abril A, et al. 2021 American College of Rheumatolos/i
recommendations for the management of gout. Ann Rheum Dis. Vasculitis Foundation guideline for the management ofgiant c€ll arteritis and
2077 ;76:29 42. [PMID, 274575741 doi:10.1136/annrheumdis-2016 209707 Takayasu arteritis. Arthritis Rheumatol. 2021;73r1349 1365. [PMID: 3a2358841
Richette E Doherty M, Pascual E, et al. 2ol8 updated European League Against Terrier B, Pagnoux C, Perrodeau E, et al; French Vasculitis Study Group. Long-
Rheumatism evidence based recommendations for the diagnosis of gout. term efficacy of remission-maintenance regimens for ANCA associated
Ann Rheum Dis. 2020;79:31-38. IPMID: 3116758] doi:10.1136/annrheumdis vasculitides. Ann Rheum Dis. 2018:77:1150-1156. IPMID: 29724729]
2019 215315 doi:10.1136/annrheumdis-2017-212768
Rosenthal AK, ryan LM. Calcium pyrophosphate deposition disease. N Engl J Wechsler ME, Akuthota R Jayne D, et al; EGPA Mepolizumab Study Team.
Med. 2o16;374:2575-84. IPMID: 27355536] doi:1o.1o56/NEJMral5l11l7 Mepolizumab or placebo for eosinophilic granulomatosis with polyangiitis.
Stanway J, Marianayagam T, Ellis S. Crystal arthropathies. Medicine. 2018; N Engl J Med.2017g76:1921 1932. [PMID: 28514601] doi:10.1056/NUMoa
46:181 6. doi:10.1016/j.mpmed.2017.12.003 1702079

U.S. Food and Drug Administration. FDA adds Boxed Warning for increased Other Rheum.tologic Diseases
risk of death with gout medicine Uloric (febuxostat). Accessed March 6,
Feist E, Mitrovic S, Fautrel B. Mechanisms, biomarkers and targets for adult
2019. Available atr https://www.fda.gov/Drugs/Drugsafety/ucm631182.htm
onset Still's disease. Nat Rev Rheumatol. 2018;14:603-618. IPMID:
302180251 doi:1O.1O38/s41584 018 0081 x
Infectious Arthritis
Harapas CR. Steiner A, Davidson S, et al. An update on autoinflammatory
Baud D, Gubler DJ, Schaub B, et al. An update on Zika virus infection. lrncet. diseases: inflammasomopathies. Curr Rheumatol Rep. 2018;20:40. IPMID:
2Ol7 ;39O:2O99 -2109. [PMIDr 28647173] doi:10.1016/SO14O 6736(17)31450-2
298468191 doi:10.1007/s11926 018 0750 4
Beam E, Osmon D. Prosthetic joint infection update. Infect Dis Clin North Am. Hatemi G, Christensen R, Bang D, et al. 2018 update ofthe EULAR recommen
2018;32:843 859. IPMID: 30241717] doi:10.1016/i.idc.2018.06.005 dations for the management of Behqet's syndrome. Ann Rheum Dis.
Hassan AS, Rao A, Manadan AM, et al. Peripheral bacterial septic arthritis: 2Ol8;77:808-818. [PMID: 29625968] doi:10.1136iannrheumdis-2018 213225
review ofdiagnosis and management. J Clin Rheumatol. 2017:23:435-442. Hatemi G, Mahr A, Ishigatsubo Y et al. Trial of apremilast for oral ulcers in
IPMID: 289264601 doi:10.1097/RHU.0000000000000588 Behqet's sl,ndrome. N Engl J Med. 2019:381:1918-1928. [PMID: 31722152]
Hogan JI, Hurtado RM, Nelson SB. Mycobacterial musculoskeletal infections. doi:10.1056/NEJMoa1816594
Infect Dis Clin North Am. 2017;31:369-382. IPMID: 28292541] doi:10.1016/j. Perugino CA, Wallace ZS, Meyersohn N, et al. l"arge vessel involvement by
idc.20U.O1.007 lgG4-related disease. Medicine (Baltimore). 2016;95:e3344. IPMID:
Kapadia BH, Berg RA, Daley JA, Fritz J, Bhave A, Mont MA. Periprosthetic joint 274281811 doi: 10. I 097/ M D. oo000ooo00003344
infection. Lancet. 2016;387:386-394. IPMID: 26135702] doi:1O.1016/SO140- Vitale A, Sota J, Rigante D, et al. Relapsing polychondritis: an update on patho
6736(14)61798 O
genesis, clinical features, diagnostic tools, and therapeutic perspectives. Curr
l^antos PM, Rumbaugh J, Bockenstedt LK, et al. Clinical [tactice Guidelines by Rheumatol Rep. 2016;18:3. IPMID: 26711694] doi:10.10o7/s11926-ol5-0549-5
the Infectious Diseases Society of America (IDSA), American Academy of Wallace ZS, Naden RB Chari S, et al; American College ofRheumatologr/European
Neurologr (AAN), and American College of Rheumatolog/ (ACR): 2020 l€ague Against Rheumatism IgG4 Related Disease Classification Criteria
guidelines for the prevention, diagnosis, and treatment of Lyme disease.
Working Group. The 2019 American College of Rheumatologr/European
Arthritis Rheuma tol. 2027 ;7 3 12- 20. I PMID : 33251716]
l.eague Against Rheumatism classification criteria for IgG4 related diseas€.
Pathak H, Mohan MC, Ravindran V. Chikungunya arthritis. Clin Med (Lond). Arthritis Rheumatol. 2O2O :72i -19. [PMID: 31793250] doi:10.1002/art.41120
2019;19:381-385. [PMID: 31530685] doi:10.7861/clinmed.2O19-0035
West SG. Current management ofsarcoidosis I: pulmonary cardiac. and neu
arthritis of native joints. Infect Dis Clin North Am. 2O17;31:203
Ross JJ. Septic rologic manifestations. Curr Opin Rheumatol. 2018;30:243 248. [PMID:
218. [PMID: 28366221] doi:10.1016/j.idc.2017.01.001 293898281 doi:10.1097/BOR.000000oo00000489

96
t
t
I

t
I
It
t
i
matology Self-Assessment Test o,
I

I
Rheu F
t (l,
i
vt
UI
This self-assessment test contains one-best-answer multiple-choice questions. Please read these directions carefully o
r^
I before answering the questions. Answers, critiques, and bibliographies immediately follow these multiple-choice ut
questions. The American College of Physicians (ACP) is accredited by the Accreditation Council for Continuing
q,
Medical Education (ACCME) to provide continuing medical education for physicians. t

The American College of Physicians designates MKSAP 19 Rheumatology for a maximum of 24 A MA PRA Category 1.
t
Credits'". Physicians should claim only the credit commensurate with the extent of their participation in the activity.

Successful completion of the CME activity, which includes participation in the evaluation component, enables
the participant to earn rp to 24 medical knowledge MOC points in the American Board of Internal Medicine's
Maintenance of Certiflcation (MOC) program. It is the CME activity provider's responsibility to submit participant
completion information to ACCME for the purpose of granting MOC credit.

Eqrn Credits or MOC Points Onltne


To earn CME credits or to apply for MOC points, MKSAP users need to answer at least one of two questions correctly
(earning a score of at least 50%) and click the Submit CME button. Each single MKSAP 19 self-assessment question
qualifles for one quarter of a CME credit hour or ABIM MOC point.

MKSAP 19 Subscribers can enter their self-assessment question answers and submit for CME/MOC in two ways:

1. Users of MKSAP 19 Complete who prefer to use their print books and a paper answer sheet to study and
record their answers can use the printed answer sheet at the back of this book to record their answers. The
corresponding online answer sheets, which are available on the MKSAP 19 Resource Page, may be used to
transcribe answers onto the online answer sheets. Users may then submit their answers to qualif,i for CME
credits or MOC points (see below for information on Opting in for MOC). Users who prefer to record their
answers on a paper answer sheet should save their answer sheet for future use. Users who study with MKSAP
19 print can also submit their answers directly within MKSAP 19 Digital by accessing the self-assessment
questions dashboard and selecting the preferred subspecialty section to begin answering questions.

2. Users of MKSAP 19 Digital can enter their answers within the digital program by accessing the self-assessment
questions dashboard and selecting the preferred subspecialty section to begin answering questions and click-
ing the Submit CME button once they quali$z for CME and are ready to submit. Learners should keep in mind
their yearly CME and MOC deadlines when determining the appropriate time to submit.
Learners' CME/MOC submission progress will be shown on the MKSAP 19 Digital CME/MOC/CPD page.

Optinginfor MOC
MKSAP 19 users can opt in for simultaneous submission of CME and MOC points as they answer self-assessment
questions. To opt in, users will be required to complete a form requesting their name, date of birth, and ABIM
number. The MOC Opt-in Form will be presented upon the user's flrst CME submission and needs to be completed
only once.

97
Directions
Each of the numbered rtems is followed by lettered answers. Select th e ONE lettered answer that is BESI in each case. tl
o,

o
UI
Item 1 joints. There are similar flndings on the thumb and second rrt
flnger on the left hand. Several of the affected proximal o
ta
A 23 year-old man with Behqet syndrome is evaluated for vt
recurrent oral and genital ulcers. He is currently not hav- interphalangeal joints have a slight purple discoloration.
There is a right knee effusion. The second and fourth toes on
ing a flare. The episodes typically last a few days to a few (I,
weeks. He reports no eye or skin symptoms or neurologic the right foot are diffusely swollen, as are the flrst and third tl
disease. He is taking a therapeutic dosage ofcolchicine to toes on the left foot. Two small psoriatic plaques one in the
prevent disease flares but has had three episodes ofpainful umbilicus and one on the scalp are noted.
oral ulcers in the last 6 months during this treatment. He Laboratory evaluation reveals a negative rheumatoid
has received tapering doses of prednisone to control the factor result.
disease during these episodes.
On physical examination, vital signs are normal. There Which of the following is the most likely diagnosis?
are no active mucocutaneous ulcerations. Healed scars in (A) Osteoarthritis
the scrotal area are present. Joints are not warm or swollen.
(B) Psoriaticarthritis
Laboratory evaluation reveals a blood C-reactive
protein level of 1.2 mg/dl (tz mglL) .
(C) Seronegativerheumatoidarthritis
(D) Systemic lupus erythematosus
Which of the following is the most appropriate
treatment?
(A)
(B)
Apremilast
Ixekizumab
Item 4
A 67-year-old woman is evaluated in the hospital fbr mal tr
aise. fatigue, arthralgia, and rash of B weeks' durationl
(C) Leflunomide dry cough and sinus congestion ol6 weeks' dura[ion; and
(D) It4ycophenolatemofetil painless eye redness and dyspnea that began several days
ago.
oC
On physical examination, temperature is 38.2
Item 2 (100.8 'F), blood pressure is 122176 mm Hg, pulse rate is
A 23 year old man is evaluated for fever, abdominal pain, 104/min, respiration rate is 24lmin, and oxygen saturation
rash, and arthritis of the right knee of 3 days' duration is 92'li with the patient breathing ambient air. Bilateral,
that resolved 1 week ago. He has had more than 20 similar localized ocular injection is seen. Scattered rhonchi are
episodes, the last three occurring in the past year. The flrst heard on lung auscultation. and petechiae and purpura are
episode occurred at age 5 years and presented as abdominal visible on the legs.
pain; the patient underwent appendectomy but no appen Laboratory studies:
dicitis was found. His paternal grandfather and maternal Erythrocyte 87 mm/h
grandmother had a similar syndrome. sedimentation rate
Physical examination flndings, including vital signs, Creatinine 2.1mg/dl (185.6 pmoli L)
are normal. ANCA Pending
Laboratory evaluation shows an erythrocyte sedimen- Urinalysis 3+ blood; 2+ protein; 20-30
tation rate of 23 mm/h, a normal serum creatinine level, dysmorphic erythrocytes/hpfl 5 10
and 1+ protein on urinalysis. Ieukocyteslhpf; erythrocyte casts
Chest CT scan is shown.
Which of the following is the most appropriate
treatment?
(A) Canakinumab
(B) Colchicine
(C) Indomethacin
(D) Prednisone

Item 3
A 50 year oldwomanisevaluatedfora6 monthhistoryof
pain and swelling in the hands, knees, and feet. She reports
t hour of joint stiffness in the morning.
On physical examination, vital signs are normal. The
thumb and second and third flngers on the right hand are
diflusely swollen, with tenderness and swelling at the meta-
carpophalangeal and proximal and distal interphalangeal

99
Self-Assessment Test

tn
.D
l[ Which of the following is the most appropriate diagnostic behavioral therapy, land and aquatic therapy, and tai chi.
ut E testtoperformnext? She also has depression, treated with escitalopram.
UI
(D
CONT.
On physical examination, vital signs are normal. There
vt (A) Kidneybiopsy
is tenderness at multiple soft-tissue sites with soft palpa-
(B) Sinus biopsy
a

tion. All other flndings are normal.


o (C) Skin biopsy
(D) Ihoracoscopic lung biopsy Which ofthe following is the most appropriate additional
(D
vr treatrnent?

Item 5 (A) Duloxetine


(B) Medical cannabis
A 32-year-old man is evaluated forlowbackpain of 6 months'
duration. He experiences back pain at night and back pain (C) Oxycodone
and stifftress in the morning for more than 30 minutes. He (D) Prednisone
feels less back pain when he exercises. He had an episode of (E) Pregabalin
left Achilles tendinitis 4 months ago and continues to have
some pain and swelling at the Achilles insertion.
On physical examination, vital signs are normal. Item 8
Peripheral joints are normal. Lumbar spine flexion is lim- A 56-year-old man is evaluated for a 2-year history of mid
ited, and hip flexion, abduction, and external rotation to lower back pain and stiffness. He reports no buttock pain.
cause discomfort in the buttocks bilaterally. He has tried home-based exercises, with minimal benefit.
Laboratory evaluation reveals normal erythrocyte On physical examination, there is diffuse tenderness
sedimentation rate and blood C-reactive protein level. to palpation of the lower thoracic spine and upper lumbar
Anteroposterior pelvic radiograph shows no evidence spine, with reduced flexion and extension. There is no sa-
ofsacroiliitis. croiliac joint tenderness.
taboratory evaluation reveals a normal blood C-reactive
Which of the following is the most appropriate diagnostic protein level.
test to perform next? Radiograph of the spine is shown. Sacroiliac joint
(A) Additional radiographic views of sacroiliac joints radiographs are normal.
(B) CTofsacroiliacjoints
(C) HLA-B27 antigen testing
(D) MRI of sacroiliac joints
(E) Rheumatoid factor and anti-cyclic citrullinated peptide
antibodies

Item 6
A 2S-year-old woman is evaluated for a 3-year history of
joint pain involving her hands with more than L hour of
morning stiffness. She also has intermittent photosensitive
facial rash, fatigue, and intermittent subjective fever.
On physical examination, vital signs are normal. There
is tenderness to palpation with swelling of the second and
third metacarpophalangeal joints bilaterally.
Complete blood count, serum creatinine level, and urinal-
ysis are normal. Erythroclte sedimentation rate is 35 mm/h.

Which of the following is the most appropriate diagnostic


test to perform next?
(A) Anti-double-stranded DNA antibodies
(B) Antinuclearantibodies
(C) Anti-Ro/SSA, anti-La/SSB antibodies
(D) Anti-Smith antibodies
(E) Anti-Uf-ribonucleoprotein antibodies

Item 7
A 43-year-old woman is evaluated for flbromyalgia. She
continues to be symptomatic with poor sleep qualit5r and
diffuse stiffness despite her participation in cognitive

100
Self-Assessment Test

t
o,
Which of the following is the most likely diagnosis? experienced unintentional weight loss of 3 kg (0.0 lb). the
abdominal pain is poorly localized and unrelated to delcca
(A) Ankylosingspondylitis o
tion. Stools are loose but not greasy, mucoid, or bloody. Shc tr
(B) Diffuse idiopathic skeletal hyperostosis has been taking naproxen twice daily, with good control of' tt
t
(C) Spinal calcium pyrophosphate deposition disease her ankylosing spondylitis. She reports no recent travel or q,
t,l
(D) Spondylosis deformans antibiotic therapy. She has no other medical problems and tt
t:rkes no additional medictrtions.
On physical eramination, vital signs are normal. o
rrl
Item 9 There is mild nonlocalizing abdominal tenderness. Range
A 36 year-old woman is evaluated during a routine well- of motion of cervical and lumbar spine is normal. Sacroiliac
joints are not tender.
ness visit. She is asymptomatic and has no medical prob
lems. She works as an accountant and rarely engages in
physical activity. She drinks a glass of red wine with dinner Which of the following is the most likely diagnosis?
five times weekly. Her diet is rich in protein and starches (A) Celiac disease
and low in fruits, vegetables, and nuts. She has smoked
(B) Inflammatory bowel disease
one pack ofcigarettes per day for 5 years. Her mother has
rheumatoid arthritis, and the patient is concerned that she (C) Irritable bowel syndrome
might also develop the disease. Her only medication is a (D) Small intestinal bacterial overgrowth
combined oral contraceptive.
Physical examination, including vital signs, is normal.
Item 12
Which of the following is most likely to reduce this patient's A 68-year-old man is evaluated for recent onset of mild
risk for developing rheumatoid arthritis? pain and signiflcant swelling of his left knee. He lives in
(A) Alcohol cessation eastern Pennsylvania and is an avid golfer. Several months
(B) Discontinuation of combined oral contraceptive ago he had an illness characterized by an enlarging ery
thematous annular skin lesion in the left popliteal fossa
(C) Increased physical activity
associated with fever, arthralgia, and myalgia that sponta
(D) Mediterranean diet neously resolved after several days.
(E) Smoking cessation On physical examination, vital signs are normal. Joint
examination shows diffuse swelling of the left knee with
minimal warmth but no erythema, slight reduction in full
Item 10 flexion of the knee but pain-free range of motion, and a
A 75-year-old man is evaluated for facial redness and hand, popliteal cyst.
elbow, and knee rash that have progressed over the past Results of serologic testing are positive for Borrelio
6 months. He has no other medical problems and takes no burgdorferi.
medications.
On physical examination, vital signs are normal. There Which of the following is the most appropriate treatment?
is an erythematous, slightly elevated, irregular rash over
(A) Ceftriaxone
both cheeks, forehead, and chin, sparing the nasolabial
folds. Eyelids also appear pink and swollen. There is faint (B) Doxycycline
erythema over the dorsal hands. An indurated rash with (C) Hydroxychloroquine
faint scale is present over the extensor surfaces of the (D) Methotrexate
elbows and knees. Muscle strength and the remainder of
the examination are normal.
Complete blood count, comprehensive metabolic
panel, and serum aldolase and creatine kinase levels are
normal.
Item 13
An 85 year old woman is evaluated ftir a 10 day history ol'
tr
right knee pain and swelling. which is impairing her ability
I to walk and climb stairs. She has a history of similar symp
Which of the following is the most likely diagnosis?
toms over the past 2 years. [iach episode has lasted about
(A) Acute cutaneous lupus erythematosus ll wecks and resolves with rest and ice.
(B) Amyopathic dermatomyositis On physical examination. vital signs are normal. She
(C) Psoriasis has moderate swelling of the right knee, with warmth and
tenderness, and decreased range of motion related to pain.
(D) Rosacea No skin lesions or tophi arc noted.
l-aboratory studies:
Erythrocyte seditnentation rate
tr Item 11
A 36 year old rvoman is evaluated lbr abdominal pain
Cornplete blood count
Comprehensive metabolic panel
65 mmrh
Normal
Normal
and diarrhea. She has a 3 year history of ankylosirtg spon
Urate Normal
dylitis. For the past 6 months. she l-ras had fbur or five
bowel movements daily and over the past month she has Radiograph ofthe right knee is shown (top ofnext page).

101
Self-Assessment Test

vt
(D
? On physical examination, temperature is 37.0 "C
vt (98.6 "F), blood pressure is 2.,101120 mm Hg. pulse rate is
lrt 90i min, respiration rate is ltllmin. and oxygen saturation
(D
ut is 96'1, with the patient breathing ambient air. The skin
art
J
over the hands. forearms, trunk, lower legs, and feet is
.D
tight and appears thickened. The fingers on both hands
show cyanosis. No nuchal rigidity is noted. and flndings on
.D
UI neurologic examination are normal.
laboratory studies:
[]ematocrit .).) t,
Leukoclle count Normal
Platelet count 90,0001pL (90 x 1O'q L)
Creatinine 2.4 mg1dl. (212.2 pmol, L)
Urinalysis 2+ protein; no blood or casts

Peripheral blood smear reveals diminished platelet


numbers and schistocytes.

Which of the following is the most appropriate intravenous


treatment?
(A) Captopril
(B) Cyclophosphamide
(C) Methylprednisolone
(l)) Metoproloi
Wtictr of the following is the most likely diagnosis?
f,l
cour. (A)
(tl)
Acute calcium pyrophosphate crystal arthritis
Gout flare
Item 16
A 78-year-old man is evaluated at a follon'-up visit. Two tr
(C) Inlectiousarthritis r.r,eeks ago, he presented n,ith severe nelv onset head
(D) Rheumatoid arthritis aches, claudicative ja$, pain, and proximal myalgia of
all four limbs. He r.t,as diagnosed with biopsy-confirmed
giant cell arteritis. and prednisone. 60 mg d, was initiated.
Item 14 Since initiation of prednisone. his presenting symptoms
have improved but he has also developed mood su,ings,
A 35 year old woman is evaluated in follow-up for a recent insomnia. significant hypergtycemia. elevated blood pres
diagnosis of ankylosing spondylitis. Despite the use of sure, and dependent edema. His other medical problems
three different NSAIDs and physical therapy, she continues are type 2 diabetes mellitus. hypertension, dyslipidemia,
to have night pain and morning stiffness/pain lasting more and atherosclerotic coronary artery disease. Other current
than t hour. medications are aspirin. metfbrmin. metoprolol. atorvas
On physical examination, vital signs are normal. There tatin, losartan. and hydrochlorothiazide.
is pain at the sacroiliac joints with hip flexion, abduction, On physical examination, blood pressure is 172,'94 mm I Ig
and external rotation. Lumbar spine range of motion is and pulse rate is 90/min. Other than pitting pedal edema. the
mildly limited. Peripheral joints are normal. cardiopulmonary examination is normal.
Initial anteroposterior radiograph of pelvis showed Point-of-care blood glucose level is 310 mgrdl
bilateral sacroiliitis. (t2.2 mmol;L).
Changes in the patient's current medical regimen.
Which of the following is the most appropriate next including prednisone taper, are made to control glucose.
treatment? blood pressure. and edema.
(A) Etanercept
(B) Methotrexate Which of the following is the most appropriate additional
treatment?
(C) Prednisone
(D) Sulfasalazine (A) Cyclophosphamide
(B) Infliximab
(C) Methotrexate
tr AItem 15
36 year-old r,r,oman is evaluated in the emergency
(D) 'focilizumab

department after experiencing a tonic-clonic seizure. For


I week she has had severe headaches. Eight months ago, Item 17
Raynaud phenomenon of the hands and feet developed. A 62 year old man is evaluated for worsening exertional
Current medication is sustained release nifedipine. dyspnea and nonproductive cough over the past year. He

102
Self-Assessment Test

Ut
has a 6 year history of seropositive rheumatoid arthri- Item 20 €,
F
tis. Current medications are tofacitinib, methotrexate, and
A 66 year-old man is evaluated fbr a 10-year history of
folic acid.
pain affecting the flngers and bases of the thumbs, hips,
o
On physical examination, vital signs are normal. Car-
and knees. Morning stiffness lasts less than l5 minutes, and UI
t
diac examination reveals normal jugular venous pressure c,
pain is worse with activity. UI
and S, without extra sounds. There are reduced breath vl
Physical examination reveals bony hypertrophy of
sounds throughout the lung flelds, with bibasilar inspi
the distal and proximal interphalangeal joints of the hands
ratory crackles. Lung percussion is resonant throughout. o
and squaring of the thumb bases bilaterally. Both knees are ta
Joint examination shows advanced changes of rheumatoid
tender to palpation medially and laterally. Bony enlarge,
arthritis involving the hands and wrists. The remainder of
ment of medial knees and crepitus on range of motion
the examination is unremarkable.
are noted. There is no soft tissue swelling, warmth, or
effusion.
Which of the following is the most likely cause of this
patient's exertional dyspnea?
Which of the following is the most appropriate test?
(A) Heart failure (A) Antinuclear antibodies
(B) Interstitial lung disease (B) Rheumatoid factor
(C) Pulmonaryarterial hypertension (C) Serologic testing for Borrelia burgdorferi
(D) Rheumatoid arthritis pleural eflusion (D) Synovial fluid analysis
(E) No additional laboratory studies

tr Item 18
A 50 year old man is evaluated for severe pain and swell Item 21
EI
ing in the left knee of 2 days' duration. He has a long A 56 year old woman is evaluated at a folkrw-up visit.
history of psoriasis and polyarticular psoriatic arthritis. She has polyarticular tophaceous gout affecting her hancls,
He reports no fever, chills. or pain in other joints. Current Ieet, and knees bilaterally. She is receiving a tapering dose
medications are methotrexate, lolic acid, and topical clo- of prednisone following an acute gout flare. She had previ
betasol propionate. ously received allopurinol but discontinued it atter devel
:
On physical examination, vital signs are normal. There oping a rash. She has hypertensicln and chronic kidney
:
are psoriatic plaques on the elbows. sacrum, and anterior disease. Current medications are losartan and prednisone.
shins. The lelt knee is swollen and warm; the patient holds s mgid.
it at 45 degrees olflexion, and he is unwilling to further flex Laboratory studies show a serum creatinine level ol
I or extend. There is no othcr joint swelling. 1.8 mgidL (159.1pmol/L), a serum urate level of 10.1 mg/dl
Arthrocentesis of the left knee reveals a synovial fluid (0.60 mmolrL), and an estimated glomerular filtration rate
lcukocl,te count of 40,0001pL (a0 x 10eiL) with 90'7, neu- of 31 ml-imin 11.73 m).
trophils. Gram stain is negative, and synovial fluid analysis
for crystals and bacterial culture are pending. Which of the following is the most appropriate additional
treatment?
Which of the following is the most likely diagnosis?
(A) Allopurinol
(A) Gouty arthritis (B) Colchicine
I (B) lnf'ectir-rusarthritis (C) Febuxostat
(C) Osteoarthritis (D) Pegloticase
(D) Psoriatic arthritis (E) Probcnecid

tr
L

!
Item 19 Item 22
A 3O-year-old woman is seen in follow up. She was diag- A 35 year old woman is evaluated in the emergency
nosed with systemic lupus erythematosus 12 years ago. department lbr sudden vision loss in the lefl eye. Two years
1 She was treated for lupus nephritis, which is now quies ago, she had developed intermittent fever. myalgia, and
I cent. Clinical and laboratory flndings have been stable for
! chronic tatigue. She reports recent postprandial abdominal
5 years, with no disease flares. Medications are hydroxy pain and eflbrt related left arm pain.
chloroquine and azathioprine. On physical examination, bltnd pressure is 160/100 mm
Ilg in the lefl arm and 130i 80 mm I Ig in the right arm. Pulse
Which of the following health risk assessments should be rate is 88/min. Radial pulse in the left arm is abscnt; left-sided
performed now? subclavian bruit is present.
:
Laboratory studies reveal an erythrocyte sedimenta
(A) Breast cancer
tion rate o1 76 mm/h and a blood hemoglobin level o1'
I (B) Cardiovasculardisease e.2 gtdL (e2 s,tL).
i
(C) Iron overload Magnetic resonance angiogram of the chest and
(D) Pulmonary disease abdomen shows focal luminal narrorving in several areas.
:
103
Self-Assessment Test

t,l
ID

t,l
UI
tr
CONT,
including the ascending aorta. lelt subclavian artery. supe
rior mesenteric artery. and right renal artery.
Item 25
A 26 year-old woman is evaluated for a 3-month history of
increasing left hip pain that is worse with prolonged stand-
o
6
UI Whieh of the following is the most likely diagnosis? ing. Systemic lupus erythematosus was diagnosed 5 years
=
(D (A) Cryoglobulinemic vasculitis ago, characterized by small joint arthralgia, malar rash,
cytopenias, and Raynaud phenomenon. At diagnosis she
(B) Giant cell arteritis
{
.D (C) Polyarteritisnodosa
required high-dose prednisone, hydroxychloroquine, and
an azathioprine therapy. Prednisone dosage was tapered over
(D) Takayasu arteritis 6 months. Periodic flares have been treated with increased
dosages ofprednisone. Current medications are hydroxy
chloroquine, azathioprine, and low-dose prednisone.
Item 23 On physical examination, internal rotation and
A 62 year-old man is evaluated in the hospital for an arthro- full flexion of the left hip are limited and elicit pain. The
centesis conflrmed diagnosis of acute polyarticular gout remainder of the examination is normal.
involving the right knee, left ankle, and forefoot. Joint fluid Complete blood count, erythrocyte sedimentation
Gram stain and culture were negative. He was hospitalized rate, and serum complement levels are normal. Anti
4 days ago with heart failure complicated by pulmonary double-stranded DNA antibody titer is low and has been
embolism. He has a history of recurrent acute gout. The flrst unchanged for 6 months.
night ofhis hospital stay, he developed polyarticular gout Radiograph of the left hip is normal.
that has not responded to appropriate prednisone doses. He
also has hypertension, hyperlipidemia, type 2 diabetes mel- Which ofthe following is the most appropriate
litus, and stage G3a chronic kidney disease; he underwent management?
kidney transplantation 10 years ago. Current medications
(A) Change azathioprine to mycophenolate mofetil
include low molecular-weight heparin, metoprolol, Iosar
tan, furosemide, insulin glargine, and tacrolimus. (B) Increase prednisone dosage
The right knee, left ankle, and forefoot are swollen, (C) MRI of left hip
tender. and warm. (D) Physical therapy

Which of the following is the most appropriate


treatment? Item 26
(A) Anakinra A 48 year old woman is evaluated for sicca. She has a2 year
(B) history of Sjdgren slmdrome. Chewing sugarless gum and tak
Colchicine
ing frequent small sips ofwater are increasingly less helpful for
(C) Intra-articulartriamcinolone oral dry,rress. Current medications are artificial tears and oph
(D) Naproxen thalmic cyclosporine. She has no other conceming symptoms.
On physical examination, vital signs are normal.
Other than dry oral mucosa, the examination is normal.
Item 24
A 66 year-old woman is evaluated for a 10 year history Which of the following is the most appropriate treatment? I

of pain in the hands and knees. She has tried occupa-


tional therapy, with minimal beneflt; she is enrolled in a (A) Cevimeline !

weight loss program. She has hypertension and chronic (B) Hydroxychloroquine
kidney disease. Current medications are lisinopril and (C) Low-doseprednisone
amlodipine. (D) Rituximab
On physical examination, Heberden and Bouchard :

nodes are present and there is carpometacarpal joint ten-


derness. The medial joint line of each knee is tender to
palpation. There is bony enlargement of the medial knees,
with crepitus on range of motion.
ltem 27
A 58-year-old \\"oman is evaluated fbr brief episodes of
pain, swelling. and rcdness in thc right wrist for ,1 days.
tr
Laboratory evaluation shows a serum creatinine level
of 2.2 mgldL (194.5 pmouL) and an estimated glomerular During the past 2 years, she has had similar episodes in her
flltration rate of lcf't wrist and left third metacarpophalangeal joint, which
45 ml/min/l.73 m2.
resolved without treatment.
On physical ex:rmination, vital signs are normal. Joint
\
Which of the following is the most appropriate
examination reveals swelling. warmth, and redness olthe
treatment?
Ieft wrist and redness and bony enlargement of'the left
(A) Hydrocodone third metacarpophalangeal joint. Othcr joints are normal.
(B) Meloxicam Laboratory studies show norntal complete blood count.
(C) conrprehensive metabolic panel, and serum calcium, mag
Topical capsaicin
nesium, and thyroid stimulating hormone levels.
(D) Topical diclofenac
Radiographs of wrist and melacarpophalangeal joir.rts
(E) Topical lidocaine are sholvn (top ofnext page).
:

1(J4
Self-Assessment Test

vt
6'
h
E
o
=
t!
UI
c,
UI
l,t

€,
tt

Fl 51 novial ftuid from tl.re left nrist shonrs a leukocyte to control rheumatoid arthritis. She has no sicca symp
l*l c,runt tl1 30.000 pl. (30 x 10'),'1.) u,ith 90')1, neutrophilsl torns. Current medications are preclnisone. methotrexatc.
coNT pollrizing microscopy shou,s nunrerous positively bire ftrlic acid. and adalimumab.
fiingent rhomboid crystals within ncutrophils. Synovial On physical examination, vital signs are normal.'lhere
Ilnid Cram stain and cullure are ncgative. are rheumatoid nodules over the olecranon processes. Thc
spleen tip is palpable. Joint examination reveals ulnar
Which of the following is the most appropriate laboratory deviation, subluxation at the mct:rcarpophalangeal joints,
study to perform next? rcduced range of motion at the wrists, and bilateral swell
ing of the \,\,rists and Ieft ankle.
(A) Antinuclelr anlihodies
Laboratory studies:
(B) llrythrocytesedimentation rate
Absolute neutrophil count 1l00rpL (1.1 x 10'r,L)
(C) Itheumatoid factor l-cr-rkocyte count :l800i pL (3.8 x 10'q,'l-)
(l)) Serum ferritin Absolute lymphocyte count Normal
count
Platelet Normal
Item 28 Urir-ralysis Normal

A 52 year-old woman is evaluated at a follow-up visit. Der Which of the following is the most likely diagnosis?
matomyositis was diagnosed 4 weeks ago; she was positive
(A) AA amyloidosis
fbr anti Mi 2 antibodies. Prednisone has improved helio-
trope and photosensitive rashes and proximal muscle weak (B) Fclty syndrome
ness. She is currently asymptomatic. Current medication is (C) Sjdgren syndrome
prednisone, 60 mg/d. She is participating in physical therapy. (l)) Systemic lupus erytl.renrlitosus
On physical examination, vital signs are normal. There
is no rash. Muscle strength is normal.
Serum creatine kinase level is 200 U/L, decreased from Item 30
520 U/L I month ago. A 27-year-old woman is evaluated for a 4-year history of
Results of screening colonoscopy, mammography, progressive achy and stiff low back pain that wakes her
and cervical cancer screening are all normal. up at night and a 2-year history of intermittent, severe,
sharp bilateral buttock pain. She has stiffness for 90 min
Which of the following is the most appropriate treatment? utes each morning. Exercise and ibuprofen help relieve
pain. She has a history of unilateral uveitis. She has no
(A) Add hydroxychloroquine
children.
(B) Add methotrexate On physical examination, vital signs are normal.
(C) Add rituximab Range of motion of the lumbar spine is decreased in all
(D) Continue current therapy directions. The eye examination, occiput-to-wall distance,
chest expansion, and peripheral joints are normal.
Laboratory studies reveal an elevated blood C-reactive

tr Item 29
A :)9 year-old woman is evaluated for newly discovered
protein level and negative HLA- 827 antigen result.
Radiographs of the pelvis and lumbar spine are
normal.
ncutrcpenia. She has a 10 year history of severe, diflicult

105
Self-Assessment Test

l/r
.D
Which of the following is the most appropriate diagnostic On physical examination, vital signs are norrnal. Exam-
D
Ut test to perform next? ination ofthe hands reveals bilateral polyarticular joint swell-
u) ing and tenderness involving the metacarpophalangeal joints.
lD
u! (A) Bone scanning
vt
(B) CT of pelvis Which of the following is the most likely cause of this
(D
(C) MRI of pelvis patient's joint pain?
(D) Rheumatoid factor and anti-cyclic citrullinated peptide
.D (A) Ankylosingspondylitis
u) antibodies
(B) Erosive osteoarthritis
(C) Reactive arthritis
Item 31 (D) Rheumatoidarthritis
A22 year old woman is evaluated for a l-month history of
progressive rash on the face, chest, and arms following sun
exposure. She also has experienced wrist arthralgia and
malaise for the past 2 weeks.
On physical examination, blood pressure is l42l
Item 33
A 73 year-old woman is hospitalized for progressive dys-
tr
pnea. Six months ago, she developed chronic sinusitis and
82 mm Hg, pulse rate is 88/min, and respiration rate is
nose bleeds. Four months ago, a persistent dry chronic
16/min. The facial rash is shown.
cough developed, followed by myalgia and distal paresthe-
sia. She has lost 13.6 kg (30 lb).
On physical examination, temperature is 38.2 'C
(100.8 "F), blood pressure is 148/96 mm Hg, pulse rate is
104/min, respiration rate is 24lmin, and oxygen saturation
is 94% with the patient breathing ambient air. Dry crack-
les are heard at the lung bases. There is loss of sensation
to light touch in the left foot. Numerous small palpable
red-purple lesions are present on the lower legs.
laboratory studies:
Ery.throcyte 120 mm/h 't
I
sedimentation rate I

Creatinine 1.7 mgldL (150.3 pmol/L)


ANCA Positive
Antiproteinase-3 70 antibody index (normal 1
I
antibodies <1 antibody index)
Urinalysis 3+ blood; 2+ protein; 20-30
dysmorphic erythrocltes / hpfi
5-10 leukocytes/hpf
Chest radiograph shows peripheral pulmonary paren
Similar-appearing patchy skin lesions are seen over chymal opacities. Chest CT scan shows multiple opacities
the upper chest in a V-neck distribution and the dorsal and nodules throughout both lungs.
forearms. Active and passive wrist extension elicits dis- Kidney biopsy results are pending.
comfort. The remainder of the examination is normal. High-dose glucocorticoids are started.

Which of the following is the most likely diagnosis? Which of the following will most likely be the appropriate
additional treatment?
(A) Acute cutaneous lupus erythematosus
(A) Azathioprine
(B) Lupus pernio
(C) Rosacea
(B) Methotrexate
(C) Mycophenolatemofetil
(D) Subacute cutaneous lupus erythematosus
(D) Rituximab

Item 32
A 33 year-old woman is evaluated for a 1-month history of Item 34
pain in the hands. She also has stiffness in the hands upon A S0-year-old woman is evaluated for a 2-year history
arising in the morning that lasts 90 minutes. She otherwise of pain affecting the knee joints. The pain is associated
has been well. She reports no tingling or numbness in the with walking or standing and occurs 1 to 2 days per week.
flngers; changes in the color of the digits; or rash, oral She has tried acetaminophen with minimal beneflt; she
ulcers, or alopecia. Her vaccinations are up to date. She is enrolled in a weight loss program. She prefers a home-
works in her home as a computer software engineer, is not based self-management treatment option.
sexually active, has no contact with children, and has not On physical examination, vital signs are normal. BMI
traveled. is 33. there is mild tenderness to palpation of knee medial

106
Self-Assessment Test

UI
joint lines bilaterally. Knee joints are not warm or swol- coronary artery disease, COPD, and a history of diverticuli Fo,
len and have normal range of motion. No other joints are tis. Current medications are aspirin, lisinopril, metoprolol,
involved. a tiotropium inhaler, methotrexate, and sulfasalazine. €,
On physical examination, blood pressure is 136/84 mm E
UI
vt
Which of the following is the most appropriate Hg. BMI is 29. Multiple metacarpophalangeal joints are (u
U!
treatment? tender to palpation, and there is active synovitis. UI

(A) High-impact aerobic exercise Result of an interferon-gamma release assay is


negative. (l,
(B) Tai chi Hand radiographs show joint space narrowing and t
(C) Transcutaneouselectrical nerve stimulation three new erosions.
(D) Vegan diet
Which of the following is the most appropriate
treatment?

tr Item 35
An 85-year-old woman is evaluated lbr worsening chronic
(A) Abatacept
(B) Adalimumab
right shoulder pain of 1 year's duration. 'lhe pain is
more severe with use. She has no history of trauma to the
(C) Anakinra
shoulder. (D) Tocilizumab
On physical examination, vital signs are normal. The (E) Tofacitinib
right shoulder is swollen and tender, with no warmth or
redness. There is pain-related decreased range of motion
of the shoujder in all directions. Other joints are normal. Item 37
Ervthrocfie sedimentation rate and blood C reactive A 42 year old woman is evaluated for 3 months of pain
protein level are normal.
and swelling in her hands and multiple other joints, as
Radiograph of right shoulder is shown. well as morning stiflness lasting more than t hour. She has
increasing difficulty with daily activities, such as walking
and making the bed. She has no other symptoms or medi-
cal problems and takes no medication.
On physical examination, vital signs are normal. She
has swelling and tenderness of the second and third proxi-
mal interphalangeal joints bilaterally, right second and left
third metacarpophalangeal joints, left wrist, and bilateral
ankles.
laboratory studies:
C reactive protein 6.4mgldL(64mglL)
Rheumatoid factor Positive
Antinuclearantibodies 1:80(>1:160positive)
Anti-cyclic citrullinated Positive
peptide antibodies

Which of the following is the most likely diagnosis?


(A) Chikungunya virus infection
Synovial fluid from the right shoulder is blood-tinged;
(B) Parvovirus 819 infection
analysis of fluid reveals a lcukocyte count of 500rpl
(0.5 x 10'i L), negative Gram stain and culture results. and (C) Rheumatoidarthritis
no crystals. (D) Systemic lupus erythematosus

Which of the following is the most likely diagnosis?


Item 38
(A) Adhesivecapsulitis
A 65 year-old man is evaluated for a 4-month history of
(B) Basic calcium phosphate associated arthritis
a rash over the face and hands and a 3 month history of
(C) Calcium pyrophosphate deposition disease progressive proximal muscle weakness. He also reports
(D) Rheumatoidarthritis a 6.8-kg (15-lb) unintentional weight loss. He has had no
fevers, respiratory symptoms, or sensory changes.
On physical examination, vital signs are normal. There
Item 36 is weakness in the proximal muscles of the arms and legs.
A 58 year-old man is evaluated at a follow-up visit. He has A thickened, red rash is seen on the cheeks, forehead, and
had rheumatoid arthritis for 5 years. Increasing morning upper chest. Photograph ofhand is shown (top ofnext page);
stiffness, fatigue, increasing joint pain, and swelling of similar flndings are seen on both elbows. There is no muscle
small hand joints have developed in the past 6 months. His tenderness, joint swelling, or edema. The remainder of the
disease activity score shows moderate activity. He also has examination is normal.

107
Self-Assessment Test

vr
TD
Which of the following is the most likely diagnosis?
D
UI (A) Adult-onset Still disease
ra
.D
ta (B) Familial Mediterranean fever
art
(C) Infectiousendocarditis
(D
(D) Systemic lupus erythematosus
{
.D t
(,l
Item 40
A 47-year-old woman is evaluated for a 3-year history of
pain in all of her muscles and joints. She is a commercial
truck driver. Her sleep is poor, and she awakens unre-
freshed. She completes a flbromyalgia diagnostic ques-
tionnaire and meets the criteria for flbromyalgia. She has
anxiety and depression, with a history of opioid use disor-
der. She takes no medications.
On physical examination, vital signs are normal.
Physical examination flndings are limited to widespread
muscle tenderness with normal strength.
Laboratory evaluation reveals an erythrocyte sedi-
mentation rate of 19 mm/h, a blood C reactive protein level
of 0.3 mg/dl (3 mg/L), and a thyroid-stimulating hormone
level of 1.6 pU/mL (1.6 mU/L).
Low-impact aerobic exercise is recommended.
Laboratory evaluation reveals a hemoglobin level of
Which of the following is the most appropriate additional
12.5 g/dl (125 g/L) and serum creatine kinase Ievel of4500 U/L.
treatment?

Which of the following is the most appropriate test to (A) Amitriptyline


perform next? (B) Diclofenac
(A) Antinuclear antibody testing (C) Duloxetine
(B) Colonoscopy (D) Pregabalin
(C) Muscle biopsy (E) Tramadol
(D) Pulmonary function testing
Item 41
Item 39
A 46 year-old woman is evaluated at a follow-up visit to
A 24-year-old man is evaluated for fever of 6 weeks' dura-
discuss results of bone mineral density testing. She has
tion. He also has joint pain, myalgia, and occasional sore a l0-year history of myasthenia gravis and is receiving
throat. The fever begins in the early evening and resolves long-term glucocorticoid therapy. Current medications
by morning. His most recent temperature in the eve- are azathioprine, prednisone, calcium/vitamin D, and
ning was 39.5 'C (103.1 'F). The fever is accompanied by a pyridostigmine.
salmon-pink macular rash on the trunk and arms, which Dual energy x ray absorptiometry shows T-scores of
resolves with the fever. He has no history of travel. Current 2.1 at the lumbar spine and 1.9 at the femoral neck, with
medication is acetaminophen. corrected Fracture Risk Assessment Tool scores of 3.5u1, at
On physical examination, temperature is 37.5 'C the hip and17% overall.
(99.6'F). The remaining vital signs are normal. There is
bilateral cervical lymphadenopathy. A friction rub is heard
Which of the following is the most appropriate
bilaterally at the lung bases. Abdomen is tender without
treatment?
guarding. The knees have eflusions. Cardiac examination
is normai. (A) Alendronate
[.aboratory studies: (B) Calcitonin
Erythrocyte sedimentation rate 125 mm/h (C) Romosozumab
Leukocyte count 22,OOOl1tL (22xtoe lL) (D) Teriparatide
Hemoglobin 11.5 g/dl (11s g/L)
Ferritin 5200 ng/ml (5200 pg/L)
Blood cultures are pending. Item 42
A chest CT scan shows small pleural effusions bilater- A 32 year-old woman is evaluated for pain in the right elbow
ally with moderate pleural thickening. An abdominal CT of a few months' duration. She has no history of injury. The
scan shows a small amount of ascitic fluid with evidence of pain worsened after she began a new exercise program that
peritoneal thickening. includes weights, and it is exacerbated with use. The pain

108
Self-Assessment Test

Ul
makes it hard to lift things. She also has increased fatigue. On physical examination, vital signs are normal. Sym att
Acetaminophen partially alleviates the pain. metric proximal and distal weakness in both arms is noted.
There is local tenderness over the right lateral epicon Hip flexor strength is diminished, and he has a waddling c,
dyle, and active wrist extension against resistance elicits gait. Otherwise, Iower extremity strength is normal. Upper ra
pain. She has full range of motion of the shoulder, elbow, t!
extremity examination is notable for atrophy of the fore o
vt
and wrist. There is no warmth, redness, or swelling. All arm muscles and a weak grip. No muscle tenderness, joint tt
other joints are normal. swelling, edema, or rash is observed.
Serum creatine kinase level is 310 U/L. Erythrocyte
+q,
Which of the following is the most likely cause of the sedimentation rate, serum aminotransferase levels, and
(a
patient's symptoms? thyroid-stimulating hormone level are normal.
(A) Degenerat ive joint disease
Which of the following is the most likely diagnosis?
(B) Disorder ofcenlral pain processing
(C) Inflammatoryarthritis (A) Inclusion body myositis
(D) Mechanical soft-tissue injury (B) Polymyositis
(C) Statin myopathy
(D) Systemic lupus erythematosus
Item 43
A Sl-year old woman is evaluated for an immigration
physical examination. She has a 2 year history of rheuma
toid arthritis. Immigration requirements include multiple
Item 46
A 28 year old man is seen in lbllow up for management of tr
vaccinations, and she has no recollection or record ofher a multisystem febrile illness. Up until 6 neeks ago he was
immunizations. Current medications are oral methotrex in good health. He takes no medications.
ate, tofacitinib, and folic acid. On physical examination, temperature is ll8.2 'C
(10o.8 'F), and blood pressure is 1SBi 84 nlm Hg. Bilateral
Which of the following vaccines is contraindicated in this 2+ ankle edema and bilateral tenderness and swelling of
patient? the metacarpophalangeal joints and wrists are observed.
'lhe remainder of the examination is normal.
(A) Hepatitis B virus
Laborato4r studies:
t (B) Measles, mumps, and rubella Henroglobin B.e gi dL (Be g/L)
(C) Quadrivalentinfluenza Leukocyte count 3U00/pt- (3.8 x 10erL)
(D) Tetanus toxoid, reduced diphtheria toxoid, and acel- Platelet count 90.000iirl (90 x 10ei L)
lular pertussis Cl) complement l.ow
(E) 13-Valent pneumococcal conjugate
04 cornplement Low
(lreatinine 1.6 mgrdl. (t41.4 pn-rolrL)
Anti double-stranded Positivc
Item 44 DNA antibodies
Antinuclear antibodies I :1 280 (speckted pattern)
; A 40 year old woman is evaluated fbr a 6 month history Urinalysis 2+ blor-rd; 3+ protein; dysmorphic
of bilateral hand and wrist pain and swelling. She has erythrocytes; no casts
morning stiflness lasting for t hour. Her hands and wrists Urine protein -creatinine :150 mg/g
i are swollen. ratio
Soft tissue swelling and warmth and tenderness of the
second through fourth metacarpophalangeal joints and Kidney biopsy results are pending.
t Prednisone is initiated.
wrists bilaterally are noted. Range of motion of these joints
i is normal but painful.
A diagnosis of rheumatoid arthritis is being considered. Which of the following is the most appropriate additional
;
treatment?
Which of the following will be most helptul in establishing (A) Azathioprine
It the diagnosis?
(B) Ilydroxychloroquine
(A) Anti-cyclic citrullinated peptide (CCP) antibodies (C) I Iydroxychloroquine and mycophenolate mofetil
(B) Anti-CCP antibodies and rheumatoid factor (D) No additional therapy
5
(C) C3 and C4 complements
(D) Rheumatoid factor
i Item 47
A S7-year-oldwomanisevaluated forfatigue, generalDed ach
Item 45 iness, and oral dryness without ocular dryness. Her review of
A 73 year old man is evaluated lor a 3-year history of systems is unremarkable. She has hypothyroidism, migraine
progressive weakness. He has a 7-year history of hyper- headaches, major depressive disorder, and chronic allergic
lipidemia and hypertension treated with atorvastatin and rhinitis. Current medications are levothyroxine, amitripty
Iisinopril. line, citalopram, intranasal glucocorticoids, and cetirizine.

109
Self-Assessment Test

ln
(D
On physical examination, vital signs are normal. Oral Which of the following is the most likely diagnosis?
D
vt dryness is noted. There is no ocular inflammation or dry- (A) Eosinophilic granulomatosis with polyangiitis
UI ness or salivary gland enlargement. The remainder of the
t,D
gr (B) Hypersensitivityvasculitis
tt examination is normal.
(C) Polyarteritisnodosa
.D
Laboratory studies:
Ery.throcyte sedimentation rate 18 mm/h (D) Rheumatoidvasculitis
{
.D
Thyroid-stimulating hormone 1.6 pU/mL (1.6 mU/L)
Ut Rheumatoid factor Negative
Antinuclear antibody 1:80 Item 49
Anti-La/SSB antibody Negative A ss-year old man is evaluated for urate-iowering ther-
Anti Ro/SSA antibody Negative apy. He has experienced three gout flares over the past
year. He also has hypertension and chronic stable angina.
Which of the following is the most likely cause of this Current medications are low dose aspirin, metoprolol,
patient's oral dryness? atorvastatin, and lisinopril. He is of Han Chinese descent.
(A) Hypothyroidism Laboratory evaluation reveals a serum urate level of
9.8 mg/dL (0.s8 mmol/L).
(B) Medication adverse effects
(C) Sjogren syndrome
Which ofthe following is the most appropriate management?
(D) Systemic lupus eqthematosus
(A) Allopurinol
(B) Discontinuation of aspirin
Item 48 (C) Febuxostat
A 63-year-old woman is evaluated for fatigue and a leg rash
(D) HLA B.57:01 allele testing
that began 4 days ago. She has no other symptoms. She has
recently diagnosed hypertension, and hydrochlorothiazide
(E) HLA B-58:01 allele testing
was initiated 2 weeks ago.
On physical examination, vital signs are normal. The
rash is shown. The larger lesions are palpable. The remain Item 50
der of the examination is normal. A72 year old man is evaluated for a 15-year history of pain
and stiffness in the knees. He can walk only 50 yards before
needing to stop because ofthe knee pain. He has tried exer-
cises and physical therapy without relief. Intra articular
glucocorticoids provide minimal relief. He has no other
medical problems. Current medications are maximum
dosages of celecoxib, duloxetine, and acetaminophen.
On physical examination, the medial joint line of
each knee is tender to palpation. He has crepitus r,l'ith
movement.

Which of the following is the most appropriate treatment?


(A) Arthroscopic knee surgery
(B) Chondroitin sulfate and glucosamine
(C) Intra-articular hyaluronic acid injection
(D) Intra articular platelet rich plasma injection
(E) Total knee replacement

Item 51
A 53 year old man is evaluated in the emergencl'depart tr
ment for abdominal pain. Four rt'eeks ago. fatigue and
malaise developed. followed 1 u,eek later by a rash. Trvo
r,r,eeks ago. he began tripping or,er his left fbot. Three da1,s
ago. he developed abdominal pain after a meai; abdominal
pain is nolt constant.
On physical examination. temperature is 38.0 "C
(100.,1 "F). and blood pressure is 154 96 mm Hg: other vital
signs are normal. The abdomen is soft and nontender. N{us
cle strength at dorsiflexion of the left ankle is 315. Erythem
Laboratory evaluation reveals normal complete blood atous nodules are present on the lower legs. The remainder
count, comprehensive metabolic panel, and urinalysis results. of the examination is normal.

110
Self-Assessment Test

vt
o
tr ffia.;fl;**o'
g
coNT reactiveprotein
8.7 gidL(87 s,tLj
Item 54
A 56 year old woman is evaluated for a 2 year history of
F
€,
13.8 mg/dl mg/L)
(138
osteoarthritis involving the flngers, bases of the thumbs,
Creatinine 7.7 mgldL (rSO.: pmol/L) hips, and knees. She has tried physical and aquatic therapy UI
(,}
Urinalysis Normal (u
with minimal beneflt, and she is enrolled in a weight loss vt
Hepatitis C virus serology Negative program. She has no other medical problems and takes no t!
HIV testing Negative
medications.
(u
Chest radiograph is normal. On physical examination, vital signs are normal. vt
BMI is 29. Heberden and Bouchard nodes and squaring
Which of the following is most likely to establish the of the bilateral flrst carpometacarpal joints are present.
diagrrosis? The flrst carpometacarpal joints and medial joint line of
both knees are tender to palpation. There is pain in the
(A) ANCA panel anterior left groin on internal rotation of the hip. There
(B) Hepatitis A virus serology is bony enlargement of the knees, with crepitus on range
(C) Kidney biopsy of motion.
(D) Magnetic resonance angiography of abdomen
Which of the following is the most appropriate
treatment?

tr Item 52
A 45-year old man is evaluated 2 weeks after an ernergency
(A) Acetaminophen
(B) Duloxetine
department visit for gouty olecranon bursitis. Naproxen was
initiated at 500 mg twice daily; after clinical improvement,
(C) Gabapentin
the dosage was tapered. He has a 5 year history of acute gout (D) Piroxicam
that occurs three times a year. He also has a history of kidney (E) Tramadol
stones. Current medication is naproxen, 250 mg twice daily.
There is left olecranon bursal thickening without redness,
warmth, orfluid. Tophi are presentbilaterallyinthe flrst meta- Item 55
tarsophalangeal joints without swelling, redness, or warmth. A 65-year-old man is evaluated for "pain everyr,vhere."
Kidney f'unction is normal. Serum urate level is Two months ago, he developed neck and upper back pain,
10.5 rng/dl (0.02 rnmolll). fatigue, and malaise. Over the next few weeks, stiffness and
achiness developed in the shoulders and upper arms. He has
Which of the following is the most appropriate additional no headaches, visual changes, or jaw discomfort. He takes
treatment? no medications.
(A) Allopurinol On physical examination, vital signs are normal. There
is no scalp tenderness. Temporal artery pulses are normal.
(B) Colchieine There are no swollen or tender joints. Discomfort limits
(C) Febuxostat range of shoulder motion bilaterally and causes difficulty
(D) Probenecid with standing from a seated position. Extremity strength
and reflexes are normal.
Laboratory evaluation shows a normal complete
Item 53 blood count, normal serum thyroid stimulating hormone
A 35-year-old man is evaluated for a 3-day history of level, and blood C-reactive protein level of 6.8 mg/dl
abrupt polyarticular joint pain in both hands. His S-year- (os mg/L).
old daughter recently had a low grade fever and malaise
with 1 day offrequent loose stools; 2 days later she had an Which of the following is the most appropriate
erythematous rash over her cheeks. management?
On physical examination, vital signs are normal. Joint
(A) Measurement of rheumatoid factor, antinuclear anti
examination shows tenderness of the right second, third,
bodies, and ANCA
and fourth proximal interphalangeal joints; tenderness
ofthe right second and third metacarpophalangeal joints; (B) Prednisone, 15 mg/d
tenderness of the left second and third proximal interpha (C) Prednisone, 60 mg/d
langeal joints; and tenderness ofthe left second and third (D) Pregabalin
metacarpophalangeal joints.
Results of serologic tests are pending.
Item 56
Which of the following is the most appropriate treatment? A 63-year old man is evaluated for an S-week history
(A) Diclofenac
of a spreading rash on the feet and legs, fatigue, and
arthralgia.
(B) Etanercept On physical examination, vital signs are normal. The
(C) Hydroxychloroquine rash is shown (top of next page). Some lesions are palpable.
(D) Methotrexate He cannot extend his left wrist. Findings on pulmonary,

111
Self-Assessment Test

vr
(D
cardiac, and gastrointestinal examinations are unremark- Item 58
vt able. There are no swollen or tender joints. A 62-year-old woman is evaluated for joint pain. She has a
l/}
(D
UI
10-year history ofpsoriasis and a 3-year history ofarthritis.
t Her psoriasis has been controlled with adalimumab, but
(D joint symptoms persist. She has stiffness in the affected
joints for less than 30 minutes every morning, and pain
.D
increases with activity. There is no dactylitis. Current
t^ medication is adalimumab.
On physical examination, there are small psoriatic
plaques on the left elbow and right knee. She has bony
enlargement, without swelling, of most distal and prox
imal interphalangeal joints of the hands. the left knee
has a small effusion. The remainder of the examination is
normal.
Radiographs of the hands show joint-space narrow-
ing and osteophytes at the distal and proximal interpha-
Iangeal joints of both hands. Radiographs of the knees
show medial joint-space narrowing and subchondral
sclerosis of the left medial compartment.
laboratory studies:
Erythrocyte sedimentation rate 110 mm/h Which of the following is the most likely cause of this
Alanine aminotransferase 93UIL patient's joint pain?
Aspartate aminotransferase 89 U/L (A) Chronic gout
C3 complement 113 mg/dl (rrao mg/L) (B) Osteoarthritis
C4 complement Not detected
Creatinine 2.r mgl dL (18s.6 pmol/L) (C) Psoriaticarthritis
Rheumatoid factor 118 U/mL (118 kU/L) (D) Rheumatoidarthritis
Anti-cyclic citrullinated Not detected
peptide antibodies
Urinalysis 2+ blood; 2+ protein; Item 59
dysmorphic A 40 year-old woman is evaluated for a 9-month history
erythrocytes; no casts of Raynaud phenomenon. She reports no dilficulty swal-
lowing, dyspnea on exertion, or hand stiffness. She has
Which of the following is the most likely diagnosis?
recently developed gastroesophageal reflux disease.
(A) Cryoglobulinemicvasculitis On physical examination, vital signs are normal. The
(B) Hlpersensitivityvasculitis skin between the distal and proximal interphalangeal
joints ofthe flngers is difficult to tent. Ihree telangiectasias
(C) Polyarteritisnodosa
on the right hand and four on the left are noted. She also
(D) Rheumatoidvasculitis has two small telangiectasias on the face and one on the
inside bottom lip. No digital pitting is seen.
Laboratory evaluation reveals a positive antinuclear
Item 57 antibody titer (1:SZO) and positive anticentromere antibody
A 36-year-old woman is evaluated for preconception titer (1:160).
counseling. She has a 3 year history of rheumatoid arthri-
tis well controlled with leflunomide, hydroxychloroquine, Which of the following is the most likely diagnosis?
and certolizumab. Her most recent disease activity score
showed remission. In the past, attempts to discontinue her
(A) Diffuse cutaneous systemic sclerosis
medications have been unsuccessful because ofrecurrent (B) Eosinophilicfasciitis
active disease. (C) Limited cutaneous systemic sclerosis
On physical examination, vital signs are normal. BMI (D) Mixed connective tissue disease
is 24. Range of motion is normal, and joints are not warm
or swollen.
Item 6O
Which of the following is the most appropriate
A 60-year-oldwoman is evaluated for achiness and fatigue.
preconception management?
She reports dry mouth, irritated eyes, and discoloration of
(A) Discontinue all drugs flngers after exposure to cold temperature that rapidly
(B) Discontinuecertolizumab responds to rewarming.
(C) Discontinuehydroxychloroquine
On physical examination, vital signs are normal.
Parotid glands are enlarged, and salivary pooling is
(D) Discontinueleflunomide decreased. There is no lymphadenopathy. All appendicular
(E) No changes are needed joints have preserved range of motion without discomfort.

112
Self-Assessment Test

ta
Laboratory evaluation reveals erythrocyte sedimen On physical examination, respiration rate is 18/min c,
F
tation rate of 77 mmlh, a blood C-reactive protein level of and oxygen saturation is 97% with the patient breathing
0.23 mg/dl (2.3 mglL), and a rheumatoid factor level of ambient air. On a 6 minute walk test, oxygen saturation €,
202UtmL(202kUtL). drops to 90%. Crackles are heard at both lung bases. Heart tt
vt
sounds are normal. Skin changes consistent with diffuse (l,
ra
Which of the following is the most appropriate diagnostic cutaneous systemic sclerosis are present. t,
test to perform next? A high-resolution CT scan shows ground glass
changes in the lower lung flelds. (l,
(A) ANCA Pulmonary function tests show an FVC of 82% of tt
(B) Anti-cycliccitrullinatedpeptide antibodies predicted and a Dr-co of 65% of predicted.
(C) Anti Ro/SSA antibodies
(D) Cryoglobulins Which of the following is the most appropriate treatment?
(A) Hydroxychloroquine
Item 61 (B) Methotrexate
A 70 year-old woman is evaluated for a S-year history of (C) Mycophenolatemofetil
tophaceous gout in both hands. Previous treatment with allo- (D) Nintedanib
purinol resulted in rash. Over the past year, maximum-dose
febuxostat has decreased serum urate levels to 9.5 mg/dl
(O.SO mmol/L). She also has hlpertension and stage G3b
chronic kidney disease. Current medications are febuxostat
Item 63
An 18 year old woman is evaluated for f'ever, chills. and tr
and losartan- malaise that developed 5 days after her last menstrual
Radiograph of the right hand obtained before initia period. The following day, she developed arthralgia that
tion of febuxostat therapy is shown. first involved the left wrist and then the right knee: the left
flfth toe became exquisitely painful and diffusely swollen,
with limited range of motion. She is sexually active with
one male partner. Her only medication is a combined oral
contraceptive.
\ On physical examination, temperature is 38.1 'C
(i00.6 'F). Her feet and left hand are shown.
i

Which of the following is the most appropriate treatment? Which of the following is the most likely diagnosis?
(A) Addhydroxychloroquine (A) Disseminated gonococcalinfection
(B) Addprobenecid (B) I{epatitis B virus infection
(C) Stop febuxostat and start pegloticase (C) HtV associated ar-thritis
I

(D) No treatment changes (D) l.yme arthritis


(E) Systemic lupus erythematosus
Item 62
A 52 year old man is evaluated for treatment of intersti Item 64
tial lung disease. Over the last 6 months, he develgped a A 50 year-old woman is evaluated for a 6-month history of
nonproductive cough and dyspnea on exertion. He has progressive difficulty getting out of a chair and raising her
a 3 year history of diffuse cutaneous systemic sclerosis arms above her head. She has a l0-year history ofRaynaud
and gastroesophageal reflux disease. Current medication phenomenon and gastroesophageal reflux. Current medi-
is pantoprazole. cations are omeprazole and acetaminophen.

113
Self-Assessment Test

vr
(D
On physical examination, vital signs are normal. Fin- the proximal interphalangeal joints, metacarpophalangeal
la gers are pufflz bilaterally, without digital ulcers or skin joints, wrists, elbows, ankles, and metatarsophalangeal joints.
UI thickening. Neurologic examination shows bilateral prox
(D
UI imal muscle weakness; distal muscle strength in upper and
UI Infection with which of the following is the most likely
lower extremities is normal bilaterally. The remainder of diagnosis?
.D
the examination is normal.
(A) Chikungunya virus
Iaboratory studies:
(D
tt (B) Hepatitis B virus
Complete blood count Normal
C3 complement Normal (C) Hepatitis C virus
C4 complement Normal (D) HIV
Creatine kinase 2000 u/L
Antinuclear antibodies Strongly positive
Anti-U1-ribonucleoprotein antibodies Positive Item 67
Anti-Ro/SSA antibodies Negative A 26-year-old man is evaluated for a 1 week history of
Anti-La/SSB antibodies Negative pain and swelling in the left ankle, along with pain in
Anti-smooth muscle antibodies Negative the toes and tenderness in the left heel. Three weeks ago,
Anti-double-stranded DNA antibodies Negative he returned from a vacation in Central America. Before
Urinalysis Normal returning home, he experienced 4 days of diarrhea that
spontaneously resolved. Current medication is acetamin-
Which of the following is the most likely diagnosis?
ophen as needed.
(A) Mixed connective tissue disease On physical examination, vital signs are normal. The
(B) Rheumatoidarthritis left ankle is swollen, with reduced range of motion. Several
(C) Systemic lupus erythematosus toes are swollen, as shown, and the Achilles insertion on
the left is swollen and tender.
(D) Undifferentiated connective tissue disease

Item 55
An 82-year-old man is seen in follow-up for inclusion body
myositis. A prednisone taper over the past 4 months was
completed without clinical improvement.
On physical examination, vital signs are normal. There
is weakness in the shoulders, forearms, hand grip, hip gir-
dle, and knee extensors that is unchanged from previous
examinations.
Serum creatine kinase level is 370 U/L, minimally
decreased from 4 months ago.
Laboratory evaluation reveals a blood C-reactive pro-
tein level of 3.5 mg/dl (ss mg/L).
Which of the following is the most appropriate treatment?
(A) Addcyclophosphamide Which of the following is the most appropriate
(B) Add intravenous immune globulin management?
(C) Add methotrexate (A) Azithromycin
(D) Initiate physical therapy (B) HLA-B27 testing
(E) Restart prednisone (C) Piroxicam
(D) Stool cultures for enteric pathogens

tr Item 56
A 45 year-old man is hospitalized for incapacitating poly Item 58
tr
arthralgia and fever. One week before hospitalization, pain A 72-year-old woman is evaluated for a 3-month history
in multiple joints of the hands, wrists, elbows, ankles, and of abdominal and back pain and 4-month history of an
feet developed over a 24-hour period. Five days before increasingly rounded face.
hospitalization. fever, malaise, and intensification of the On physical examination, vital signs are normal. BMI
joint pain developed. Two days before hospitalization, a is 29. Bilateral lacrimal and parotid glands are enlarged.
rash appeared on the extremities and trunk and spread to Laboratory evaluation shows a hemoglobin A,. value
the face. He lives in southern Florida and has not recently of 5.4"1,, an alkaline phosphatase level of 255 UlL, and an
traveled outside the United States. estimated glomerular filtration rate of 59 mliminll.T3 m2
On physical examination, temlxrature is 39.1'C (102.4.F) (same as l year ago).
and pulse rate is 116/min. He has bilateral conjunctivi An abdominal CT scan shows periaortitis with a retro
tis and a maculopapular rash on the face, extremities, and peritoneal mass that extends bilaterally and encircles both
trunk. Joint examination shows swelling and tendemess of ureters without hydronephrosis.

114
Self-Assessment Test

tt
c,
m
llJ
cONT
Needle biopsy of the mass reveals dense fibrous tissue
in a storifbrm pattern, interspersed with an inflammatory
irrg11."1., including increased numbers of IgG4 positive
Item 71
A 38-year old woman is evaluated fbr pain in multiple tr (u
fingers. She fractured her left second digit 1 year ago after
plasmablasts. Serum IgG,l levels are not elevated. |a
a minor trauma. For the past 2 years, she has occasionally |n
(l,
experienced f'ever and night sweats. a
In addition to prednisone, which of the following is the most On physical examination, vital signs are normal. Parotid
vt
appropriate treatment? and lacrimal glands are eniarged. Multiple coin-sized pink.
(u
(A) Cevimeline red, and violaceous papular skin lesions are present on the t
(B) arms and anterior chest. The right second and flfth digits are
Methotrexate
swollen and tender. Hepatomegaly is present.
(C) Rituximab
l-aboratory studies:
(D) Surgicaldebridement
Complete blood count Normal
Erythrocl.tesedimentationrate 62mmlh
Item 69 Alkaline phosphatase 166U lL
Alanine aminotransferase 58 UIL
A 50 year-old woman is evaluated for exertional dyspnea Aspartate aminotransferase 64UlL
of 6 months'duration. Limited cutaneous systemic sclerosis FIIV test Negative
was diagnosed 6 years ago. She reports no other cardiopul- Purified protein derivative skin test Negative
monary symptoms. Other medical problems include sclero
dactyly, Raynaud phenomenon, and gastroesophageal reflux Chest radiograph shows hilar lymphadenopathy.
disease. Current medications are nifedipine and omeprazole. Abdominal ultrasound reveals liver enlargement with
On physical examination, oxygen saturation is 97%
multiple infiltrating nodules. A hand radiograph is shown.
with the patient breathing ambient air. Other vital signs
are normal. Sclerodactyly of the flngers is noted. Telangi-
ectasias are present on the hands and face. Lungs are clear
to auscultation. Heart examination is normal.
Twelve months ago, a baseline high resolution chest
CT scan was normal.
FVC is 96% of predicted, and Dr-co is 58% of predicted.
Twelve months ago, these values were 97"/. and 75ok of
predicted, respectively.

Which of the following is the most appropriate diagnostic


test to perform next?
(A) Cardiac magnetic resonance imaging
(B) Repeat high resolution chest CT
(C) nignt heart catheterization
(D) Transthoracic echocardiography

Item 70
A 40 year old woman is evaluated for several months of
joint pain and stiffness in the hands and feet. Over the past
7 weeks, she has developed swelling in several of her flnger
joints and in her right wrist.
On physical examination, vital signs are normal. Joint
examination shows bilateral swelling and tenderness of the
second metacarpophalangeal joints, third proximal inter
phalangeal joints, and right wrist, along with tenderness
of the metatarsals.

Which of the following will be the most helpful diagnostic


tests?
(A) C-reactive protein and erythrocyte sedimentation rate
(B) Lyme disease enzyme linked immunosorbent assay
and Western blot serologies
(C) Rheumatoid factor and anti-cyclic citrullinated pep-
tide antibodies
(D) Rubella and parvovirus serologies

115
Self-Assessment Test

tt
(D

D
l^
UI
tr
CONT.
Which of the following is the most appropriate diagnostic
test to perform next?
Item 74
A 70 1'ear-old man is evaluated for a 3 n'eek history of
tr
.D
U! (A) Bone biopsy gradually increasing pain ar-rd swelling in the left knee.
vt
(B) Bronchoscopicbiopsy I{e has osteoarthritis. Six months ago he underu'ent total \
=
o joint arthroplastl, of the left knee. Current medication is
(C) Lip biopsy
acetaminophen.
(D) Skin biopsy On phl,sical examination, vital signs are normal.
.D
tt Examination of the left knee shows slight n'armth. a small
Item 72 effusion u'ithout erythema, and a well healed midline scar.
A 60-year-old man is evaluated for a l-year history of Examination olthe right knee is normal.
lower extremity edema. He has a 30-year history of poorly Laboratory evaluation reveals an er1'throc1,'te sedi
controlled ankylosing spondylitis. He also has intermit- mentation rate of 40 mm/h, leukocl,te count of 8000 pl.
tent uveitis. His only medication is naproxen; he has been (8.0 x 10'r,'L). and blood C-reactive protein level of 1.2 mg
reluctant to initiate biologic agents. dL (12 mg'l-).
On physical examination, blood pressure is 158/90 mm Hg.
Other vital signs are normal. He has kyphosis with immo Infection with which of the following organisms is most
bility of the cervical, thoracic, and lumbar spine. There is likely?
decreased range of motion of the shoulders and hips with
(A) Neisserio gonorrhoeae
30-degree flexion contractures at both hips. There is 2+
bilateral swelling of the lower extremities.
(B) Pseudomonas oeruginosa
(C) Stophylococcus epidermidis
laboratory studies:
Albumin 2.5 gldL(2s glL) (D) StreptococcLts pAogenes
C-reactive protein 6.3 mg/dl (0S mg/L)
Creatinine 1.8 mg/dl (fSg.f pmol/L)
Urinalysis 3+ protein; no erythrocytes, Item 75
leukocytes, casts, or eosinophils 19 year old woman is evaluated for recent onset of a
A
Protein-creatinine ratio 5200 mg/g malar rash and arthralgia, which Iimits her daily activities.
On physical examination, vital signs are normal. She
Which of the following is the most likely diagnosis?
has an edematous, erythematous malar rash and small joint
(A) Analgesicnephropathy tenderness and swelling in several proximal interphalan
(B) IgAnephropathy geal joints, metacarpophalangeal joints, wrists, and ankles.
(C) Interstitialnephritis The remainder of the physical examination is normal.

(D) Renal amyloidosis laboratory studies:


Erythrocyte sedimentation rate 35 mm/h
Platelet count 92,OOOl1tL(92xr}elL)
Item 73
Comprehensive metabolic panel Normal
A 36-year-old woman is evaluated for a recent diagnosis C3 complement Normal
of rheumatoid arthritis. Over the past 2 months she has C4 complement Normal
had increasingly severe pain and swelling in the hands and Anti-double stranded DNA Positive
wrists and pain in the feet. Her only medication is a com antibodies
bined oral contraceptive. Antinuclear antibodies 1:1280 homogenous
On physical examination, vital signs are normal. Joint Urinalysis Normal
examination reveals swelling and tenderness of the left
second, third, and flfth proximal interphalangeal joints; Chest radiograph is normal.
right second and third proximal interphalangeal joints; Sun protection, including use of a broad spectrum
bilateral second, third, and fourth metacarpophalangeal sunscreen, is recommended.
joints; right wrist; left ankle; and left second and third
metatarsophalangeal joints. Which of the following is the most appropriate additional
Radiographs of hands and feet demonstrate juxta treatment?
articular osteopenia without erosions. (A) Prednisone
Laboratory evaluation reveals a hemoglobin level of
10.8 g/dl (108 g/L), leukocyte count of s4O0/pL (5.4 x
(B) Prednisoneandhydroxychloroquine
l0eil) with normal diflerential, and platelet count of (C) Prednisone, hydroxychloroquine, and cyclophospha-
435,000/gL (435 x l0'/L). mide
(D) Topical hydrocortisone cream
Which of the following is the most appropriate treatment?
(A) Diclofenac
(B)
(C)
Methotrexate
Mycophenolatemofetil
Item 76
A 50 year-old man is evaluated for pain, swelling, ten- tr
derness, and redness ofthe right great toe that began last
(D) Rituximab lright. He has hacl several similar episodes over the past

116
5elf-Assessment Test

tt
pl 2years. He has type 2 diabetes mellitus and a recent diag- Which of the following is the most appropriate diagnostic
G'

l*l nosis of deep venous thrombosis. Current medications are test to perform next?
(l,
coNT apixaban, metfbrmin, and rosuvastatin.
(A) Brain MRI E
On physical examination, the right first metatarso tt
vt
phalangeal joint is tender. red. warm, and swollen. (B) ChestCT (l,
ta
Laboratory evaluation reveals a serum urate level of (C) Echocardiography ta
9.0 mg/dl (0.53 mmol/L). (D) Kidney biopsy
Radiograph of the right foot is shown. =
tto
Item 78
A 31-year old man is evaluated following a recent diagno-
sis of ankylosing spondylitis. He has low back and pelvic
pain that interrupts his sleep. Acetaminophen has not
helped.
On physical examination, vital signs are normal. The
FABER (Flexion, ABduction, and External Rotation) test of
the hip elicits pain at both sacroiliac joints. Lumbar spine
range of motion is slightly limited. There is good range of
motion of the cervical spine and peripheral joints, includ-
ing the hips. No peripheral joint swelling or tenderness is
noted.
Radiographs of the lumbar spine show bilateral
sacroiliitis.
The patient is referred to physical therapy.

Which of the following is the most appropriate additional


treatment?
(A) Etanercept
(B) Methotrexate
(C) Naproxen
Which of the following is the most appropriate initial (D) Sulfasalazine
treatment?
(A) Allopurinol
Item 79
(B) Colchicine
A Sl-year-old woman is evaluated at a follow-up visit.
(C) Indomethacin Sj0gren syndrome was diagnosed 3 years ago. For the past
(D) Prednisone 4 weeks she has experienced arthralgia, unintentional
weight loss, fever, and drenching night sweats. She has
hypothyroidism. Current medications are levothyroxine,

tr Item 77
A24 year old woman is hospitalized fbr rapidly progressive
artiflcial tears, and ophthalmic cyclosporine suspension.
On physical examination, vital signs are normal.
Parotid glands are enlarged, and the oropharynx is dry.
lower extremity edema and rising serum creatinine level. She
Lungs are clear to auscultation. There are no skin lesions or
is diagnosed with new-onset systemic lupus ery,.thematosus.
On physical examination, blood pressure is 162/94 mm
evidence of joint synovitis.
Hg, and pulse rate is 98/min. An erythematous malar rash Laboratory studies:
is present. The metacarpophalangeal joints are swollen Erythrocyte sedimentation rate 73mm/h
bilaterally. Lower extremity edema and facial puffiness Hemoglobin ro.2gldL (102 g/L)
are present.'lhe cardiopulmonary examination is normal. C3 complement 102mg/dl (1020 mg/L)
C4 complement 8 mg/dl (80 mg/L)
Laboratory studies:
Thyroid-stimulating hormone 3.0 pU/mL (3.0 mU/L)
C3 complement Low
Rheumatoid factor 442UlmL(442kUlL)
C4 complement Low
Anti-cyclic citrullinated peptide Negative
Creatinine 1.3 mg/dl (114.9 pmol/L)
antibodies
Anti double stranded DNA Positive
Anti-La/SSB antibodies Negative
antibodies
Anti Ro/SSA antibodies >8 U/L (normal, <1 U/L)
Antinuclear antibodies 1:1280 (speckled pattern)
Urinalysis O 1+ protein; no
Urinalysis 3+ protein; 2+ blotidl 35
erythrocytes, Ieukocytes,
erythrocytes/hpf: no
or casts
leukocytes; no casts
Spot urine protein creatinine 600 mg/g Immunoflxation studies show a monoclonal IgM
ratio gammopathy (r type).

117
Self-Assessment Test

ur
.D
+
D
Which of the following conditions is most likely (C) Polymyalgia rheumatica
U! responsible for her recent symptoms? (D) Rheumatoid arthritis
Ut
.D
UI (A) AL amyloidosis (E) Systemic lupus erythematosus
(a
J (B) Hypothyroidism
.D (C) Lymphoma Item 82
(D) Rheumatoidarthritis
.D A72 year-old man is evaluated for chronic right knee pain
t^ with morning stiffness lasting 15 minutes. He is experi
encing increased difficulty walking because of knee pain.
tr Item 8O
An 18 year old woman is evaluated lbr a I week history
He has no other medical problems other than overweight.
His only medication is acetaminophen, which provides
of'f'ever and dilfuse myalgia, and pain and slvelling in the minimal pain relief.
hands and knees that have progressed over the last month. On physical examination, BMI is 34. There is bony
She also has acnc. Current medicatiorrs are minocl,cline. hypertrophy of the knee without effusion or signs of
topical tretinoin, and an oral contraceptive. inflammation. He has moderate pain with lull knee flexion
On physical examination, temperature is 38.3 'C but no restriction of motion. The remainder of the exam
(100.9 "F)i other vital signs are normal. She has synovitis ination is noncontributory.
in her hands and wrists and inflammatory acne on the Iace Topical diclofenac is prescribed.
and upper back.
laboratory studies: Which of the following is the most appropriate additional
Complete blood count Normal treatment?
Erythrocyte sedimentation rate 36mm h
Normal
(A) Exercise and weight loss
Creatinine
Anti cyclic citrullinated peptide Negative (B) Massage therapy
antibodies (C) Mobilization/manipulation and passive range of
Antinuclear antibodies 1 :320 (speckled pattem) motion
ANCA Positive (p ANCA (D) Transcutaneous electrical nerve stimulation
pattern)
Rheumatoid factor Negative
Urinalysis Nornral Item 83
Test results ft-rr antihistone and othcr specific autoanti A 42 year-old woman is evaluated for increasingly severe
bodies are negative. and serum complement levels are normal pain and swelling in both hands for the last 6 months. She
has morning stiffness that lasts t hour. Current medication
Which of the following is the most likely diagnosis? is ibuprofen.
On physical examination, vital signs are normal. Joint
(A) Drug induced lupus erythematosus examination reveals bilateral swelling and tenderness of
(B) Granulonratosis'^'ith polyangiitis the second and third metacarpophalangeal joints and third
(C) Rheumatoidarthritis proximal interphalangeal joint.
(D) Systemic lupus erythematosus laboratory studies:
Hemoglobin 10.8 gi dL (108 g/L)
Rheumatoid factor Positive
Item 81 Anti-cyclic citrullinated Positive
A 37 year-old woman is evaluated for a l-year history of peptide antibodies
widespread joint and muscle pain, fatigue, poor sleep, Antinuclear antibodies Negative
and difflculty focusing. She also has irritable bowel syn
Which of the following is the most appropriate test to
drome and migraine headaches. Current medications are
perform next?
sumatriptan and topiramate.
On physical examination, vital signs are normal. No (A) ANCA
rash is present on the face or extremities. Joint examina (B) Anti double stranded DNA antibodies
tion shows normal range of motion and no joint swelling. (C) MRI of hands
Most soft tissue is tender to light palpation. Muscle strength
is normal.
(D) Plainradiographyofhands
Laboratory evaluation shows a normal complete blood
count, serum thyroid stimulating hormone level, and
urinalysis as well as an erythrocyte sedimentation rate of Item 84
20 mm/h. A 3O-year old woman is evaluated for a 2 month history
of skin changes, primarily on her chest and arms. She
Which ofthe following is the most likely diagnosis? has no other symptoms. Her only medication is an oral
contraceptive.
(A) Fibromyalgia On physical examination, vital signs are normal. The
(B) Generalized osteoarthritis rash on her chest is shown (top ofnext page).

118
)
Self-Assessment Test
t
I v!
I 6'
swollen and warm, with erythema, tenderness to palpation,
and limited range of motion due to pain. Other joints are
I

t (u
normal.
t r/l
r,l
I Which of the following is the most appropriate diagnostic (l,
t
test to perform next? vt
t
t (A) Aspiration of the left knee o
(B) Erythrocyte sedimentation rate arl
t
(C) Radiographyoftheleft knee
t (D) Serum urate level
t
t
t
Item 87
A S5-year old woman is evaluated for a 5,day history of tr
There is no evident scarring or lesions in the scalp increasing pain, swelling,, and warmth in the left knee.
She has a 30-year history of rheumatoid arthritis. Current
t or ears, hair loss, or joint swelling. The remainder of the
medications are etanercept, prednisone, methotrexate,
examination is normal.
t Laboratory evaluation reveals an antinuclear antibody folic acid, and meloxicam.
titer of 1:640 with speckled pattern; result for anti-Ro/SSA On physical examination, temperature is 37.5 "C
t antibody is positive.
(99.s "F); nther vital signs are normal. Joint examina-
tion of the hands reveals nonpainful deformities and
L
restricted movement characteristic of advanced but
Which of the following is the most likely diagnosis?
quiescent rheumatoid arthritis. The left knee exhibits
t (A) Acute cutaneous lupus erythematosus warmth and a moderate effusion without erythema; flex-
t (B) Cutaneousleukocytoclasticvasculitis ion is restricted because of pain. The right knee shows
I
(C) Discoid lupus erythematosus bony hypertrophy.
L Laboratory evaluation shows an erythroqrte sedimen,
(D) Subacute cutaneous lupus ery,thematosus
I tation rate of 7O mmlh and leukocyte count of 13,500/pL
L (13.5 x lOelL).

t
I
I
tr Item 85
A 78-year-old woman is evaluated for constant bilateral
Which of the following is the most appropriate diagnostic
test to perform next?
headache of I week's duration. Two months ago, she was
diagnosed with polymyalgia rheumatica. Symptoms were (A) C'reactive protein
;
:
relieved with prednisone, 15 mgld. Headache appeared (B) Serum procalcitonin
i shortly after the dosage was tapered to 5 mg/d. She also (C) Synovial biopsy
has experienced new-onset intermittent muscular jaw dis, (D) Synovial fluid analysis
; comfort with chewing.
On physical examination, vital signs are normal. The
scalp and temporal arteries are not tender to palpation. No
bruit is heard over the great vessels; temporal artery pulses
Item 88
are intact. A 3O-year-old woman is seen for preconception coun-
Laboratory evaluation shows a blood C reactive pro- seling. Three years ago, she was diagnosed with systemic
tein level of 12.8 mg/dl (128 mg/L). lupus erythematosus and nephritis. She responded rapidly
to induction therapy with combination immunosuppres-
Which of the following is the most likely diagrrosis? sants and has been maintained on mycophenolate mofetil
and hydroxychloroquine for the past 2 years. An intrauter
(A) Giant cell arteritis ine device is used for contraception.
(B) Granulomatosiswith polyangiitis Laboratory evaluation is consistent with quiescent
(C) Microscopicpolyangiitis disease, and there is no evidence of kidney dysfunc-
(D) Polyarteritisnodosa tion.

Which of the following is the most appropriate


preconception management?

tr Item 86
A 72-year-old man is evaluated for severe left knee pain
(A) Advise against pregnancy
(B) Continue current medications
with swelling that has worsened over the past 5 days. Pain
is exacerbated with weight bearing. He has no history of (C) Discontinue hydroxychloroquine and mycophenolate
trauma. Current medication is acetaminophen. mofetil; add prednisone
On physical examination, temperature is 38.2 "C (D) Discontinue mycophenolate mofetil; add azathio-
(100.8 "F). Other vital signs are normal. The left knee is prine

119
Self-Assessment Test

t/t
o
t^
t^
tr Item 89
A 60-year old man is evaluated in follow up after an
l-aboratory studies:
I Iematocrit
Leukocyte count
30"1,
4000/prl (4.0 x 10e,il). \^,ith lymphopenia
.D
t, episode of podagra. which u,as treated with colchicine.
UI This r.r,as his first gout flare. f{e also has hypertension and Platelet count T2,OOO StL (72 x 10'q L)

hyperlipidemia. Current medications are hydrochlorothi C reactive protein s.s mg/dl (55 mgrL)
.D
azide and atorvastatin. Creatinine 1.4 mg/dL (12:l.B pmollL)
Urinalysis 1+ protein, 1+ blood; 2 3 ery'throclles:
(D On physical examination. blood pressure and other
u) findings are normal. no leukocytes; no casts
Laboratory evaluation reveals a semm urate level of' Which of the following is the most likely cause of this
9.0 mgldl. (o.se mmoliL). patient's pericarditis?

Which of the following is the most appropriate (A) Adult onset Still disease
treatment? (B) Coxsackievirus infectior-t
(A) (C) Idiopathicpericarditis
Start allopurinol
(B) (D) Systemic lupus erythenlatosus
Start vitamin C
(C) Stop atorvastatin and start fenofibrate
(D) Stop hydrochlorothiazide and start losartan Item 92
A 4S-year-old woman is evaluated for progressive short
ness of breath for 6 months, hearing impairment, and knee
Item 90 pain. She has no additional medical problems and takes no
A 57-year-old woman is evaluated for a 2.5 year history medications.
I
of pain and swelling in the hands and wrists. She also has On physical examination, vital signs are normal; oxygen
joint stiffness for over an hour after awakening. saturation is 98% with the patient breathing ambient air. She
On physical examination, vital signs are normal. She has swelling and redness of the helices of the ear, with sparing
has swelling and tenderness of the following: second, third, of the lobule; diminished hearing bilaterally; bilateral con-
and fourth proximal interphalangeal joints of the right junctivitis; and redness, tendemess, and flattening of the nasal
hand and third and fourth proximal interphalangeal joints bridge. Lung examination reveals inspiratory stridor loudest
of the left hand; the left second and right third metacarpo- over the trachea. Knees show joint-line tendemess bilaterally.
phalangeal joints; and both wrists. The remainder of the physical examination is normal.
Laboratory evaluation reveals a blood C*reactive pro Laboratory tests, chest radiography, and CT of the
tein level of 4.8 mg/dl (+s mg/L) and positive results on upper airways and chest are ordered.
tests for serum rheumatoid factor and anti cyclic citrulli-
nated peptide antibodies. Which of the following is the most likely diagnosis?
Radiographs of the hands show periarticular osteo-
penia and joint-space narrowing of the proximal inter-
(A) Cryoglobulinemia
phalangeal joints, periarticular osteopenia and marginal (B) Granulomatosiswithpolyangiitis
erosions at the second proximal interphalangeal and meta- (C) Relapsingpolychondritis
carpophalangeal joints of both hands, and radiocarpal (D) Rheumatoidarthritis
joint-space narrowing.

Which of the following imaging studies should be done Item 93


next? A 58-year-old woman is evaluated for a 4-month history
(A) Bone scanning of diarrhea and weight loss. Diffuse cutaneous systemic
sclerosis was diagnosed 10 years ago. She reports explosive
(B) MRI of hands diarrhea soon after eating a meal. She has lost 5 kg (11 lb)
(C) Ultrasonographyofhands since the symptoms began. She also has Raynaud phe
(D) No further imaging nomenon and gastroesophageal reflux disease controlled
with pantoprazole once a day. Current medications are
sustained release nifedipine and pantoprazole.

tr Item
A
91
40 year-old man is hospitalized for acute pericarditis. He
On physical examination, vital signs are normal. Skin
changes consistent with diffuse cutaneous systemic scle
rosis are present. Findings on the abdominal examination
also reports pain in the wrists, ankles, and knees for I week
are normal.
but no other symptoms. He has otherwise been well and
takes no medications.
On physical examination, temperature is 38.0 'C Which of the following is the most appropriate treatment?
(10o.+ "F): the ren.rainder of'the vital signs are normal. (A) Cholestyramine
A pericardial friction rub is present. The wrists, knees. (B) Ciprofloxacin
and ankles are tender. There are small bilateral wrist
and knee effusions. The remainder of the examination is (C) Increase in pantoprazole dosage
unrcmarkable. (D) Loperamide

120
Self-Assessment Test

ut
q,
Item 94 Which of the following is the most appropriaG
A 29-year-old man is evaluated after an episode of anterior management?
(u
uveitis. He has 2 year history of low back stiffness at night (A) Cyclophosphamide E
and in the morning that improves with activity. Current (B) Immediate delivery ut
t
medication is meloxicam as needed. (l,
(C) Magnesium sulfate vt
On physical examination, he cannot touch his toes, UI

and there is flattening of the normal lumbar lordosis. Inter- (D) Prednisone
nal rotation and flexion elicit pain in the right hip; range (u
r/t
of motion of the right hip is diminished. Flexion, external
rotation, and abduction ofthe hips cause pain in the right Item 96
hip and low back. The remainder of the examination is
unremarkable. A 46-year old man is evaluated for a S-year history of
pain in the lower spine, knees, and hands. The hand
pain is associated with occasional swelling. Morning
Which of the following is the most appropriate diagnostic
stiffness lasts less than 30 minutes. Type 2 diabetes
test to perform next?
mellitus was diagnosed 2 years ago. His only medication
(A) Anteroposteriorradiographyofpelvis is metformin.
(B) Antinuclear antibody assay On physical examination, there is bilateral enlargement
(C) HLA-B27 testing of the second, third, and fourth metacarpophalangeal joints
as well as all proximal and distal interphalangeal joints. The
(D) Sacroiliac joint MRI medial joint line of each knee is tender to palpation.
Iaboratory studies:
Alanine aminotransferase
tr Item 95
A 2S-year-old woman is hospitalized at 26 weeks of preg-
48UlL
Aspartate aminotransferase 52U lL
Calcium Normal
nancy for lower extremity swelling, proteinuria, and elevated
C-reactive protein Normal
blood pressure. Systemic lupus erythematosus was diagnosed Ferritin Elevated
last year. Her disease has been well controlled, with stable Transferrin saturation Elevated
laboratory values and no recent clinical evidence of disease
activity. Her only medication is hydroxychloroquine. Radiograph of both hands is shown.
On physical examination, blood pressure is 152/
90 mm Hg, and pulse rate is 80/min. Facial swelling, bilat-
eral lower extremity edema, swelling of both hands, and
tenderness over the metacarpophalangeal joints are pres-
ent. She has a gravid abdomen with normal fetal heart
tones.
Iaboratory studies:
8 Weeks Ago Current
Platelet count Normal 85,000/pL (85 x
t}e tL)
Alanine Normal Normal
aminotransferase
Aspartate Normal Normal
aminotransferase
Serum urate 4.5. mg/dl
(0.30 mmoliL)
Anti-double 50 250
stranded DNA
antibodies
C3 complement 85 mg/dl 60 mg/dl
(8so mg/L) (600 mg/L)
C4 complement 16 mg/dl 4mgldL (aO mg/L) Which of the following is the most appropriate diagnostic
(160 mg/L) test to perform next?
Urinalysis Normal 3+ protein; no (A) HFEgenetesting
erythrocytes; no (B) Insulin-like growth factor 1
leuko€ytes; no casts
(C) Parathyroidhormone
Protein-creatinine 50mglg 32Omglg
ratio (D) Rheumatoid factor

121
I
'l
I

't
I
'l
I
\
i
t
I
'I
!
I
'l
I

I
I
I
I

I!
t
1
I
1
'l
t
t
I
iI
I
l
I
!
I
tt
j
I I
'l
I

I
l
I I
1
I
.t
I

I
'l
I
I
!
I

I
1
,
1
I
II
!
i
l{

1
Answers and Critiques

Item 1 Answer: A Item 2 Answer: B


Educational Objective: Treat mucocutaneous Educational Objective: Treat familial Mediterranean fever.
ulcerations in Behqet syndrome with apremilast.
The most appropriate treatment is colchicine (Option B).
The most appropriate treatment is apremilast (Option A). Autoinflammatory syndromes are monogenic diseases that
Behqet syndrome is a systemic vasculitis characterized by result in periodic episodes of system inflammation. This
recurrent oral aphthae. Other manifestations include gen, patient demonstrates multiple episodes of a systemic inflam-
ital aphthae, ocular disease, skin lesions, gastrointestinal matory syndrome characterized by fever, abdominal pain, Ut
(t,
involvement, neurologic disease, vascular disease, and rash, and arthritis, each lasting only several days. This constel ET
arthritis. The treatment of oral and genital ulcers is guided by lation is characteristic of familial Mediterranean f'ever (FMF),
the severity of symptoms and the presence of other disease a disease driven by several mutations in the MEFV gene. tJ
.C,
manifestations. This patient's disease does not seem to be This gene encodes pyrin, a protein important to the pro
responding adequately to colchicine, which is an appropriate duction and/or overproduction of interleukin 1B. Although C,
vl
flrst line therapy for mucocutaneous ulcerations. Hence, an the genetics continue to be studied, from a clinical point of (l,
alternate or additional treatment should be tried. Apremilast view the disease is most commonly found in Jewish, Arab,
Ul
is an oral phosphodiesterase 4 inhibitor approved for treat and Turkish populations and follows an autosomal recessive =
ment of psoriasis, psoriatic arthritis, and prevention of oral pattern. Other symptoms may include other forms of serositis,
ulcers in Behget syndrome. It is a reasonable alternative to including pericarditis. During episodes, inflammatory mark
colchicine as a glucocorticoid-sparing agent for patients ers are elevated; in between episodes the physical examina
with recurrent oral ulcers. Apremilast must be started at a tion and laboratory results may be normal. Diagnosis is made
low dose and uptitrated over a few days to achieve a main clinically and can be confirmed definitively in most patients
tenance dosage of 30 mg twice daily. Clinical trial data also through genetic testing. One long term consequence of the
suggest a positive impact on genital ulcers, with a trend episodic inflammation of FMF is amyloidosis, including of the
toward improvement, but the studies were not powered to kidneysr this patient's low level proteinuria may be an early
assess this outcome. Apremilast is usually well tolerated but manifestation. Other autoinflammatory syndrornes are rarer
is associated with adverse events, such as diarrhea, nausea, and have similar but nonidentical features that allow them to
and headache. It should be used cautiously in patients with be distinguished from FMF. First-line treatment oIFMF is lile
depression. Systemic glucocorticoids in tapering doses can long daily prophylaxis u,'ith colchicine. which in most cases
be used in patients who are refractory to colchicine and prevents attacks, prevents renal amyloidosis, and can inhibit
apremilast. However, if the patient requires continued sys neutrophils and suppress interleukin 1 B generation.
temic glucocorlicoid therapy, azathioprine. thalidomide, or Canakinumab (Option A) is a monoclonal antibody to
tumor necrosis factor inhibitors are most appropriate. interleukin-lB that has been reported to successfully treat
Ixekizumab (Option B) is an interleukin-l7a inhibitor patients with FMF, including those in whom colchicine has
approved for psoriasis, psoriatic arthritis, and ankylosing spon- failed. However, canakinumab is expensive and immuno
dylitis. It has not been studied for or shown to be eflective in suppressive and has not been FDA approved for FMF treat
treating mucocutaneous manifestations of Behqet syndrome. ment. Given the long experience with colchicine and its
There are no data on use of leflunomide (Option C) in efficacy, canakinumab should be reserved for patients with
the treatment of mucocutaneous manifestations of Behget FMF in whom colchicine has failed or is not tolerated.
syndrome. NSAIDs, such as indomethacin (Option C), may alle-
Mycophenolate mofetil (Option D) is not effective in viate symptoms during FMF attacks but do not address the
the treatment of mucocutaneous manifestations of Behget underlying mechanisms of FMD and are not appropriate
syndrome and should not be used unless the patient has preventive or long term therapy.
tried and not responded to other agents. Glucocorticoids. such as prednisone (Option D). n.ray
decrease the duration of attacks but also increase their fre
XEY POIT{I
quency. Thus, they are not appropriate for long-term or
. Apremilast is a reasonable alternative to colchicine as preventive FMF therapy.
a glucocorticoid-sparing agent for recurrent oral
xtY P0l1lr5
ulcers in Behqet syndrome.
. Familial Mediterranean fever is a systemic inflamma
Bibliography tory syndrome characterized by fever, abdominal pain,
Hatemi C, Christensen R, Bang D. et al. 2018 update ofthe EULAR recom rash, and arthritis, each lasting only several days.
menditions fbr the management of Behqet's syndrome. Ann Rheum Dis. (Continued)
2018;77:8o8 gtA. IPMlD, 296259681

123
i

Answers and Critiques


:

!
l(EY P0lllTS (confnrcd) flndings, including malar rash, discoid rash, and alopecia.
r This patient's presentation is not compatible with SLE.
First-line treatment of familial Mediterranean fever is
I

lifelong daily prophylaxis with colchicine, which in IEY POITTS


most cases prevents attacks, prevents renal amyloido- o Five clinical subtypes of psoriatic arthritis, which may ;

sis, and can inhibit neutrophils and suppress overlap, are recognized: symmetric polyarthritis,
:
i
interleukin-1 p generation. asymmetric oligoarthritis, distal interphalangeal- :
predominant disease, spondyloarthritis, and arthritis !

Bibliography I
mutilans.
Georgin-Lavialle S, Ducharme-Benard S, Sarrabay G, et al. Systemic autoin
flammatory diseases: Clinical state of the art. Best Pract Res Clin o Enthesitis, tenosynovitis, and dactylitis often occur in
Rheumatol. 2020r34:101529. IPMID: 32546426] psoriatic arthritis; dactylitis is found in 40"/,' to 50% of
patients and should always prompt consideration of a
spondyloarthropathy.
TA
Item 3 Answer: B
(D
= Ed ucati o na I O bjective : Diagnose psoriatic arthritis. Bibliography
UI
Ritchlin CI, Colbert RA, Gladman DD. Psoriatic arthritis. N Engl j Med.
o,
The most likely diagnosis is psoriatic arthritis (PsA) (Option 2017 ;376:957 -97O. [PMID: 28273019] doi:10.1056 NEJMral505557
CL
B). Five clinical subtypes of psoriatic arthritis, which often
n overlap, are recognized: symmetric polyarthritis, asym-
.i
llg metric oligoarthritis, distal interphalangeal-predominant Item 4 Answer: A
(D disease, spondyloarthritis, and arthritis mutilans. Enthesi-
ta Educational Objective: Diagnose granulomatosis with
tis, tenosynovitis, and dactylitis often occur. Dactylitis is
polyangiitis.
found in 40'X, to 50% of patients with PsA and should
always prompt consideration of a spondyloarthropathy. The most appropriate diagnostic test to perfbrnr next is kid
This patient has a classic distribution for PsA. The distal ney biopsy (Option A). lhis patient's presentation is highly
interphalangeal joints and an entire digit are involved (i.e., suspicious lor granulomatosis r.r,ith polyangiitis (GPA).
dactylitis), with sparing of other digits. This patient has evi- The disease course began with nonspecific symptoms but
dence of limited psoriasis, but the extent of psoriatic skin progressed to involve the upper and lower ainvays, eyes.
involvement may not correlate with the extent of arthritis. skin. and kidneys. lhe skin rash. glomerulonephritis.
In patients with PsA, affected peripheral joints sometimes and nodular lung disease are consistent with a vasculitic
take on a purplish discoloration due to hypervascularit5r. process. In GPA lvith kidney involvement. kidney biopsy
The Classification Criteria for Psoriatic Arthritis (CASPAR) is expected to show pathognomonic findings of pauci
include evidence of psoriasis (current, past, and family imnrune crescentic necrotizing giomerulonephritis.
history in first or second degree relatives), nail dystrophy, Whenever possible. it is important to confirm a diagnosis
dactylitis (current or past), radiographic flndings showing of GPA r,vith tissue pathology before beginning treatment.
new bone formation, and a negative rheumatoid factor given the many risks that accompanv intensive immuno
result. Each of these is assigned 1 point except for current suppression; prognostic data can help guide therapy as
psoriasis, which is assigned 2 points. These criteria are well. Guideline directed initial therapy fbr active. severe
91% sensitive and 98% speciflc for the diagnosis of PsA if GPA is either intravenous pulse glucocorticoids or high-
3 or more points are present. This patient's score is greater dose oral glucocorticoids plus rituximab (prelerred) or
than 3. cyclophosphamide. If the diagnosis is clear and urgent
This patient's symptoms are inflammatory making treatment is necessary. however, biopsy may be bl,passed
osteoarthritis (Option A) unlikely despite involvement of (in this case. for example, if the ANCA panel u'ere positive
the distal interphalangeal joints. In addition, osteoarthritis and the patient \ rere more acutely ill).
does not cause dactylitis. biopsy (Option B). although less invasive than
Sir.rus
Rheumatoid arthritis can be seronegative (Option C) kidney or lung biopsy, has poor sensitiviry- and therefore
in 15% to 2O"1, of patients classifled as having the disease, shoulcl not be performed ur.rless the diagnosis is in question
but rheumatoid arthritis is typically a symmetric arthri- and no other tissue is alailable.
tis and does not affect all of the joints in an entire digit, Althor.rgh skin biopsl, (Option C) should shon abnor-
including the distal interphalangeal joint, as seen in this malities collsistent rvith a small vessel vasculitis. the find
patient. ings lvould be nonspecific.
Joints are affected in 90'1, of patients with systemic Thoracoscopic lung biopsy (Option D) might show vas
lupus erythematosus (SLE) (Option D). The most common culitic changes consistent with GPA. but this procedure
involvement is polyarthralgia, with lrank arthritis occur- has greater morbidiry than kidnel, biopsli particularly in
ring in 40%. Typical distribution involves the small periph a patient u'ith tenuous respiratory status. Transbronchial
eral joints. Dactylitis is not seen in SLE. Most patients with biopsy is more benign thar.r open lung biopsy but is generally
SLE and arthritis also have characteristic mucocutaneous not perfbrmed. given its low sensitivity (about l0'7,).

124
Answers and Cr itiques

XEY POIl{TS I(EY POITIS


o It is important to confirm a diagnosis of granulomato- o A positive HLA-P.27 test result in the setting of a com-
sis with polyangiitis by using tissue pathologr before pelling clinical picture confirms the diagnosis of axial
beginning treatment, given the many risks that spondyloarthritis.
accompany intensive i m munosuppression. . If a patient has many features of spondyloarthritis,
o Standard therapy for granulomatosis with polyangiitis then a positive HLA-827 result adds little to the post-
includes high-dose glucocorticoids plus rituximab test probability; ifthere are few features ofspondy-
(preferred) or cyclophosphamide. loarthritis, then a positive HLA-B27 result is helpful.

Bibliography Bibliography
Geetha D. Jefferson JA. ANCA-associated vasculitis: core curriculum 2020. Danve A, Deodhar A. Axial spondyk)arthritis in the USA: diagnostic chal
Am J Kidney Dis. 2020;75:124 137. [PMID: 3135$11] doi:10.1053/j.ajkd. lenges and missed opportunities. Clin Rheumatol. 20l9rll8:625 634 U!
2019.0,1.031 IPMID: 305885551 doi:10.1007/slO067 O18 4397 3 o
ET

Item 5 Answer: C Item 6 Answer: B (J


E'
Ed u catio n a I O biective : Evaluate for spondyloarthritis Educational Objective: Evaluate a patient by using
tu
with HIA-B27 testing. antinuclear antibody testing. Ut
(l,
The most appropriate diagnostic test to perform next is HLA The most appropriate laboratory test to perform next is anti-
ta
=
B27 antigen testing (Option C). The test assesses for the nuclearantibody (ANA) measurement (Option B). This patient E
presence ofthe HLA B27 antigen on the surface ofcells and has speciflc features of autoimmune disease, inflan.rmatory
not the presence of the actual gene. This patient has inflam- polyarthritis, and photosensitive facial rash, along with fever (a
matory low back pain manifesting as night pain, morning nonspecific systemic feature). ANA is directed against nuclear
stiffness, and pain that improves with activity. He also has antigens and associated with systemic lupus erythematosus
enthesitis of the Achilles tendon, which is seen in spon (SLE). Up to one third of the healthy population has a low
dyloarthritis. A positive HLA P.27 antigen test result in the titer (1:40-1:80) for ANA, and up to 5'7, has a titer of 1:160 or
setting of a compelling clinical picture conflrms the diagno higher. ANA can also be seen in other autoimmune condi
sis of axial spondyloarthritis. If the HLA,-827 antigen result tions, infection, and malignancy or may be drug induced. An
is negative, MRI of the pelvis to evaluate for sacroiliac joint isolated positive ANA result with nonspecific symptoms and
inflammation is warranted. In a patient with inflammatory normal clinical examination does not establish the diagnosis of
low back pain and a positive HLA-827 antigen result but no a connective tissue disease. In a patient with flndings strongly
other features of spondyloarthritis, MRI is also indicated. In suggesting an underlying rheumatologic disease, such as this
general, HLA B27 antigen testing adds probabilistic certainty patient with features of SLE, ANA testing is appropriate. ANA
to the evaluation oflow back pain. Ifdiagnostic certainty is testing should be done befbre any subserolog, testing. Il the
already high (i.e., the patient has many features of spon- ANA test result is positive, then specific subserologies are indi
dyloarthritis), then a positive test result adds little to the cated on the basis ofthe clinical scenario. Ifthe ANA test result
posttest probability; if there are few features of spondyloar is negative, then the ANA subserologic findings are typically
thritis, then a positive HLA-827 antigen result is helpful. negative. Therefore, if the initial ANA result is negative, then
An anteroposterior radiographic view of the pelvis is subserologr testing is not indicated.
useful and is the recommended view for detecting sacro- ANA specificity or subserologr testing (i.e., testing lor
iliitis. However, plain radiographs may not show evidence of antibodies to specific nuclear components, such as DNA or
sacroiliitis for 1 to 2 years after symptoms develop. Thus, if centromeres) should be reserved for patients with a positive
the anteroposterior radiograph is unremarkable, additional ANA result and a clinical syndrome suggesting an underly
views (Option A) are not needed. ing rheumatologic disease. lf the ANA result is positive and
In cases of suspected sacroiliitis, low dose CT (Option clinical flndings suggest a specific autoimmune inflamma
B) is useful to clarify ambiguous changes seen on plain tory disease, then subserolos/ testing is appropriate. ln this
radiographs, whereas MRI (Option D) is more sensitive and patient, if the ANA result is positive, SLE specific autoan
can identiflz sacroiliac inflammation even in the absence of tibodies (anti-double stranded DNA [Option A], anti Ro/
radiographic changes. Neither imaging test is indicated at SSA and anti LalSSB antibodies [Option C], anti Smith
this time, pending results of HIA B27 testing. [Option D], anti Ul ribonucleoprotein [Option E]) should
Rheumatoid arthritis is an inflammatory polyarthritis be obtained to further characterize the disease. Anti double
with a predilection lor the small joints of the hands and feet. stranded DNA antibodies are 95'1, specific for SLE and
Rheumatoid arthritis can affect the cervical spine but not the approximately 50'7, to 60'1, sensitive. They are typically found
lumbar spine or sacroiliac joints. This patient has no clinical in more severe disease, especially kidney disease. Antibody
flndings that suggest rheumatoid arthritis; thus, serologic levels commonly follow disease activity and are useful to
testing (Option E) is not indicated. monitor during therapy. Anti-Smith antibodies are the most

125
Answers and Critiques

speciflc for SLE (99%) but have low sensitivity. Antibody Medical cannabis (Option B) is increasingly available for
levels do not correlate with disease activity. Anti-Ul- use in patients with chronic pain. Many states have passed
ribonucleoprotein antibodies (particularly high titers) are laws legalizing the use of cannabis or allort'ing its use for
sensitive for mixed connective tissue disease. Antibody levels certain medical conditions, although it is still classifled by
do not correlate with disease activity. In patients with SLE, the U.S. Drug Enfbrcement Administration as a schedule I
anti-Ro/SSA and anti-La/SSB antibodies are associated with agent. Current data on the effectiveness of medical cannabis
photosensitive rash. In addition, offspring of mothers who for chronic pain are characterized by signiflcant heterogene
are positive for anti-Ro/SSA or anti LalSSB are at increased ity in both patient populations and cannabis preparations.
risk for neonatal lupus erythematosus (rash and congenital There are no high quality data on the efficacy of medical
heart block). However, none of these antibodies should be cannabis speciflcally for fi bromyalgia.
obtained until the ANA test result is conflrmed positive. Although tramadol may be beneficial in moderating
symptoms in patients with fibromyalgia, possibly because
ltY PottT
D of its inhibition of serotonin and norepinephrine reuptake.
(a . Antinuclear antibody (ANA) specificity or subserologz other opioids, such as oxycodone (Option C). have no evi
testing (testing for antibodies to specific nuclear com- dence of efficacy. In patients with flbrom),algia. the use of
=
.D
ponents) should be reserved for patients with a posi-
IA opioids other than tramadol is more likely to result in harm
o, tive ANA result and a clinical syndrome suggesting an than in beneflt.
underlying connective tissue disease. Patients with flbromyalgia do not respond to anti-
n inflammatory agents. Trials consisting of therapeutic doses of
.st Bibliography naproxen, ibuprofen, and prednisone (Option D) have found
(D
Mulhearn B. Tansley SL. McHugh NJ. Autoantibodies in connecti\e tissue each to be equivalent to placebo in randomized clinical trials.
disease. Best Pract Res Clin Rheumatol. 2020r3.1:101.162. IPMID,
UI These agents are not indicated in patients with fibromyalgia.
318,180551 doi: t0. 1016, j.berh.2019.101.162

xEY ?OTXTS
. FDA approved medications for fibromyalgia include
Item 7 Answer: E
pregabalin, duloxetine, and milnacipran.
Educationa I Objective: Treat fibromyalgia with o Patients with fibromyalgia do not respond to anti
pregabalin.
inflammatory drugs, including NSAIDs and glucocor
The most appropriate additional treatment is pregabalin ticoids. and do not respond to opioids, with the
(Option E). Optimal management of fibromyalgia requires a exception of tramadol.
holistic approach, including education. exercise, and psycho-
social support. Pharmacotherapy is often warranted, although Bibliography
nonpharmacologic measures remain a cornerstone of treat- Nlacfarlane (iJ. Kronisch C. Dean L[:. et al. F-ULAR revised recommendations
ment. Pregabalin, an antiepileptic drug. improves quality of fbr the management of fibromlalgia. i\nn Rheum Dis. 2017:76:318 :128.
[P1\,1 I D: 27:]7781 5l doi : I 0. I I i]6 annrheumdis 2J16 2097 21
life and decreases pain. It is one of only a few medications that
are FDA approved for treating flbromyalgia. The medication
should be initiated at low doses at night (i.e.. 50 or 75 mg at Item 8 Answer: B
bedtime) and slowly titrated as tolerated up to a maximum
Educational Obiective: Diagnose diffuse idiopathic
dosage of 225 mg t\,vice daily. Common adverse effects include
skeletal hyperostosis.
drowsiness, dizziness, and peripheral edema. Weight gain of
7% or more is seen in nearly 10'/. of patients. Off label use The most likely diagnosis is diffuse idiopathic skeletal hyperos
of gabapentin. another antiepileptic drug, has demonstrated tosis (DISH) (Option B). DISH is a noninflammatory condition
effectiveness in a randomized controlled trial and is frequently that occasionally is associated w,ith elevated inflammatory
used for this disorder in dosages up to 600 mg twice daily. markers and causes calcification and ossification of spinal
Prescribing duloxetine (Option A) would be appro ligaments (especially the anterior longitudinal ligament) and
priate in a patient with flbromyalgia. particularly one with entheses (tendon and ligament attachments to bone). This
depression, but not in one receiving escitalopram. Adding a leads to pain and stiffness as well as reduced range of motion
serotonin-norepinephrine reuptake inhibitor atop a selec of the spine. DISH is more common in men aged 45 years and
tive serotonin reuptake inhibitor would pose an unaccept older and most often involves the thoracic spine. DISH does
able risk for serotonin syndrome. Duloxetine is otherwise not affect the sacroiliac joints. This patient presents u,ith clas
an appropriate medication to use in Iibromyalgia, at a dos sic epidemiologic features: he is an older man. he has typical
age of up to 60 mg daily; consultation with the patient's symptoms and findings of DISH consisting of back pain and
psychiatrist would be essential if duloxetine needed to be stiffness without sacroiliac pain. and his radiograph shows
considered. Other medications that would be appropriate the distinctive finding of flowing linear calcification and ossi
for a patient not already taking a selective serotonin reuptake fication along the anterolateral aspects of the vertebral bodies
inhibitor would include milnacipran and off label use of (Figure [top of next pagel). These distinctive radiographic
tricyclic antidepressants (amitriptyline and nortriptyline). findings are most easily visualized on lateral images.

126
I

Answers and Critiques


t
I

Item 9 Answer: E
Educational Objective: Advise smoking cessation in a
I
patient at risk for rheumatoid arthritis.

The intervention most likely to reduce this patient's risk for


: developing rheumatoid arthritis is counseling for smoking ces-
sation (Option E). One of the most provocative environmental
: factors for the development of rheumatoid arthritis is smoking.
i A dose dependent relationship exists between smoking and
I

the development of seropositive rheumatoid arthritis. Smoking


can lead to lung inflammation, which activates such enzymes
I

as peptidylarginine deiminase, which deaminates arginine to


fbrm citrullinated peptides. HtA D alleles code for the shared UI
epitope. The shared epitope, a five amino acid sequence, pref-
o
erentially binds and presents citrullinated peptide antigens. ET

This leads to the production of anti-cyclic citrullinated peptide


rr,
antibodies that could initiate inflammation by fixing comple- E'
ment in the tissues. Patients who smoke are at increased risk for tg
rheumatoid arthritis, particularly those with a tamily history of la
rheumatoid arthritis, and should be counseled about smoking o
Arkylosing spondylitis (AS) (Option A) is an inflamma-
cessation. The risk for RA increases with an increasing number
tory arthritis ofthe spine, usually presenting as chronic back
ofsmoking pack years, and the risk decreases as the interval of
cUt=
pain and stiflness, typically before age 45 years. Radiographic
smoking cessation increases. Combining behavioral counseling
flndings include squaring of the vertebral bodies due to ante
with pharmacotherapy is more effective than either modality
rior and posterior inflammation, bone erosions, and formation
alone in achieving long term smoking abstinence.
of syndesmophytes, which are typically thin and vertically ori
A reduced risk for rheumatoid arthritis has been
ented and lead to anlrylosis. Involvement of the sacroiliac joints
associated with moderate alcohol consumption in some
is a hallmark of AS, and affected patients have elevated serum
observational studies. There is no reason for this patient to
inflammatory markers. This patient does not have the clinical,
discontinue alcohol consumption (Option A) to reduce her
radiographic, or laboratory findings characteristic ofAS.
risk for rheumatoid arthritis.
Calcium pyrophosphate deposition disease (Option C)
This patient should not be counseled to discontinue her
can involve the spine and has been associated with spine
combined oral contraceptive pill (Option B). Observational
stiffness and pain, as well as cartilage calcification and bony
studies have detected either a reduced risk fbr rheumatoid
an$osis. However, thick, flowing osteophytes are not typ
arthritis or no impact on its development in patients taking
ical ofthis disease.
a combined oral contraceptive.
Spondylosis defbrmans (degenerative disk disease with
lncreasing physical activity (Option C) has been incon-
osteophl.te formation) (Option D) is an important disorder
sistently associated with a reduced risk for developing rheu
to be considered in the differential diagnosis of DISH. This
matoid arthritis. There are other reasons to recommend
condition is usually seen in patients older than 55 years.
increased physical activity to this patient, but this recom
Although the spurs in the cervical and lumbar spine can
mendation should not be based on the possibility of reduc
resemble those seen in DISH. involvement of the anterior
ing the risk for rheumatoid arthritis.
longitudinal ligament in the thoracic spine characteristic of
Consumption of a Mediterranean diet (Option D) is not
DISH is not observed in spondylosis.
specifically shown to reduce the risk for rheumatoid arthri
xEY Potx?s tis. Higher intake of flsh may be associated with a lower risk
. Diffuse idiopathic skeletal hyperostosis is a nonin- for rheumatoid arthritis, but studies on this association are
flammatory condition that causes back pain and inconclusive, as are studies on the effect of red meat, coflee
consumption, and use of' antioxidants.
stiffness without sacroiliac pain, typically in men aged
45 years and older.
f,tY P0lft!,. .:.:..
o Diffuse idiopathic skeletal hyperostosis is associated
. Patients who smoke are at increased risk for rheuma-
with distinctive radiographic finding consisting of
a
toid arthritis, particularly those with a family history
flowing linear calcification and ossification along the
of rheumatoid arthritis, and should be counseled to
anterolateral aspects of the vertebral bodies.
stop smoking.

Bibliography
. Exercise, alcohol use, and dietary factors are not
Mader R, Verlaan JJ, Eshed l, et al. DifTuse idiopathic skeletal hyperostosis firmly associated with either increased or decreased
(DISH): where we are now and where to go next. RMD Open. 2017;3: risk for rheumatoid arthritis.
eOOO472. IPMID: 28955488] doi:10.1136/rmdopen 2017 000472

127
Bibliography Bibliography
Salliot C. Nguyen Y Boutron Ruault MC, et al. Environment and lifestyle: DeWane ME. Waldman R, Lu J. Dermatomyositis: clinical teatures and
their influence on the risk of RA. J Clin Med. 2020;9. IPMID: 32993091] pathogenesis. J Am Acad Dermatol. 2o2o;82:267 -281. IPMID: 312798081

Item 10 Answer: B Item 11 Answer: B


Educational Objective: Diagnose amyopathic Educational Objective: Diagnose inflammatory bowel
EI
dermatomyositis. disease in a patient with an$osing spondylitis.

The most likely diagnosis is amyopathic dermatomyosi The most likely diagnosis is inflammatory bo$'el disease
tis (Option B). Some patients with dermatomyositis never (lBD) (Option B). IBD occurs in up to 14'L of patients with
develop muscle involvement (amyopathic dermatomyositis) ; anl<11osing spondylitis (AS), a much higher rate than the
these patients can be diagnosed with skin biopsy conflrma- approximately 0.5'7, seen in the general population. Among
D tion of dermatomyositis in the setting of at least 6 months patients \\.ith AS who develop IBD. Crohn disease and ulcer
(a without muscle involvement. A characteristic skin rash ati\€ colitis occur with equal frequencyl IBD in AS is slightll,
E
.D should prompt consideration of dermatomyositis even with more common among men and occurs early in the course
UI normal muscle strength and a normal serum creatine kinase ofAS. By 10 years ofAS, the risk lor IBD has returned to the
o,
level. This patient has a typical photosensitive distribution general population level. The bowel inflammation and gut
TL
rl of dermatomyositis rash on the face, including a probable dysbiosis that are common in IBD may drive the inflam
early heliotrope rash over the eyelids. He also has Gottron mation in the spine, leading to AS. Clinical nranifestations
4t papules on the elbows and knees. Skin biopsy would likely of IBD in patients with combined AS and IBD are similar tir
.D show characteristic interface dermatitis. Further evaluation those in patients with IBD alone.
U!
can include antibody and additional testing for subtle mus Celiac disease (Option A) is an immune mediated dis
cle inflammation, using electromyography or MRI, but even ease that primarily affects the small intestine in response
if the results are normal, this patient should be treated for to dietary gluten. It is one of the most common causes of
dermatomyositis. Patients with amyopathic dermatomyo malabsorption. Celiac disease has no association u,ith AS
sitis also have a high risk for interstitial lung disease and or other rheumatologic diseases. Although patients with
cancer; screening for these conditions should also be part of chronic diarrhea should be evaluated lor celiac disease. IBD
the initial evaluation. is a more likely diagnosis in a patient with AS.
Acute cutaneous lupus erythematosus (Option A) can Irritable bowel syndrome (lBS) (Option C) is a func
be difficult to distinguish clinically from dermatomyositis, tional bor,tel disorder defined by the presence of abdominal
especially on the face, because it can present in the same pain in association with defe'cation and, or a change in bolr,el
distribution and can have similar flndings on skin biopsy. habits. The diagnosis of IBS requires symptoms ol recurrent
However, lupus is not likely to cause a heliotrope rash or abdominal pain at least I day a u,eek fbr :l months, along
Gottron papules on the hands, elbows, and knees. with at least two ol the fbllou,ing three additional crite
This patient's Gottron papules on the elbows and knees ria: pain related to defecation. change in stool fiequency
can be difficult to distinguish from psoriasis (Option C) or change in stool consistenc'y This patient's pain is not
by description or physical examination, but psoriasis is related to defecation. and her recent unintentional weight
less likely to cause the photosensitive rashes seen in this loss makes IBS unlikell:
patient. Small intestinal bacterial overgronth (SIBO) (Option D)
Rosacea (Option D) is the most common cause of a is caused by various conditions. including impaired motility
red rash in a malar distribution in an adult. Rosacea is an strictures. or blind loops. Diagnosis requires t1.'pical symp
inflammatory skin condition that produces small pink pap toms and a confirmatory test (usually breath testing). SIBO
ules and pustules on the central face with a variety of speciflc is associated with systemic sclerosis but l.ras not been irssoci
patterns. The erythema ol the cheeks can mimic the malar ated with AS unless there is another predisposing condition.
rash of systemic lupus erythematosus or dermatomyositis
XEY POIf,IS
but would not be expected to cause skin changes on the
extremities. o Inflammatory bowel disease occurs in patients with
ankylosing spondylitis at a much higher rate than in
(tY P0rxTs
the general population.
o Some patients with dermatomyositis never develop o Patients with ankylosing spondylitis who develop
muscle involvement (amyopathic dermatomyositis)
abdominal pain, diarrhea, or a change in bowel habits
and are diagnosed by using skin biopsy in the setting
should be evaluated for inflammatory bowel disease.
of at least 6 months without muscle involvement.
o A characteristic skin rash should prompt considera-
Bibliography
tion of dermatomyositis even with normal muscle Fragoulis GE, Liava C, Daoussis D, et al. Inflammatory bowel diseases and spon
strength and a normal serum creatine kinase level. dyloarthropathies: from pathogenesis to treatment. World J Gastroenterol.
2019;25:2162 2176. IPMID: 31143068] doi:10.3748/wjg.v2s.i18.2162

128
I

t
Answers and Critiques
L

t
I

L
Item 12 Answer: B
Educational Objective: Treat Lyme arthritis.
Item 13 Answer: A
Educationa I Objective: Diagnose acute calcium
tr
pyrophosphate crystal arthritis.
t The most appropriate treatment is doxycycline (Option B).
Although arthralgia and myalgia often occur at the earlier 'lhe most likely diagnosis is acute calcium pyrophosphate
t (CPP) crystal arthritis (pseudogout) (Option A). Acute CPP
stages of Lyme disease, Lyme arthritis is a late-stage manifes
i
tation. It is typically monoarticular, most commonly in the crystirl arthritis is an episoclic nrthritis that most commonly
I knee. It should be suspected in patients who may have had occurs in older women. tt usually develops over several days
L
untreated or incompletely treated Lyme disease, although and may last fbr weeks. The knee and wrists are common
not all patients will have a history compatible with previous sites. 'lhe pre'sentation is usually inflammatory with pain,
Lyme disease. All patients with Lyme arthritis, however, swellir.rg. and warmth <-rf the inv'olved joint. Systernic markers
I
should have positive results on enzyme-linked immunosor of ir.rf lammation may be elevated. such as tl-re erythrocyte sed
;
bent assay and Western blot confirmatory serologies; this is imentation rate in this patient. Plain radiographs, like the one l,t
(l,
the primary means of diagnosing Lyme arthritis, although a shown fbr this patient, may demonstrate cht.rndrocaicinosis.
:
ET
I second enzyme immunoassay can be used as a confirmatory Chonclrocalcinosis is seen as a thin white Iine that tracks below
test. In addition, Borrelia burgdorJ'ert DNA can be detected the surface layer ol the cartilage because it represents CPP (,
by polymerase chain reaction in synovial fluid, but this test deposition within the cartilage. A definitive diagnosis n'ould -t
t offers no advantage to serologic testing. Randomized con- include aspiration of the joint fluid and the clemonstration of ag
trolled trials have suggested that 90'1, of patients with Lyme CPP crystals, which are weakly positively biretiingent (blue Ul
arthritis are eflectively treated with a 28-day course oforal when parallel to and yellow wher.r perpendicular kr the polar o
antibiotics. Doxycycline is the most widely recommended izing axis of'an optical filter) and classically rhon.rboid shaped. vt
=
I E
first choice in antibiotic treatment of Lyme arthritis. Treat- (lout flare (Option B) is a less likely diagnosis given the
ment with a 28-day course of amoxicillin is an alternative involvement of the knee without current or prior lypical
flrst line therapy. For patients whose arthritis improves but joint i nvolvement (first metatarsophalangeal joi nt, ibrefeet.
does not completely resolve, a second 28-day course of the ankles, ancl flngers). The ll week duration of the previ-
same antibiotic can be prescribed. For pregnant and lactat ous episodes is longer than that usually seen with gout
ing women, tetracyclines are generally avoided in favor of a (t-z weeks). The normal serum urate level also makes gout
p Iactam antibiotic. unlikely, although serum urate levels may occasionally lall
Ceftriaxone (Option A) is unnecessary in most cases into the normal range during an acute flare. Monosodiun.r
of Lyme arthritis because patients generally respond to oral urate crystals are needle shaped and negatively birefrin
antibiotics. Celtriaxone is not the first choice for treatment gent uncler polarized light; they appear lellow when par
of Lyme arthritis unless concomitant neurologic symptoms allel ar.rd blue when perpendicular to the polarizing a.xis.
are present. It may be given for moderate to severe persistent lnfectious arthritis (Option C) is unlikely because the
arthritis after the failure of oral antibiotics. patient has no fever or other systemic sympt(nns afler 10
A small subset of patients may have persistent arthri days, as well as a normal complete biood count. IIer history
tis that suggests a reactive arthritis, which may respond of similar episodes also argues against in{ection as the cause.
to immunosuppressive therapy. For moderate to severe Nevertheless. a1l patients with acute monoiirticular arthritis
persistent symptoms, treatment with hydroxychloroquine should undergo synovial fluid analysis that inclucles a Gram
(Option C) or methotrexate (Option D) may be appropriate, stain anct culture to help cxclude inf'ectious arthritis.
but only after failure of treatment with oral and intravenous Ill.rcumatoid arlhritis (Option D) is not a likcly diag-
antibiotics. nosis because the patient has monoarticular knee afihritis,
without the symmetric joint involvement of hancls. wrists, or
XEY POIXTS
feet typical of rheumatoid arlhritis. Finally, chonclrocalcino-
. All patients with Lyme arthritis should have positive sis is not a feature of rheumatoid arthritis.
results on enzyme linked immunosorbent assay and
Western blot confirmatory serologies; this is the XEV POI]ITI
primary means of diagnosing Lyme arthritis, although . Acute calcium pyrophosphate cr).stal arthdtis (pszudogout)
a second enzyme immunoassay can be used as a
is characterized by the sudden onset of pain, warmth, ten-
conflrmatory test.
demess, and swelling of the affected joint, usually a knee
. Ninety percent of patients with Lyme arthritis are or wrist; flares are typicaUy longer than those ofgout.
effectively treated with a 28-day course of oral . Definitive diagnosis of calcium pyrophosphate crystal
doxycycline or amoxicillin.
arthritis would include aspiration of the joint fluid and
demonstration of calcium plrophosphate crystals,
Bibliography which are weakly positively birefringent (blue when
ArvikarSL, Steere AC. Diagnosis and treatment of Lyme arthritis. Infect Dis
parallel to and yellow when perpendicular to the polar
Clin North Am. 2O15;29:269-80. IPMID: 259992231 doi:10.1016/j.idc.2015. izing axis of an optical filter) and rhomboid-shaped.
02.004

129
Answers and Critiques

Bibliography underll-ing sl,stemic sclerosis, which is best treated rvith


Rosenthal AK. Ryan l.M. Calcium pyrophosphate deposition disease. N Engl an ACE inhibitor. Captopril in particular is usefirl in this
J Med. 2016;37.1:2575 tt.1. IPMID: 27355536] doi:10.1056'NEJMral511117
situation because it has a relatively short half lif'e and
can be titrated rapidly to control blood pressure. Sclero
Item 14 Answer: A derma renal crisis can occur in Iimited or dilluse cutaneous
systemic sclerosis but is more common in dilluse dis
Ed ucati o na I O bjective : Treat ankylosing spondylitis with
ease. Risk factors ir.rclude recent onset of sl'sternic sclerosis
a tumor necrosis factor inhibitor.
((4 years). diffuse skin discase, presence olanti RNA pol1,
The most appropriate next treatment is etanercept (Option nrerase III antibodies, and recent use of glucocorticoids.
A). This patient has a clear diagnosis of ankylosing spon- 'lhe pathophl,'siology is thought to be related to endothe
dylitis (AS). and first line therapy with NSAIDs has failed. lial injury leading to vascular constriction. intirnal thick
The American College of Rheumatolory (ACR) recommends ening. ar-rd fibrin deposition. 'uvith endothelin I plaf ing
the use of a tumor necrosis factor (TNF) inhibitor after fail an important role in tl.ris process. Also inrportant in the
la ure of a trial of at least two NSAIDs for 2 tri 4 weeks each. pathophl-siology is activatior.r of the renin angiotensin
(D
= or if NSAIDs cannot be tolerated. TNF inhibitors. such as irldosterone s),stemr interrupting this pathual'hrs been a ke1'
UI etanercept (a fusion protein), adalimumab, or infliximab to therap)'. Manifestations ol scleroderma renal crisis are
q,
(monoclonal antibodies), can be used. The selection may those seen in patients with hypertensive emergenc)'. such
CL depend on the presence of comorbid disease, such as uve as headache, encephalopathy, retinopath)'. acute kidney
n itis or inflammatory bowel disease, where the monoclonal injury'. and heart tailure. Anemia. thrombocl'topenia, and
lt antibodies would be more appropriate. 'fNF inhibitors have proteinuria are comnlon laboratory features. Evidence of
been shown to be disease modiffing in AS. Interleukin-l7 rnicroangiopathic hernoll'tic anemia u'ith schistoc)'tes on
rD
UI inhibitors, such as secukinumab, could be used in patients the peripherai blood smear is a clue to the diagnosis. ACE
in whom a TNF inhibitor fails or in whom a TNF inhibitor inhibitors are disease modiff ing agents ftrr this pcltentialll'
may be contraindicated. The ACR conditionally recommends Iirtal complication, regardless of the serum creatinine level.
TNF inhibitors over interleukin 17 inhibitors in patients in Patients may need telnporary diirlysis. and kidney function
whom NSAIDs fail or who cannot tolerate NSAIDs. may improve during therapy' fbr up to 2.1 months after
Nonbiologic disease-modifying antirheumatic drugs, presentation. Prophylactic use of'ACE inhibitors does not
such as methotrexate (Option B) or sulfasalazine (Option prevent scleroderma renal crisis and n.ra1' lead to poorer
D), have no ellicacy in axial disease and limited efflcacy in renal outcomes.
peripheral disease. Treatment with sulfasalazine is recom Immunosuppressior.r has no role in modiff ing the
mended primarily fbr patients with prominent peripheral course of scleroderma renal crisis. so neither intravenous
arthritis and few or no axial skeleton symptoms and in these cyclophosphamide (Option B) nor intravenous methylpred
patients, it may be more eflective than methotrexate. How nisolone (Option C) is indicated. Recent use ot glucocor
ever, a TNF inhibitor may be the best option fbr these patients. ticoids is a risk factor fbr scleroderma renal crisisr tl.tus.
The ACR strongly recommend against treatment with glucocorticoids should be avoided.
systemic glucocorticoids, such as prednisone (Option C). Intravenous rnetoprolol (Option D) does not address
Glucocorticoids have numerous adverse efl'ects. which are the pathophl,siolory of scleroderma renal crisis. p Blockers
more likely to occur with higher doses and longer treatment, are generally not first line therapy ftir rnost causes of'
and lower doses are not as efficacious as TNF inhibitors. hl,pertensive emergencies other than episcldes associ
ated with eclampsiai preeclampsia, acute coronary syn
XEY POIXI
drome. and aortic dissection: in these cases, labetalol or
o Tumor necrosis factor inhibitors are useful in patients a short acting agent such as esmolol is typically pref'erred
with ankylosing spondylitis in whom NSAIDs have to metoprolol.
failed or cannot be tolerated.
f,EY POIilTS
Bibliography . Patients with scleroderma renal crisis may exhibit
Ward MM, Deodhar A, (lensler LS, et al. 2019 update of'the American College anemia, thrombocytopenia, proteinuria, and schisto-
ol Rheumatolos//Spondylitis Association ol America/Spondyloarthritis cytes on the peripheral blood smear.
Research and Treatment Network Recommendations fi)r the treatment of
ankylosing spondylitis and nonradiographic axial spondyloarthritis. o ACE inhibitors (typically captopril) can be lifesaving
Arthritis Rheunlatol. 2019:71:1599-1613. IPMID: l]t+3OO:lOl doi:10.1002
irrt.41042 in patients with scleroderma renal crisis, and dosages
should be titrated to control blood pressure regardless
of serum creatinine level.

tr Item 15 Answer: A
Educational Objective: Treat scleroderma renal crisis. Bibliography
Zanatta E, Polito P, lavaro M, et al.'lherapy ofscleroderma renal crisis: state
The most appropriate intravenous treatment is captopril ot the art. Autoimmun Rev 2018;17:882-889. [PMID: 30005860] doi:
(Option A). This patient has hypertensive crisis due to 10. l016ii.autrev2018.03.0l2

130
I
L
t
I
t Answers and Cr i!iqg9_s
1

17 Answer: B
tr Item 16 Answer: D
Educational Objective: Treat giant cell arteritis with a
Item
Ed u cati ona I O bj ective: Diagnose interstitial lung disease
glucocorticoid-sparing agent. in a patient with rheumatoid arthritis.

lhe most appropriate aclditional lreatment is tocilizumab The most likely cause of this patient's exertional dyspnea is
(Option D). Ciant cell arteritis (GCA) is a medical enrer interstitial lung disease (lLD) (Option B). ILD may develop
ger.rcy that requires inrmediate therap1,. the risk fbr sr-rdden in 507, of patients with rheumatoid arthritis, particularly
and irreversible loss of vision is imminent in the absence those who are seropositive for rheumatoid factor and anti-
:

ol appropriate treatnlerlt. Synrptoms ancl inflamrlabry cyclic citrullinated peptide antibodies; clinically signifl
nrarkers usualll,' respond rapiclll to gluc<lc'orticoids. Hort, cant disease is seen in 10'1, of patients and contributes to
ever. glucocorticoids ha'n'e nrany adverse eflects, ancl this excess mortality. ILD types include bronchiolitis, organiz
patient will henefit Iron.r the adclition rit a glucocorticoid- ing pneumonia, nonspeci{ic interstitial pneumonia, and
sparing agcnt. Generally acceptccl indicalior.rs for a gluco- usual interstitial pneumonia. Patients may also have pul U!
corticoid sparing agent include the presence of comorbid monary rheumatoid nodules. Patients with rheumatoid
o
ET
cliseases that are negatively afiected by glucocorticoids, arthritis treated with methotrexate are also at risk for
development of glucocorticoici related adverse efI'ects, and possible drug-induced parenchymal lung disease. Nonpro-
prolonged neecl lbr treiltnlent. 'lhis patient nreets all indi- ductive cough and dyspnea are the most common present
t,
!,
cations firr a glucocrlrticoid sparing ager.rt. Tocilizumab, an ing symptoms of ILD. Lung examination findings vary and tr
.!
interleukin 6 blocker, is highly eflective for the treatnlent may be normal. Inspiratory fine crackles suggest intersti- Ut
(l,
of' GCA n,hen administered during glucocurticoid tapering tial flbrosis. The evaluation of suspected ILD includes full
or :rs monotherapy fbllor,r,ing discontinuation of' gluco pulmonary function testing (restrictive physiologr), chest ta
=
E
corticoids. 'lhe 2021 American College of Rheumatologr radiography (which may be normal), and high-resolution
guidelines make a conditional recomme'nclation fbr gluco CT of the chest.
corticoids r,r,ith tocilizumab over oral glucocorticoids alone Heart failure (Option A) is more common in patients
in patients u,ith newly diagnosccl (iCA. with rheumatoid arthritis than in the general population
Cyclophosphamide (Option A) is an immunosuplrres.- and may contribute to increased mortality. The increased
sive agent used to treat systelnic virsculitis, such as gran prevalence of heart failure may be due to the presence of
ulomatosis poll,ar.rgiitis. Its use and ellectiveness in inflammatory mediators, antirheumatic drug therapy, amy
patients with "t'ith
GCA are unproven, consistir.rg m:rinly of loidosis, or ischemic heart disease. Heart failure is unlikely
l f'ew uncontrollecl studies involving small nunrbers of in the absence of jugular venous distention, edema, or an Sr.
patients. Patients with rheumatoid arthritis rarely develop pul-
A randomized clinical trill demonstrated that inflix monary arterial hypertension (Option C); that condition is
imab (Option B) dicl not reduce the proportion of patients often associated with rheumatoid vasculitis. In patients with
with GCA relapses or increase the proportirtn of'paticnts long-standing ILD, secondary pulmonary hypertension can
uho could taper glucocorticoid dosages. Infliximab is not develop. Pulmonary arterial hypertension is unlikely in this
an eflective agent fbr thc treatmellt of GCi\ and should not patient with a normal S, and normal jugular venous pres
be used. sure. The presenceofinspiratory crackles also argues against
Methotrexate (Option C) is a glucocorticoid sparing pulmonary arterial hypertension.
drug used in the treatment of GCA. Evidence on its eflec Pleural disease (Option D) occurs in up to 5% of patients
tiveness is mixed. ancl clinical eftect seerns to be r.nodest with rheumatoid arthritis; pleural effusions are exudative
irt best. and can be large. Rheumatoid arthritis pleural effusions are
characterized by low glucose, pH, and complement levels,
XEY POIf,IS
as well as elevated levels of total protein, rheumatoid factor,
. Generally accepted indications for a glucocorticoid- and Iactate dehydrogenase. A pleural effusion large enough
sparing agent in patients with giant cell arteritis to cause dyspnea on exertion is most likely to be associated
include the presence of comorbid diseases that are with absent breath sounds and dullness to percussion over
negatively affected by glucocorticoids, development of the affected lung fleld. Inspiratory crackles would not be
glucocorticoid reiated adverse effects, and prolonged expected as the only flnding.
need flor glucocorticoids.
. Tocilizumab, an interleukin 6 blocker, is an effective XEY POIlITS
glucocorticoid sparing agent in the treatment of giant r Interstitial lung disease may develop in 50% of
cell arteritis. patients with rheumatoid arthritis; clinically signifi-
cant disease is seen in 10% of patients and contributes
to excess mortality.
Bibliography
Maz M, Chung SA. Abril A. et al. 2021 American College ol Rheumatolory/
. Nonproductive cough and dyspnea are the most com-
Vasculitis lbundation guideline for the management ol giant cell arteritis mon presenting symptoms of interstitial lung disease.
and Takayasu arteritis. Arthritis Rheumatol. 2021. IPMID: 342358841

131
Answers and Critiques

Bibliography Bibliography
Spagnolo p lung in rheumatoid arthritis:
Lee JS, Sverzellati N, et al. The MerolaJF, Wu S. Han J, et al. Psoriasis, psoriatic arthritis and risk ofgout
focus on interstitial lung disease. Arthritis Rheumatol. 2018;70:1544 in US men and women. Ann Rheum Dis. 2015:74:1495 500. [PMID:
1554. IPMID: 29806092] doi:10.1002/art.40574 246516201 doi:10.1136/annrheumdis 2014 205212

tr Item 18 Answer: A
Educational Objective: Diagnose gouty arthritis in a
Item 19
Educational Objective:
Answer: B
Assess a patient with systemic
patient with psoriasis and psoriatic arthritis.
lupus erythematosus for cardiovascular risk.
'lhe most likely diagnosis is gout!' arthritis (Option
A). This The most appropriate assessment to perform is for cardio-
patient has an acute painful joint in the setting ofpsoriasis vascular disease risk (Option B). Patients with systemic
and psoriatic arthritis (PsA). Hyperuricemia and gout are lupus erythematosus (SLE) have a high risk for cardiovascu
D comorbidities associated lvith psoriasis. It is thought that lar disease compared with the general population, including
ul the rapid tllrnover of skin cells in patients r,r'ith psoriasis a greater risk for myocardial infarction and ischemic stroke,
E drives protein metabolism and increases serum urate levels.
.D even in younger patients. Control of the underlying SLE
tt A recent study oftwo large cohorts, the Health Professionals may be the most important way to minimize this risk, but
o, tbllorv up Study ancl the Nurses' Health Study: fbund that patients should also be screened for other modifiable risk
EL the risk fbr gout in participar.rts with psoriasis rvas almost factors. Assessment of diet, exercise, hypertension, diabetes
r.t double that of participants witl.rout psoriasis; the risk was mellitus, smoking, and potentially lipids are indicated in
flve times greater in participants nith PsA than in those patients, including young patients, with SLE.
.ct
n,ithout either conclition. This Ope of joint preser.rtation Patientswith SLE have a higheroverall riskformalignan-
rD
t^ (acute swelling, severe pain. unwillingness to flex or extend
cies (particularly hematologic) but not cancer-related mor
joint) can have one ol three ciuses: infectious arthritis. tality; the risk for non Hodgkin lyrnphoma is at least two to
hemarthrosis. or crystalline arthritis. Joint fluid analysis is three times higher than in the general population. There may
necessary to rnake thc correct diagnosis. be an increased risk for cancers ofthe lulva, cervix, lung, thy-
Inf'ectious arthritis (Option B) is always possible in roid, and possibly liver. Malignancy risk in SLE is tied to the
a patient with acute monoarticular arthritis. particu- use of immunosuppressive agents. Breast cancer risk (Option
larly r,lhen the knee is involved. Hortever. r,r'ith intectious A) does not seem to be increased, but appropriate screening
arthritis, fluid leukocyte counts
f'ever is common, synovial should be initiated in the future as indicated by guidelines.
rrlnge f'rom 50.000 to 100.0001pL (50 100 x 10'),'L) or In patients with SLE, normocytic, normochromic
higher. and the Gram stain is positive in up to 50')i, of inflammatory anemia is common; autoimmune hemoll'tic
patients. In PsA. joint infection is much less common than anemia occurs in approximately 10% and correlates with
gouty arthritis. SLE activity. Lymphopenia/leukopenia is also common but
Patients r,l,ith osteoarthritis (Option C) usually usually mild. Thrombocltopenia occurs in 30% to 50% of
describe an insidious onset of intermittent symptoms, patients with SLE, and approximately 10'7, develop severe
which become more persistent and severe over time. thrombocytopenia (platelet count <50,000/pL [50 x 10e/L])
Patients with osteoarthritis generally do not have systemic in isolation or in conjunction with hemolytic anemia. Iron
features. 'lhe lvpical synovial tluid analysis rtill shor,v 200 overload syndromes (Option C), such as hemochromatosis,
to 2000 lcukocytes'pl. (O.Z-Z.O x l0q'L). 'lhis patient's joint are not a morbidity of SLE unless associated with increased
fluid count of 40,000 leukocytesi gL (40 x 10"/l-) with 90'7,, transfusion requirements.
neutrophils indicatc's that he has an inflammatory arthritis. Some patients with SLE or overlap syndromes can have
not osteoarthritis. interstitial lung disease that would warrant additional test
Flares ol PsA (Option D) are not typically this pain- ing with pulmonary function testing or chest CT. However,
ful and usually develop more skrwly. In addition. an acute this patient is asymptomatic, and assessment for pulmonary
arthritis of this magnitude is unlikely in someone \{,ith PsA disease (Option D) is not warranted.
rtho is receiving methotrexate and has no other afl'ected
joints. Acute monoarticular arthritis morc strongly suggests XEY POIXIS
hemarthrosis. infectious arthritis, or gout. o Patients with systemic lupus erythematosus have a
high risk for cardiovascular disease compared with
I(EY POITIS the general population, including a greater risk for
o Acute monoarticular arthritis is due to infectious myocardial infarction and ischemic stroke, even in
arthritis, hemarthrosis, or acute crystalline arthritis; younger patients.
joint fluid analysis is necessary to make the correct o Control of the underlying systemic lupus erythemato-
diagnosis. sus may be the most important way to minimize car-
. Hlperuricemia and gout are comorbidities associated diovascular risk, but patients should also be screened
with psoriasis and psoriatic arthritis. for other modifiable risk factors.

132
Answers and Critiques

Bibliography t(EY POIilIS (onttnaed)


Andrades C. Fuego C. Manrique Ariia S. et al. Management ofcardiovascular o Laboratory testing and imaging are usually not neces-
risk in systemic lupus erythematosus: a systemxtic review. Lupus. 2017i
26:1407 7419. [PMID: 284571 97] sary to diagnose osteoarthritis but are helpful if other
causes of arthritis are being considered or to help
Item 20 Answer: E define the safety of potential therapies.
Educational Objective: Diagnose osteoarthritis with
history and physical examination. Bibliography
Martel-Petletier J, Maheu E, Pelletier JR et al. A new decision tree fbr diag
No additional laboratory studies are needed (Option E). nosis of osteoarthritis in primary care: international consensus of
Osteoarthritis (OA) diagnosis is based on history and physi experts. Aging Ctin Exp Res. 2019:31:19 30. IPMID, 305395'111

cal examination; radiography is conflrmatory but may not be


necessary. In early OA, clinical flndings may not be accompa
vt
nied by radiographic changes; conversely, some patients with
prominent radiographic changes may have minimal or no
symptoms. Patients with OA are typically older than 50 years
Item 21 Answer: C
Educational Objective: Treat tophaceous gout in a
tr c,
ET

patient with contraindications to allopurinol.


of age; diagnosis at an earlier age should prompt inquiry into |.,
a history of previous joint damage, endocrine or metabolic 'lhe most appropriate additional trcatment is f'ebuxtlstat -,-
disorders, or a genetic proclivity fbr early disease. Joints most (Option C). this patient has tophaceous gout, which is an |!
tt
commonly affected are the hands (first carpometacarpal inclication fbr urate lor,r,ering therap!: ll.re American Gtllege o
joints, distal and proximal interphalangeal joints), feet (first of' Rheumatology (ACR) strongly recotnmencls treatntent
la
=
metatarsophalangeal joint and mid foot), knees, hips, and with allopurinol (Option A) as the prel'erred first line agent
spine, but the distribution varies. Localized OA, in which a over all other uratc lor,r,ering therapies, includitrg in patients
single joint is affected, is more often a consequence of injury u,ith moderate'to severe chronic kidney clisease (CKD)
or joint asymmetry and often occurs in weight-bearing joints (stage >3). \,\,ith appropriate dose adjustment. Ilort'ever. she
(hip or knee). Furthermore, this patient has noninflamma ciinnot take allopurinol because of a history ol a rasl.r rt'ith
tory arthritis, as evidenced by minimal morning stiffness previous use. An uncommon but scrious complication of'
and no evidence of warmth, soft tissue swelling, or effusion; alkrpurinol is a hypersensitivity reaction. which may lead
these lindings further support the diagnosis of OA. Addi to organ dysfunction and death. 'lherefore. allopurinol is
tional tests would be helpful if other causes of arthritis were not appropriate, and therapy with fbbuxostat. a nonpurine,
being considered or would aid in decisions about therapeutic noncompetitive xanthine oxidase inhibitor. is indicltecl.
options. No additional tests are needed in this patient. Beciruse only 3'l'" of fbbuxostat is renally cleared, it n-ray be
The cardinal signs of inflammation are pain, erythema, uscd in patients with CKD without dose adjustment if the
swelling, and warmth; with the exception of pain, non estimated glomerular filtration rate is 30 to 60 ml-/mini
inflammatory conditions usually lack these features. This 1.7i1 m). The starting I'ebuxostat dosage is ,10 mgrd, r'r,hich
patient's history and clinical flndings do not suggest inflam nr:ry be increased to up to itO mg/d if needed on the basis of
matory arthritis, in particular inflammatory autoimmune scrum urate lelel. Achieving and n-raintaining a serum urate
disorders (such as systemic lupus erythematosus or rheuma target ollcss than 6.0 mg,'dl. (0.35 mmol, L) are strongly rec-
toid arthritis). Evaluation for these conditions with testing ommended Ibr all patients receiving urate-lowering thcrapy.
of antinuclear antibodies (Option A) or rheumatoid factor Prophyiaxis with prednisone should be continued along
(Option B) is not needed. u,ith febuxostat for 3 to 6 months or until the serum urate
Lyme arthritis is an infectious arthritis with an inflam level is at target.
matory synovial response. It is usually monoarticular and Colchicine (Option B) is used lbr treatir.rg acute gout
occasionally polyarticular. This patient has polyarticular flares and fbr long term prevention of flares while urate
arthritis and does not have flndings suggestive of inflam- lowering therapy is begun. Colchicine dcles not aftect serum
matory arthritis; serologic testing for Borrelia burgdorferi urite concentrations. Colchicine requires dose adjustmcnt
(Option C) is unnecessary. fbr kidney disease and rthen it is takcn concurrently rt'ith
Synovial fluid analysis (Option D) is helpful if effusion drugs that aflbct its n.retabolism.
is present and there is concem about other causes of arthri 'Ihe ACR strongll, recommends against pegloticase
tis, such as concurrent crystal arthritis, infections, or other (Option D) as a first line urale lor,vering therapy: Pegloticase
inflammatory causes. This patient does not meet these criteria. is indicated in paticnts in whom allopurinol and febuxostat
have failed or cannot be tolerated. It is inlravenously adtnin -
t(EY PO!ltrS
istered and expensive and thus should be reserved lbr an
o The cardinal signs of inflammation are pain, ery- appropriilte clinical setting. Ircbuxostat has not lailed in this
thema, swelling, and warmth; with the exception of piitientr thus. pegloticase is not indicated.
pain, noninflammatory conditions such as osteoar- Probenccid (Option E) lolvers serum urate by bbck
thritis usually lack these features. (continued)
ing uric acid reabsorption in the kiclney proximal tubule.
'll-re ACR strongly recommends allopurinol or tebuxostat

133
Answers and Critiq_ues

tr
CONI
over probenecid fbr urate-lowering therapy in patients
rt,ith moderate to severe CKD. Probenecid is not cftective
in patients with CKD (estimatecl glomerular filtration rate
iirtery stenosis leading to hypertension, as r,rell as mesenteric
ischemia. it affects ntedium-sized vessels and so lr,ould not
:

:
invoive the subclavian artery. ln addition, the most common
<50 mLiminr1.73 nt2) and is contraindicated in patients \\,ith n.ranifbstations of PAN are cutaneous findings (such as pal
I
nephrolithiasis. pable purpura) and neurologic involvernent. Roughly 7O"/,, of
patients \,vith PAN have neurologic disease. most commonly \
r(lY P0rilIS
. Febuxostat is recommended for patients with indica-
rnanifesting as mononeuritis multiplex. Unlike in Takayasu \
arteritis, pulselessness is r.rot chlmcteristic of PAN.
tions for urate-lowering therapy who have contraindi-
cations to allopurinol therapy. t(EY P0ilrIS 1

o Allopurinol and febuxostat are both strongly recom


. Takayasu arteritis is a rare large vessel vasculitis caus
i
mended over probenecid as urate-lowering therapies ing inflammation and subsequent stenoses in the
:
D in patients with moderate-to-severe chronic kidney large arteries, including the aorta and its branches.
Ut disease. o Manifestations of Takayasu arteritis can include limb
E
.D ischemia and associated diminished (or absent)
u! Bibliography pulses, mesenteric ischemia, and hypertension due to
o,
FitzGerald JD. t)albeth N. Mikuls T, et al. 2020 American College of' renal artery stenosis.
EL Rheunratolopgr guideline lbr the management ol gout. Arthritis Care Res
rt (Hobrrken). 2O2O:72:741 760. Il,MID: 32391934] doi:10.1002iacr.24180
Bibliography
.€l Keser C. Aksu K. l)iagnosis and diflerential diagnosis of large vessel vascu

.D Item 22 Answer: D litides. Rheum:ltol Int. 2019:i]9: 169 1 8.5. IPMI D: 3022]3271 doi: 10.1007r
la
EI Ed ucationa I Objective: Diagnose Takayasu arteritis.
s00296 018 .11.57 ll

lhe most likely diagnosis is Takayasu arteritis (Option D). Item 23 Answer: A
lhis rare large vessel vasculitis disproportkrnately affects
Educational Objective: Treat refractory gout with
young women ancl is much more conrmon in East Asia
anakinra.
(prev:rlence is 40imillion in Japan and up to B/million else
where). Onset is usually indolent. rvith nonspecific symp The most appropriate treatment is anakinra (Option A).
toms and signs presenting months to years before onset of' Anakinra is a recombinant human interleukin (lL)-1 recep
frank vasculitis. The disease causes infiammation and subse tor antagonist. IL 1 cytokine activity is an important part
quent stenoses in the large arterics, including the aorta and of the inflammatory process in acute gout. This patient has
its branches. Manif'estations can include limb ischenria and a severe polyarticular gout flare likely provoked by diure
associated diminished (or absent) pulses, mesenteric isch sis. In patients with severe refractory gout that has not
emia, and hypertension due to renal aftery stenosis. Clues responded fully to other agents or in r,r'hom other agents are
on exalnination include bruits over large vessels, absent contraindicated, anakinra may be used as ofllabel treat-
pulses, discrepancy in blood pressure between limhs. and ment. It has a short half life and is given by subcutaneous
hypertension. Laboratory findings indicate active systemic injection, 100 mg/d, usually for 3 to 5 days. The dose is
inflammation but are not diagnostic. lmaging of the aorta decreased in patients with stage 4 or 5 chronic kidney dis
and its branches is diagnostic: biopsy of an involvecl vessel ease (CKD). Canakinumab, a monoclonal antibody to ll-lp,
is not fbasible. Thirty percent of patients can present with which blocks binding to the IL 1 receptor, is a longer-acting
ocular ir-l,olvement, much olwhich is attributed to'lakayasu IL I inhibitor that may be used in refractory gout flares but
retinopathy and hypertensive retir.ropathy. is more expensive than anakinra.
this patient does not have the cutaneous features (pal Colchicine (Option B) is not appropriate because this
pable purpura, digital ischemia, ulcers, necrosis, livedo patient has CKD and is receiving tacrolimus. Colchicine
reticularis) that are the nrost conlmon symptoms ol cryo use should be avoided in patients taking concomitant cyto-
globulinemic vasculitis (Option A), seen in mrtre than 90',1 chrome P450 3A and P glycoprotein inhibitors, particularly
of patients. Other common manif'estations ir.rclude periph in the setting of CKD. Long term use of colchicine with tac
cral ncuropathy and glomerulonephritis. rolimus or cyclosporine may result in neuromyopathy.
Ciant cell arteritis (CCA) (Option B) is not present. Intra-articular, intravenous, or intramuscular gluco-
Nearly all patients rvith GCA are older than 50 years. Absent corticoids (Option C) may be used to treat acute gout in
pulses are typical in Takayasu arteritis but do not occur in hospitalized patients in whom both NSAIDs and colchicine
GCA. In addition. GCA rarely aflbcts the descending aorla are contraindicated or those who cannot take medications
and its branches: thus, renovascular hypertension tronr orally. This patient has not responded to appropriate doses
renal ar1ery steltosis and nlesenteric ischemia dre not man of oral glucocorticoids, and his comorbidities (hypertension,
ifbstations of GCA. hyperlipidemia, type 2 diabetes mellitus, heart failure) pres-
This patient's findir.rgs do not suggest polyarteritis ent relative contraindications to additional or higher dose
nodosa (PAN) (Option C). Although IAN can cause renal glucocorticoid use in any form. In addition, the polyarticular

134
I

Answers and Critiques


t
t
I

nature of his gout flare makes the use of intra articular r(EY P0tllTS
glucocorticoid injection more challenging. . Topical NSAIDs are safe and effective for treatment of
! Naproxen, an NSAID (Option D), is contraindicated
knee and hand osteoarthritis and should be consid
in this patient because he has heart failure and is taking
ered before oral NSAIDS.
I
Iow molecular weight heparin. NSAIDs can contribute to
i worsening heart and kidney failure and increase the risk for . Oral NSAIDs are recommended for the treatment of
i
bleeding in patients taking anticoagulants. hand, knee, and hip osteoarthritis but should be
avoided in patients with comorbidities, such as peptic
TEY POIXT
ulcer disease, cardiovascular disease, and chronic
o In patients with severe refractory gout that has not kidney disease.
responded fully to other agents or in whom other
agents are contraindicated, anakinra, a recombinant Bibliography
human interleukin l receptor antagonist, may be Kohsinski Sl.. Neogi 1', Hochberg MC. et lr1. 2019 An.rerican Oollege ol t,t
(u
used as off-label treatment. Rheumat(rogi 'Arthritis Founclation guideline fbr tlte manrgement {)l'
osteoarthritis of the hand. hip, and knee. Arthritis Rheuntttol. 2020r ET
72:220 23:1. LPMID: :ll 908163 | doi:10.1 002/art.4l l.l2
Bibliography L'
Fitz(ierald JD. Dalbeth N, Mikuls 1l et al. 2020 American College ot !t
Rheumatology guideline fbr the management of gout. Arthritis (lare Res E
I
(l loboken).')O2O ;7 2:74 760. IPM ID: 3239 19:]41 doi : I 0. 1002iircr.24180 Item 25 Answer: C ag
a

Educational Objective: Diagnose cause ofjoint pain in a (I,


patient with systemic lupus erythematosus. tt
Item 24 Answer: D =
The most appropriate management is MRI of the left hip
Educational Objeaive: Treat osteoarthritis of the knees
(Option C). Isolated new pain in the hip is an uncommon
and hands with a topical NSAID.
manifestation of a systemic lupus erythen-ratosus (SLE) flare.
The most appropriate treatment is topical diclof'enac (Option A serious complication ol SLE is osteonecrosis, which most
D). Topical NSAIDs, such as diclofbnac, are saf'e and ef fective commonly affects the hips but can also involve other large
for treatment of knee and hand osteoarthritis (OA). For OA joints, and should be suspected when there is otherwise unex
in those locations, the 2019 American College of Rheuma plained pain and/or reduced range of motion in a single joint.
tolory (ACR)/Arthritis Foundation (AF) guideline suggests Long-term and high prednisone dosages (>zO mgld), severe/
that topical NSAIDs should be considered betbre oral NSAIDs active SLE. and vasculitis are all associated with increased
because of f'ewer safety concerns. Topical NSAIDs are not risk. In patients with osteonecrosis, the plain radiograph can
effective for hip OA and have limited eflicacy in OA of other remain normal for months after symptoms begin. MRI is the
sites. modality of choice for sensitive evaluation of early disease,
There is minimal benefit of opioid therapy, including with plain radiography uselul to diagnose and monitor later
hydrocodone (Option A), for chronic pain control in patients stages. Small lesions can improve and resolve spontaneously,
with OA. Opioids pose a high risk lor toxicity and depen but larger lesions usually lead to bony collapse and stmctural
dence and should not be used to treat OA. sequelae. New hip pain in patients with a history of SLFI
Oral NSAIDs, such as meloxicam (Option B). are an should prompt evaluation for osteonecrosis with N4RI.
ACR/AF guideline recommended first-line treatment fbr If the patient were experiencing a more global flare
hand, knee, and hip OA. However. they should be used or intolerance to her current medications, switching her
cautiously in patients older than age 50 years and avoided maintenance therapy from azathioprine to mycophenolate
in those with comorbidities, such as a history of peptic ulcer mof'etil (Option A) could be reasonable. However, she has
disease or gastrointestinal bleeding, and in patients with no other signs of SLE activity, and so this change r.t'ould not
hypertension, cardiovascular disease, or chronic kidney dis be appropriate.
ease, as in this patient. This patient has a normal erythrcicyte sedimenta
The ACR/AF guideline conditionally recommends topi tion rate, normal serum complement levels, and low titer
cal capsaicin (Option C) for patients with knee OA and con anti double strandedDNA antibodies. Becausethispatient's
ditionally recommends against topical capsaicin in patients isolated hip pain is unlikely to be caused by increased inflam
with hand OA. The guideline notes limited data supporting mation. and because her disease is otherwise well controlled.
the efficacy of topical capsaicin for knee OA. No direct evi increasing her prednisone dosage (Option B) would not be
dence supports topical capsaicin in the treatment ol hand appropriate and could accelerate damage fiom osteonecrosis.
OA, and eye contamination is possible when it is used on Physicat therapy (Option D) can be useful fbr reha
the hands. bilitation of joint pain in SLE and may be helpful for this
Other topical agents are available, such as lidocaine patient, but diagnosing the cause of this patient's hip pain
(Option E) and methyl salicylate preparations, but they are should take priority because the diagnosis and stage of
not as well studied or as eflicacious as topical NSAIDs. They osteonecrosis are important for determining the need for
may be used as adjunctive measures. potential surgical intervention.

135
:
.:

Answers and Critiques


1

TEY POITIS balanced by any benefits. Glucocorticoids are used in the


)
o Patients with systemic lupus erythematosus, espe-
treatment of Sjogren syndrome related systemic disease,
such as interstitial lung disease, demyelinating diseases,
cially those who have received high cumulative doses
autoimmune hepatitis, and arthritis. It is recommended that
ofglucocorticoids, are at risk for osteonecrosis, espe-
glucocorticoids be used at the minimum dose and length of
cially in the hip.
time necessary to control active systemic disease.
. MRI is the imaging modality of choice for suspected Rituximab (Option D) is not approved for the treat
osteonecrosis at earlier stages of disease. ment of SjOgren syndrome and has shown no efficacy in
treating oral dryness. It is appropriately prescribed to the
Bibliography small subset ol patients with Sjogren syndrome who have
Tse SM, Mok CC. Time trend and risk factors ofavascular bone necrosis in cryoglobulinemic vasculitis, but this patient has no signs
patients with systenric lupus erythematosus. Lupus. 2017)6:715 722.
of vasculitis (cutaneous ulcers, necrosis, palpable purpura,
lPMtD,27831s.101
D livedo reticularis, or Raynaud phenomenon).
ta
€ I( EY PO t TIS
(D Item 26 Answer: A . Nonpharmacologic therapies are the first-line treat
Ut
o, Educational Obiective: Treat oral dryness of Sjdgren ment of Sjdgren syndrome-related oral dryness,
a syndrome. including local moistening with water, sugar-free
rl The most appropriate treatment is cevimeline (Option A). The acidic candies, lozenges, and/or mechanical stimu-
patient's primary symptom is worsening oral dryness associ lants (such as sugar free chewing gum).
ll
E
(D ated with Sjogren syndrome. There is no disease modifying o Pharmacologic stimulation of salvia with cevimeline
l,I treatment to globally address manifestations of Sjogren or pilocarpine is indicated for oral dryness not relieved
syndrome. and no medication alters the natural history of with nonpharmacologic therapies.
chronic oral and ocular dryness. However, treatments can
alleviate xerostomia and xerophthalmia. For oral dryness, Bibliography
behavioral management may be enough to allay symptoms. Ramos Casals M, Brito Zenin P, Bombardieri S, et al; EULAR Sjogren
This can include chewing sugarless gum or sucking on sugar Syndrome Task Force Group. EULAR recommendations for the manage
ment of Sjogren's syndrome s,ith topical and systemic therapies. Ann
free sour candies or lozenges to stimulate saliva, taking fre Rheum Dis. 2O2O:79:3 18. [PMID: 316727751
quent small sips of water, or using a spray bottle to moisten
mucosal surfaces (to limit oral intake). Saliva substitutes are
available commercially in the form of oral sprays, gels, and llem27 Answer: D
rinses for more severe symptoms but are often not eflective.
Educational Objective: Diagnose a secondary cause of
EI
Cevimeline, a cholinergic muscarinic agonist, stimulates
acute calcium pyrophosphate crystal arthritis.
salivation and is appropriate for moderate oral dryness that
does not improve with the aforementioned interventions. The laboratory study most appropriate to perform next is
Pilocarpine, another muscarinic agonist, is an equally effica- measurement of serum f'erritin (Option D). This patient's
cious alternative to cevimeline. Cholinergic adverse efl'ects of recurrent acute flares of pain and srvelling in tl-re r,l'rists
cevimeline and pilocarpine can include sweating, abdominal and third metacarpophalatrgeal (MCP) joint are consistent
pain, nausea, flushing, and increased urination. Ocular dry with acute calcium pyrophosphate (CPP) crystal arthritis
ness can be treated with over the counter remedies. such (pseudogout). 'lhe radiograph ol the MCP joints shows a
as artificial tears (used during the day) and eye lubricants small hooked osteophyte of the lef t tl-rird MCP joint. Hooked
(usually used betbre bedtime). Refractory or severe ocular osteophytes ol the second and third MCP joints are a charac
dryness may be managed by using topical immunosuppressive teristic feature of CPP crystal arthritis. The radiograph of the
containing drops, such as cyclosporine (a calcineurin inhib wrist shows calcification along the flbroarticular cartilage
itor) or lifitegrast (an integrin inhibitor). collsistent u,ith chonclrocalcinosis. Synovial fluid analysis
Hydroxychloroquine (Option B) does not alter the nat shows CPP crystals, nhich confirm the diagnosis. Given the
ural history of Sjdgren syndrome or improve oral or ocular patient's age (<60 years), secondary causes ofCPP deposition
dryness. This medication was widely favored by Sjogren (sr.rch as hyperparathyroidism. hypothyroidism, hypophos
syndrome specialists until a randomized controlled trial phatasia, hypomagnesemia. and hemochromatosis) should
published in2Ol4 demonstrated no efficacy for oral or ocular be sought. Iler norrnal comprehensive metabolic parlel,
dryness or fatigue after 24 weeks compared with placebo. serunr magnesium level, and tlryroid stimulating hormone
Hydroxychloroquine can, however, beneflt patients with level exclude all secondary causes other than henrochroma
Sjogren syndrome who experience inflammatory arthritis or tosis. Measurements of serun-r f'erritin and transferrin satu
subacute cutaneous lupus erythematosus. ration are appropriate screening tests for hemochromatosis.
There is no indication to prescribe prednisone (Option Henrochromatosis typically presents in men age 40 years or
C). Glucocorticoids will not improve oral or ocular dryness, older and in women generally after menopause because of
and therefore the risks of prednisone in this patient are not slower iron accumulation in the premenopausal years.

136
Answers and Critiques

m Testing fbr antinuclear antibodies (Option A) would be therapy or with more severe disease. There is no guidance
IIJ .pptnpriate il the patient displayed characteristic signs or for optimal duration of therapy, but in the absence of flares,
toNl ,ynrptoms ot
sysl5'n-r1a lupus erytlrernatosus, such as malar most patients can discontinue treatment in 9 to 12 months.
msh, photosensitivity, inflammakiry arthritis, weight loss, Physical therapy may help maintain muscle function and
and fever. Howevel this test is not indicated because the his, should be used in all patients.
tory, examination. ancl Iaboratory findings arc not consistent Hydroxychloroquine (Option A) may be helpful as
with system ic lupus erythematosus. adjunct therapy for skin rashes in some patients with der
Erythrocyte sedintentation rate (Option B) is a nonspe matomyositis who do not want, tolerate, or obtain adequate
cific test fbr inflammation. It may be elevated in any inflam relief from topical agents and other systemic therapies. How
nratory state and \"'ould not help determine the cause of this ever, antimalarial agents do not impact muscle disease and
pat ient's in flummation. are not sufflcient as glucocorticoid-sparing therapy.
Rheumatoid factor (Option C) is useful in the diagnosis Rituximab (Option C) is sometimes used if the disease
of rheumatoid arthritis (RA). Although RA often afl'ects the does not respond adequately to glucocorticoids plus either ut
(t,
hands and \ /rists, it typically presents with morning stiff azathioprine or methotrexate, or ifthose agents are not tol
ness and the gradual onset of pain, stiflncss, and swelling ET
erated. However, this patient should receive a trial of meth
in a symmctric distribution of hands and feet. Plain radio otrexate or azathioprine and intravenous immune globulin (J
graphs in early RA would not show a hooked osteophlrte or before using rituximab. !t
calcifi cations in the fi brocartilage. Although this patient has had a good response to pred .!
nisone, symptoms are likely to recur if the glucocorticoid tt
rEY POITIS o
dose is tapered without glucocorticoid sparing therapy.
. For patients with acute calcium pyrophosphate crystal Long-term use of moderate or high doses of glucocorticoids r^
=
arthritis who are younger than age 60 years, labora- (Option D) should be avoided if possible because of cumula-
tory evaluation for contributory metabolic disease tive adverse effects, including bone loss, weight gain, meta-
(hyperparathyroidism, hemochromatosis, hypophos- bolic changes, and risk for cataracts and glaucoma.
phatasia, hypomagnesemia) is warranted.
r(EY P0ll{T5
o Hooked osteophytes of the left third metacar-
pophalangeal joints and cartilage calcification are
. In patients with dermatomyositis, initiation of metho-
trexate or azathioprine at diagnosis or after an early
radiographic findings consistent with calcium
pyrophosphate crystal arthritis.
initial response to glucocorticoids is the most appro-
priate next step to achieve long-term disease control
and to allow effective tapering ofthe prednisone dose.
Bibliography
Rosenthal AK, Ryan LM. Calcium pyrophosphate deposition disease. N Engl . Physical therapy may help maintain muscle function
J Med. 2016;374:2575 84. IPMID: 27355536j doi:10.1056/NEJMralSlll17
and should be used in all patients with inflammatory
myopathy.

Item 28 Answer: B Bibliography


Educational Objective: Treat dermatomyositis with the Oddis CV Aggarwal R. Treatment in myositis. Nat Rev Rheumatol. 2018;
14:279 289.lPMlD : 295933431 doi:10.1038/nrrheum.2018.42
addition of a glucocorticoid-sparing agent.
The most appropriate treatment is to add methotrexate
(Option B). This patient has classic clinical and laboratory
29 Answer:
leatures of dermatomyositis with myositis-associated auto-
antibodies and no evidence of associated malignancy or
Item
Ed ucationa I Objective :
B
Diagnose Felty syndrome.
tr
other complications. In particular, the presence of anti Mi-2 'lhe most likely diagnosis is Felty syndronre (Option B).'this
antibodies predicts a good prognosis with a low risk for rare syndrcme is a complication of r.tell established rheu
interstitial lung disease and cancer. Her symptoms have nratoid arthritis characterized by neutropenia and spleno
completely resolved with an initial course of oral prednisone, megaly. Patients witlr t'elty syndrome are at risk for serious
typically initiated at 1 mg/kg/d. Combination therapy with bacterial intbctions. lower extremity ulceration. lymphoma.
glucocorticoid-sparing agents is important for maintaining a ancl r,,asculitis. Patients may present rt''ith an asympk)nlatic
clinical response and for minimizing glucocorticoid adverse clecline in the neutrophil cor.rnt that is detected on routine
effects in patients with dermatomyositis. Initiation of meth- laboratory testing ibr n-redication toxicity. Because a decline
otrexate or azathioprine at diagnosis or after an early initial in leukocyte count can also be irn adverse ellbct of nunterous
response to glucocorticoids is the most appropriate next nredications used to treat rheumatoid artl.rritis. it is import-
step to achieve Iong term disease control and to allow effec ant to evaluate this possibility. llowever, medications are far
tive tapering of the prednisone dose. Intravenous immune less likely kr specifically reduce only the ncutrophil count
globulin can be added as flrst or second-line treatment for and are not likely to be associated with splenomegall,: Some
patients with an inadequate response to initial combination patients prcsent with an infection related to neutropenia.

137
tr
CON].
In this instance. the total leukocyte count mav be normal irt
the time of presentation but fall ."r'ith treatment of the inf'ec
tion. Patients rtith rhcun.ratoid arthritis can also have large
the basis of history and physical examination, MRI of the
pelvis may reveal erosions, edema, fatty metaplasia, and
changes of inflammation (synovitis, enthesitis, or capsuli
granullr lymphocyte syndrome, w'hicl.r can progress to lirrgc tis) along the sacroiliac joints. These changes are diagnos-
granular lymphocytic ler-rkemia. Finclings overlap rvith Felty tic of inflammatory spondylitis. Later in the disease, plain
synclrome and include ncutropenia. ancrnia. thromlloc),to radiographs of the sacroiliac joints may show narrowing,
penia. splenotnegallt ancl recurrent inlections. erosions, sclerosis, or fusion. Later flndings on plain radio-
AA amyloidosis (Option A) is a rare complication of' graphs of the lumbar spine may reveal early squaring of the
long standing and poorly controlled rheumatoid arthritis. vertebral bodies, followed by syndesmophyte formation.
In a minority of patiellts, it can leacl to splenomegaly as a Bone scanning (OptionA) is sensitive in the diagnosis of
consequence of infiltration of the spleen b1, amyloid protein. inflammatory arthritis, and a negative scan excludes inflam
Hovvever. AA aml,loiclosis r'r,ould not be erpected to result in mation; however, this test is rarely used in the diagnosis of
D neutropenia. making it an unlikely diagr.rosis in this p:rtient. sacroiliitis or other inflammatory arthritides because it is
UI Paticnts rvith rhcumatoid arthritis may develop an nonspecific. MRI is more speciflc than bone scanning in the
E
(D overlap syndrome characterized by features olanother auto detection of early sacroiliitis and is preferred when plain
UI immune disease. such as Sj6gren s1,'ndrome (Option C) or radiography is nondiagnostic.
o, s),stelnic lupus er1'themakrsus (SLU) (Option D). Hou'ever. A CT scan of the pelvis (Option B) may show bony
EL the patient r'r,ould not be designated as having an overlap changes of inflammatory spondylitis, such as bony erosions.
a.t
syndronre in the absence ol t-vpical features of the other sclerosis, and ankylosis, but CT exposes the patient to more

4t autoimmune disease. When SLE occurs in patients with radiation. Therefore, it is not appropriate unless the patient
rheumrrtoicl arthritis, a more typical hematologic picture cannot undergo MRI.
.D
t^ u,oulcl include l1,mphopenia. hemolytic anemia. or throm Rheumatoid factor and anti-cyclic citrullinated peptide
bocl'topenia. A positive antinuclear antibodl'test result xncl antibodies (Option D) are diagnostic tests for rheumatoid
clinicirl rnanifestations secn in SLII but not in rheunratoid arthritis. Results of these tests are not typically positive in
arthritis, such as kiclncy disease, might appear. Similarly. patients with ankylosing spondylitis. Rheumatoid arthritis
Sjogren syndrome is diagnosed in patients with rheumatoid does not cause low back pain because it does not affect the
arthritis r,r'hen sicca symptoms are prcsent. Typicallyl this lumbar spine. This patient has no peripheral inflammatory
r,r,ould be accompanied by anti-Ro SSi\ antibodies anci occa joint symptoms consistent with rheumatoid arthritis, so
sionally anti La SSII antibodies. these tests are not indicated. Although adults with rheu
matoid arthritis may have eye involvement, it is typically
KEY POIl{I5
scleritis, not uveitis as in this patient.
. Felty syndrome is a complication of well-established
rheumatoid arthritis characterized by neutropenia r(tY PolilT5
and splenomegaly. . When plain radiographs are normal and the clinical
. Patients with Felty syndrome are at risk for serious suspicion is high, MRI of the pelvis may reveal ero-
bacterial infections, lower extremity ulceration, lym- sions, ankylosis, fatty metaplasia, and inflammatory
phoma, and vasculitis. changes along the sacroiliac joints that are diagnostic
of inflammatory spondylitis.
Bibliography . A CT scan of the pelvis may show bony changes of
Gazitt T, Loughran TP Jr. Chronic rreutropenia in l.Gl. leukemia and rheuma inflammatory spondylitis, but CT exposes the patient
toid arthritis. Hematololl/ Am Soc Hematol [.]cluc Program. 2017 l)ec 8;
2017(l):181 186. I PMID: Z9'l'lZ'ZS,l] doi:10.1182/asheducation 20l7.l.l8l
to radiation and is reserved for patients who cannot
undergo MRI.

Item 30 Answer: C Bibliography


Educational Objective: Diagnose inflammatory Maksymonlch WP lmaging in axial spondyloarthritis: evalu:rtion of
inflammatory and structural changes. Rheum Dis Clin North Am. 2016:
sacroiliitis by using MRI. 12:645 662. IPMID; 27712019] doi:10.1016ii.rdc.2016.07.003

The most appropriate diagnostic test to perform next is


MRI of the pelvis (Option C). This patient has sympton.rs Item 31 Answer: A
of inflammatory back pain with sacroiliitis consistent with
ankylosing spondylitis. She has developed symptoms at a Educational Objective: Diagnose acute cutaneous lupus
young age (<45 years) and has morning stiffness, nighttime erythematosus.
pain, improvement of symptoms with exercise and ibupro- The most likely diagnosis is acute cutaneous lupus erythe-
fen, and a history of uveitis. Her decreased range of motion matosus (ACLE) (Option A). Cutaneous lupus erythematosus
of the lumbar spine in all directions is typical of this disease. can exist with or without systemic disease. There are three
Early in the disease, radiographs may be normal. When plain primary patterns of lupus-speciflc skin rashes: ACLE, sub-
radiographs are normal and the clinical suspicion is high on acute cutaneous lupus erythematosus, and chronic cutaneous

138
Answers and Critiques

lupus erythematosus. The lupus speciflc skin diseases are not and, if MCP joints on one hand are tender and swollen,
found in other conditions and have varying associations with there are usually MCP joints on the other hand (although
systemic lupus erythematosus (SLE). Identifing the pattern not always the same digits on each side) that are symptom
of the skin disease can be important for guiding additional atic and abnormal on physical examination as well. Other
evaluation and treatment. ACLE often presents with a pho frequently involved joints are the proximal interphalangeal
tosensitive malar or butterfly rash, with patchy, edematous, joints and wrists. Corresponding joints in the feet (interpha
e4,.thematous areas on the cheeks and over the nose. sparing langeal joints. metatarsophalangeal joints. and ankles) are
the nasolabial folds. Similar rashes can occur on other sun also often affected.
exposed skin, including the neck, upper chest, and arms. Ankylosing spondylitis (Option A) is a chronic inflam
Nearly all patients with ACLE have SLE, and as such tend to matory disease affecting the axial skeleton, entheses, and
have systemic symptoms and positive serologic results (typi peripheral joints. [n contrast to other forms of spondyloar
cally antinuclear, anti Ro/SSA, and anti-La/SSB antibodies). thritis, sacroiliac joint involvement is considered an essential
Lupus pernio (Option B) is a skin manifestation seen in feature of ankylosing spondylitis. About one third ofpatients UI
(,
some patients with sarcoidosis. Skin lesions can occur on the develop hip joint involvement, and about one third develop
ET
face, including the cheeks, nose, lips, forehead, or ears, and peripheral arthritis elsewhere, often in the shoulders but not
are firm and purple. These skin lesions are not photosensi the N4CP joints. L'
tive and do not appear on the chest or arms. Erosive (inflammatory) osteoarthritis (OA) (Option E'
Rosacea (Option C) is the most common cause of an B) is a subset of primary hand OA. Clinically, erosive OA .E
erythematous rash in the malar distribution. Rosacea is mostly afl'ects the distal interphalangeal and less commonly Ul

more likely to include pustular lesions and visible telangiec affects the proximal interphalangeal joints. Erosive OA is
o
tasias and does not spare the nasolabial fold. more inflammatory and painlul than typical hand OA and is ta
=
Subacute cutaneous lupus erythematosus (SCLE) more common in women. Erosive OA is not associated with
(Option D) appears as one of two variants: annular and generalized OA and usually spares the thumb bases. Erosive
polycyclic photosensitive plaques on the back. chest, and arthritis does not involve the MCP joints.
extremities, or psoriasiform scaly plaques in a similar distri Reactive arthritis (Option C) is a rare cause of inflamma
bution. SCLE does not scar but may resolve with temporary tory arthritis that can occur following speciflc genitourinary
pigmentary changes. Fewer than one in f,our patients with and gastrointestinal infections. Arthritis is asymmetric and may
SCLE have SLE, and as many as one in three cases are drug be monoarticular or oligoarticular; joints commonly affected
induced. The latter may resolve with withdrawal of the caus include the knee, ankle, and wrist. Enthesitis is especially com
ative agent. This patient's rash does not match the typical mon; Achilles tendinitis and plantar fasciitis occur in up to 90'X,
presentation ofSCLE. of cases. l)actylitis can also occur. The spine and sacroiliac joints
are less commonly involved. Symmetric involvement of the
l(EY POl]tT5
MCP joints is not characteristic of reactive arthritis.
o Acute cutaneous lupus erythematosus often presents
with the classic photosensitive malar or butterfly rash, t(EY P0ilaIS
with patchy, edematous, erythematous areas on the . In rheumatoid arthritis, both hands are usually
cheeks and over the nose, sparing the nasolabial folds; involved, most commonly manifesting as symmetric
similar rashes can occur on other sun exposed skin, involvement of the MCP joints.
including the neck, upper chest, and arms. o In rheumatoid arthritis, the interphalangeal joints,
. Nearly all patients with acute cutaneous lupus erythe metatarsophalangeal joints, and ankles are often also
matosus have systemic lupus erythematosus and as symmetrically affected.
such tend to have systemic symptoms and positive
serologies. Bibliography
Sparks JA. Rheumatoid arthritis. Ann Intern Med. 2019r170:lTC1 ITCl6.
IPMID: 110.5968791 doi: 10.71126 rAITC2OI90l010
Bibliography
\t'enzelJ. Cutaneous h-lpus erythematosus: new insights into pathogenesis

33
and therilpeutic strNtcflies. Nat Rev Rheumatol. 201911.5:519 532. [PMID:
31 39971 1 I
Item Answer: D
Educational Objective: Treat newly diagnosed
tr
ANCA associated vasculitis.
Item 32 Answer: D
The most appropriate treatnlent is high dose glucocorticoids
Educational Objective: Diagnose rheumatoid arthritis
and rituximab (Option D). This patient has granulomatosis
by the pattern ofjoint involvement.
n,ith polyangiitis (CIA). ANCA-associated vasculitis is sug-
The most likely cause of this patient's joint pain is rheuma gestecl lry the scvere ancl inflammatory multisystemic il]ness.
toid arthritis (Option D), given joint involvement that is poly Clues that this is GPA specifically include sinus, kidney. and
articular and includes the metacarpophalangeal (MCP) joints. h-rng involvcnrent; the skin flndings (palpable purpura is not
In rheumatoid arthritis. both hands are usuallv involved, specilic fbr GPA but strongly sugqests a small vessel vasculitis);

139
Answers and Critiques

tr
CONT,
peripheral nervous svsteln inr,olvement (left foot senson, bss):
and the presence of antiproteinase 3 antibodies. To conflrn.r
the diagnosis irnd guide management, tissue biopsl' (in this
for the management of OA includes educational. behav-
ioral, psychosocial, and physical interventions. as r.tell as
pharmacologic therapy. In some patients, physical. psycho
case. kidnel') rnay be done i{'possible. High dose glucocorti social, mind-body interventions may be adequate to con-
coids plus either rituximab (preferred) or c),clophosphamide trol symptoms; others may need sequential or combination
is the appropriate treirtment for induction of remission in approaches. For a motivated patient like this one, mind body
selere orgarl threatening or lif'e threatening GPA. Selecteci interventions, such as tai chi, are appropriate to consider
patients nray benefit frorn plasma erchange. Once rernission as an adjunct option. Tai chi is a traditional Chinese mind
is achieved, patients should receive maintenance therapl.con body practice that combines meditation with slon. gentle,
sisting of rituxinrirb, methotrexate. or azathioprine: rituximab graceful movements; deep diaphragmatic breathing: and
appears to be most effective for prer,,enting relapse and is relaxation. Tai chi has been shown in a randomized trial to
guideline pret.erred. On their olr,n. high dose glucocorticoids be as beneficial as physical therapy for knee and hip OA pain
are not suf ficient fbr induction and maintenance treatment ol and is strongly recommended by the American College of
tt ANCA associated vasculitis. Mortality rates in patients receiv- Rheumatologz (ACR)/Arthritis Foundation (AF) guideline
(D ing only glucocorlicoids are high. Observational studies from for the management of knee and/or hip osteoarthritis.
=
gt the 1970s ar.rd l9U0s demonstrated that glucocorlicoids plus Strong evidence supports the use of exercise in the
o, cyclophosphamide was associated r,r,ith a significant improve treatment of knee and hip OA. Most studies that assessed the
CL ment in sunival. fivc times that of historical controls. and a role ol aerobic exercise in the management of OA evaluated
n lou,er frequency ol relapse. walking and found it to be beneficial. However, high-impact
lt Clucocorticoicls plus azathioprine (Option A) is not exercises (Option A) are not tolerated by patients with knee
adequate ftrr inducing remission of GPA. Azathioprine has OA and may be harmful. Thus, they are not appropriate for
.D
vl been usecl to maintain remission once achiel.ed (although this patient.
rituximab is superior firr maintenance o{'remission). but this Patients frequently use transcutaneous electrical nerve
combination has never been shou'n tcl induce remission in stimulation (TENS) (Option C) to help relieve the pain asso-
active ANCA associated vasculitis. ciated with knee and/or hip OA. Holr,'ever, studies have
Although glucocorticoids plus methotrexate (Option B) demonstrated a lack of benefit. and TENS should not be
are pref'erred for active. nonsevere GPA. this patient has recommended to the patient.
se\ere slistemic CPA ancl tl.rerefore requires high-dose glu Weight loss is associated with a meaningful improve
cocorlicoids plus rituxin.rab. ment in knee, hip, and hand OA symptoms that is propor
Althougl.r glucocorlicoids plus mlcophenolate moletil tional to the loss of weight. The efficacy of weight loss for OA
(Option C) can induce remission in ANCA associated \ascu symptom management is enhanced by use of a concomitant
litis, patients treated u,ith this therap-v" ha'"e a higher rate of exercise program. Beyond that, speciflc diets, such as a vegan
relapse than those treated with glucocorticoids plus rituximab. diet (Option D), have not been shown to decrease OA related
Because ANCA associated vasculitis is a chrcnic condition. the symptoms and should not be recommended.
treatment least associated with relapse is the most preferable.
XEY POI lITS
XEY POIilIt . A comprehensive plan for the management of osteoar-
o High-dose glucocorticoids alone are not sufficient for thritis includes educational, behavioral, psychosocial,
induction and maintenance treatment of ANCA- and physical interventions, as well as pharmacologic
associated vasculitis. therapy.
. High-dose glucocorticoids plus rituximab are the pre- o Tai chi is as beneficial as physical therapy for patients
ferred treatment for induction of remission in severe with osteoarthritis of the knee and/or hip.
organ-threatening or life-threatening granulomatosis
with polyangiitis. Bibliography
Kolasinski SL, Neogi T, Hochberg MC. et al. 2019 American College of
Rheumatologl/Arthritis Foundation guideline for the management of
Bibliography osteoarthritis of the hand. hip. and knee. Arthritis Rheumatol. 2020:
Chung SA, Langford CA, Maz M, et al. 2021 American College of 72:220 - 233. IPMID: 319081 63] doi:10. 1002 art.41142
Rheumatolos//Vasculitis Foundation Guideline for the management of
antineutrophil cytoplasmic antibody-associated vasculitis. Arthritis
Rheumatol. 2021. IPMl D: 34235894]
35
Item 34 Answer: B
Item
Ed
Answer: B
ucational Objective: Diagnose basic calcium
tr
phosphate-associated arthritis.
Educational Objective: Manage osteoarthritis of the knee
The most likely diagnosis is basic calcium phosphate asso
with tai chi.
ciated arthritis (Option B). Deposition of basic calcium
The most appropriate treatment is tai chi (Option B). This pl.rosphate crystals (hydroxyappatite) occurs in older per
patient has knee osteoarthritis (OA). A comprehensive plan sons and targcts pcriarticular tendons and ligaments. In

140 q
Answers and Critiques

fi rare instances, basic calcium phosphate crystal deposition Rheumatoid arthritis (Option D) rarely presents in the
E is associated with inflammatory arthritis and periarthritis. shoulder in older patients. It is associated with in{lamma
GONI
most classically manifesting in older women as "Milwau tory joint fluid (leukocyte count, 2000 20.000/pl. [2.0 20 x
kee shoulder." '[his inflammatory state causes progressive fOe/L]) and is not associated with periarticular calcifications
destruction of the glenohumeral joint. and destruction of on plain radiographs. Monoarticular presentations of rheu
the rotator cuff as shown in the radiograph below. The red matoid arthritis are uncommon.
arrow pointing to the humeral head, illustrates aggressive
KTY POIXTS
bone destruction at the glenohumeral junction. The blue
arrow illustrates distended subacromial/subdeltoid bursa o Deposition of basic calcium phosphate crystals
with fluid and calcific debris. Basic calcium phosphate crys- (hydroxyappatite) occurs in older persons and targets
tals are not seen on routine synovial fluid analysis because periarticular tendons and ligaments.
visualization requires special stains and/or electron micros- . In rare instances, basic calcium phosphate crystal
copy. Unlike in other crystal induced diseases. pain onset is deposition is associated with inflammatory arthritis vt
(u
gradual.
and periarthritis, most classically manifesting in older ET
women as "Milwaukee shoulder," an inflammatory
state that causes progressive destruction of the rotator rJ
cuff and glenohumeral joint. !,
.E
ul
BibHography (l,
McCarthy GM, Dunne A. Calcium crystal deposition diseases beyond gout.
t=
Nat Rev Rheumatol. 2018;14:592-602. IPMID: 30190520] doi:10.1038/
s41584 018 0078-5

Item 35 Answer: B
Educational Objective: Treat rheumatoid arthritis with
the most appropriate biologic therapy.

The most appropriate treatment is adalimumab (Option B).


Rheumatoid arthritis treatment is aimed at controlling clin-
ical disease and joint damage. Patients whose condition is
Adhesive capsulitis (frozen shoulder) (Option A) usually unresponsive to initial treatment with methotrexate, with
occurs in patients aged 40 to 70 years. The cause appears to or without another nonbiologic disease modifying anti-
involve glenohumeral joint capsular thickening and fibrosis. rheumatic drug, could receive triple therapy (methotrexate,
Patients describe shoulder pain that is often constant but sulfasalazine, and hydroxychloroquine) or a biologic agent.
is worse at night and in cold weather: shoulder stiffness The choice of a drug regimen in such patients depends on
is common. Adhesive capsulitis can be idiopathic but is many factors, including disease activity, comorbid condi-
also associated with prolonged immobilization, anteced tions, patient preferences for route of administration and
ent shoulder surgery or injury. diabetes mellitus, hypo frequency ofdosing, adverse prognostic features, and regu-
thyroidism, and autoimmune disorders. On examination. latory and cost barriers to drug access. This patient has active
patients have limited passive and active range of motion in clinical disease and radiographic evidence ofworsening joint
all directions. Plain radiographs are of limited diagnostic use damage despite combination treatment with sulfasalazine
in patients with adhesive capsulitis: most often the radio and methotrexate. Escalation to additional therapy with a
graphs are normal, except for osteopenia in some cases. This tumor necrosis factor (TNF) inhibitor, such as adalimumab,
patient's abnormal shoulder radiograph is not consistent is an appropriate choice. Despite multiple comorbidities, this
with adhesive capsulitis. patient has no contraindications to treatment with a biologic
Calcium pyrophosphate deposition (CPPD) disease (such as active infection or heart failure). All TNF inhibitors
(Option C) is an unlikely diagnosis. Although it may involve provide similarbeneflt and maybe used in this patient based
the shoulder joint, it is typically associated with osteoar- on the factors outlined above. These regimens may have a
thritis, not severe erosive joint destruction. Calcium pyro- faster onset of action than triple therapy with nonbiologic
phosphate crystals, which are weakly positively birefringent disease-modi$zing antirheumatic drugs.
(blue when parallel to and yellow when perpendicular to Reasonable alternatives to a TNF inhibitor include sev
the polarizing axis ol'an optical filter) and classically rhom' eral other parenterally administered agents, including aba
boid shaped, are usually identified in the joint fluid. Chon. tacept (Option A), the anti-interleukin-6 receptor antibody
drocalcinosis, visible as a thin white line that tracks below tocilizumab (Option D), and the oral targeted synthetic
the surface layer of the cartilage within the joint. may be Janus kinase inhibitor tofacitinib (Option E). However,
visible on plain radiograph. Flxtensive soft tissue calcifica these agents are usually used after failure or intolerance of
tion such as seen in this radiograph is not typical of CPPD. a TNF inhibitor. Furthermore, this patient has signiflcant

141
Answers and Critiques

comorbidities that should prompt caution with consid Arthritis is common in patients with systemic lupus
eration of these therapies. Abatacept has been associated ery,thematosus (SLE) (Option D). SLE arthritis can have an
with exacerbations of COPD in clinical trials but not in acute or subacute onseti can be mono . oligo . or poll,artic
real world practice. However. it must be given cautiously ular; frequently affects small joints of the hands. elbon's. and
in patients with COPD. Tocilizumab increases the risk for knees: and can lead to malalignment deformities similar to
bowel perforation in patients with a history of diverticuli- rheumatoid arthritis. However. SLE arthritis does not lead
tis, and tofacitinib poses an increased risk for thrombotic to underlying bony erosions as seen in rheumatoid arthritis.
events. The number of joints involved is usually fbrt'er than seen in
Anakinra (Option C) is an interleukin-1 blocker rheumatoid arthritis. Characteristic mucocutaneous flnd
approved for rheumatoid arthritis. but it is less efficacious ings of SLE accompany arthritis onset. including malar rash.
than other biologics and rarely used for this indication. discoid rash, and alopecia. Systemic involvement (e.g.. leu
kopenia. thrombocytopenia, nephritis) may also be present.
TEY POIXI
D Almost all patients with SLE will be positive for antinuclear
vt
o Patients with rheumatoid arthritis not responding to antibodies. Because this patient lacks any additional features

.D
methotrexate, with or without another nonbiologic of SLE. and because positivity for anti cyclic citrullinated
UI disease modiffing antirheumatic drug, could receive peptide antibodies is not a feature of SLE arthritis. a diagno
o, triple therapy (methotrexate, sulfasalazine, and sis of SLE is unlikely.
EL hydroxychloroquine) or a biologic agent.
n f,EY POITIS

Bibliography
o Rheumatoid arthritis is characterized by the subacute
lr onset of pain and swelling in multiple small joints of
Smolen JS, Landewd RBM, Bijlsnla lWJ. d al. EULAR recommendations fbr
.D the management of rheun'latoid rrthritis with synthetic and biological
ut the hands and feet, associated with prolonged morn-
disease-modifying antirheumatic drugs, 2019 update. Ann Rheum Dis.
2O2O:79 :685 699. IPMl D: 319693281 ing stiffness and physical examination evidence of
polyarticu lar synoviti s.

. Laboratory studies supporting the diagnosis of rheu


Item 37 Answer: C matoid arthritis include rheumatoid factor, anti-cyclic
Educational Objective: Diagnose rheumatoid arthritis. citrullinated peptide antibodies, and elevated inflam-
matory markers.
The most likely diagnosis is rheumatoid arthritis (Option
C). Rheumatoid arthritis is characterized by the subacute Bibliography
onset of pain and swelling in multiple joints, including the Sparks JA. Rheumatoid arthritis. Ann lntern l\{ed. 2019:170:lTCl ITCl6.
proximal interphalangeal joints of the hands and feet, meta IPMID: 305968791 doi: 10.7326 AITC2OI90l0l0
carpophalangeal and metatarsophalangeal joints, wrists,
knees, and ankles. Prolonged morning stiffness is common;
physical examination reveals polyarticular synovitis. Joint Item 38 Answer: B
involvement is characteristically symmetric. Distal interpha-
Educational Objective: Screen for cancer in a patient
langeal joints are rarely. ilever, involved. Laboratory studies
with dermatomyositis.
supporting the diagnosis include rheumatoid factor (present
in 7O"/,, of patients. but of limited specificity), anti-cyclic The most appropriate test to perform next is a colonos
citrullinated peptide antibodies (present in 70'l,, of patients copy (Option B). This patient has dermatomyositis. A mus-
and 9511, specific), and elevated inflammatory markers. cle biopsy is not necessary to diagnose dermatoml,ositis in
Arthritis due to chikungunya virus (Option A) may patients with characteristic clinical and laboratory findings,
mimic rheumatoid arthritis with polyarticular involvement including proximal muscle weakness. strikingly elevated
of small and large joints. Onset is typically more rapid than serum creatine kinase levels. and skin manifestations that
in rheumatoid arthritis; follows travel to endemic areas: and are unique to dermatomyositis. The most typical feature is
is preceded by fever, headaches, and rash. Autoantibodies the Gottron rash (Gottron papules and Gottron sign), which
would not be expected to be positive, although inflamma is characterized by erythematous to violaceous areas over the
tory markers may be elevated. metacarpophalangeal and proximal interphalangeal joints.
Parvovirus B19 infection (Option B) causes a symmet There is an association between risk for malignancy and
ric inflammatory arthritis involving the hands that mimics dermatomyositis (3- to 12-fold higher in patients with new
rheumatoid arthritis. Although polyarticular synovitis may dermatomyositis than in age-matched populations) and, to a
be present initially, joint swelling resolves in days to weeks lesser extent, polymyositis. The risk for malignancy is great
in most cases. Onset typically occurs after exposure to a par est in the 3 years following the diagnosis of dermatomyositis
vovirus-infected child, such as in a daycare setting. Adults or polymyositis. Common associated cancers are ovarian.
with parvovirus B19 arthritis usually do not display the char lung, pancreas, stomach, and colon cancer and lymphoma.
acteristic "slapped cheek" rash seen in children. Anti cyclic The most important initial step is symptom-directed test-
citrullinated antibodies are typically absent. ing and age and sex-appropriate cancer screening because

142
Answers and Critiques

patients with active malignancy associated with dermato- anakinra and canakinumab, has been successful. Anti inter-
myositis need prompt. appropriate treatment fbr both can leukin 6 therapies have shown promise.
cer and the autoimmune disease. lf an initial directed cancer Like AOSD, familial Mediterranean f'ever (FMF) (Option
screen is normal, many experts will order CT of the chest, B) is a disease of systemic inflammation; it is also associated
abdomen, and pelvis. Because this patient has unintentional with fever, joint pain, abdominal pain, and rash in the set
weight loss and anemia, the flrst test should be a colonos- ting of high inflammatory markers. However, patients with
copy to assess for colorectal cancer. FMF typically experience lifelong recurrent intermittent
Results of antinuclear antibody testing (Option A) are episodes of inflammatory disease, typically lasting several
positive in about 50'X, of patients with dermatomyositis, but days and occurring approximately monthly, rather than the
a positive result would not meaningfully add to the diagnos persistent inflammatory state experienced by this patient.
tic or prognostic evaluation. Patients with infectious endocarditis (Option C) expe
Muscle biopsy (Option C) would likely confirm the rience fevers, petechial rashes, and arthritis/arthralgia.
diagnosis and can be useful to improve understanding Although infectious endocarditis should always be consid UI
o
of prognosis. However, this test is not necessary to diag- ered in patients with fever ol unknown origin, the pattern
ET
nose dermatomyositis when the clinical features are this of daily evening fevers accompanied by salmon pink rash is
typical. not characteristic, and the lack of a cardiac murmur argues (J
Pulmonary function testing (Option D) and other against the diagnosis. -t
evaluation for lung disease may also be warranted in most Patients with systemic lupus erythematosus (SLE) tg
patients with dermatomyositis. especially those with symp (Option D) may experience a wide constellation of symp- UI
(l,
toms or findings, antisynthetase antibodies, or factors pos toms, including rash, arthritis, and serositis. Fever in SLE
ing high risk for interstitial lung disease. In a patient without UI
may be present but is not typically as high as seen here, and =
E
active pulmonary symptoms, however, this testing is less it does not cycle daily. Moreover, the salmon pink rash seen
urgent than is an evaluation for cancer. here, directly associated with fever, is not characteristic of
SLE. SLE is much less common in men, n.raking it less likely
I(EY POITTS
in this patient.
. A muscle biopsy is not necessary to diagnose dermat
omyositis in patients with characteristic clinical and f,EY POII{T
laboratory findings, including proximal muscle weak- . Adult onset Still disease, a condition of unknown
ness, strikingly elevated serum creatine kinase levels, cause, is characterized by daily evening fevers;
and skin manifestations that are unique to dermato- salmon pink rash; inflammatory arthritis; sore throatl
myositis. serositis; and Iaboratory findings of high inflamma-
o The risk for malignancy is 3- to 12 fold higher in tory markers, elevated ferritin level, and leukocytosis.
patients with new dermatomyositis than in age
matched populations; symptom-driven testing Bibliography
Giacomelli R. Ruscitti P, Shoenfbld \'. A comprehensi\.e review on ,dult
and age- and sex appropriate cancer screening are onset Still's disease. J Autoimmun. 2018193:2.1 36. IPMID: 300771'251
indicated.

Bibliography Item 40 Answer: C


Qiang JK. Kim WB. Biribergenova A. et tl. Risk of maligniinc) in dermato Educational Objective: Treat fibromyalgia in a patient
myositis and polymyositis. J Cutan Med Surg. 2017;21:l3l 136. [PMID:
'lts:tltzgl with depression and anxiety.
The most appropriate treatment is duloxetine (Option C).
Item 39 Answer: A ln addition to discussing the diagnosis at length and pro
Ed u cati o n a I Obj ective : Diagnose
viding education regarding nonpharmacologic recommen
adult-onset Still disease.
dations, such as low impact aerobic exercise, a medication
The most Iikely diagnosis is adult onset Still disease (AOSD) is warranted. Choice of pharmacologic therapy is based
(Option A). This patient's symptoms (daily evening fevers on symptom proflle, patient comorbidities, and medication
accompanied by salmon-pink rash, inflammatory arthritis, adverse effects because few trials have directly compared
sore throat, and serositis) and laboratory evidence ol high the efflcacy of medications. Of the available choices, duloxe
inflammatory markers and leukocytosis are most consistent tine (a serotonin norepinephrine reuptake inhibitor) is most
with AOSD, a condition of unknown cause. His high serum appropriate tbr a patient with fibromyalgia and comorbid
ferritin level, although not specific, is fairly characteristic of depression and anxiety because it is FDA approved for these
AOSD. Because no speciflc test exists fbr AOSD, it remains a indications as well. Duloxetine should be started at 30 mgrd
diagnosis of exclusion, and other causes should be consid and titrated to 60 mg/d after a week or so to mitigate nausea,
ered. Past treatment options for AOSD, including glucocor a frequent early adverse effect that usually dissipates over
ticoids and methotrexate, were frequently inadequate. The time. If the drug is stopped, a slow taper should be used
use of interleukin-p directed biologic therapies, including because withdrawal symptoms often occur with sudden

143
Answers and Critiques

discontinuation. Patients should be made aware that ben- major osteoporosis-related fracture). Thus, treatment for
efits of the medication will not be seen for several weeks. glucocorticoid-induced osteoporosis is appropriate. This
To measure interval improvement, the Fibromyalgia Impact approach also applies to patients at moderate or interme
Questionnaire (a validated clinical tool) or other validated diate risk for fracture (FRAX risk score for the hip ofl?, to
clinical tool should be completed before and an appropriate 3%); those with a score of 10% to 19'1, overall would also be
time after an intervention to assess change in clinical status. a candidate for preventive treatment. Oral bisphosphonates,
Although amitriptyline (Option A) is an acceptable such as alendronate, are flrst-line therapy because of their
off label therapy for flbromyalgia, it is sedating and not efficacy, cost, availability, and ease of administration. They
appropriate for someone with a job such as truck driving. are preferred in guidelines not only because ofcost, safety,
Amitriptyline is not an effective antidepressant at the low and e{ficacy but also because ofa lack ofsuperiority ofother
doses recommended for flbromyalgia. osteoporosis medications. Clinical trial data show that both
Diclofenac (Option B) is an NSAID. Anti-inflammatory alendronate and risedronate have elficacy in glucocorticoid
D medications, including NSAIDs and glucocorticoids, are not induced osteoporosis. For patients who cannot tolerate oral
Ut more effective than placebo in treating flbromyalgia. bisphosphonates or have other circumstances precluding

.D Pregabalin (Option D), although an FDA-approved med their use, intravenous zoledronic acid may be used.
ar. ication for flbromyalgia, is not the best option fbr someone Calcitonin (Option B) is not typically used to treat or
o, with a physically hazardous occupation because both seda prevent glucocorticoid induced osteoporosis because effi
EL tion and dizziness are frequent adverse effects. Pregabalin cacy data are lacking and other, more effective drugs are
n would also not address this patient's depression or anxiety. available.
It Although tramadol (Option E) is helpful fbr treating Romosozumab (Option C) is a dual anabolic and anti
E fibromyalgia in the short term, prescribing an opioid is inap- resorptive agent that works by inhibiting sclerostin. No data
.D
UI propriate in a patient with a history of opioid use disorder or yet support the use of romosozumab for preventing or treat
someone with a job that requires a constant state of alertness. ing glucocorticoid-induced osteoporosis.
Other options are teriparatide (a parathyroid hormone
I(EY POI]ITS
analogue) (Option D) and denosumab (a receptor activator
r Choice of pharmacologic therapy for fibromyalgia is of nuclear factor rB [RANK] ligand inhibitor). Teriparatide
based on symptom profile, patient comorbidities, and is the next best option for patients who cannot tolerate
medication adverse effects. bisphosphonates or who continue to sustain fractures while
o Duloxetine is appropriate for a patient with fibromy taking bisphosphonates. However, it is not the preferred first
algia and comorbid depression and anxiety because it choice for this patient. Denosumab is used as a backup to
is approved for these indications as well. bisphosphonates and teriparatide, primarily because of the
increased risk for vertebral fracture after discontinuation.

Bibliography t(EY POll{TS


Mactarlane GJ. Kronisch C. Dean LE. et al. I:ULAR revised recommendations
Ibr the management ol fibromyalgia. Ann Rheum Dis. 2017;76:318-328.
o Glucocorticoid therapy is a major risk factor for bone
IPMID, 27377815) doi:10.11 36/annrheumdis 2016 209724 loss and increased fracture risk.
. Patients receiving long-term glucocorticoids should
Item 41 Answer: A be assessed for bone loss and fracture risk and treated
on the basis of the assessment.
Educational Objective: Treat glucocorticoid-induced
osteoporosis with an oral bisphosphonate.
Bibliography
The most appropriate treatment is alendronate (Option A). Buckley L, Cuyatt G, Fink HA, et al. 2017 American College ofRheumatologr
Glucocorticoid therapy is a major risk factor for bone loss guideline fbr the prevention and treatment of glucocorticoid induced
osteoporosis. Arthritis Rheumatol. 2017 ;69:1521 1537. IPMID: 28585373]
and increased fracture risk. In addition, fractures occur at
higher bone mineral density values than in patients with
postmenopausal osteoporosis. The risk increases with dos Item 42 Answer: D
age and duration, but even low dosages of5 to 7.5 mg/d are
Educational Objective: Diagnose a mechanical
associated with increased risk for fracture. Thus, patients
soft-tissue injury.
receiving long term glucocorticoids should be assessed for
bone loss and fracture risk and treated on the basis of the This patient has a mechanical soft tissue injury (Option D),
assessment. Both bone density measurement and gluco likely lateral epicondylitis. Common nonarticular sources
corticoid-corrected (addition of 15'2, to the risk), Fracture of musculoskeletal symptoms are the soft tissues (tendons,
Risk Assessment Tool (FRAX)-calculated, 10 year proba ligaments, and bursae) around or away from the joints.
bility can be helpful in this assessment. This patient has Isolated tendon and/or ligament involvement usually sug-
a corrected FRAX score for the hip of 3.5'1,, placing her at gests noninflammatory disorders, such as mechanical
a high risk for fracture and above the treatment thresh- injury/irritation, overuse, or degeneration (e.g., rotator cuff
old to prevent hip fracture (>s'2, at the hip or >20% for disorders or tennis elbow).

144
Answers and Critiques

The elbow is a very unlikely location for a degenera- with vaccinations 2 to 4 weeks before initiating immu-
tive joint disease (Option A), such as osteoarthritis. Localized nomodulatory agents, but this is not easy to accomplish.
osteoarlhritis, in which a single joint is affected, is more often a Evidence suggests attenuation of response to vaccines in
consequence of injury or joint asymmetry and often occurs in patients receiving biologic agents. Patients receiving tra
weight bearing joints (hip or knee), not the elbows or shoul ditional immunomodulatory agents (disease modifying
ders. Because this patient has no history of injury and has full antirheumatic drugs) in doses that cause only low grade
range of elbow motion, localized osteoarthritis is unlikely. immunosuppression can receive any immunizations, killed
Disorders of central pain processing (Option B), such or live, without any problems. Low-grade immunosuppres
as fibromyalgia, have hyperresponsive pain centers in the sion is deflned as short term prednisone use of less than
brain and spinal cord. Patients characteristically note allody- 20 mgld, methotrexate dosage of less than 0.4 mg/kg/wk,
nia (a heightened sensitivity to stimuli that are not normally and azathioprine dosage of less than 3.0 mg/kg/d. Patients
painful) and hyperalgesia (an increased response to painful who are receiving these immunosuppressive agents at higher
stimuli). An additional characteristic feature is "wind up" dosages should not receive live vaccines. Similarly, patients t
o
or temporal summation. When repeatedly exposed to a receiving biologic or small-molecule therapies should not
ET
mildly uncomfortable stimulus, patients with flbromyalgia receive live vaccines, such as measles, mumps, and rubella,
experience progressively additive pain, indicating that the or live attenuated vaccines, such as influenza and herpes (,
pain stimuli are both persistent and inadequately damped. zoster, because ofrisk for infection by the vaccine agent. In -tE
Finally, patients fypically have difluse rather than localized this patient, methotrexate at a low dosage (0.2 mg/kg/wk) ag
pain. This patient has no features characteristic of a central would not constitute a contraindication to administration vt
L
pain processing disorder. The local nature of her pain also !,
of any vaccination. However, she is also is being treated
argues against flbromyalgia. which is a disease of wide with tofacitinib, and vaccination with measles, mumps, and vt
=
E
spread pain. rubella is therefore contraindicated.
The differentiation between inflammatory and non- The hepatitis B virus (Option A), quadrivalent influenza
inflammatory signs and symptoms is central to the evalu (Option C), tetanus toxoid, reduced diphtheria toxoid, and
ation of patients with musculoskeletal pain. Autoimmune acellular pertussis (Option D), and 13 valent pneumococcal
conditions typically present with inflammation, whereas conjugate (Option E) are killed vaccines and are not contra
mechanical or degenerative disorders are characteristically indicated in this patient.
noninflammatory. The cardinal signs of inflammation are
f,EY POII{T
pain, erythema, swelling, and warmth; with the excep,
tion of pain, noninflammatory conditions usually lack these . Patients receiving biologic or small-molecule thera-
features. These features are absent in this patient, making pies should not receive live vaccines (such as measles,
inflammatory arthritis (Option C) an unlikely diagnosis. mumps, and rubella) or live attenuated vaccines (such
as influenza and herpes zoster) because of risk for
I(EY POITTS
infection by the vaccine agent.
. Isolated tendon and/or ligament involvement usually
suggests noninflammatory disorders, such as mechan- Bibliography
ical injury/irritation, overuse, or degeneration. Furer V, Rondaan C, Heijstek MW, et al. 2019 update of EULAR recommen
. The cardinal signs of inflammation are pain, ery-
dations for vaccination in adult patients with autoimmune inflammatory
rheumatic diseases. Ann Rheum Dis. 2020;79:39 52. IPMID: et+tgOOSl
thema, swelling, and warmth; with the exception of
pain, noninflammatory conditions usually lack these
features. Item 44 Answer: B
Educational Objective: Confirm the diagnosis of
Bibliography
rheumatoid arthritis with autoantibody testing.
Ilatten R. Coady D. The rheumatological examination. Medicine.
2018;46:161 165. doi.orgi 10. l0161j.mpmed.2O17.12.OO2 The combination of anti-cyclic citrullinated peptide (CCP)
antibodies and rheumatoid lactor (Option B) will be
most helpful in establishing the diagnosis. Among some
Item 43 Answer: B 100 genetic loci currently recognized as associated with
Educational Objedive: Avoid live vaccines in a patient rheumatoid arthritis (RA) risk, the most important is the
receiving biologic therapy. class II HLA group, especially HLA D alleles. These alleles
code for the shared epitope, a five-amino acid sequence that
The measles, mumps, and rubella vaccine (Option B) is binds and presents citrullinated peptide antigens important
contraindicated in this patient. Patients with autoimmune in the pathophysiologz of RA. Because citrulline is an amino
rheumatologic diseases, such as rheumatoid arthritis, are acid that does not normally occur in humans, citrullinated
at increased risk for infections because of multiple factors, proteins are immunogenic, especially in people who also
and infection prevention and vaccination are crucial. Ide- have the shared epitope. Citrulline is formed by the action of
ally, patients with autoimmune diseases should be updated the enzyme peptidylarginine deiminase, which is found at

145
Answers and Critiques

sites olinflammation and deiminates arginine to form citrul patients with inclusion body myositis have cricopharyngeal
Iinated peptides. Anti-CCP antibodies are closely associated muscle involvement, leading to dysphagia and increased risk
with the pathogenesis of RA and are highly speciflc (9S'1,). fbr aspiration. Patients with inclusion body myositis can have
In addition, anti-CCP antibodies may be positive in patients elevated serum creatine kinase levels. but elevations are usu
with RA who are negative for rheumatoid factor. Rheumatoid ally more modest, less than 10 times the upper limit of normal,
factor is less speciflc tbr RA, is present in only about 70'l. of as compared with other forms of idiopathic inflammatory
patients with RA, and may be seen in other diseases (such as myopathies in which serum creatine kinase levels are gener
hepatitis C virus infection, endocarditis, and systemic lupus ally higher. A muscle biopsy can help to conflrm the diagnosis
erythematosus). However, a f'ew patients, particularly early but is not needed when clinical fbatures are characteristic.
in the course of the disease, may be negative for anti CCP Polymyositis (Option B) can also present with prcigres
antibodies but positive for rheumatoid factor; thus, both anti sive muscle weakness. However, it generally progresses more
CCP antibodies and rheumatoid factor, rather than either rapidly over weeks or months and involves only the proximal
D alone (Options A, D), should be measured to increase the muscles. Activities such as arising from a chaiq climbing
UI diagnostic speciflcity and sensitivity of testing. When they stairs, and lifting objects above shoulder height are typically
E are present together, their specificity is further increased. afl'ected. Onset over several years with a more modest degree
.D
-
U) tsoth antibodies are included in the diagnostic criteria for RA. of weakness, as seen in this patient, would be unusual in poly
q,
The complement system is an essential part ol the myositis. In addition, polymyositis would not be expected to
CL immune response, promoting vasodilation, attracting leu cause weakness of the hands or the distal arm muscles.
a.l
kocytes, and assisting in the lysis of opsonized bacteria Statins are generally safe and well tolerated. 'lhey can
ll during humoral immunity. Complement components, such cause asymptomatic. dose related elevations in aminotrans
E as C3 and C4 (Option C), are acute phase reactants that t'erase levels in approximately 1'l, of patients, but liver injury
.D
l^ are synthesized in the liver and rise in many inflammatory occurs in less than 0.001'X, ofpatients. The incidence ofstatin
states. However, in response to diseases that lead to immune associated muscle symptoms is no greater than 1u1,, and the
complex formation (systemic lupus erythematosus; cryo incidence of myopathy (Option C) and rhabdomyolysis is
globulinemic and urticarial vasculitis) and other states, such less than 0.1'ln. However, some patients can have muscle
as inlections (bacterial endocarditis, sepsis, viremia) and symptoms from statin therapy without an elevation in serum
glomerulonephritis, complement cascades are activated, and creatine kinase levels. and it can then be difficult to be cer
serum C3 and C4 levels fall because of excessive consump tain whether muscle symptoms are due to statin therapy.
tion. Although serum C3 and C,1 levels may be elevated in '[he patient's prolonged course, minor serum creatine kinase
patients with RA, this nonspecific finding will not be helpful elevation. and muscie weakness distribution are more char
in establishing the diagnosis. acteristic of inclusion body myositis than statin myopathy.
Other autoimmune disorders, such as systemic lupus
rEY POIIITS
erythematosus (Option D), can cause myositis. However,
r Anti cyclic citrullinated peptide antibodies are a diag- they generally have other characteristic features, such as
nostic marker of rheumatoid arthritis and have a high arthralgia and skin rashes, that are not seen in this patient.
specificity (95%) but may be absent in early disease
I(EY POII{T
when rheumatoid factor is positive.
o lnclusion body myositis can be distinguished from
. The presence of both anti-cyclic citrullinated peptide
other forms of idiopathic inflammatory myopathy by an
antibodies and rheumatoid factor increases the diag
insidious onset, slower progression, and involvement of
nostic likelihood of rheumatoid arthritis.
distal muscle groups as well as proximal muscles.

Bibliography
Bibliography
\Vlrburtor.r L, Hider St.. Mallen CD. et al. Suspected very early inflammatory
rheumatic diseases in prinrirry cart. Best Pract Res CIin Rheumatol. l.undberg lE, de \risser M. Werth Vll Chssilication ol m),-ositis. Nat Rev
2019:33:101,119. IPMID: 318105501 doi: 10. lOl6r'j.berh.2019.06.001 Rheunratol. 2O18:14i269 278. lPMll): 2965112i1 doi:l0.1038inrrheunr.
201rJ.4l

Item 45 Answer: A
Ed u cati o na I O bj ective : Diagnose inclusion body myositis.
'lhe most likely diagnosis is inclusion body myositis (Option
ttem 46 Answer: C
Educational Objective: Treat systemic lupus
tr
erythematosus with nephritis.
A). h.rclusion body myositis is an uncommon cause of weak
'lhe most appropriate treatlrent is prednisone plus hldrory
ness generally seen in older adults, characterized by a slow
insidious onset. It can be distinguished clinically from poly chlorocluine and mycophenolirte mofetil (Option C)' this
myositis by the involvement of distal muscle groups in addition prtient has new onset s1'sten-ric lupus erythematosus (SI.F.)
to proximal muscle groups. Weakness in flnger flexors and r,r,ith nephritis. t.upr.rs nephritis can present with rrlini
atrophy of tbrearm muscles are characteristic features. Muscle r.r.ral laboratory abnorntalities (non nephrotic proteinuria.
distribution is usually but not always symmetric. Up to half of hcmaturia), fiank neph ril is ( hypertensiort, lower extreln i ty

146
Answers and Critiques

tr
CONT.
edema, active urine sediment, and elevated serum creati
nine level), and/or nephrosis (nephrotic range proteinuria.
dependent edema, and thrombosis). Anti double-stranded
and mouth breathing are among the most common causes of
dry mouth. Her symptoms do not suggest a speciflc rheuma-
tologic disorder. In particular, she is unlikely to have Sjogren
DNA antibody titers are a marker for risk, and complement syndrome (Option C). Oral dryness is a common symp
consumption is a common phenomenon during active kid tom, whereas Sjdgren syndrome is relatively rare. She has
ney disease. Untreated active disease may progress to kidney no ocular dryness; has a serologic proflle that does not com
tailure. He has f'ew other symptoms ot'SLE, but laboratory port with a diagnosis of Sj0gren syndrome (absence of high
findings and clinicai presentation strongly suggest this diag titers of antinuclear antibodies and/or rheumatoid factor,
nosis. Although kidney biopsy is indicated to determine anti Ro/SSA, and anti LalSSB autoantibodies); and has no
histologic subtlpe and chronicity, treatment should not be other symptoms, physical examination flndings, or labora
delayed. I{igh dose or "pulse" glucocorticoicls are indicated tory results to suggest the disease. Although it is not impos-
fbr new or recurrent active nephritis and should be fblloued sible that she has Sj6gren syndrome, the oral dryness is more
by a glucocorticoid taper. Ilydroxychloroquine is indicated likely the result of another cause. Several medications (such ur
(u
in almost all patients rn'ith newly diagnosed SLE. In addition, as antidepressants and antihistamines) frequently cause oral
ET
fbr moderate or severe disease that is organ threatening, dryness. The patient takes three such medications (amitripty
i
additional immunosuppressive therapy is indicated to con- line, cetirizine, and citalopram). It is important that patients (J
trol disease and to allow {br tapering of glucocorticoid ther like this are not misdiagnosed with rheumatologic disease !t
apy. For most patients with new nephritis, induction with and treated inappropriately with immunosuppressive agents. rE
mycophenolate mof'etil is the preferred agent. lntravenous Hypothyroidism in adults (Option A) is often associated U!

cyclophosphamide can also be used if patients have severe with a buming sensation of the mouth, but oral dryness is o
disease or are intolerant to mycophenolate. not a commonly reported symptom. This patient has appro- =
a

Prednisone plus azathioprine (Option A) could be used priately treated hypothyroidism, with a normal thyroid-
as maintenance therapy; typically along with hydroxychlor stimulating hormone level, making this diagnosis unlikely.
oquine. but azathioprine is not first-line therapy fbr ner.r, This patient's history review of systems, and physical
onset lupus nephritis. examination do not suggest systemic lupus ery.thematosus
Prednisone plus hydroxychloroquine (Option B) (Option D). Prominently absent from this patient's history
is appropriate initial therapy for mild SLE. However, in are indications ofskin disease, serositis, and kidney disease;
a patient with organ threatening disease, such as lupus other than dry mouth, the physical examination is normal.
nephritis, combination immunosuppressant tl-rerapy that The normal ery.throcyte sedimentation rate argues against a
includes mycophenolate mofetil or cyclophosphamide is system inflammatory or autoimmune condition. An antinu
rreeded tocontrol the disease. clear antibody titer of 1:80 is nonspecific and unremarkable
Prednisone alone (Option D) will help control immedi and may be seen in up to 50% of persons tested. Even at much
ate symptoms, but nithout additional immunosuppressive higher titers, a positive antinuclear antibody result alone does
agents, return of disease activity is likely as the prednisone not imply the presence of systemic lupus erythematosus.
dosage is tapered. Prolonged use ofmoderate or high-dose
t(EY PO$tT5
glucocorticoids is associated r.tith a r,ariety of adl.erse out
comes and should be avoided. o ]\{edications, such
as antidepressants and antihista-
mines, and mouth breathing are common causes of
(EY POITT'
drymouth.
. For systemic lupus erythematosus that is organ- . In the absence of additional suggestive symptoms,
threatening, combination immunosuppressive physical findings, and supporting laboratory data, oral
therapy is indicated to control disease.
dryness should not be attributed to a rheumatologic
o For most patients with newly diagnosed lupus nephri- condition.
tis, induction with glucocorticoids and mycophenolate
mofetil is the preferred immunosuppressive treatment. Bibliography
Shiboski CH, Shiboski SC, Seror R, et al; International Sjogren's Syndrome
Bibliography Criteria 'Working Group. 2016 American College of Rheumatolos//
European League Against Rheumatism classification criteria fbr primary
Fanouriakis A, Kostopoulou M, Alunno A, et al. 2019 update of the EULAR Sjdgren's syndrome: a consensus and data driven methodolos/ involving
recommendations for the management of systemic lupus erythematosus. three international patient cohorts. Ann Rheum Dis. 2ol7;76:9 16.
Ann Rheum Dis.2019;78:736 745. [PMID: 30926722] IPMID : 27 7 a9 4661 doi : 1 0. 113 6 /annrheumdis 201 6 21057 7

Item 47 Answer: B Item 48 Answer: B


Educational Objective: Diagnose medication-induced Ed u cati o na I O bj ective : Diagnose hypersensitivity
xerostomia. vasculitis.

The most likely cause of this patient's oral dryness is medi- The most likely diagnosis is hypersensitivity vasculitis
cation adverse effects (Option B). Medication adverse effects (Option B), a limited cutaneous small vessel vasculitis.

147
Answers and Critiques

This patient has a vasculitic rash, with a large crop of pete- Bibliography
chiae and palpable purpura. The appearance of the rash Frumholtz I-. I-aurent Roussei S, Lipsker D, et al. Cutaneous vasculitis:
suggests small-vessel rather than medium vessel involve- reviei( on diagnosis and clinicopathologic correlations. CIin Rev Allerg'
Immunol.2020. [PMID:32378145] doi:10.1007 s12016 020 08788 l
ment (for example, there are no subcutaneous nodules or
necrosis). Several types ofsmall vessel vasculitis can affect
the skin. The first step in the evaluation of a cutaneous Answer:
Item 49 E
vasculitis is to determine whether systemic symptoms or
flndings are present. In this case, no flndings suggest a sys- Educational Objective: Screen for Hl-{-B's8:Ol allele
temic illness. The physical examination is unremarkable before starting allopurinol therapy.
aside from the rash, and results of laboratory testing are The most appropriate management is HIA 8'58:01 allele test
similarly unrevealing. This strongly suggests that the diag ing (Option E). Presence of this allele has been associated with
nosis is hypersensitivity vasculitis. This condition is often a higher incidence of allopurinol hlpersensitivity Testing for
triggered by medications (thiazide diuretics are among this allele is conditionally recommended by the American Col-
Ut the most lrequent culprits) or infections, although about lege of Rheumatologr (ACR) in high-risk populations (includ

(D 50'1, of patients have no known precipitant. Drug-induced ing persons ofKorean, Han Chinese, Thai, and African descent,
ut hypersensitivity vasculitis most often occurs 7 to 10 days in whom HLA-B'58:01 is more common) before initiation of
o, after the introduction of a new medication, and recovery allopurinol given the potential severity of a hypersensitivity
EL is likely with discontinuation of the drug. If symptoms reaction. This reaction is heralded by a rash that may progress
n persist or recur, anti-inflammatory agents, topical or low to DRESS (drug reaction with eosinophilia and systemic rymp
dose systemic glucocorticoids, colchicine, or dapsone may toms) syndrome (DRESS is also increasingly being referred
,tr
E be helpful. to as drug induced hypersensitivity syndrome [DIHS] and
.D
.,t This presentation is not congruent with eosinophilic drug hypersensitivity syndrome [DHS]). Hypersensitivity to
granulomatosis with polyangiitis (Option A). This systemic allopurinol may also include Stevens Johnson syndrome and
small vessel vasculitis invariably involves the lungs (usually toxic epidermal necrolysis. This patient has risk factors for
with a prodromal phase of rhinitis and asthma and other allopurinol hypersensitivity, including Han Chinese descent.
pulmonary manifestations with disease progression) and and should undergo HLA-B'58:01 allele testing. Additional
often affects the peripheral nervous system and skin. Lab risk factors for severe cutaneous reactions include female sex,
oratory testing reveals the presence of ANCA antibodies history of rash with allopurinol, and concurrent use of thia-
(myeloperoxidase subtype) about half the time, and periph- zide or loop diuretics. Drug hypersensitivity can still occur in
eral eosinophilia is expected. patients negative for the HLA 8'58:01 allele, and patients must
Polyarteritis nodosa (Option C) is unlikely. Although be counseled to stop the drug immediately if a rash occurs.
it can present with palpable purpura, as in small-vessel If the allele is present, allopurinol (Option A) should
vasculitis, other skin flndings indicating medium-vessel be avoided and a different urate-lowering agent, such as
involvement would be present (e.g., livedo reticularis, sub febuxostat (Option C), should be used. However, allopurinol
cutaneous nodules, painful deep ulcers, or skin necrosis). remains the urate lowering drug of choice, and caution
This patient has no other features of polyarteritis nodosa, should be used when prescribing febuxostat to patients with
such as constitutional and musculoskeletal symptoms, a history of cardiovascular disease. Allele testing remains
peripheral nervous system involvement, or gastrointestinal the best choice for this patient. Allopurinol desensitization
disease. is conditionally recommended for patients with a previ-
Rheumatoid vasculitis (Option D) is rare. It develops ous allergic response to allopurinol who cannot be treated
in patients with long-standing and severe rheumatoid with other oral urate-lowering agents, such as febuxostat or
arthritis and is almost nonexistent in properly treated probenecid.
patients. It most often affects the skin but is a medium- Low-dose aspirin can raise the serum urate level
vessel vasculitis. This patient does not have rheuma through renal uric acid retention. However, the ACR con-
toid arthropathy; thus, rheumatoid vasculitis is not a ditionally recommends against stopping aspirin (Option B)
possibility. in patients with gout who are taking it for appropriate indi
cations. This patient should continue his low dose aspirin
as secondary prevention of coronary artery disease events.
XEY POIilTS
Abacavir is a nucleoside analog used in the treatment ol
o The first step in the evaluation of a cutaneous vasculi HIV infection. Abacavir is associated with a severe hypersen
tis is to determine whether systemic symptoms or sitivity reaction in individuals identifled by the HLA-B-57:01
findings are present. allele (Option D). Patients who are prescribed abacavir must
. Hypersensitivity vasculitis, a limited cutaneous small- flrst undergo testing to show they are HLA 8'57:01 negati've
vessel vasculitis, is often triggered by medications or
in order to reduce the risk for a hypersensitivity reaction.
As with allopurinol, absence of the allele does not protect
infections, although about 50% ofpatients have no
patients from hypersensitivity reactions, and appropriate
known precipitant.
counseling should be provided.

148
Answers and Criti ques

rEY POI IIIS t(EY POlt{IS


o Presence ofthe HLA-B.58:01 allele has been associ- . Total joint replacement is a curative option for patients
ated with a higher incidence of allopurinol hypersen- with osteoarthritis in whom conservative therapy has
sitivity and is more prevalent in persons of Korean, failed; it relieves pain and improves function.
Han Chinese, Thai, and African descent. . The American College of Rheumatologr/Arthritis
o Testing for the HLA-B.58:01 allele is recommended in Foundation recommends against the use of chondroi-
high-risk populations before initiation of allopurinol tin sulfate (with or without glucosamine), biologics,
given the potential severity of a hypersensitivity intra-articular hyaluronic acid, platelet-rich plasma,
reaction. stem cells. or botulinum toxin in the treatment of
knee osteoarthritis.
Bibliography
FitzGerald JD, Dalbeth N, Mikuls 1l et al. 2020 American College of Bibliography 9t
Rheumatolos/ guideline for the management ol gout. Arthritis Care Res
c,
(Hoboken). 2020;72:744 760. [PMID: 323919341 doi:10.1002/acr.24180 Kolasinski SL. Neogi 1. llochberg M(1, et al.2019 American (irllege ol'
RheumatololyiArthritis I,bundation guideline tbr the management of E
osteoarthritis ol the hand. hip. ancl knee. Arthritis Rheumatol. 2020: L
7 2:220 233. I PM I D: 3 I 9081 63 I doi : I 0. l002irrt..4 I 1.'12 rr,
Item 50 Answer: E =,
.!
Educational Objective: Treat advanced, symptomatic ah
osteoarthritis with joint replacement surgery. Item 51 Answer: D o
Educational Objective: Diagnose polyarteritis nodosa. ut
The most appropriate treatment is total knee replacement =
(Option E). This patient has osteoarthritis (OA) that has Magr.retic resonilnce angiographv ol the abtiomen (Option
not responded to nonpharmacologic and pharmacologic D) is nrost liliely to establish the diagnosis. lhe nrost lil<cly
approaches and is limiting function. Total joint replace cliagnosis is polyarteritis r.rodosir (PAN), a nredium vessel
ment is a curative option for patients in whom conservative r,asculitis. The patient hls an inllarnrnatory disease n,ith
therapy has failedl it relieves pain and improves function. neu, onset kidney disease but rtrithout evidence of glonrcru
Overall, outcomes are excellent, with small risk for hardware lar involvernent. Unlike small vessel '"'asculitis, r,r'l-riclr causes
loosening and Iate infection. Osteotomy is an alternative for glomerulonephritis, t']AN afiects the renal virsculirturcr there
younger patients with varus or valgus knee and predomi fbre. it does not generallv cause rn active urinary secli
nantiy unicompartmental disease. rnenl but can crusc rcnal insufficiencl- rnd hypertension.
Multiple high quality studies have shown that 'lhe nodular skin lesions sLlggest r-neclium vessel virsculitis.
arthroscopic surgery for knee OA (Option A) provides no The postprlr.tdial abdominal pain points to nrcscnteric iscl.r
advantage over conservative management unless joint buck emia. 'lhc patient has a left fbot drop. sr-rggesting periphcral
ling, instabili0r, or locking is present. This patient does not nenropathl,. Mononeuritis multiplex. which c:rn be seen in
have these indications. small or medium vessel disease. is cornmon in PAN. Whcn
Because of lack of efhcacy, the 2019 American College a patient suspectecl ot having PAN has abdominal sympbrns.
of Rheumatologz (ACR)/Arthritis Foundation (AF) guideline inraging of the mesenteric vasculature c:rn be diagnostic. In
strongly recommends against the use of chondroitin sulfate this casc. rxignctic resonrnce angiography lr,ould clenron
in patients with knee and/or hip OA, as well as combination stratc characteristic findings of tAN allecting the gut: lnulti
products that include glucosamine and chondroitin sulfate plc iureurysms and lnminul irregularities (stenosis) in bigger
(Option B). irrteries. r,t ith occlusive lesions in smaller ones.
The ACR/AF guideline recommends against the use of The clinical scenario cloes not suggest any ol the
hyaluronic acid injections (Option C). The guideline recog- ANCi\ rssociated vasculitides. AN(lA associirtcd vasculiti
nizes the limited evidence for the benefit of this treatment. des involve small vessels, and paticnts may havc sympk)rns
particularly when other alternatives have failed to provide relatecl kr the upper and loner airw:iys, kidneys (glon.rerr-rlo
beneflt. Debate about the role of these injections contin- rrephritis). and eyes, ears. iu'rcl skin (pllpablc'purpura): cor.r
ues because of conflicting efhcacy data across trials and stitutional s],'nlptonrs arc also comrnon. Therefbre, testing fbr
meta-analyses, as well as their high cost. However, they are ANCi\ (Option A) is unlikcly to be helpful.
frequently used in practice for knee OA. llepatitis A virus testing (Option B) nould not aicl in
Intra-articular platelet-rich plasma (Option D) is not diagr.rosis.llepatitis IJ vims (llBV) and tllV. Ibr r,vhich the
efflcacious in the treatment of knee OA. The ACR/AF guide paticnt is ncgativc. cau cilLlse PAN (and. rnure rarelyl so car-r
I
line strongly recommends against it; the lack of standardiza- hepatitis C virus). Heprtitis A vims docs not. Llecause IlllV
tion in availabie preparations of platelet-rich plasma makes prevalence has declir.rccl since tl.re advent of the I IBV r,accir.re
it difficult to identifiz exactly what is being injected. The ancl antiviral treatmcnt, the proportion ot' paticnts with
guideline also recommends against other similar therapies, HBV associated PAN hls dcclitred fiom :16'2, to less than
such as intra-articular injection of stem cells, biologics, or 5')1, of all IAN cascs; most contemporxry cascs are tlterefbre
botulinum toxin. considerccl to irc prirnarily autoimnrunc.

149
Answers and Critiques

m PAN spares the glomerulus but can affect the larger Like allopurinol, f'ebuxostat (Option C) is a urate 1or,r.
IIJ 1qns1 vessels, leading to hypertension and renal insuffi- ering agent that inhibits xanthine oxidase. It is indicated in
coNT
cien.y. A biopsy of the kidney (Option C) would therefore be patients in n,hom allopurinol has failed or lvho cannot toler
nondiagnostic (unless medium vessels were included in the ate allopurinol. This patient has ner,er receired urate lowering
biopsy, which would result in bleeding). therapy; therefore, febuxostat is not indicated.
Probenecid (Option D) is not a preferred medication
{EY POIIIIS
for this patient. Allopurinol is strongly recommended as the
. Polyarteritis nodosa is a medium-vessel vasculitis that preferred urate lowering agent. In addition. probenecid is
affects the renal vasculature and causes nodular skin contraindicated in patients with nephrolithiasis.
lesions, abdominal pain, and mononeuritis multiplex.
TEY POtl{T5
. In a patient who has polyarteritis nodosa with
abdominal symptoms, magnetic resonance angiogra-
. Urate lowering therapy is strongly recommended for
D patients with gout and any of the following indica-
phy can be diagnostic.
UI tions: one or more subcutaneous tophi; evidence of
(D radiographic damage (any modality) attributable to
= Bibliography
l^ gout; or frequent gout flares, with "frequent" being
De Virgilio A. Greco A. Magliulo G. et al. Polyarteritis nodosa: a contempo-
q,
rary overvie$,. Autoimmun Rev 2016;15:56.1 70. [PMID: 2688.1100] defined as tvvo or more episodes annually.
doi:10.1016/j.autrev2016.02.015
CL
rl . Allopurinol is strongly recommended as the preferred
urate lowering agent, over all other agents, including
l!
(D
art
tr Item 52
Educational Objective:
Answer: A
Treat chronic recurrent gout with
in patients with moderate to severe chronic kidney
disease (stage >3).
urate-lowering therapy.
Bibliography
The most appropriate treatment to add is aliopurinol (Option Fitzcerald JD. Dalbeth N, Mikuls T. et al. 2020 American College of
A). this patient has a recent history ol gouty bursitis as well Rheumatolos/ guideline lbr the management of gout. Arthritis Care Res
(Hoboken). 2O2O:72:714 760. LPMID: 32391%41 doi:10.1002racr.24180
as a history ofrecurrent episodes ofgouty arthritis. tophi. and
elevated serum urate. The American College of Rheumatologr
(ACR) strongly recommends initiating urate-lowering therapy Item 53 Answer: A
for patients with gout and any of the following indications:
Educational Objective: Treat parvovirus B19 arthropathy
one or more subcutaneous tophi; evidence of radiographic
damage (any modalifi) attributable to gout; or frequent gout The most appropriate treatment is an NSAID, such as
flares, with "frequent" being deflned as tvvo or more annuaily diclofenac (Option A). The patient has an acute onset of
This patient has two indications for urate-iowering therapy: polyarticular arthralgia with minimal flndings of inflam
t'wo or more recurrent episodes per year and tophi. Allopu- matory synovitis on physical examination. The history of
rinol, a xanthine oxidase inhibitor, is strongly recommended a recent febrile illness with the subsequent appearance of
by the ACR as first-line urate-lowering therapy in all patients, a facial rash in the patient's child suggests parvovirus B19
including those with moderate to severe chronic kidney dis- infection (Iifth disease). Up to 60% of adults with parvovirus
ease, with appropriate dose adjustments. In patients tak- B19 infection experience arthritis. It often presents acutely,
ing aliopurinol, the HLA-B.58:01 allele is associated lvith a is symmetric and polyarticular, and typically involves the
markedly elevated risk for acute hypersensitiviS reaction, proximal small joints of the hands. Parvovirus B19 arthri-
also known as DRESS (drug reaction with eosinophilia and tis should be suspected when appropriate clinical features
systemic symptoms) syndrome. Testing for the HLA-B-58:01 are present in someone who has exposure to children.
allele before starting allopurinol is conditionally recom- Acute parvovirus B19 inlection is diagnosed by detecting
mended for patients of southeast Asian (e.g., Han Chinese. anti parvovirus IgM antibodies in the serum. The patient's
Korean, Thai) or African descent. It is appropriate to continue symptoms could also suggest rheumatoid arthritis or pso
naproxen as prophylactic therapy for 3 to 6 months along with riatic arthritis, but the brief duration of symptoms does not
the initiation of urate lowering therapy to prevent flares whiie permit diagnosis of either of these entities at this time. It is
the serum urate level is lowered. appropriate to relieve symptoms with an NSAID but not to
Colchicine (Option B) may be used for treatment of start immunosuppressive therapy now.
acute gouty arthritis or for long term prevention of flares Etanercept (Option B) is a tumor necrosis factor inhib
while urate-lowering therapy is initiated. If it is used to itor that can be chosen as flrst line therapy for rheumatoid
prevent flares during urate lowering therapy, it should be arthritis. The diagnosis of rheumatoid arthritis would need
continued for 3 to 6 months or until serum urate is at target to be conflrmed by a longer duration of symptoms and
level. This patient has responded to naproxen and therefore additional laboratory testing (such as complete blood count.
does not need additional acute treatment ofhis gout or addi- erythrocy.te sedimentation rate, C-reactive protein, rheuma-
tional medication for prevention. Colchicine does not affect toid factor, and anti cyclic citrullinated peptide antibodies)
the serum urate level. and radiographic studies.

150
Answers and Critiques

Hydroxychloroquine (Option C) is indicated for the Systematic reviews and meta-analyses suggest that acet
treatment of signs and symptoms of systemic lupus erythe aminophen (Option A) provides no beneflt for hip or knee
matosus, dermatoml,ositis, and rheumatoid arthritis. In the OA. Acetaminophen may be considered as add on therapy
absence of a diagnosis of one of these conditions, hydroxy for short term and episodic use but is not appropriate as
chloroquine is not appropriate empiric treatment fbr an initial therapy for this patient.
acute polyarthritis. Duloxetine (Option B), a serotonin-norepinephrine
lVlethotrexate (Option D) is appropriate flrst line ther reuptake inhibitor with central neryous system activity, has
apy for rheumatoid arthritis and psoriatic arthritis. How shorn,n efllcacy for OA pain and would also be reasonable
ever, before initiation of an immunosuppressive therapy. the for this patient. However, it should be used only if NSAIDs
diagnosis of rheumatoid arthritis or psoriatic arthritis must are inadequate and is not the best choice for initial therapy.
be confirmed. When symptoms are present for only a few Gabapentin (Option C) and pregabalin are more effec
days, causes ofjoint pain and swelling, such as viral inf'ection tive than placebo in the treatment of neuropathic pain
and drug reactions, are in the diftbrential diagnosis. When conditions, such as postherpetic neuralgia and diabetic t
(u
symptoms are present for at least 6 weeks, these diagnostic neuropathy. There is no evidence of their eflectiveness for
ET
possibilities become less likely and higher suspicion for chronic OA pain. Furthermore, they may be associated with
rheumatologic diseases will emerge. On the basis of the time dose dependent dizziness and sedation. Thus, gabapentin is U
frame over which this patient reports symptoms, initiation not appropriate for this patient.
of methotrexate is not appropriate. Opioid therapy provides limited benefit fbr chronic
=,
IE
pain control in patients with OA and poses a high risk for la
I(EY POIIITS (t,
toxicity and dependence. Opioid use should be avoided for
o Parvovirus arthropathy often presents acutely, is
B19 OA treatment. Tramadol (Option E), a partial opioid with UI
=
symmetric and polyarticular, and typically involves fewer adverse effects and less addictive potential than pure
the proximal small joints of the hands. opioids, may be considered in some patients, but it should
o Parvovirus B19 arthropathy should be suspected when be used only in limited circumstances, and never as an initial
appropriate clinical features are present in someone therapy.
who has exposure to children and is diagnosed by KEY POI]IIS
detecting anti parvovirus IgM antibodies in the serum.
. NSAIDs are often the initial treatment of choice for
osteoarthritis if used judiciously in low risk patients,
Bibliography
with minimizing and monitoring of adverse effects.
Mauermann M. Ilochiluf Stange K. Klelntrrnn A, et al. Paru^,irus inli,ction
in early arthritis. Clin Exp Rheumirkrl. 2016 Mar Apr:3.1:207 13. It,l\'llI): o Acetaminophen provides no benefit for hip or knee oste
268866871
oarthritis; it may be considered as add on therapy for
short term and episodic use but not as initial therapy.
Item 54 Answer: D
Educational Objective: Use NSAIDs for initial Bibliography
osteoarthritis treatment. Kolasinski SI-. Neogi T. Hochberg MC. et al. 2019 Americilr.r College of
Rheumrtololl/ Arthritis Foundirtion guideline for the mi:lncgement of
ostcorrthritis of the hand. hip. and knee. Arthritis Rhcr"lnliltol. 2020i
The most appropriate initial treatment for this patient with 72:220 233. IPMID: 31908163] doi:10.1002,'art.411,12
osteoarthritis (OA) is an NSAID, such as piroxicam (Option
D). NSAIDs inhibit cyclooxygenase enzymes, blocking gen
eration of the lipid prostaglandin E,,,. NSAIDs are efhca Item 55 Answer: B
cious in OA and are strongly recommended by the 2019
Educational Objective: Treat polymyalgia rheumatica.
American College of Rheumatology/Arthritis Foundation
for hand, hip, and knee OA. However, their adverse efl'ect The most appropriate management is prednisone, l5 mg/d
proflles make sustained use problematic, especially in older (Option B). This patient has polymyalgia rheumatica (PMR)
persons and patients with comorbidities. Topical NSAIDs are characterized by symmetric proximal myalgia and stiflhess,
effective and because of their saf'ety profile are preferred to accompanied by constitutional symptoms and an elevated
oral NSAIDs. However, topical NSAIDs are ineffective in the C reactive protein level. Symptoms are most pronounced
treatment of hip OA. For this patient with hand, knee, and in the morning and improve after several hours. Although
hip OA, oral NSAIDs are the initial treatment of choice. This some patients also have peripheral arthritis (usually of the
patient has no risk factors for common contraindications knees and wrists), most do not. Physical flndings are usu-
and adverse effects, such as peptic ulcer disease or kidney ally limited to difficulty moving the affected areas (shoul
disease. Hence, piroxicam would be a reasonable choice as ders and hips). There is no diagnostic test specific fbr PMR.
initial treatment and may adequately relieve her symptoms. With a reasonable degree ol suspicion for PMR, an empiric
It would also be prudent lbr the patient to take piroxicam glucocorticoid trial is appropriate. Initiating prednisone at
regularly for a limited time and consider taking it as needed 12.5 to 20 mg/d is appropriate for this generally self limited
in the future. illness. Symptoms usually improve dramatically within

151
Answers and Critiques

48 hours (diagnostically helpful), and tapering may begin IgG. Type III is usually associated with autoimmune disease
after 2 weeks. and is characterized by polyclonal IgG and polyclonal IgM.
This patient has a clear diagnosis of PMR, and there is This patient has severe cryoglobulinemia, r,r'ith ne\\' onset
no need to seek an alternative diagnosis that is not suggested kidney failure, palpable purpura, and peripheral neuropathy
by the clinical presentation. Testing for rheumatoid factor, affecting the left radial nerve. Cutaneous involvement (pal-
antinuclear antibodies, and ANCA (Option A) is unnecessary pable purpura, digital ischemia, ulcers, necrosis. and livedo
because this patient shows no evidence of diseases such as reticularis) occurs in about 90'7, of patients. Other common
rheumatoid arthritis, systemic lupus erythematosus (SLE), manifestations include glomerulonephritis (usually mem
or granulomatosis with polyangiitis. Rheumatoid arthritis branoproliferative) and arthralgia without arthritis. Pulmo
is characterized by a chronic inflammatory polyarthritis nary and or gastrointestinal involvement is rare. In addition
affecting large and small joints, with a predilection for the to detection of cryoglobulins, laboratory abnormalities fre
small joints of the hands and feet. SLE is a multisystem auto quently include depressed C4 complement and low CHro,
immune disease affecting mainly women (90"1,), and skin as well as a positive rheumatoid factor result; this patient's
gt rash and joint symptoms aflect 90'1, of patients. Granuloma undetectable C4 level in conjunction with a high rheumatoid

(} tosis with polyangiitis aflects the upper and lower airways, factor level therefore strongly support a cryoglobulinemia
vt kidneys, eyes, and ears. diagnosis. Most cases of mixed cryoglobulinemia result from
q,
A prednisone dosage of 60 mg/d (Option C) is much too another underlying condition, the most common of which
EL high and unnecessary for PMR. This dosage is used to treat is hepatitis C virus infection (suggested by elevated amino
('!
giant cell arteritis. transferase levels). Patients with cryoglobulinemia should
Although the patient's primary concern is "pain every undergo hepatitis C virus testing and be treated ifthe result
,tr
(} where," further questioning reveals that the patient has pain is positive. Severe cryoglobulinemia requires urgent treat
gl in distinct areas. It is crucial to pinpoint areas of discomfort ment r,r,ith high-dose glucocorticoids plus rituximab.
via history because different disease processes result in dis Hypersensitivity vasculitis (Option B) can appear as
tinct patterns of pain. Patients with chronic widespread pain showers of purpura on the extremities but does not lead to
syndromes, such as flbromyalgia, may indeed have difluse systemic disease (glomerulonephritis, hypocomplemente-
pain. However, patients with other, more localized issues mia, neuropathy, and nephropathy).
often report "pain every.where," and in the absence ofa good Polyarteritis nodosa (Option C), a medium vessel
history premature closure may lead to an incorrect diagno- vasculitis, most commonly aflects the skin (tender erythem
sis. In this case, diagnosing fibromyalgia and prescribing atous nodules akin to erythema nodosum, palpable pur
pregabalin (Option D) would be incorrect. pura, livedo reticularis, ulcers), peripheral nervous system
(mononeuritis multiplex), and gut. However, polyarteritis
IEY POITIS
nodosa does not cause glomerulonephritis, hypocomple
. Polymyalgia rheumatica is characterized by symmet- mentemia. or an elevated rheumatoid factor. It can cause
ric proximal myalgia and stiffness, accompanied by purpuric lesions, but not to the degree seen in cryoglobu
constitutional symptoms and elevated infl ammatory linemia.
markers. Rheumatoid vasculitis (Option D) is seen in patients
. For polymyalgia rheumatica, initiating prednisone at with long-standing and severe rheumatoid arthritis. This
12.5 to 20 mg/d is appropriate treatment. patient has no history of arthropathy, and, despite his
arthralgia, does not have fiank arthritis. The rheumatoid fac
Bibliography tor level is elevated, but this test is not speciflc (results can be
Matteson El-, I)ejaco C. Polymyalgia rheumatica. Ann lntern Med. 2017:
elevated in chronic bacterial or certain viral infections and
166:l'IC65 ll'(180. [PMID: 284603951 doi: 10.71]26 AITC2Ol7O5O2O Sjdgren syndrome, for example). Anti-cyclic citrullinated
peptide antibodies, which are more speciflc for rheumatoid
arthritis. are not present. Finally, kidney involvement is pres
ent in this case but rare in rheumatoid vasculitis.
Item 55 Ansyyer: A
Educational Objective: Diagnose cryoglobulinemic XEY POITTS
vasculitis. r Clinical manifestations of cryoglobulinemia include
The most likely diagnosis is cryoglobulinemic vasculitis cutaneous involvement (palpable purpura, digital
(Option A). Cryoglobulinemic vasculitis results from cryo- ischemia, ulcers, necrosis, and livedo reticularis),
globulin containing immune complexes depositing in small- peripheral neuropathy, arthralgia, and glomerulone-
and medium-sized arteries, leading to an inflammatory phritis (usually membranoproliferative).
response. Of the three types of cryoglobulinemia, only types o In addition to detection of cryoglobulins, cryoglobu-
II and III (mixed cryoglobulinemia) cause vasculitis. Type II,
Iinemia-associated laboratory abnormalities fre
which usually results from certain viral infections and auto
quently include depressed C4 complement and low
immune disease, is typifled by a monoclonal IgM with activ
CHro, as well as a positive rheumatoid factor result.
ity against IgG (i.e., a rheumatoid factor) as well as polyclonal

152
Answers and Critiques

Bibliography Bibliography
Fuentes A, Mardones C, Burgos PI. Understanding the cryoglobulinemias. Curr Sammaritano LR, Bermas BL, Chakravarty EE, et al. 2020 American College
Rheumatol Rep. 2019;21:60. [PMID: 317,11077] doi:10.1007/sll926 019 of Rheumatologr guideline fbr the management of reproductive health
0859 0 in rheumatic and musculoskeletal diseases. Arthritis Rheumatol. 2020r
72:529 556. [PMtD, 32090480]

Item 57 Answer: D
Ed ucationa I Objective : Treat rheumatoid arthritis Item 58 Answer: B
during pregnancy. Educational Objective: Diaglose the cause ofjoint pain
The most appropriate preconception management is
in a patient with psoriasis.
discontin-
uation of leflunomide (Option D). Leflunomide is highly tera- The most likely diagnosis is osteoarthritis (OA) (Option B).
togenic and absolutely contraindicated with pregnancy. During Psoriatic arthritis (PsA) is one of the few forms of inflam
the preconception phase, leflunornide should be discontinued. matory arthritis that affects the distal interphalangeal joints. Ut
The active metabolite of leflunomide (teriflunomide) under
o
This Iocation is also commonly aflected by OA. This patient
goes enterohepatic circulation and has a half life of nearly ET
has a classic history for OA, with minimal morning stiffness,
3 weeks. Because 5 half-lives are required for a metabolite to pain with increasing use, and a classic distribution in the (,
be significantly cleared, leflunomide levels have been detected hands (distal interphalangeal and proximal interphalangeal
in the serum months following discontinuation of the drug. joints). Her radiographs are also typical for OA and show no
=t
E
tu
Demonstration of undetectabie blood levels is needed befbre r,l
erosions. The literature conllrms thal 47n/,' of patients with q,
conception. An ll day cholestyramine washout must be used psoriasis who were thought to have PsA had another cause
to remo\e teriflunomide and should be followed up with mea of joint pain, most commonly OA.
vl
=
g
surement of leflunomide and its metabolite levels to ensure Hyperuricemia is reported to have a high prevalence
removal of drug. Management of rheumatoid arthritis (RA) in patients with psoriasis. This metabolic derangement can
during pregnancy is complicated. be associated with gout development. In chronic recur
Discontinuation of medications used to treat RA rent gout (Option A), patients experience increasingly fre-
(Option A) may be feasible in some patients whose disease quent and severe, and often polyarticular, arthritic attacks.
is well controlled, but most patients need to continue ther These attacks may eventually evolve into a persistent chronic
apy during pregnancy especially if de escalation of therapy arthritis. The characteristic radiographic changes of estab-
reactivates disease, as in this patient. She is likely to require lished disease include punched-out lesions with overhang-
continued therapy during pregnancy. ing edges ofcortical bone. This patient's clinical history and
Patients with active disease who are receiving tumor radiographic flndings are not consistent with chronic gout.
necrosis factor inhibitors can continue these medications Patients with PsA (Option C) have an inflammatory
through conception; the duration of use during pregnancy form of arthritis that manif'ests with prolonged morning
depends on the speciflc agent. Certolizumab (Option B) stiffness (>30 minutes) and symptoms that improve with
may be continued throughout pregnancy because it crosses activity. PsA affects up to 25'X, of people with psoriasis, and
the placenta in low to undetectable amounts. Other tumor the associated joint disease develops after the appearance
necrosis factor inhibitors, such as etanercept, adalimumab, of skin changes in 85% of patients. The joint manifestation
infliximab, and golimumab, may be continued safely until can be pauciarticular or polyarticular and can be accompa-
the start of the third trimester. nied by axial involvement. Enthesitis (inflammation where
Hydroxychloroquine (Option C) crosses the placenta. tendons/ligaments insert into bone) is seen in almost half
However, the dose typically used to treat systemic lupus ery- of patients with PsA, and dactylitis is equally common. Nail
thematosus or other rheumatologic diseases does not pose changes are common and are associated with distal inter-
a risk for letal toxicity. It is safe to continue hydroxychloro phalangeal arthritis and enthesitis. These flndings are not
quine in this patient. present in this patient, making PsA unlikely at this time.
Because leflunomide is absolutely contraindicated Rheumatoid arthritis (Option D) characteristically affects
during any stage of pregnancy, continuation of all three the metacarpophalangeal joints, metatarsophalangeal joints,
medications is inappropriate (Option E). and proximal interphalangeal joints of the hands and feet but
spares the distal interphalangeal joints ofboth the upper and
l(EY por{Ts
lower extremities. This patient's pattern of joint involvement
o Leflunomide is highly teratogenic and absolutely con- is most compatible with OA, not rheumatoid arthritis.
traindicated during any stage of pregnancy; during
the preconception phase, leflunomide should be dis- rEY POIl{II
continued and cholestyramine used to remove its o Psoriatic arthritis is one of the few forms of inflamma-
metabolite. tory arthritis that affects the distal interphalangeal joints.
. Hydroxychloroquine can be continued throughout . The most common missed diagnosis in patients with
pregnancy. psoriasis and arthritis is osteoarthritis.

153
Answers and Critiques

Bibliography (EY P0IXTS (orilinued)


Cladman DD. Clinical features and diagnostic considerations in psoriatic
arthritis. Rheum Dis Clin North Am. 2015:41:569 79. [PMID: 26.1762191
o Anti-Scl 70 antibodies are generally associated with
doi: I 0.1016/j.rdc.20l 5.07.003 diffuse cutaneous systemic sclerosis, with a prevalence
approaching 30'7,, and these antibodies are associated
Item 59 Answer: C with a higher risk for interstitial lung disease.
Ed ucational Objective: Diagnose limited cutaneous
systemic sclerosis. Bibliography
Stochmal -\. Czuu'ara J, 'lrojanor,r,ska M. et al. Antinuclear antibodies in
The most likely diagrosis is limited cutaneous systemic scle s)stemic sclerosis: an update. CIin Rev Allerg, Immunol. 2o20:58:-10 51.
rosis (LcSSc) (Option C). Anticentromere antibodies (titer of lP\llD:306077491 doi:10.1007 s12016 018 8718 8
1:80 or higher) are rare in healthy people. The finding ofanti-
centromere antibodies (ACAs) in the setting of Ralnaud phe
D nomenon is 60'X, sensitive and 98'7, speciflc for LcSSc, which Item 60 Answer: C
t was previously knov,n as the CREST slmdrome. Patients with
E Ed ucationa I Objective: Diagnose Sjogren syndrome.
(D LcSSc have manifbstations that can include calcinosis, Ray
ta naud phenomenon, esophageal dysmotility, sclerodactyly, and The most appropriate diagnostic test to perform next is
A'
3 telangiectasia (CREST); all flve conditions may not be present. anti Ro/SSA antibodies (Option C). This patient has flnd
EL
The presence of anticentromere antibodies increases the risk ings highly suggestive of Sjogren syndrome, oral and ocular
n for pulmonary hypertension, which occurs in 20"1, of these dryness and enlarged parotid glands. She also has other non
tt patients. Patients who have only Raynaud phenomenon and specific symptoms (achiness, fatigue. and Raynaud phenom
(D ACA must be followed for development of LcSSc. This patient enon) that are common in patients with Sjdgren syndrome.
t^ with recent onset of Ra,,naud phenomenon is showing features Anti Ro/SSA antibodies are the most speciflc diagnostic test.
ofLcSSc. Although gastroesophageal reflux disease is common The patient also has an elevated rheumatoid factor lerel.
in the general population, the recent onset oftelangiectasias and a flnding seen often in patients with Sj6gren syndrome.
sclerodactyly suggests LcSSc. The telangiectasias seen in LcSSc Although rheumatoid factor is more common in Sj6gren
are matted; these clusters of dilated vessels appear as small pink syndrome than in rheumatoid arthritis, it is not speciflc to
or red macules that blanch on light pressure. They are most either disease.
commonly seen on the face, mucous membrane, and hands. ANCA testing (Option A) would not be informative for
Patients with diffuse cutaneous systemic sclerosis (DcSSc) this case because the patient's presentation does not suggest
(Option A) have skin involvement that is more extensive than systemic vasculitis. Clinical manifestations of vasculitis are
that found in LcSSc and additionally includes the arms, trunk, the result of tissue ischemia associated with the involved
and lower extremities. Anti Scl 70 antibodies are generally vessels. ANCA associated vasculitides involve small vessels.
associated with DcSSc, with a prevalence approaching 30'1,, and patients may have symptoms related to the upper and
and these antibodies are associated with a higher risk for inter Iower airways, kidneys, eyes, ears, and skin; constitutional
stitial lung disease. ACAs are uncommon in DcSSc. symptoms are also common.
Eosinophilic fasciitis (Option B) is associated with Anti cyclic citrullinated peptide (CCP) antibodies
orange peel induration (peau d'orange) ofproximal extrem (Option B) are more speciflc than rheumatoid factor for
ities, with sparing of hands and face, and the presence of diagnosing rheumatoid arthritis. Ho'nrever, in a patient who
peripheral eosinophilia. The presence of Raynaud phenom seems likely to have Sjdgren syndrome, anti CCP antibody
enon and the positive ACA result make eosinophilic fasciitis testing is not helpful. Only a small proportion of people
an unlikely diagnosis. presenting with joint discomfort have rheumatoid arthritis.
Mixed connective tissue disease (Option D) is a speciflc The normal joint examination provides another clue that
overlap syndrome that includes clinical manif'estations of at rheumatoid arthritis is unlikely.
least two of the following: systemic lupus erythematosus, Cryoglobulinemia can be caused by Sjdgren syndrome,
polymyositis, and systemic sclerosis. Positive results for anti- but there is no reason to suspect cryoglobulinemia in this
Ul-ribonucleoprotein antibodies are the primary laboratory patient. Cryoglobulinemic ischemia is fixed rather than
feature in many patients. The absence of flndings associated reversible (digital tips often appear gangrenous). Cryoglob
with another connective tissue disease and a positive ACA ulins (Option D) cause small and medium vessel vasculitis
titer support the diagnosis of LcSSc rather than mixed con by depositing in vessel walls and inciting an inflammatory
nective tissue disease. response. Cryoglobulinemia almost always causes a rash (e.g..
leukocytoclastic vasculitis manifesting as palpable purpura)
XEY POIf,IS
and often causes mononeuritis multiplex and arthralgia.
. The finding of anticentromere antibodies in the set Kidney disease, usually membranoproliferative glomerulo
ting of Raynaud phenomenon is 60% sensitive and nephritis, is also frequently seen. Rheumatoid factor and an
98% specific for the development of limited cutaneous elevated erythrocyte sedimentation rate are expected in cryo
systemic sclerosis. globulinemia, but the rest of this patient's clinical picture
(Continued)
does not support the diagnosis of cryoglobulinemia.

154
Answers and Critiques

XEY POITIS narrowi ng. Thus, adding hydroxychloroquine to febu xostat


e Anti would not be appropriate.
Ro/SSA antibodies are the most specific labora-
Probenecid (Option B), a uricosuric drug, is not eflec
I tory test for diagnosis of Sj6gren syndrome.
tive as a urate-lowering agent in patients with chronic kid
. Although rheumatoid factor is more common in ney disease (estimated glomerular llltration rate <50 mL/
Sjdgren syndrome than in rheumatoid arthritis, it is min/1.73 m2) and should not be added to febuxostat in this
not specific to either disease. patient.
Continuing febuxostat alone (Option D) wiil not further
Bibliography lower serum urate levels; the patient has been receiving this
Shiboski (lFI, Shiboski SC, Seror R, et ali lnternrtional Sjdgren's Syndrome therapy for 1 year and has steady state levels, and therefore
Criteria Working Group. 2016 American College of Rheumatolos//
she will not likely benefit further from it.
European l-eague Against Rheumatism classification criteria for primary
Sjogren's syndromei a consensus and data driven methodolos/ involving
three international patient cohorts. Ann Rheum Dis. 2017;76:9 16- I(EY POIilI ta
q,
IPMID : 277 89 1661 doi : I 0. 1 I 36 /a nnrheumdis 2O1 6 21 O 57 1 . Switching to pegloticase rather than continuing cur
rent urate-lowering therapy is strongly recommended ET
.P
for patients with gout for whom allopurinol, febux
Item 61 Answer: C U
Ed u cati ona I Objective : Treat refractory tophaceous
ostat, or uricosurics have failed to achieve the serum E
urate target and who continue to have frequent gout tli'
gout with pegloticase. UI
flares (two or more flares per year) or who have non-
The most appropriate treatment is to stop febuxostat and resolving subcutaneous tophi.
o
(,t
start pegloticase (Option C). This patient has chronic topha =
E
ceous gout with a persistently elevated urate level above Bibliography
target (<6.0 mg/dl [O.ss mmol/L]) despite maximum dose Fitzcerald lD. Dalbeth N. Mikuls T. et al. 2020 American College of
Rheumatologr guideline fbr the management of gout. Arthritis Cirre Res
febuxostat. Switching to pegloticase rather than continuing (Hoboken). 2020;72:714 760. IPMID: 3239193.11 doi:l0.lOO2/acr24luo
current urate lowering therapy is strongly recommended
for patients with gout for whom allopurinol, febuxostat,
or uricosurics have failed to achieve the serum urate target Item 62 Answer: C
and who continue to have frequent gout flares (two or more
Educational Objective: Treat interstitial lung disease in
flares per year) or who have subcutaneous tophi. Because
systemic sclerosis.
this patient is allergic to allopurinol and febuxostat has
failed, pegloticase is indicated. Pegloticase is recombinant The most appropriate treatment is mycophenolate mofetil
pegrlated uricase administered intravenously every 2 weeks. (Option C). This patient with systemic sclerosis has classic
All patients should be screened for glucose 6 phosphate manif.estations of interstitial lung disease (lLD), which occurs
dehydrogenase (G6PD) activity before administration of in 50'2, of patients with difluse cutaneous systemic sclerosis.
pegloticase; low activity of G6PD poses a risk for hemo The prevalence is even higher in those who have anti Scl
lytic anemia and methemoglobinemia, and pegloticase is 70 antibodies (857,). This patient has dyspnea on exertion,
contraindicated in that setting. Pegloticase rapidly lowers cough, a reduction in FVC and DIco, and a high resolution
the serum urate level, and prophylactic therapy (colchicine, CT scan th:rt is typical for nonspecific interstitial pneumo
NSAIDs, or glucocorticoids) must be administered concur nitis. Patients may also have usual interstitial pneumoni-
rently with treatment. Formation of antibody to pegloticase tis. A usual interstitial pneumonitis pattern demonstrates
may occur (30% 50% of patients) and cause severe infusion honeycombing on high resolution CT and is less responsive
reactions. Serum urate level must be checked before each to therapy than nonspeciflc interstitial pneumonitis. The
infusion. After the initial dose, a serum urate level greater 6-minute r,r,alk test provides afunctional assessment; if oxy
than 6.0 mg/dl (0.35 mmol/L) on t\,vo consecutive assess- gen desaturation of 88'2, or below occurs while the patient is
ments indicates loss of efficacy due to antibody formation. walking, the patient is a candidate for home oxygen therapy.
necessitating discontinuation of pegloticase. Pegloticase Some studies suggest that aggressive treatment of gastro
should not be given with any other urate lowering therapy intestinal reflux disease may have a positive effect on ILD
so that interpretation of the serum urate level before each progression. Mycophenolate mofetil is a mainstay of therapy
infusion is clear. Therefore. this patient should discontinue for ILD in systemic sclerosis. Data from open-label trials sug
febuxostat. gest that mycophenolate mofetil can stabilize and, in some
Hydroxychloroquine (Option A) is used in the treat- cases, improve lung function. The Scleroderma Lung Study ll
ment of rheumatoid arthritis. This patient does not have a compared oral cyclophosphamide for 1 year to mycopheno
history of rheumatoid arthritis. The erosions seen on her Iate mofletil for 2 years. Both agents were equally eflicacious
radiographs are typical of gout with sclerotic borders and in improving and stabilizing FVC; however, mycophenolate
overhanging edges. Typical rheumatoid arthritis radio mofetil was better tolerated and can be used for many years.
graphic changes include periarticular osteopenia, mar whereas cyclophosphamide has a time limited utility due to
ginal erosions without overhanging edges, and joint space toxicity. Expert opinion places mycophenolate mofetil flrst

155
An swe-11
-1 1
d 9.!!'q.! :t
for both induction and maintenance therapy in patients with inflammatory lower extremity arthritis, reactive arthritis.
systemic sclerosis and ILD. and psoriatic arthritis. Upper extrenriry findings make lllV
Hydroxychloroquine (Option A) may be useful for infbction less likely. this patient's skin finding is typical of'
arthritis in systemic sclerosis but does not play a role in other dissen.rinated gonococcirl infection rather than I I I\r associated
manifestations of the disease, including ILD. arthritis, nrhich may have no skin nrani{estations or may be
Methotrexate (Option B) has utility in skin disease and associated with kerakrderma blenorrhagicum, cir-cinate bal
arthritis of systemic sclerosis, but no data suggest it is useful anitis. ancl psoriasis.
for ILD. Arlhritis due to lyrne disease (Option D) tends to have
Nintedanib (Option D), a tyrosine kinase inhibitor, slows a subacute onset and nronoarticulirr presentation. typically
the progression of idiopathic pulmonary flbrosis and ILD in causing a swollen but not exquisitely painful knee. Patients
the setting of systemic sclerosis. However, the patients who will sometinres note accompanying firtigue but not constitutional
beneflt the most, when to initiate therapy, and optimal duration symptoms. No skin nranifestations are associated Lyme
"vith
D of therapy are unclear. Most experts reserve nintedanib for irrthritis because it is a late n.ranifestation of Borrelia burgdor
UI patients who cannot take mycophenolate mofetil or cyclophos lbri inl'ection and is not associated with erythema nrigratrs.
E phamide or who progress while receiving these treatments. The cliflerential cliagnosis of nerv onset oligoarticular
.D
ul inflanrmatory arthritis in a 1'oung \r'oman should include
o, XEY POIXIS
systemic lupus ery4hematosus (SLE) (Option E). Arthralgia and
CL . Interstitial lung disease occurs in 50'/. of patients with arthritis are presellt in up to 95',1, ot patients with SLE at some
f.l diffuse cutaneous systemic sclerosis; the prevalence is point in their disease. l]ouever, the abrupt onset of symptonrs
85% in the presence ofanti-Scl 70 antibodies. in this case and the absence of skin n.ranifbstations lvpical of
It
(D
. Mycophenolate mofetil is a mainstay of therapy for SLE (alopecia. discoid lesions. malar rash) make Sl.E less likely
tn interstitial lung disease in systemic sclerosis. X EY PO Il{IS
o Gonococcal arthritis may present as an acute inflam
Bibliography
matory monoarthritis.
Volkmann ER. Tashkin Dq Li N. et al. Mycophenolate mofetil versus placebo
tt)r systemic sclerosis related interstitial lung disease: an lnttlysis of o An arthritis-dermatitis syndrome characterized by
Scleroderma l.ung Studies I and ll. Arthritis Rheumatol. 2ol7t69:1451
1.160. IPMID: 28376288] doi:10.1002/art.,1011,1 acutely painful tenosynovitis, migratory arthralgia,
and pustular or vesiculopustular skin lesions is the
most common manifestation of gonococcal arthritis.

tr Item
Ed
63
ucationa
Answer: A
I Objective : Diagnose disseminated Bibliography
Ross JJ. Septic arthritis of native ioints. lnfect Dis Clin North Am. 2017;
gonococcal infection.
31:2O3 218. IPMID: 28366221]
'[he most likely diagnosis is disseminatecl gonococca] infec
tion (Option A). Although gor.rococcal arthritis may present
as an acute inflammatory monoarthritis, the most typical Item 64 Answer: A
llresentation is an arthritis dermatitis syndrome charac Educational Objective: Diagnose mixed connective
terized by fbver, chills, malaise, acutely painful tenosynovi
tissue disease.
tis, migratory arlhralgia. and pustular or vesiculopustular
skin lesions. More rarely, hemorrhagic nracules or papules The most appropriate diagnosis is mixed connective tissue dis
rnay be seen. Most patients have 2 to 10 skin lesions, most ease (MCTD) (Option A). MCTD is an overlap syndrome that
commonly on the distal extremities. Women nlay present includes clinical features of systemic sclerosis, systemic lupus
cluring or immediately after menstrual periods. However. erythematosus, andior inflammatory myositis. It is associated
genital s1'mptoms are often absent. Disserninated gonococcal with anti-Ul ribonucleoprotein (RNP) antibodies. Clinical
infbction should be part of the difl'erential diagnosis fbr any features may present sequentially over time, as in this patient
sexually active patient presenting'nvith possible inf'ectious presenting with proximal muscle weakness consistent with
arthritis or inf'ectious tenosynovitis, rcgardless of age. inflammatory myositis. Ralmaud phenomenon is common
Acute l.repatitis B virr.rs (HBV) inf'ection (Option B) is in patients with MCTD. Gastroesophageal reflux may be due
included in the difl'erential cliagnosis of'disseminated gono to esophageal dysmotility, a systemic sclerosis-like condition;
coccal infection. Patients with acute IIUV infection can also the presence of diffusely puffi fingers is also a common find
develop fever, chills, polyar-thritis, tenosynovitis. and rash. ing in early systemic sclerosis. Treatment of MCTD is based on
Hortever, the arthritis of ircute HBV inf'ection is usually the speciflc organ system involved. The presence ofnew myo-
polyarticular and symmetric. In addition. the rash may be sitis requires treatment with immunosuppressive medication.
urticarial. lichenoid, or palpable purpura, but not pustular. Rheumatoid arthritis (Option B) causes an inflamma-
tllV infection (Option C) has been associatecl with a tory arthritis, typically involving the metacarpophalangeal
variety of musculoskeletal presentations, including a short joints and wrists, and may be associated with Raynaud phe
lived (<24 hour) painful :rrticular syndrome. oligoarticular nomenon. However, it is not associated with anti UI-RNP

156
t
Answers and

antibodies or myositis and thus is an unlikely diagnosis in Cyclophosphamide (Option A) can be considered as sal
I

this patient. vage therapy for patientswith severe inflammatory myositis


Systemic lupus ery.thematosus (SLE) (Option C) is asso refractory to other therapies but is not considered flrst or
ciated with such clinical features as photosensitive rash, sero- second Iine treatment. The potential for harm outweighs the
: sitis, kidney disease, cytopenias, and hypocomplementemia. expected benefit in patients with inclusion body myositis,
Myositis may be a feature of SLE, but the lack of other features and cyclophosphamide should not be initiated.
of SLE in this patient, coupled with the features of systemic Intravenous immune globulin (Option B) can be used
sclerosis (puffu fingers and gastroesophageal reflux), makes in patients with polymyositis or dermatomyositis who have
this a less likely diagnosis. The presence ofantinuclear anti- an inadequate response to initial combination therapy with
bodies helps establish a diagnosis of SLE but is not speciflc glucocorticoids and traditional immunosuppressive agents,
for SLE, and antinuclear antibodies are commonly fbund in such as methotrexate or azathioprine. However, it would
other rheumatologic diseases, including MCTD. not be likely to add beneflt in patients with inclusion body
Undifferentiated connective tissue disease (Option D) is myositis without a glucocorticoid response. a^
o
associated with milder symptoms of arthritis and Raynaud For patients with most forms of idiopathic inflamma-
ET
phenomenon. It is not associated with myositis or systemic tory myopathies, adding methotrexate (Option C) to gluco
sclerosis features. Although patients are positive for antinu- corticoids for combination therapy is a good initial practice IJ
clear antibodies, they are usually negative for the specific that can reduce the total amount and duration ofglucocor t,
extractable nuclear antigen subsets (e.g., anti-double-stranded ticoid use. Ilowever, there is little evidence for methotrexate .E
DNA, anti Ro/SSA, anti LalSSB, anti UI-RNP antibodies, or other glucocorticoid sparing therapy in patients with UI

and anti smooth muscle antibodies). inclusion body myositis, especially if they have no meaning o
ful clinical response to high-dose glucocorticoids. vt
=
xtY PotilIs
Restarting prednisone (Option E) in this case is unlikely
r Mixed connective tissue disease is an overlap syndrome to benefit the patient because he had little to no meaningful
that includes clinical features of systemic sclerosis, sys response to prednisone when initially instituted. Adverse
temic lupus erythematosus, and/or inflammatory elfects associated with long-term glucocorticoid use, includ
myositis; it is associated with Raynaud phenomenon ing glucocorticoid myopathy, could exacerbate his reduced
and anti-Ul ribonucleoprotein antibodies. muscle function.
. Treatment of mixed connective tissue disease is based t(EY POtl{TS
on the specific organ system involved.
I
. Patients with inclusion body myositis often have little
improvement while receiving immunosuppressive
Bibliography
therapies; these therapies should be discontinued ifthey
Cunnarsson R, tletlevik SO, Lilleby V et al. N,lixed connective tissue disease.
Best Pract Res Clin Rheumabl. 2016;30:9.5 lll. [PMID: 27121219) doi:t}. are not effective.
l0l 6ij.berh.2016.03.002
o Physical therapy may help patients with inclusion body
myositis to maintain muscle function for activities of
Item 65 Answer: D daily living.
Educational Objective: Treat inclusion body myositis.
Bibliography
The most appropriate treatment is to initiate physical ther Creenberg SA. Inclusion body myositis: clinical features rrnd pathogenesis.
apy (Option D). Inclusion body myositis is an insidious con Nat Rev Rheumatol. 2019 May;15(5):257 272. I PMID: 308377081 doi:
10.10i18/s41584-Ol9-O186 x
dition affecting older adults. It is distinguishable from other
forms of myositis by its slow progression and different pat
tern of muscle involvement. Both proximal and distal muscle 66
involvement occur; finger and forearm flexor involvement
is nearly pathognomonic. Up to half of patients with inclu
Item
Ed ucationa
Answer: A
I Objective : Diagnose Chikungunya arthritis.
tr
sion body myositis have cricopharyngeal muscle involve- Inf'ection with chikungunya virus (Option A) is the most
ment, leading to dysphagia and increased risk for aspiration. likely diagnosis. Chikungunya is an arbovirus spread by
Serum creatine kinase levels are elevated to a lesser extent nrosquitocs. Acute infbction is heralded by the abrupt onsct
than seen in other forms of idiopathic inflammatory myopa- of serere. polyarticuhr arthralgiir artcl fever, an exanthen-r:r
thies. Autoantibodies are less often present, but anti NTSc1A lhat may bc fbcal or difluse, ancl conjunctivitis. 'lhe distri
autoantibodies are found in up to half of patients and rarely bution o{ the arthritis is often bilaterally syrnmetric and
in healthy persons. Patients with inclusion body myositis, usually involves 10 or nrore joints. Ihe temperature is olten
unlike those with other forms of idiopathic myositis, often greater than 119.0 "C (102.2 "F). ancl fel.er persists fbr 3 to
have little improvement while receiving immunosuppressive 5 days. l,aburatory testing may rcvcal leukopcnia, lymph
therapies. Therapeutic trials should not be continued if they openia, thrombocytopenia. ancl elevated scrunr arninr.r
are not effective. Physical therapy may help patients to main translerase and creatinine levels. Chikungunya virus can be
tain muscle function for activities of daily living. detected b1, polymerase chain reaction for 1 u,eek after onset

157
Answers and Critiques

ot syll.lptoms. after lt,hich testing becrtmes unreliable. Spe to 3 weeks after an enteric infection or a bout of nongono
EX cific lgM antibody is detectable ill 5 to 10 da1,'s afler symp coccal urethritis or cervicitis. loint inflammation can be
CONT.
tolr onset and remains positivc fbr up kt 2 to 3 rnonths. This intense, with high leukocyte counts in the synovial fluid.
patient's intensity of pain, lnrgc trunrber of joints involvcd, The sacroiliac joints and spine may be involved. Enthesitis
higl.r temperature. and residence in an endemic area make usually affects the heel (plantar fasciitis or Achilles ten-
chikungunya the most likely diagn<tsis. dinitis) or may present as dactylitis of the flngers or toes.
Acute hepatitis B virus infection (Option B) ma1' be Extra articular features can include nonpainful oral ulcers:
lccompanied by an abmpt. severe onset olpoly'articular joint ocular inflammation (conjunctivitis, keratitis, episcleritis.
pain that antedates and ternrinates r,r'ith the onset of jaunclicc. or anterior uveitis)r and a psoriasiform rash (keratoderma
Hort'eler. it is not associatecl r'r'ith irn ircr-rte f'ebrile onset or blennorrhagicum), typically on the hands and feet. Some
'uvidespread maculopapular rash: other skin findings. such as patients also develop circinate balanitis on the glans penis
urticaria, small vessel vasculitis. and ery.tl.rema multifbrme, or erythema nodosum on the lower extremities. Reactive
D have been associated with acute hepatitis B virus infection. arthritis is treated with NSAIDs at anti inflammatory doses
la I lepatitis C virus inf'ection (Option C) is largely asynrp tbr at least 2 weeks. If relief is incomplete, intra-articular
E bmatic but occasionally is acctimpanied by polyarthritis. glucocorticoid injections and oral glucocorticoids can be
lD
aa llowever, it is not associated with an acute febrile onset or used. If symptoms persist beyond 3 to 6 months, the use of
o, nracukrpapular rash. When hepatilis C virus inlection is disease modifliing antirheumatic drugs. such as sulfasala
CL associated with cryoglclbulinenria. arthralgia ma1' be seen zine, methotrexate, or tumor necrosis factor inhibitors. may
rl but the r:rsh is purpuric. Other skin mar.ritestations of hep be necessary for symptom control and to prevent joint ero
lt atitis C virus infection nrav inclutle livedo reticularis. ery sion. Therapy is discontinued 3 to 6 months follor,r'ing dis-
(D
thema multiforme. and lichcn ltlanus. ease remission. Most cases of reactive arthritis last 6 weeks
Ut HIV infbction (Option D) is associirted u,ith sereral mus to 6 months; only 25'X, of patients have persistent disease.
culoskeletal syndromes. Although the painful articular syn [.ong term complications can include erosive arthritis, usu
clrome is characterized by an abrupt onset of arthralgia and ally of the metatarsophalangeal joints; aortitis with aortic
nryalgia, it is not associatecl witl.r fl'ver, is oligoarticular and valve insufficiency; atrioventricular heart block; and uveitis.
asymnretdc, most lrequently involves the joints of the lower Antibiotics, such as azithromycin (Option A), generally
cxtrcmities, and typically resolves r,r'ithin a day HlV associated are not indicated in reactive arthritis because they do not
arthritis, leactive afillritis. and psoriatic arthritis are some affect illness outcomes. Antibiotics may be used for per
times seen in patients u,ith tllV infbctior.r but do not hare ln sistent enteritis and for patients with symptoms of acute ure-
acute onset and are not associated \\'ith fe\,er and rash. thritis or persistent Chlomydio associated reactive arthritis.
This patient has no indication for antibiotic therapy
I(EY POITIS
Between 50% and B0'7, of patients with reactive arthritis
. Acute infection with chikungunya virus is heralded by are HLA B27 positive. but testing for HLA B27 (Option B) is
the abrupt onset of severe, polyarticular arthralgia, useful only if the diagnosis is uncertain. In this patient. the
high temperature, rash, and conjunctivitis and often diagnosis is not in question and HLA B27 results would not
with bilaterally symmetric polyarticular arthritis. change treatment.
. Chikungunya virus can be detected polymerase chain Stool cultures (Option D) would be useful only if the
reaction for 1 week after onset of symptoms; specific diarrhea persisted. This patient had short-lived enteritis, so
IgM antibody is detectable in 5 to 10 days after symp- stool studies are not indicated.
tom onset and remains positive for up to 2 to 3 months. I(EY POITTS
. Reactive arthritis typically entails arthritisienthesitis
Bibliography
in the lower extremities 2 to 3 weeks after an enteric
Pathak H. Mohan MC, Ravindran V Chikungunya arthritis. Clin Med (Lond).
2019;19:381 385. [PMID: 31530685] doi:1o.7861 clinmed.2019-0035 infection or a bout of nongonococcal urethritis or
cervicitis.
. Reactive arthritis is treated with NSAIDs at anti-
Item 67 Answer: C inflammatory doses for at least 2 weeks.
Ed ucationa I Objective : Treat reactive arthritis.
Bibliography
The most appropriate management is an NSAID, such as
Schmitt SK. Reacti\€ arthritis. Intl,ct [)is Clin North Am. 2017:31:265 277
piroxicam (Option C). The patient has post enteric infection lPl\'llD: 282925.101 doi:10.1016 i.idc.20l7.ol.002
reactive arthritis, with asymmetric pauci arthritis involv
ing the lower extremities, enthesitis involving the Achilles
tendon, and dactylitis of the right fourth toe and left third
Item 68 Answer: C

and fourth toes. Reactive arthritis is the least common form


Educational Obiective: Treat lgG4-related disease.
of spondyloarthritis (2'l,,
of cases). Reactive arthritis typi 'llre rnost irppropriate trealnrent is rituximab (Option C). The
cally entails arthritis/enthesitis in the lower extremities 2 ir.rfiltrative involvement of'the parotid and lacrimal glands,

158
Answers and Critiques

tr
CONT
together \,ith retroperitonell fibrosis and periaortitis. is
nrost consistent \\rith [gG.1 relatecl disease. IgG.l positive
plasnrablasts on biopsy con[irnt thc dirrgnosis. The patient's
Pulmonary arlerial hypertension (PAH) occurs in roughly
10'7, of patients with systemic sclerosis and is more prevalent
in the limited form (in this patient, evidenced by distal only
clevated alkaline phosphatasc lcvcl suggests associated bil skin involvement). PAH is a major cause of morbidity and
iary involvement. 'lhe absencc of elevated serum IgG4 anti mortality. Risk factors include anticentromere antibodies,
boclies is not exclusior-rary because serum antibodies are antibodies to U1 ribonucleoprotein, antiphospholipid anti
often not present. Treatment of lgG4 related disease r.nost bodies, disease duration greater than 6 years, and telangi
often consists of initial high dose glucclcorticoids lbllo"vecl ectasias. TTE provides an estimation of pulmonary arterial
bv a slort, taper. Beciruse the cliserrse often recurs ciuring systolic pressure and assessment of both right and left
tape'r. nr-rd because glucocorticoirls bear significtrnt toxic heart size and function. Current recommendations based
itr': co:rdministration ol rituxinrab (irn anti CD20 antibody on expert opinion vary. but PAH screening with pulmo
that indirectly depletes plasnrablirsts) is olten initiated to nary function tests and TTE at baseline and at the time of'
pernlit accelerated glucocorticoid taper. l{or,r,ever. ritux development of any new signs or symptoms is reasonable. ta
o
inrab nronotherapy is also eltec'tive and can be used alonc ln addition, most experts obtain pulmonary function tests
ET
in pltients with absolute or relative contraindications tcl and TTE annually. PAH is typically associated with a Dt.c<r
glucocorticoid theraplr that is less than 60'X, of predicted and is disproportionately (J
Cevimeline (Option A) is rr lrarirsympathetic musca low compared with the decrease in lung volumes (FVC/ .E,
rinic agonist used to trcrt sicca in patients with Sjogrerr Dr.co ratio >1.6) or a decrease in Dt.co ctf 2O''1, or greater in .E
syndrune. Because this paticnt has parotid enlargement. 1 year. In this patient, an elevated FVC/Dr-co ratio (1.66) tl
(,
shc n.rigl.tt hare sicca and rnight bcnefit fiom cevimeline. suggests the possibility of PAH and mandates the need fbr
Ilou,e."'er. cevimeline h:rs not becn stuclied in the context of 'l"l'E. Because TTE may be more sensitive than specific. fur vt
=
E
IgG.1 parotitis, and tlre patient hls r.rot reported dry nrouth. ther testing is required il the results indicate possible PAH.
Methotrexate (Option B) is a clisease r.r-rodify-ir-rg anti Cardiac magnetic resonance imaging (Option A) can
rheumatic drug used as x glucocorticoici sparing agent fbr measure right ventricular mass, and right ventricular hyper
nrrrly diseases, including ANCA vlsculitis. Methotrexate hrs trophy can be a clue to the presence of PAH. However, this
llso been used extensively lbr icliopathic retroperitoneal test is expensive and cannot estimate pulmonary artery
li brosi s. I Ioweveq limited evidence supports methotrL'xxtc pressure. TTE, on the other hand, can be used fbr that
to xssist in glucocorlicoid trpering or as a single agent in purpose.
Ig(i1 rclated disc'ase. [t rryor-rlcl therefbre not be the first High resolution CT o1' the chest (Option B) is most
choicc in this instance. useful to Iook for underlying interstitial lung disease (which
Surgical debridement of the retroperitoneal mrrss is more prevalent in dilluse than in limited cutaneous sys-
(Option D) may'be necessary if the mlss threatens to erode temic sclerosis). However, the normal pulmonary examina
into thc aorta. but thirt is not the cirse in this patient.'lhe tion, normal FVC, and significantly diminished DLco more
rnass is \ryrapped around both uretersl however, there is nt-r strongly suggest PAH and the need fbr TTE.
hydnrnephrosis or deterioration ot'kidney function to sug Right hearl catheterization (Option C) is the gold stan
gest obstructive nephropathy, which would require ureteral dard fbr conflrming PAH, but it is invasive and is typically
stcnt plrcement. At this ciisease stage, medicai treatnlent is done after noninvasive tests suggest PAH. Right heart cath
nlore appropriate. eterization should be performed in most patients with an
abnormal TTE result to confirm the diagnosis of PAH and tcl
s
I( EY POI TI guide therapy.
. Treatment of IgG4-related disease most often consists
of initial high-dose glucocorticoids and rituximab;
t( EY P0 t ilI3
rituximab can be used alone in patients with absolute o In systemic sclerosis, pulmonary arterial hypertension
or relative contraindications to glucocorticoid therapy. is tlpically associated with a Dlco less than 60%
of predicted and is disproportionately low compared
Bibliography with the decrease in lung volumes (FVC/Drco
I)erugino CA, Stone lFI. IgG4 relxted disease: rn Lrpdate on pathophysiologl ratio >1.6)
and inrplications for clinicll clre. Nrt Rcv Rheumatol. 2020;76:702-714.
lPMll):1129390601
r In patients with systemic sclerosis, transthoracic
echocardiography can estimate pulmonary arterial
systolic pressure and assess both right and left heart
Item 69 Answer: D size and function.
Educational Objective: Diagnose pulmonary arterial
hypertension in a patient with limited cutaneous systemic
Bibliography
sclerosis. Sundaram SM, Chung L. An update on systentic sclerosis associated pulmo
nJry arterial hypertension: a review ol the current literature. (lurr
The most appropriate diagnostic test to perform next Rhcumatol Rep. 2018;20:10. I PMlt): 29,1880161 doi:10.1007is11926 ol8
is transthoracic echocardiography (TTE) (Option D). o7o9 5

159
Answers and Critiques

Item 70 Answer: C Bibliography


Martinez Prat L. Nissen MJ, Lamacchia C, et al. Comparison of serological
Ed u catio n a I O bj ective: Evaluate for rheumatoid biomarkers in rheumatoid arthritis and their combination to impro\€
arthritis using laboratory testing. diagnostic performance. Front tmmunol. 2018:9:1113. lPlvllD: 29928272]
doi:10.3389 fimmu.2018.01113
The most helpful diagnostic studies will be rheumatoid
factor (RF) and anti cyclic citrullinated peptide (CCP)
antibodies (Option C). Rheumatoid arthritis presents Item 71 Answer: D
with symmetric, polyarticular inflammatory synovitis and Educational Objective: Diagnose sarcoidosis.
EX
prominently involves the small joints of the hands and
feet. These include the proximal interphalangeal, metacar- The most appropriate diagnostic test to perform next is
pophalangeal, and interphalangeal joints of the toes and skin biopsy (Option D). The combination of inflltrative and
metatarsophalangeal joints. RF is found in approximately destructive bone lesions. skin lesions. parotid and lacrimal
70% of patients with rheumatoid arthritis and may be enlargement. l-rilar lymphadenopath)', and hepatic nodules
present at the time of disease onset; however, it has limited strongly sllggests sarcoidosis, an inflammatory disease tl-rat
ta
E speciflcity. Anti CCP antibodies are also present inTO% of is characterized by formation of noncaseating granulomas
.D
patients with rheumatoid arthritis but have a speciflcity that can infiltrate almost any organ. Sarcoidosis is best diag
UI
o, of 95%. Positivity for both tests increases the likelihood of nosed b-r- histologic assessment of a biopsl' specimen o{'
EL rheumatoid arthritis and may assist in early diagnosis. afl'ected tissue. In this case. skin is the most accessible tissue
rl C reactive protein and ery.throcyte sedimentation rate and is clinically involved; granulomatous skin inrolrement
(Option A) are inflammatory markers that can be elevated is common in sarcoidosis and aflects approximatell' 25'li, of
.tt patients.
in a host of infectious conditions, as well as reflect systemic
(D
I nfi ltratir,e involvement ol bone suggests the possibilig'
ta inflammation in diseases such as rheumatoid arthritis. How-
ever, they have no diagnostic specificity for any particular of a malignant condition. potentialll' a lymphoma. Such a
disease and do not distinguish among forms of inflamma diagnosis could be supported by the hilar l1'mphadenopa-
tory arthritis. thy or-r chest radiograph, as rvell as by the parotid (but not
Enzyme linked immunosorbent assay and Western blot commonly lacrimal) enlargement. Horverer. the patient has
serologies (Option B) can be useful in the diagnosis of Lyme no other li,mphadenopathli her complete blood cour-rt is
disease, including Lyme arthritis. However, this testing is nomal. and the nodular ir.rvolvement of the liver and skin
not appropriate in this patient. The patient has po$articular r,r,oulcl be atypical. Moreover. sympton.l duration ot 2 vears
symmetric inflammatory arthritis, which is not a pattern isr-rot cor-rsister-rt lvith 11,'n.rphoma. Bone biopsy (Option A)
of presentation of Lyme disease. The Spical presentation of and bronchoscopic biopsl ol'a hilar t; mph node (Option B)
Lyme arthritis is a subacute onset of monoarticular inflam coulcl distinguish lymphoma f'rom sarcoidosis. bnt both tests
matory arthritis of the knee. are irlasive and are not needed if a skin biopsv confirms
Viral infections, such as rubella and parvovirus B19, sarcoidosis.
are associated with an acute polyarthritis syndrome that Parotid and lacrimal enlargement couid be consistent
may mimic rheumatoid arthritis. These viral syndromes u,ith Sjogren syndrome, an infiltrative disease of exocrine
lzpically last only a few days to several weeks, usually less glands. Sj6gren s_v-.ndrome can affect the skin and joints;
than 6 weeks. Serologic testing (Option D) can help identify honever. in the skin it q,pically produces a non nodular
viral syndromes in some individuals with early disease. rash, and in the joints it produces a nonerosi'ue arthritis
Anti-CCP antibodies are usually negative in patients with rather than iin infiltratire disease of bone. Nodular liver
viral syndromes causing po\zarthritis. Early identiflcation of involvement would not be expected. Diagnosis of Sj6gren
rheumatoid arthritis is important for initiation of effective syndrorne could involve obtaining a lip biopsy (Option C).
therapy to prevent joint damage. Initial measurement of RF along r,vith anti Ro/SSA rtr anti LaTSSB antibodl testing:
and anti CPP antibodies is the best diagnostic test option in houever. sarcoidosis is more likely and a skin biopsy u'ould
this patient with a 7 week history of symmetric, polyartic be definitive.
ular inflammatory synovitis that prominently involves the l(tY Porlrls
small joints of the hands.
o Sarcoidosis is an inflammatory disease that is charac
I(lY POlttrs terized by formation of noncaseating gtanulomas and
can infiltrate almost any organ, including bone.
. The most useful laboratory studies to aid in the diag
nosis of rheumatoid arthritis are rheumatoid factor
o Sarcoidosis is best diagnosed by histologic assessment
and anti-cyclic citrullinated peptide antibodies. of a biopsy specimen of affected tissue; when skin is
o Rheumatoid factor and anti involved, biopsy is simple and can be definitive.
cyclic citrullinated pep-
tide (CCP) antibodies are approximately 70% sensitive
in the diagnosis of rheumatoid arthritis, but anti-CCP Bibliography
antibodies are 95"L specific. Kuchaz EJ. Osseous manifestations of sarcoidosis. Reumatologia. 2020;
58:93 100. IPMID: 324766821

160
Answers and Critiques

Item72 Answer: D Bibliography


Educational Objective: Diagnose renal amyloidosis Mercieca C, van der Horst Bruinsm, IE, Borg AA. Pulmonary, renal irnd
neurological comorbidities in patients with ankylosing spondylitis;
associated with poorly controlled anlgdosing spondylitis. implications fbr clinical practice. Curr Rheumatol Rep. 2O14;16:434.
IPMID: 249255891 doi:10.1007/sll926 o14 O4:r4 7
The most likely diagnosis is renal amyloidosis (Option D).
Patients with long-standing, poorly controlled inflammatory
diseases, such as ankylosing spondylitis (AS) or rheumatoid Item 73 Answer: B
arthritis, can develop renal amyloidosis (AA amyloidosis). Educational Objective: Treat rheumatoid arthritis with
The prevalence of kidney disease in AS is 2',/, to 13'l.; in the
disease modiffing antirheumatic drugs.
past, AA amyloidosis was the leading cause of kidney disease
(and remains a major cause) in patients with uncontrolled AS. The most appropriate treatment is methotrexate (Option B).
With the advent of biologic therapy and subsequent control The patient has rheumatoid arthritis, and methotrexate is
ot inflammation in AS, the prevalence of AA amyloidosis has considered the flrst line treatment ol a patient with a new I,l
diagnosis of rheumatoid arthritis. Methotrexate can be used o,
declined. AA amyloidosis most commonly aflects the kid
neys, manifesting as proteinuria and eventually end stage alone for rheumatoid arthritis, and the dose can be titrated ET
kidney disease. It less commonly affbcts the gastrointesti up over a period of weeks, with periodic assessment of .E
the clinical response and laboratory measures of systemic
(J
nal tract, heart, and peripheral nerves. In AS, AA amyloi
dosis is most often seen in men, patients with peripheral inflammation (erythrocyte sedimentation rate, C reactive =,
E
.!
joint disease, and patients with a history of uveitis. Affected protein) and monitoring fbr medication toxicity. Contra' vt
patients will usually manilest high levels of inflammatory ceptive use in women of child bearing age is recommended q,
markers, such as erythrocyte sedimentation rate and C because methotrexate can be associated with pregnancy loss U!
=
E
reactive protein, indicating insufficient control of disease and fetal abnormalities. Il methotrexate alone is inadequate
related inflammation. Treatment with efl'ective medications. after a trial for an appropriate length of time, it can also
such as tumor necrosis factor inhibitors, may stabilize or be used in combination with additional disease modifying
improve the kidney manif'estations of AA amyloidosis in agents, including tumor necrosis factor inhibitors, abata
patients with AS. When AA amyloidosis is suspected, Congo cept, tofacitinib, and rituximab.
red staining of biopsy specimens from the kidney, rectum, or NSAIDs, such as diclofenac (Option A), are not indicated
abdominal subcutaneous fat will reveal amyloid deposition. fbr the treatment of rheumatoid arthritis and should not be a
Analgesic nephropathy (Option A) may develop after substitute for initiating treatment with a disease modifying
years of NSAID use and typically presents with abnormali ergent. NSAIDs may be used in patients with rheumatoid
ties on routine urinalysis, including pyuria, proteinuria, and irrthritis and other fbrms of inflammatory arthritis to relieve
hematuria. Chronic kidney disease and end stage kidney symptoms. However, they would not be expected to alter the
disease can develop in some patients. This patient's urine natural history of the disease or prevent permanent deformi
findings are not compatible with analgesic nephropathy. ties or radiographic damage. NSAID use with glucocorticoids
lgA nephropathy (Option B) is seen in patients with should be avoided. and caution should be exercised with
AS but typically manifests as hematuria and proteinuria, concomitant methotrexate.
not isolated proteinuria. lgA nephropathy in patients with IVlycophenolate mof'etil (Option C) is an immunosup
AS tends to be milder, with less renal insufficiency, than pressive drug that reversibly inhibits inosine monophos
in those with idiopathic IgA nephropathy. More aggressive phate dehydrogenase and aflects lymphocyte stimulation
treatment of AS does not seem to afl'ect the clinical course of and proliferation. It is important in the treatment of systemic
the AS-related IgA nephropathy. lupus erythematosus and in transplant recipients but has no
Interstitial nephritis (Option C) can be caused by a role in the management of rheumatoid arthritis.
variety of medications, including NSAlDs. antibiotics, and Rituximab (Option D) is indicated for the treatment ol
proton pump inhibitors. Symptoms may include fever, rash, rheumatoid arthritis. However, it is not an appropriate choice
ancl eosinophilia/eosinophiluria. All patients will have a rise fbr initial therapy in this patient. Rituximab is indicated fbr
in serum creatinine levels as well as pyuria, hematuria, and treatment of patients who have had an inadequate response
proteinuria. Leukocyte casts are typically seen on urinalysis. to methotrexate alone or in whom multiple other agents have
This patient lacks these laboratory findings. firiled. Rituximab is given via intravenous administration ot'
two doses 2 weeks apart every 6 months. Thus, the time
KEY POI l{TS liame over which it would be expected to work in rheumatoid
arthritis would be inappropriate for a patient with new onset
o Patients with long-standing, poorly controlled anky-
disease who has not yet been treated with methotrexate.
losing spondylitis can develop renal (AA) amyloidosis,
the major cause of kidney disease in these patients. I(EY POITTS
r AA amyloidosis most commonly affects the kidneys, o Methotrexate first-line treatment of a patient
is the
manifesting as proteinuria and, eventually, renal with a new diagnosis of rheumatoid arthritis.
insufficiency. (Continued)

161
Answers and Critiques

XEY POlllTS (otldiawd) Streptococci (Option D) are less commonll intpli


cated in prosthetic joint in{'cctittrts than are staph}lococci.
. If methotrexate alone is inadequate, it can be used in
accollnting tbr about i0')1, of positive sy'noYial fluid culturcs
combination with additional disease-modifying
from infbcted prosthetic joiltts.
agents, including tumor necrosis factor inhibitors,
abatacept, tofacitinib, and rituximab. XEY POIXTS
r Early onset (<3 months) and late-onset (>12 months)
Bibliography prosthetic joint infections are most commonly due to
Singh JA. Saag KG. Bridges St- Jr. et ill. 2015 American College of Staphylococcus oureus; delayed onset (3 to 12 months)
Rheumatologl guideline lbr the treatment of rheumatoid arthritis.
Arthritis Rheumatol. 2016:68:l 26. IPNIID: 265.159.10] doi:10.1002 infection is most often due to coagulase-negative
art.39.180 staphylococcus.
o Delayed-onset (3 to 12 months) prosthetic joint infec-
D 74
UI
E
(D
tr Item
Ed
Answer: C
ucational Objective: Diagnose prosthetic joint
tion is characterized by an insidious onset and subtle
physical examination findings (absence of fever, local
pain only, slight warmth, mild effusion).
infection with coagulase-negative staphylococcus.
vt
o,
Irrfection \ ,ith Stoph.Ulococ'cris epirlermidis. a coagulasc Bibliography
a- negative staphylococcus. is nlosl likel1, (Option C). Earll
n onset (<:l nlonths after surgerv) prosthetic joint infectior.rs
Beanr E. Osmon D. Prosthetic joint infection update. Infect Dis Clin North
Anr. 2018:32:843 859. [PMID: 3o2-ll7 17 | doi:10.1016 i.idc.2018.06.005

lt are often due to Stciph.qkrc'oc'clls olu?Lls. gram negirtivc


(D bacilli. anaerobes. or pollmicrobial infection. Delalecl
UI
onset (3 to 12 months after surgery) infections are ofter.r Item 75 Answer: B
dtrc to coagulase-negative staphylococci. Cutibocteriunt
Educational Obiective: Treat mild systemic lupus
species, or enterococci. [.:rte onset infections (>12 rnonths)
erythematosus.
irre often due to S. crurzu.s, gram negative bacilli. clr p
hcnrolytic streptococci. '[ris patient de'veloped ne\\r symp The most appropriate treatment is prednisone and hydroxy
toms in a prosthetic knec about 6 nronths after tot:rl joint chloroquine (Option B). This patient has clear flndings of
arthroplasty: His history is t-v-pical of delayed prosthetic new but relatively mild systemic lupus erythematosus (SLE).
joint infection, with an insidious rather than abrupt olrset She has skin and joint involvement and thrombocytope
of initially mild but then persistr.rlt s) mptoms. The phl,sical nia without evidence of other organ involvement. First line
e\ilmination Iindings are subtle: an absence of fever. krcal therapy for mild SLE includes hydroxychloroquine. often
pain onl1,. slight r'r,armth. and mild efiirsion. The orglnisrn with glucocorticoids if a more rapid response is needed.
most commonly irnplicatecl in prosthetic joint ir.rfection Many patients will need additional immunomodulatory
in this setting with subacutc onsct. subtle findings :l to 12 therapy at some point in their disease. Hydroxychloroquine
months trfter surgery is coirgulirse negative staphylococ is a mainstay of treatment in SLE because it reduces disease
cus. Because of the fiequency of staphylococcal infectior.rs, associated damage, prevents disease flares, and improves kid
it is ah^,ays appropriate to initiate ernpiric treatment ibr ney and overall survival. In addition, hydroxychloroquine
infcction r.r,hile awaitirrg culturc rcsults: r,ancomlcin is au may reduce the risk for thrombosis, liver disease, and myo
appropriate first choice. Whcn aspiratior-r ol a prosthetic cardial infarction and improve lipid profiles. Glucocorticoids
joint is required. the procedure shoulcl be perfbrn.recl b1'an are also used in most patients with SLE. particularly in acute
orthopedic surgeon. disease. After disease stabilizes, glucocorticoids are tapered
Prosthetic joint infection uith Neisseria gonorrhoeae to the lowest effective dosage. pref'erably to no more than
(Option A) almost never occurs. Ihe arthritis associated \\'it11 7.5 mgrd within 4 to 6 months. and should be discontinued
gonorrheal infection is generirlly abrupt in onset and oftell entirely if possible. Glucocorticoid exposure. especially at
acconrpanied by exquisitely p:iinful tenos)inovitis, fever. ancl high doses, should be limited because of associated risk for
skir.r lesions. The lesions rangc fiont single pustules to a le."l, organ damage, infection, and premature mortality. Azathio
lesions that might be papular. pustular. ur hemorrhagic lnd prine or methotrexate is recommended if patients with mild
ltre nrost fiequently firund on the distal extremities. Thesc disease have refractory symptoms. All patients with SLE
findings are not seen in this patient. should be counseled on sun protection, vaccinations. and
I)seirdomonos oeruginoso (Option B) is occ:rsionall_r' lifestyle modiflcations to optimize cardiovascular health.
implicated in prosthetic joint infcctions but is considerablv Prednisone alone (Option A) is likely to help with her
less common than grarn positivc organisms. often repre current symptoms but is unlikely to provide Iasting benefits
sented in iess than 10'.1, of positive cultures. A l.ristory o1 after it is tapered or discontinued.
gastrointestinal or genitourinlr_"- infecti<-rn or immunosup For patients with more severe SLE manifestations.
pression puts patients at higher risk fbr prosthetic joint including nephritis, central nervous system disease, or
infcction with Pseudornoncrs sltecies. 'll.ris patient does not other more severe manifestations, additional therapy is war
have a hisbry compatiblc with suclr infection. ranted. Induction with mycophenolate or cyclophosphamide

152
Answers and Critiques

(Option C) would be reasonable, in addition to glucocorti- ll1 acute flare, ntainly to improve {irtr-ire adherence' but
coids and hydroxychloroquine, in that setting. This patient only il tl-re patient is also treing adequately treated ftlr the
does not require this degree of pharmacotherapy unless her acute flare. Allopurinr-rl i,vill not 1.reat the acute flare and'
disease progresses. if adrninisterecl alone. may' actr-rally prolong the f lare dura
Alt patients with a new diagnosis of SLE probably tion. When initiating allopr-rrinol, paticnts shoulcl cotrtinue
I require some immunomodulatory therapy, such as hydroxy tiking anti inflantnratory f lare prophylaxis ft)r irt least 3 to
i
chloroquine. Those with cutaneous symptoms alone can also 6 months because urate lowcring transiently raiscs the fre
: quency of flare occurrence.
be treated with topical agents, such as hydrocortisone cream
t (Option D), and monitored carefully, but this patient has NSAIDs. such as indontcthacin (Option C). given fbr 5 t<r
I joint inflammation and hematologic Iindings that warrant 7 clays are also ef lective in treating acute gout. NSAIDs have
t intervention with additional systen.ric theraplt an antiplatclet ef I'cct atrd are associated r,r'ith risk ftrr gastro
t
I
intestinal ulcer:rtion and bleeding. This patient is receiving
(EY POIilIS UI
I
anticoagulation: thus. NSAIDs arc contraindicated. (l,
t o First-line therapy for most patients with systemic Prednisone (Option )) may be an eflective trcatment ol
ET
lupus erythematosus includes hydroxychloroquine acute gout. l-lowcver, it raiscs the blood glucose Ievel and is
t with glucocorticoids. not tl'rc best choicc in a patient rvith type 2 diabetes mellitus.
I
(J
I . Hydroxychloroquine is a mainstay of treatment in t(EY POfi{Tt g
=,
systemic lupus erythematosus because it reduces IE
i . In the absence ofchronic kidney disease, colchicine, tt
I disease-associated damage, prevents disease flares,
1.2 mg, lollowed by a single 0.6 mg dose in t hour, is
o
t and improves kidney and overall survival.
an effective treatment of acute gout within the first
yt
=
E
i 24 hours.
Bibliography
:

r Fanouriakis A. Kostopoulou M. Alunno A, et al. 2019 update of the EULAR o Administering urate-lowering therapy, such as allopu-
;
recommendations fbr the management of systemic lupus erythematosus. rinol, in the absence of anti-inflammatory therapy may
Ann Rheum Dis. 2019;78:736-745. IPMI D: 309267221
result in recurrent flares of acute gout.
:

Item 76 Answer: B Bibliography


;

t EX Educational Objective: Treat acute gout with colchicine. tritzcerald JD, Dalbcth N. l\,tikuls T, et al. 2020 American College ol'
Rheumatolos/ guideline fbr the management of gout. Arthritis Care Res
i
'lhe nrost appropriate treatnrent is colchicine (Option B). (Hoboken). 2o2o..l 2:741 760. [PMID: 323919341 doi:10.1002/acr.24180

'lhis patient's presentrtion is consistent lvith an ircute flare


of classic podagra (gor-rt ol the great toc) lvitl.r pain. swelling, Item 77 Answer: D
and reclness ol the first r.netatarsophalangeal joir.rt. His r-:rdio-
Educational Objective: Diagnose lupus nephritis with a
gruph shou's changes consistent u'itl.r chronic gout),arthritis
kidney biopsy.
in the rnetatarsophalangeal joint, with soft tissue swelling.
tophus. trnd r'rell defined erosions that havc sclerotic bor- 'lhe most appropriate diagr.rostic test to perfbrm ncxt is
dcrs and ovcrhanging margins. (lolcl'ricine is an eft'ective kidncy biopsy (Option D). This patierlt has acute systemic
trcatlxenl of acute grut u,ithin the first 24 hours of the h"tpus crythernatosus (St.E) with proteinuriir ancl hematuria
eirisoclc. [n the iibsence of chronic kidrrclr discase. the initial conccnring fbr kidne-y involvement. Iler positive results lor
dose is 1.2 mg, lbllowed by a singlc 0.6 mg close t hour later antinuclear and anti double-strandecl DNA antibodies ancl
ancl then 0.6 mg dail1, therealter if' r.reedcd. Colchicine. dcpressed serum complement values support this cliagnosis.
0.(r mg dail1,, 6ny be continued ancl ttsed as prophyhxis Additional evaluation in patients suspected ol having lupus
u'hile urate lowering therap), is initiirted. Oolchitine rnal' nephritis should include 2.1 hour urine measLrrcment fbr
result in gastrointestinal toxicity, ancl ckrsing may be limited quantitation of lrroteinuria ancl a kidney biopsy. Patients
b)'gastrointestinal tolerance. 'lbpical icc l.relps reduce clura lvith SLE with any sign of kidney involvement ilre can
tion of the flrrre and ma1, be r-rsed as adjunctive theraplr didates for kidney biops1,. Indications include proteinuria
the Americar.r College <tf Rhcumatology (ACR) strongly grcater than 500 mg.24 h, urine pnltein creatinine ratio
rccomnrencls initiating urate lornerir.rg therapy tbr patients greater than 500 mg,'g (especialll, in thc setling ol hematuria
with gout and any of thc fbllor,l,ing inclications: onc or more andior cellular casts), tir unexplained decrease in estirnatecl
subcntlneous tophi; e\riclencc of rldiographic darnage (any glomcrrrlar filtriition rate. Biops), results are important for
moclality) attributable to gout: or lreqnent gout flares. with choosing incluction thcrapy and cletermining prognosis.
"frequent" being clefined as two or more annually 'lhe ACR S[.E can cause nellropsl,chiatric rnanif'estations (cerebri
rccommends treating to a maxinrum serum uratt: levcl of tis). including seizure. delirium, psychosis, or stroke, which
lcss thirn 6.o nrg,dL (0.:15 mrnol,L). Allopurinol (Option A) can be importanl to iclentify rvhen choosirlg management
is indicirted ir-r tl'ris patient lbr long term trclltnrcnt girren strategies. Ilrair.r MRI (Option A) is sensitive tbr cletection of'
the racliographic changes ol gouty arthritis. Current ACR abnormalitics associated r,vith cerebritis buI can be difficu]t
guidelines support initiating nratc lor,r'cring therapy during to inlerpret. This paticnt has no symptr)ms to sugigest central

153
l11we_1s and Critiqtes

tr
CONT.
nervous syslent discase. so a brain MRI is less likelr,to Lle
useful in this case.
Chest CT (Option B) can be useful lbr identificalion of
maintaining range of motion, and reducing disability: transi
tion to an ongoing daily exercise program is optimal.
If NSAIDs fail or the patient cannot tolerate NSAIDs,
serositis, interstitial lung disease. or pulmona4, cmbolisru. a tumor necrosis factor (TNF) inhibitor. such as etaner
u'hich can all be associated."r ith Sl.E, but this patient has no cept (Option A), is recommended as second line disease-
cLlrrent cardiopuintonary s!mptonls or exanlination abn<tr modi$ring therapy. The eflicacy of TNF inhibitors in patients
malities. CT with contrast poses a risk fbr contrxst-induced with active AS has been demonstrated in numerous ran-
nephropathf in this patient \\'ith an increased serum creat domized controlled trials, which showed improvements in
inine level. clinical. radiographic, and MRI outcomes. Patients in whom
S[-E is associatecl n,ith pcricarditis. pericarclill or pleu a TNF inhibitor fails should receive an interleukin 17 inhib-
ral eflusions. and other nranifestations ofserositis. Echocar itor, such as secukinumab or ixekizumab.
diography (Option C) can be useful to assess fbr the presencc Nonbiologic agents, such as methotrexate (Option B) or
and severity of pericarditis lnd effusions. as uell as the sulfasalazine (Option D), do not play a role in axial disease
UI possibility of valvular clisease. In a patient u'ith a norntal but could be used as additional therapy as needed for signif
€ cardiopulrnonlrry examination ancl no cardiac s!'mptoms. icant peripheral joint disease. Sulfasalazine is the preferred
o
Ut echocardiographf is less uselul fbr guiding management agent. This patient has no evidence of peripheral joint dis-
o, than is a kidney biopsy. ease. and these nonbiologic agents are not indicated.
=
EL
n t(tY P0tilTt XEY POIIITI
=. o Recognized indications for kidney biopsy in patients . Physical therapy and NSAIDs are the recommended
=.
,EI
tr with systemic lupus ery.thematosus include glomeru- first-line therapies for ankylosing spondylitis.
(D
ta lar hematuria and/or cellular casts, spot urine . If NSAIDs fail or the patient cannot tolerate NSAIDs, a
protein-creatinine ratio greater than 500 mg/g, or tumor necrosis factor inhibitor is second line therapy
unexplained decrease in estimated glomerular for ankylosing spondylitis.
filtration rate.
. In patients with lupus nephritis, kidney biopsy results Bibliography
are important for choosing induction therapy and Ward MM. Deodhar A, Gensler I-S, et al. 2019 Update of the American
College of RheumatoloEl//Spondylitis Association ol Americar
determining prognosis. Spondyloarthritis Research and Treatment Net$ork Recommendations
for the Treatnlent ofAnkylosing Spondylitis and Nonradiographic Axial
Spondyloarthritis. Arthritis Rheumatol. 2019:71:1599 1613. IPMID:
Bibliography 314360361
Fanouriakis A. Kostopoulou M. Cheema K, et al. 2019 Update ofthe Joint
Eur()pean League Against Rheumatism and European Renal Association
European Dialysis and Transplant Association (EULAR/ERA ED'IA) rec
ommendations for the management of lupus nephritis. Ann Rheum Dis. Item 79 Answer: C
2020:79:713 723. [PMID: 322208341
Educational Objective: Identi$ probable lymphoma in
a patient with Sjdgren gmdrome.

Item 78 Answer: C Lymphoma (Option C) is most likely responsible for this


patient's recent symptoms. Patients with Sjogren syndrome
Educational Objective: Treat an$losing spondylitis
are at high risk for lymphoma (5'7, lifetime risk); 60'2, of
with NSAIDs.
these cases are mucosa associated lymphoid tissue lym
The most appropriate treatment is naproxen (Option C). This phomas. Risk factors include parotid gland enlargement.
patient has classic ankylosing spondylitis (AS), as suggested depressed C4 complement level, elevated rheumatoid fac-
by inflammatory Iow back pain with signs of sacroiliac joint tor level, elevated anti Ro/SSA and/or anti La/SSB antibody
irritation, limitation of lumbar spine motion, and changes of levels, monoclonal gammopathy, and cryoglobulinemic vas
sacroiliitis on radiograph. NSAIDs are recommended flrst- culitis. ln addition to having several of these risk factors,
line therapy for AS. In general, an NSAID must be dosed this patient has systemic "B symptoms" associated with
toward the higher end of the dosing range to modif,z new lymphoma: fever, weight loss, and drenching night sweats.
bone formation and have a significant impact on inflamma Light-chain (AL) amyloidosis (Option A) is a clonal
tion. If an NSAID is started early in the course of AS (<3 years), plasma cell dyscrasia characterized by production of amy
35%, of patients will enter remission compared with only loidogenic )" or r free light chains. These light chains can
12'1, to 15"/,, of those starting it later. Unlike with other rheu- deposit in various organs, resulting in varying clinical pre
matologic diseases, evidence suggests that NSAID use in AS sentations of the disease, most commonly the nephrotic
has a disease modi$zing effect. Patients should be aware of syndrome, restrictive cardiomyopathy, peripheral neuropa
and monitored for NSAID toxicity, such as hypertension, gas thy, and hepatomegaly. AL amyloidosis can be a rare com-
trointestinal bleeding, and kidney disease. Physical therapy plication of IgM associated monoclonal gammopathy and
is the most important nonpharmacologic intervention in AS. can rarely inflltrate the parotid glands. This patient's pre-
The goals of therapy include improving pain and stiffness, sentation, however, is most consistent with classic Sjogren

164
Answers and Critiques

syndrome, with exocrinopathy afl'ecting salivary and lacri presc'nt and are not specific. Otl'rer antibr-rdies. including ANCA
mal glands, laboratory test results consistent with Sjdgren (especially p ANCA) and F-actin, can bc seen with min(,1)'.
syndrome, and recent onset of systemic disease symptoms clinc use. Most patier.rts with dmg-induced lupus sympk)n-Is
suggesting lymphoma. r"iII improve lr,ith discontinuation of the rnedicirtiort e:tusing
Prominent symptoms of' hypothyroidism (Option B) the qrmpton,s, bLlt solre may r.reed synrpkrmatic treatmcnt.
include fatigue, weight gain, myalgia, and arthralgia. Lab Granulonrrtosis w'ith polyangiitis (Option B) is a
oratory iindings may include anemia, elevated LDL choles small vessel vasculitis associated with ANCA that ry'picrlll
terol, and hyponatremia. Hypothyroidism cannot explain causes constitutional symptoms, upper and krwer lint'ay
the patient's weight loss, low C4 complement level, or disease. and glomerulonephritis. this pirtient has an ANCA,
monoclonal gammopathy. In addition, the patient's thyroid which can be seen in a variety of aut<.rimnrune:rncl infectious
stimulating hormone level is normal, excluding hypothy states, but her clinical presentation is not suggestive of an
roidism as a cause of the patient's recent symptoms. AN(.A associ:rted rast'ulitis.
Primary Sjogren syndrome occurs in isolation; second Rheumatoid arthritis (RA) (Option C) cirn also cause la
(l,
ary Sjogren syndrome occurs in the setting of other rheu joint findings in the distribution describecl. lncl lacli of 3
ET
matologic diseases, most commonly rheumatoid arthritis rheunratoicl factor ar.rd anti cy,clic citrullinatcci pcpticle
.=
(Option D) and systemic Iupus erythematosus. Patients with antiboclies does not rule out ItA. This cliagnosis coulcl llso L'
rheumatoid arthritis and Sj0gren syndrome may have lab be considere<l if'synovitis persists aftcr discontinuation ol E'
oratory findings similar to those found in this patient, but mir.rocycline.
E
.E
this patient has Sj0gren syndrome antibodies and negative SLE (Option D) is also in the diflerer.rlirrl
diagnosis. ra
(I,
results fbr anti cyclic citrullinated peptide antibodies. In although it is lcss likely than clrug induced lupus erytl.re
addition, examination does not suggest synovitis (the patient lr:ltoslls in x patient t:rking urirrocl,clir.rc. [t shou]d be c<ln- =
Ut

has arthralgia rather than arthritis). The recent onset of sidered if syn.rptoms fail to inrprove after discontinuation of
weight loss, fbver, and night sweats in the absence of syno- minocycline.
vitis favors the diagnosis ol lymphoma over rheumatoid
XEY POIilT
arthritis.
o DruB induced lupus erythematosus is often milder
XEY POITIT
than idiopathic systemic lupus erythematosus and is
. Patients with Sjogren syndrome are at high risk for characterized by more skin and joint symptoms and a
lymphoma (5% lifetime risk). much lower risk for organ involvement or damage.
o Risk factors for lymphoma in patients with Sj0gren
syndrome include parotid gland enlargement, Bibliography
depressed C4 complement level, elevated rheumatoid Kawktt t.. Mertz tl Chasset F. et al. Charucterization of drug induced cutane
ous lupus: analysis of 199,1 cases using the WHO pharmacovigilance
factor level, elevated anti-Ro/SSA and/or anti-La/SSB database [Letter]. Autoimmun Rev 2021;20:102705. IPMll): 33188917]
antibody levels, monoclonal gammopathy, and cryo
globulinemic vasculitis.
Item 81 Answer: A
Bibliography Educational Obiective: Diagnose fibromyalgia.
Fragkioudaki S, M:rvragani CP, Moutsopoulos llM. Predicting the risk fbr
lymphoma clevelopment in Sjogren syndrome: an easy tool for clinical The most likely diagnosis is fibromyalgia (Option A). This
use. Medicine (Baltinrore). 2016i95 r3766. lPMlDt 273368631 doi:10.1097,
M D.0000000000003766
patient presents with ',r,idespread pain, fatigue, poor sleep,
and cognitive symptoms of 1 year's duration. These symp-
toms are the hallmarks of flbromyalgia, a poorly understood

tr Item
Ed
80
ucationa I Objective
Answer: A
: Diagnose drug-induced lupus
but common chronic pain syndrome affecting 2"/,, to 3"/,, of
the population. Diagnosis is suggested by history and can be
conlirmed with validated criteria (e.g., the 2016 revisions to
erythematosus.
the American College of Rheumatologz Preliminary Diag
The nrost likelv diagnosis is clrug induced lupus erythen-ra nostic Criteria). This illness is not autoimmune or inflamma
tosus (Option A). Several medications, inclucling nrinoc)- tory and extensive laboratory testing is not advised unless
cline, can cause drug induced lupus erythenratosus. This another comorbid disease is suggested. Although some
syndrome is often r.nilcler tl.ran idiopathic systemic lupus patients may have tender points, assessment of tender points
erythematosus (SLE). Patients may hirve skin ar.rd joint on examination is unreliable (particularly in men) and no
symptoms but are at a mucl.r lower risk for organ dam Ionger among the clinical criteria for diagnosis. Fibromyal-
age. Patients u,ith drug induced lupus erl,thematosus oflen gia tends to be associated with other medically unexplained
have transiently positive antinuclear arltibodies but may lack syndromes, including irritable bowel syndrome, and is espe-
typicirl aukrantibodies specific to Sl.E. Classically, paticnts cially common in patients with migraine headaches.
with drug induced lupus erytl.rematosus may have antihis Generalized osteoarthritis (Option B) affects multiple
tone antibodies. although these :rntibodies arc' not always joint groups. On joint examination, crepitus, decreased

165
Answers and Critiques

range of motion. bony enlargement, and sometimes effu program is likely the one to which the patient will adhere.
sion may be present. Ger.reralized osteoarthritis is an unlikely Walking is the most common form o1'aerobic exercise eval-
diagnosis because it would not cause the widespread pain uated in studies, either on a treadmill or as part of a super-
seen in this patient and occurs in older patients. In addition, vised community program. Strengthening exercises with
the examination demonstrates no evidence of osteoarthritis the use of isokinetic weight machines, resistance exercises,
in any joint. and isometric exercise also provide value. Aquatic exercise
Polymyalgia rheumatica (PMR) (Option C) is a clinical often encompasses aspects of aerobic fitness exercises and
diagnosis based on characteristic symptoms in a patient exercises fbr enhancing joint range of motion in a low
older than 50 years and is supported by an elevated eryth impact environment. The ACR/AF guideline notes a dose
rocyte sedimentation rate. PMR is associated with pain and response relationship of weight loss and resultant symptom
stiffness of the shoulder girdle and hip girdle. This patient's and functional improvement in patients with OA who have
age, difiuse pain, and normal erythrocyte sedimentation rate overweight or obesity. Weight loss attempts coupled with an
are not compatible with PMR. exercise program increase the benefit to the patient. Even
UI Rheumatoid arthritis (Option D) is an inflammatory small amounts of weight loss (5'1,) can have a positive elfect

o arthritis that primarily affects small joints. Patients with on pain and function.
ta rheumatoid arthritis present r,r,ith discomfort in discrete The ACR/AF recommends against the use of massage
q,
3 joints. not di{fuse pain. In addition, rheumatoid arthritis therapy (Option B) in the treatment of OA because elficacy
EL does not cause axial pain or myalgia. The musculoskele data are lacking. Massage therapy may have other benefits,
n tal examination in a ptrtient u,ho has a l-year history of' but none that pertain to outcomes speciflc to knee OA have
i. untreated rheumatoid arthritis should reveal multiple yet been demonstrated.
tt
tr swollen joints. The ACR/AF recommends against mobilization/manip
o ulation and passive range of motion (Option C) for knee and/
ta Systemic lupus erythematosus (SLE) (Option E) is an
autoimmune disease that can allect any system in the body, or hip OA. Although manual therapy can be of benefit for
including the skin, blood cells, joints, lung, central nervous certain conditions, such as chronic low back pain, data show
systenl. and kidneys. Although patients with SLE can have little additional benefit over exercise alone lor managing OA
concurrent fibromyalgia, patients with fibromyalgia rarely symptoms.
have SLE. This patient also has none ol the hallmarks ol Studies examining the use of transcutaneous electrical
SLE. such as autoimmune c).topenia, inflammatory arthritis. nerve stimulation (TENS) (Option D) have failed to demon-
rash, kidney disease, or systemic inflammation. strate benefit in patients with knee OA. The ACR/AF guide
line strongly recommends against the use of TENS.
[[Y P0lilrt
. Fibromyalgia is characterized by widespread pain, rEY POIlIIS
fatigue, poor sleep, and cognitive symptoms; diagno- . Exercise is beneficial in hip, knee, and hand osteoar
sis is suggested by history and can be confirmed with thritis; no one exercise program is superior.
a validated questionnaire. o There is a dose-response relationship of weight loss and
. Assessment of tender points on examination is unreliable slmptom and functional improvement in patients with
and no longer among the clinical criteria for diagnosis overweight or obesity and osteoarthritis that begins
of fibromyalgia. with 5'7, loss of bodyweight.

Bibliography Bibliography
Wolfe F, Clauu,l)J. Fitzcharlcs MA. et aI.2016 Revisions to the 2010/2011 Kohsinski Sl.. Neogi li llochberg MC, et al.2019 American College of
fibnrmyalgia dixgnostic criteria. Semin Arthritis Rheum. 2ol6r.t6::ll9 Rheumirkrlol5,', Arthritis Foundation guideline for the mrnagemrnt of
329. IP1\{lD:')79l6278ldoi:10.1016 j.scmarthrit.20l6.08.0l2 osteoarthritis ofthe hand. hip. and knee. Arthritis Care Res (Hoboken).
2O2OJ2:149 162. [PI\'Ill): 31908149]

Item 82 Answer: A
Ed u catio na I O : Treat osteoarthritis
bjective with exercise Item 83 Answer: D
and weight loss. Educational Objective: Evaluate rheumatoid arthritis
with hand radiography.
The most appropriate additional treatment for this patient
is exercise and weight toss (Option A). The 2019 Ameri The most appropriate diagnostic test to perfbrm next is
can College of Rheumatolory (ACR)/Arthritis Foundation plain radiography of the hands (Option D). No single
(AF) guideline strongly recommends exercise for all patients radiographic feature is diagnostic of rheumatoid arthritis.
with osteoarthritis (OA). The most robust evidence supports However, f'eatures such as periarticular osteopenia, joint
exercise in the treatment of knee and hip OA: less evi space narrowing, and bony erosions are characteristic of
dence is available for hand OA. Many helpful exercises are established disease and may help solidify the diagnosis of
available for knee and hip OA, and the evidence indicates rheumatoid arthritis when physical examination findings
that no one exercise program is superior. However. the best :rre equivocal. Ifpresent, they can also underscore the need

155
Answers and Critiques

for early treatment with disease modifying agents. which variants: annular and polycyclic photosensitive plaques
can include biologics, because the presence ofradiographic on the back, chest, and extremities, or psoriasiform scaly
damage at the time of diagnosis of rheumatoid arthritis plaques in a similar distribution. SCLE does not scar but
predicts a more severe course of disease. Thus, plain radi may resolve with temporary pigmentary changes. Fewer
ography may aid in both diagnostic and therapeutic deci than one in four patients with SCLE has systemic lupus ery-
sion making as well as help establish a baseline to assist in thematosus (SLE), and as many as one in three SCLE cases
monitoring disease activity. are drug induced. The latter may resolve with withdrawal of
ANCAs (Option A) are directed against antigens fbund the causative agent. Up to 70'7, of patients with SCLE have an
i
in the cytoplasmic granules of neutrophils and monocytes. elevated antinuclear antibody titer, with anti RoTSSA and
ANCA testing is usually performed to help diagnose or anti La/SSB being overrepresented.
exclude granulomatosis with polyangiitis and microscopic The malar "butterfly" rash is characteristic of acute
polyangiitis. However, polyarticular, symmetric inflamma cutaneous lupus erythematosus (ACLE) (Option A); holt'
tory arthritis of the small joints of the hands as seen in ever, ACLE can also manifest as a maculopapular variant t,l
G,
this patient is not a feature of ANCA positive vasculitis. that is photosensitive and distributed over the chest, upper
C'
The choice of which autoantibody tests to order should be back, and arms. Both rashes have a range ol presentation
guided by an accurate history and careful physical exanrina from pink-to-violet macules to scaly papules and plaques. tr,
tion. ln this case, ANCA testing is not the most appropriate Essentially all patients with ACI-E have SLE and as such ter.rd rE
:
diagnostic test. to have systemic symptoms and positive serologic results. .g
,
Anti double stranded DNA antibody testing (Option The rash ofACLE does not scar.
B) is not indicated. Antinuclear antibody (ANA) specificiff Cutaneous leukocytoclastic vasculitis (Option B)
o
testing (i.e., testing fcrr antibodies to specific nuclear com results from inflammation of small blood vessels and can ta
=
j
ponents, such as DNA or centromeres) should be reserved be seen in isolation or in association with a variety of
for patients with a positive ANA test result and a clinical systemic diseases, including autoimmune disorders (such
syndrome suggesting an underlying rheumatologic disease. as SI-E, rheumatoid arthritis, inflammatory bowel disease,
An anti double stranded DNA antibody test should not be and systemic vasculitides), but also infection, hypersen
perfbrmed in the absence of a positive ANA result. sitivity to medications, and malignancy. Rashes consist of
MRI (Option C) can be a valuable tool in the evaluation red or purple discrete skin lesions, are more common on
of specific concerns in patients with musculoskeietal dis the lor.r,er extremities than on the trunk or fbrearms. and
ease, but it lacks speciflcity fbr the diagnosis of rheumatoid rarely occur on the face.
arthritis. The first step in radiologic evaluatior.r of a patient Chronic cutaneous lupus erythematosus (CCLE)
with joint symptoms is usually plain radiography because defines a category of related skin diseases of which dis
it is relatively inexpensive, is readily available, and per, coid lupus erythematosus (DI-E) (Option C) is the most
mits assessment for findings characteristic of rheumatoid common. DLE lesions start as red to violet plaques that
arthritis. develop a hyperpigmented border and thick adherent scale
in the center. The centers of the lesions eventually become
r(EY POt t{IS
atrophic and depigmer.rted, causing permanent alopecia if
. No single radiographic feature is diagnostic of rheu- occurring in hair bearing areas. Plaques favor the head and
matoid arthritis, but features such as periarticular neck, particularly the scalp and the hollow ot the auricle
osteopenia, joint-space narrowing, and bony erosions of the ear. CCLE in general has a low association with Sl.E,
are characteristic of established disease. and t)LE poses approximately a 10'X, risk for underlying
. Radiographic features of rheumatoid arthritis can systemic disease.
underscore the need for early treatment with disease I(EY POIilTS
modi8zing agents because radiographic damage at
o Subacute cutaneous lupus erythematosus is a photo
the time of diagnosis predicts a more severe disease
sensitive rash occurring especially on the arms, neck,
course.
and upper trunk, usually sparing the central face and
consisting of erythematous annular/polycyclic or
Bibliography
patchy papulosquamous lesions.
Aletaha D, Smolen JS. Diagnosis ancl management of rheul.uirtoid artltritis:
a review. JAMA.2018 Oct 2:320(13):1360 1372. IPMID:3028.51831 cloi: 10. . Fewer than 25'l. of patients with subacute cutaneous
1001 iama.20lB.l3103
Iupus erythematosus have systemic lupus
ery.thematosus.
Item 84 Answer: D
Ed ucationa I Objective: Diagnose subacute cutaneous
Bibliography
lupus erythematosus.
Alniemi l)T. Gutierrez A Jr. l)ruge I-A. el xl. Subrcule cutilncous lupus ery
themrtosus: clinic:rl cluracteristics. clisease associatior.ts. treiltnlents.
The most likely diagnosis is subacute cutaneous lupus ery- and outcomes in a series ol'90 patients at Meyo Clinic. 1996 2011. l\,hyo
thematosus (SCLE) (Option D). SCLE appears as one of two CIin l)roc. 20l7;92:,106 41.1. IPl\4lD: 2U1856561

167
Answers and Critiques

tr Item 85 Answer: A
Educational Objective: Diagnose giant cell arteritis.
Item 86 Answer: A
Educational Objective: Diagnose the cause of acute
tx
monoarthritis by using arthrocentesis.
The rnost likely diagnosis is giant cell arteritis (GCA) (Option
A). 'lhis patient h:rs polymyalgia rl.reumatica. and 10'/, to The most appropriate diagnostic test to perfbrm next is
20'll, of patients n,ith polyntyalgia rheumatica have con aspiration of the left knee (Option A). This patient has
collritilnt CCA at presentatior.r or dcvelop GCA later in the acute inf lammatory arthritis of the knee characlerized by
disc:rse course. Ileadachc is present in mure than 607, o1' pain. swelling, and redness for less than 1 rveek. Physical
paticnts r,r'ith GCA and may be frontal, occipital, unilateral. examination shows swelling r,rritl-r warmtl-r ancl redness.
or gcneralized; it nra1,' be constant, intermittent, or waxing as well as tenderness with palprrtion and limited range
and u'aning. Ihe irnpofiant point is that the headache is of' motion. The clifl'erential diagnosis ol acute monoar
ner,ri A frecluent nor.rheadache manif'estation of GCA is jaw thritis includes crystal induced disease (gout or calciunr
D clauclication, or,r,ing to decreased blood flow to the muscles pl,rophosphate deposition), infection. and hemarthrosis
ut of mirstication. Jar,r'claudication is ttrund in nearly 50?; of (bleeding into the joint). Joint aspiration is necessary' to
E patients with GCA, and this symptom should always be make an accurate diagnosis and distinguish amor.rg these
.D
UI explored on revie\.t, ol svstems in a patient with polymyal causes. Synovial fluid should be analyzecl for cell count
o, gia rheun.ratica. Other key questions to ask such patients ar.rd diflerential. crystal examination, Cram stain, and
=L
C include whcther thcy havc recent changes in vision (e.g., culture.
n short lir,ecl episodes of amaurosis firgax. diplopia. or blur Erythrocyte sedimer-rtation rate (Option B) is a blood
=.
lr3.
g
riness) ancl scalp tenderness (e.g., r,r,ith hair brushing). A
blood C-reactive protein level of l2.B mg/dl (128 mgil)
test that indicates systemic inflammation. It may be elevatecl
in any inflammatory process. inclucling crystal induced dis
(D
Ut r'rru-rld be unusual lbr polvml,algia rheunratica alor.re; this ease and intectiorr. but docs r.rot distinguish betu,een the t$o 1
degree of systemic inflammation is r.nore in keeping with a diagnoses.
vasculitis. such as (lCA. Radiography ol the knee (Option C) rnay be a useful
Granulomatosis r,r,ith polyangiitis (GPA) (Option B) is test but is not adequate to distinguish the cause of acute
the n-rost common ANCA associatecl vasculitis. Typical age monoarthritis unless there is a history of traunra and frac
at onset is bctu,een .15 ancl 60 !'ears. GPA afl'ects the upper ture lvith hemarthrosis is suspected. This patient has no
and lorter ainr,ays. kidneys, eyes. and ears. At least 50u/. of hislory ol trauma.
paticnts havc constitutional symptoms. GPA can cause visual Serum urate (Option D) is not l-relpful for deterntining
chxnges bllt is not t1'picall1.'associated \vith ncrn onset head the causc of this patient's acute ntonoarthritis. Although
ache or jau, claudication. an elevated level might increase clinical suspicion of goul
Like (lPA. microscopic polyangiitis (MtA) (Option C) as the cause, only knee aspiration with fluid analysis fbr
is a small vessel r,asculitis that characteristically aflects thc cell count and crystals can determine the cause of the knee
lungs anci kidneys, along with other organ systems. Absence intlammation. The presence of soclium urate crystals in thc
of granulonras distinguishes N,IPA iiom GPA. MPA cannot synovial fluid polymorphonuclear cells is diagnostic for
account ibr tl-tis patient's syrnptoms. gout. Gout and infection may coexist. so aspirati<tn wottld
Polyrrteritis r.rodosa (Option D) is a medium be indicatcd even if the serum urate lcvcl was elevated.
l,essel lasculitis tlrat can af fect any organ in the body but Finall1,, evcn in patients with acute gout. scrum urate level
most comnronly involves the renal vasculirture (resulting rnny be normal at the tirne of the flare. Thus, a norntirl
irr renal insufficicncy and hypertcnsion), skin (produc serunl urate level would not rule out gout as the cause ol
ing nodular skin lesions). mesenteric vessels (causing acute monoarthritis.
abdominal pain, including postprandial abdominal pain),
TEY POTITI
ancl peripheral ncrves (typically presentir.rg as mononeu
ritis multiplex). The patient does not have this symp o Joint aspiration is necessary to make an accurate
toln compler, making polyarteritis nodosa an unlikely diagnosis and distinguish among causes of acute
dirrgr-rosis. monoarthritis, including crystal-induced disease
(gout or calcium pyrophosphate deposition), infec-
KEY POIilTS
tion, and hemarthrosis.
o Concomitant giant cell arteritis develops in 10'2, to . ln patients with an acute inflammatory arthritis,
20% of patients with polymyalgia rheumatica.
synovial fluid should be ana\zed for cell count and
o Common symptoms of giant cell arteritis include differential, crystal examination, Gram stain, and
headache, jaw claudication, and changes in vision. culture.

Bibliography
Maz M, Chung SA, Abril A, et al. 2021 American College of Rheumatologr/ Bibliography
Vrsculitis l.bundation guideline fbr the nlanagement ofgiant cell arte Singh N. Vrgelgesang SA. Monoarticuhr arthritis. Med (llin North
ritis and Takayasu irrteritis. Arthritis RheumrtOl. 2021. fPMID: Am. 20l7: lOl :607 6 13. IPMI I): 283727 16l doi:lo. I Ol 6,'i.mcnir.2016. 12.
3123s8841 00,1

168
t Answers and
I
9'r!qv.::
I
i

t
i
\ tr Item 87 Answer: D
Educational Objective: Evaluate new monoarticular pain
Bibliography
Ross JJ. Septic arthritis of native joints. lnfect Dis Clin North Am.
31:203,218. IpMID, ZSgOOZ:2t] doi:10.1016/j.idc.2017.01.001
2017;

I
in a patient with rheumatoid arthritis.
The nrost appropridte diagnostic test to perfbrm next
:

\ is synovial fluid analysis (Option D). 'lhe abrupt onset of


Item 88 Answer: D
acllte nronoarticular inllamnratory rrthritis in any patient Educational Objective: Treat systemic lupus
shor-rld suggest a joint inf'ection. llris patient has a history e4rthematosus in a patient planning pregnancy.
o1' krng standing rheumatoid arthritis that has damaged
The most appropriate preconception management is to
nurrerous joints. Previiius joint clamage incre:rses the risk
discontinue mycophenolate mofetil and add azathioprine
tbr joint inf'ection. 'lhe patient irlso reccives multiple inrnru- (Option D). In some very stable patients with systemic
nosupirressive irgents. u{rich also increase risk fbr joint
: lupus erythematosus (SLE), mycophenolate mofetil could
intbction. tier joint ex:rmination, apart from the left knee, ra
be discontinued with maintenance on hydroxychloroquine (l,
suggests quiescent rhcumatoicl arthritis rather than a gcn
alone. SLE is associated with increased risk for miscarriage, CT
eralizecl flirre of disease. turther raising suspicion lor an
:

premature delivery and other pregnancy complications.


inlected joint. In patients suspectecl of having infections
;
Patients with SLE who desire pregnancy should be advised rr,
arthritis. the most important cliagnostic proceclure is crpe E
to wait until disease has been well controlled for at least
ditious afihrocentesis to obtiiin joint lluid :rncl conduct syncr .g
6 months to reduce these risks. Certain therapies, such
vi:rl fluid analysis (Cram stain. culture lbacterial and. when tl
: as mycophenolate mofetil, belimumab, and methotrex o
indicated, mvcobacterial and,or fungall. leukocyte count.
ate, should be avoided in patients considering pregnancy
and crystal analysis). t
=
:
because they are teratogenic. Cyclophosphamide is associ
(l reactive protein (CRP) (Option A) is ar.r acute phase
ated with age and dose-dependent infertility and should
reactant. Although elevation oI CRP level can signal ar.r irrfec
not be used in patients contemplating pregnancy without
tion, it is a nonspecific indicator. Indeed. an elevated CRP
a compelling life or organ threatening indication. This
level can be seen in a Ilare of rheumatoicl arthritis. as well as
patient has well controlled SLE but is taking mycopheno
in other conditions. and may not help distinguish betr,r,een
late mofetil, which should be discontinued 3 months before
iniL'ction and a rheunratoid arthritis Ilare.
conception is attempted. Routine use of azathioprine in
Measurenrent of serum procalcitonin levels (Option B)
pregnancy is not advised; however, this drug is safer than
has lreen proposed to help clistinguish ini'ectious arthritis
other agents and can be used if necessary. In this patient
tiom inflan.rmatory arthritis. Ilowever, serum procalcitonin
with a history of Iupus nephritis, azathioprine can reduce
is 11ot sensitivc lor the presencc of joint infectir>n. and ir neg
the risk for flare or disease progression. Hydroxychloro
atir,e result cloes not elirr.rinate infectioLrs arthritis.
quine can be safely used during pregnancy in most patients
ln patients suspccted ot having lr joint inf'ection. a
to reduce the risk for SLE flare or other complications and
sy'no','ial biopsy (Option C) is rarell, necessaryi Syr.ulvial
should be continued.
biopsies may be helpfll in patients in r,r,hont concurrent
Patients should be advised about the risks of preg-
contiguous osteomyelitis or tubercukrsis (or infection with
nancy in the setting of an SLE diagnosis (Option A), but
arrother 5low grou'ing organism, such as a fungus) is a
given this patient's excellent disease control, a careful
diagnostic possibility. 'lhe paticrrt's shrtrt durati<ln olsymp
decision to proceed with attempts to conceive would be
toms nrakes concurrent ostcomyelitis or ir.rf'ection witl-r
reasonable.
slorv gror,.",ing organisms unlikell,. Although tulterculosis
This patient's current medications (Option B) must be
or fLngal infection callnot be absolutely excluded irr this
changed before conception given the risk for teratogenicity
patient receiving intr.nunosuppressallt clrugs. the first cliag,
associated with mycophenolate mofetil. Azathioprine is the
nostic test remains a synovial fluid analysis. A syr.rovial
best option to control the patient's lupus nephritis and may
biopsy may be consiclered in patients without an cstab
be considered following a frank discussion with the patient
lishecl diagnosis after joint f luicl analysis or those in rvhom
regarding risks and beneflts and shared decision making.
antibiotic therapy has failed.
Patients who have SLE flares during pregnancy can be
treated with prednisone or other glucocorticoids. If the risk
rEY POIilIS
for flare is considered high, empiric addition of low-dose
. The abrupt onset of acute monoarticular inflamma- prednisone (Option C) could be considered, but this patient
tory arthritis in any patient should suggest a joint currently has no indication for a glucocorticoid.
infection.
. In patients suspected of having infectious arthritis, ftY Pottrs
the most important diagnostic procedure is expedi- . Systemic lupus erythematosus is associated with
tious arthrocentesis to obtain joint fluid and conduct increased risk for miscarriage, premature delivery
synovial fluid analysis (Gram stain, culture, leukocyte and other pregnancy complications.
count, and crystal analysis). (Continued)

169
Answers and Critiques

l(E? P0IXIS (onfinucdl 'lhe ACR conditionally recornmends against su,itching


o Certain therapies used to treat systemic lupus ery-
statill agents. such as aton,astirtilt. to fenoflbrate (Option
C). I,'errofibrute has urate lowering ellects. but the ACR has
thematosus, including mycophenolate mofetil, beli,
deterrninecl that the risks (inclucling aclverse eff'ects of the
mumab, and methotrexate, should be avoided in
tnedication) outweigh potcntial benefits of sr,r,ilching among
patients contemplating pregnancy becrruse they are
patients u'ith gout. regarclless of clisease activitl:
teratogenic; cyclophosphamide is associated with
age- and dose-dependent infertility and should not XEY POIXIS
be used in patients contemplating pregnancy . For patients with gout, the American College of
without compelling life- or organ threatening Rheumatologz conditionally recommends switching
indications. hydrochlorothiazide to an alternate antihypertensive
agent when feasible.
D Bibliography . For patientswith gout, the American College of
UI Lazzaroni MG, Dall'Ara F, Fredi M, et al. A comprehensive revieu, ot the
clinical approach to pregnancy and systemic lupus erythematosus. J Rheumatologr conditionally recommends switching
E
(D Autoimmun. 2016;74:106 117. [PMID: 273774*l hydrochlorothiazide to losartan as an antihypertensive
UI
o, agent when feasible.
CL
n
4t
tr Item 89 Answer: D
Educational Objective: Treat gout by switching
Bibliography
Fitzcerald JD, Dalbeth N, Mikuls T. et al. 2020 American College of
Rheumatolos/ guideline for the management of gout. Arthritis C:lre Res
hydrochlorothiazide to losartan. (Hoboken). 2020;72:744 760. IPMID: 323919341 doi:10.1002r'acr.2,ll80
(D
Ut
lhe most appropriate treatment is to stop hl.drocl.rlorothia
zide ancl start losartan (Option D). 'lhis patient has had a first Item 9O Answer: D
episode of poclagra (gout of'tl.re gre:rt toe) treated efl'ectir.ely,
Educational Objective: Diagnose rheumatoid arthritis
with colchicine. He has a nrininrally elevated senul urate
with appropriate imaging studies.
level. Flvdrochlorothiazide lr.rd clther loop diuretics cluse
hyperuricen.ria and miry have contributed to this gout epi The most appropriate next step in diagnosis is no lurther
sode. l-osartan is an angiotensin receptor blocker that lray imaging (Option D). The history physical examination,
l.tltre sorne uricosuric effect in adciition to lorvering bkrod and laboratory studies are all consistent with a diagnosis of
pressure. For patients with gout. the Arnerican College of rheumatoid arthritis. Plain radiography of the hands and/or
Rheun.rirblory (ACR) concli t ional h, reconr mends sr,r,itch i ng feet is the standard imaging study for rheumatoid arthritis
hydroch krroth i azide to an irlt ernate anti hyperterlsive r,r,hen and can aid in diagnosis. Radiography ofthe hands and/or
feasibte.'lire ACR also conditionalh' recornmends sr,vitchir.rg feet may also detect changes suggesting other diseases that
to losartan prefbrentially as an antihypertensi\€ agent rvhen may mimic rheumatoid arthritis, such as psoriatic arthri
t'easibtc. Although this patient has had rlnly a single flure. tis, spondyloarthritis, gout, or chondrocalcinosis. Although
nrodifying his iatrogenic risk factors for gout is reasorrable early radiographs may be normal, this patient has the typi
at this time. cal radiographic changes of rheumatoid arthritis, including
Alkrpurinol (Option A) is a urate kxrering lgent. but periarticular osteopenia. marginal erosions, and joint-space
this therapy is r.rot inclicated after rr single gout episode in a narrowing. These radiographic flndings add to the diagnostic
pltienl rvithont tophi or radiographic damage. Urate lor,rerir.rg certainty that the patient has rheumatoid arthritis and are
therapy is strorrgly recomrnended by the ACR fbr patients helpful not only in establishing the diagnosis but in moni
with one or more subcutaneous topl-ri: evidence <tf raclio toring the course ofthe disease.
graphic damage (an1- ntodality) attributable to gouti or fie Bone scanning (Option A) has no diagnostic value in
quenl gout llares. with "fiecluent" being definecl ils two or rheumatoid arthritis and is not used in evaluation of this
rnore iurnually. Urate lou,ering therripl,' is conclitionallt, rec disease. No speciflc flndings on bone scanning would point
ommencied filr patients erperiencing their first flare when to the diagnosis of rheumatoid arthritis.
moderate to severe chronic kidnel, disease (stage 213). serr_rm In the setting of characteristic features of rheuma-
urate concentralion greater than 9.0 mgidl_ (0.511 nrrnoliL), toid arthritis seen on plain radiographs, MRI of the hands
or urolithiasis is present. (Option B) has no additional diagnostic value. Although MRI
l.irr patients with gor-rt. the A(.R conditionirlly recolll- can show flndings that cannot be visualized on plain radio-
nrends against aclding vitamin C sr-rpplententation (Option graphs, such as bone edema and inflammatory synovitis.
B) to decrease serulr urate levels. The ACR conclr.rdecl tl.rat these are not diagnostic of rheumatoid arthritis and are not
clrrta on vitanlin C wcre insufficierrt to sLlpport continued needed to confirm the diagnosis.
recornrnendation of its usc in paticnts rt'itlt gout. Tu'o stud Musculoskeletal ultrasonography (Option C) has
ies showed clinically insignificant changes in senlm urlte gained popularity in the diagnosis and treatment of rheu-
concentrations fbr patients rvith gout taking vitantin C sup matoid arthritis because it is noninvasive and may demon
plementation. strate a variety ofsoft-tissue flndings not visualized on plain

170
I Answers and Critiques
\
t radiographs. However, similar to MRI, when diagnostic fea supporting evidence is otten not s<lught because ot t'ost ancl
I
tures of rheumatoid arthritis are seen on plain radiographs, lack o{'efll'ct on therapy or prognosis' in this patient with
I lrthralgia, cytopcrtirt, ar.rtl llossible kidne-v" tlisease. SI-E is
musculoskeletal ultrasonography is not needed to confirm
\
I
the diagnosis. high rin the list of clilgnostic possibilities. irnd a positive SLll
cliirgnosis r,rould exclude idiopath ic pcricarditis.
\
I
XEY POI l{TS
o Plain radiography I(EY POIIIT
of the hands and/or feet is a stan
\ dard imaging study for rheumatoid arthritis and can o Pericarditis is a relatively common manifestation of
\
I
detect typical radiographic changes, including periar systemic lupus erythematosus that can be asympto
ticular osteopenia, marginal erosions, and joint-space matic (most commonly) or can cause chest pain and
r myopericarditis.
narrowing.
L . Hand and/or feet radiography can detect changes sug
Bibliography UI
gesting other diseases that may mimic rheumatoid (l,
t Dein E. DouglasI I. Petri l\,1. et al. Pericarditis in lupus. Cureus. 2019:11:e4166 3
arthritis, such as psoriatic arthritis, spondyloarthritis, IPMID: 310867511 ET
:
I gout. or chondrocalcinosis.
(J
\ Item 92 Answer: C t,
Bibliography
.9
Shiraishi M, Irukuda T. Igarashi T. et al. Differentiating rheumrtoid and Ed u cati o na I O bj ective : Diagnose rclapsing polychondritis. UI
: psoriatic arthritis of the hand: multimodality imaging characteristics. (l,
I
Rxdiographics. 2020 Sep Oct:,10:1339 1354. [PMID: i27351741
The most likely diagnosis is relapsing polychondritis (Option B
vl
\
C). This patient has a chronic inflammatory disease involv c
ing the cartilage (but not noncartilaginous structures) ofthe
tr Item 91 Answer: D
Edu cational Objective: Diagnose systemic lupus
ears, nasal bridge (saddle nose deformity), joints, and tra
chea. as well as involvement of the connective tissue of the
i erythematosus as the cause ofacute pericarditis.
eye. Hearing loss is likely related to inflammation of the car
'lhe mosl lil<ely cause ol 1his patient's pcricrrditis is systemic tilage of the middle ear. Because relapsing polychondritis is
i
lupus cn,thcrnatosus (Sl.fl) (Option D). 'lhe nranv possible a clinical diagnosis for which no speciflc tests exist (although
I causcs of' acute pericarditis includc idiopathic pericarditis, certain autoantibodies, including those to lype 2 collagen,
irrlt'ction. nredications. neolrlasm. injury fiom trrluma or may be supportive), other causes should be considered and
ischcnria. rncl irutoirnrnune irnd inf lanrmatory diseases. Fac ruled out as appropriate. Management depends on severit5z:
tors that suggest SLli as the cituse ol't his patit:nt's pericarditis NSAIDs and dapsone for mild symptoms and high dose glu
inclucle joint s,,velling, eviclence of' lrossiblc l<idney cliseasc, cocorticoids for initial management of acute and/or severe
ancl cytopcnias. particularly thrornbocykrpeniir. All of these involvement. Oral immunosuppressants (e.g., methotrexate)
arc scen in Sl.Fl. Pericarditis is a relatively colrmon manif'es may be added. The tumor necrosis factor inhibitors inflix-
tation ol SLli ancl can be asynrptorllatic (rnosl conrmonly) or imab and adalimumab and the anti-interleukin 6 agent
can ci"luse chcst pain rnd myollericarditis; tlrnponacle is rare. tocilizumab may also be effective.
Aclnlt onset Still disease (Option A) is another rheu Like relapsing polychondritis, cryoglobulinemia
mrrtologic cliagnosis that can present u,ith Icver. synovitis. (Option A) can affect the ears and nose. However, cryoglob
irncl serclsitis. including acute pericarclitis. Ilowever, patients ulinemic involvement of these tissues is typically ischemic/
lrrith aclult onset Still disease gencrally havc high platelet necrotic rather than inflammatory and usually affects the
coLlnts. suggcsting a generalized inllanrmatorv statc. Many tip rather than the bridge of the nose. Cryoglobulinemia
paticnts irlso have a rash thirt occurs ltrith ltvcrs. Thcse fea can affect the lungs but not the trachea. Moreover, most
trlres are nol present in tl-ris patier-rt. cryoglobulinemias (excepting type I cryoglobulinemia) are
(loxsackievirus (Option B) is onc of the nrost common secondary to hepatitis C virus infection or primary autoim
infbctious lgcrrts tlut can cruse acute pericarclitis: as rtith mune disease; testing for hepatitis C virus and autoimmune
milnt'other pathogens, it cirn also cause fevers and musculo antibodies can be considered.
ske le tal p:rin. It woulcl be reasonablc to considcr coxsackievi Like relapsing polychondritis, granulomatosis with
rus irrlection. but he lacks other features suc'h ls exanthem. polyangiitis (GPA) (Option B) can afiect the upper and lower
oropharyngell lesions, ancl nreningitic or encephalitic symp airways, ears, and eyes and can cause a saddle nose defor
toms. \,'iral infbctions can cause ll,mphopc'uia, but <lther mity. It can also cause arthralgia, although the arthralgia is
finclings prescnt in this paticnt. including thrombocytopenia usually migratory. However, GPA involvement of the lungs
ancl rbnornral urinalysis, wonld r.rol bc expected with a viral is typically parenchymal and does not involve the trachea.
intbction. Because 90'7, of patients with GPA are positive for antipro
Although most cases of acute pericarclitis lre icliopatl.ric teinase 3 antibodies, testing can be considered.
(Option C), a search fbr otl.rer causes is appropriate; a cause Rheumatoid arthritis (Option D) can involve the
is tirund in about 16')ti, of patients. lhe cause of isolated ircute knees symmetrically and in severe presentations can
pericarditis in most patients is presunted to be viral, but cause a systemic illness that involves the eyes and glottis

171
Answers and Critiques

(cricoarytenoid joints). However, it does not typically Pantoprazole, a proton pump inhibitor, is useful for
affect the ears or trachea. This patient also lacks the classic gastrointestinal reflux disease. This patient's reflux disease is l
symmetric polyarthritis of the hands, and his knee pain stable: flurther reducing gastric acid production by increas :
severity is not commensurate with severe rheumatoid ing the pantoprazole dosage (Option C) may contribute to
arthritis. bacterial overgrowth and may worsen the problem.
TEY POIf,I Loperamide (Option D), an opioid agonist, will slow
intestinal motility further and potentially worsen SIBO.
. Relapsing polychondritis is a chronic inflammatory
disease involving the cartilage (but not noncartilagi- TEY POII'IS
nous structures) ofthe ears, nasal bridge (saddle nose o Small intestinal bacterial overgrowth affects patients
deformity), joints, and trachea, as well as involvement with systemic sclerosis, causing diarrhea, abdominal
of the connective tissue of the eye. pain, bloating, and nausea.
D r Small intestinal bacterial overgrowth is best treated
tt Bibliography with monthly antibiotics in a rotating fashion to
E
o Borgia F. Ciuffrida R, Guarneri F, et al. Relapsing polychondritis: an updated
review. Biomedicines. 2018r6. IPMID: 3007259t]l
decrease the bacterial load in the small intestine.
Ut
o,
Bibliography
EL
a.l Item 93 Answer: B lblkou,ska Pruszynska B, Gerkowicz A. Szczepanik Kulak P et al. Small
intestinal bacterial overgrowth in systemic sclerosis: a revie\\'of the liI
Educational Objective: Treat small intestinal bacterial erature. Arch Dermatol Res. 2o191311:l 8. IPMID, 30382339] doi:lo.loo7,
a overgrowth in a patient with dilluse cutaneous systemic s00'10:l 018 1874 0
E
(D
Ut sclerosis.

The most appropriate treatment is ciprofloxacin (Option Item 94 Answer: A


B). This patient is experiencing the manifestations of small
intestinal bacterial overgrowth (SIBO), which affects 39'l,
Ed u catio na I : Diagnose ankylosing spondylitis.
Obj ective

of patients with systemic sclerosis. SIBO occurs in both the The most appropriate diagnostic test to perform next is
limited and diffuse cutaneous forms of systemic sclerosis. anteroposterior radiography of the pelvis (Option A). This
The most frequent symptoms include diarrhea, abdominal patient has had classic inflammatory low back pain for
pain, bloating, and nausea. The diarrhea is often explosive several years. Flexion, external rotation, and abduction of
and typically follows a meal. Over time, SIBO can lead to the hips elicit pain in this patient's right hip as well as lor,r'
malabsorption, malnutrition, and weight loss. Jejunal aspi back pain. These flndings, along with involvement of the
rate is considered the gold standard but is often not used in right hip, suggest ankylosing spondylitis (AS). This patient
clinical practice because ofthe requirement for endoscopy was diagnosed with uveitis. Spondyloarthritis accounts
to obtain cultures and the patchy nature of small intesti for approximately 40'l" of cases of acute anterior uveitis
nal overgrowth. Glucose and lactulose breath tests have in the United States, and most of these patients will have
acceptable speciflcity (around B0'/,) but poor sensitivity AS. Acute anterior uveitis has been reported to affect 27'f,'
(30'/,, 40"1,) fbr diagnosing SIBO. Diagnosis requires typi of patients with AS and rarely causes permanent visual
cal symptoms and a confirmatory test, although empiric damage.
antibiotic therapy with monitoring for improvement in An antinuclear antibody test (Option B) can help
symptoms may be reasonable in patients with a high prob evaluate for autoimmune diseases, such as systemic lupus
ability of SIBO, such as this patient. The protocol is to a ery.thematosus, but these diseases rarely cause uveitis. More
take a different antibiotic with activity against bowel flora over, the patient's inflammatory back pain symptoms and
(e.g., ciprofloxacin, doxycycline, amoxicillin/clavulanic right hip restriction of motion are more compatible with
acid, and rifaximin) for the flrst 10 days of the month for a spondyloarthropathy than an autoantibody associated
4 months. Systemic sclerosis affects both smooth muscle rheumatologic disease.
and autonomic nerves and leads to small intestine hypo The prevalence of AS in the HLA B27 positive popula
motility. Hypomotility results in food and liquid stasis, tion is only about 5'2,; positive results for HLA B27 may be
which then fosters bacterial overgrowth. SIBO also causes absent in up to 15'1, of patients with confirmed AS. There
bile acid deconjugation, leading to dietary malabsorption fore, HLA-B27 testing (Option C) cannot independently con
of fats and fat soluble vitamins. A low lat diet may help flrm or exclude a diagnosis of AS. According to Assessment
reduce the intensity ofthe diarrhea. of SpondyloArthritis international Society (ASAS) criteria,
Cholestyramine (Option A) is often prescribed for bile HLA-827 positivity would be helpful if the pelvis radiograph
acid diarrhea following limited ileal resection, after abdom. is normal in a patient with high likelihood of AS. The pres
inal radiation therapy, or for postcholecystectomy diarrhea. ence of back pain plus HLA-B27 positivity plus two other
Patients with SIBO have bile acid insufficiency, and treat features of spondyloarthritis (in this case, uveitis and hip
ment with a bile acid binding resin, such as cholestyramine, arthritis) would classifz this patient's disease as an axial
is likely to worsen this patient's symptoms. spondyloarthritis.

172
Answers and ues

MRI of the sacroiliac joint (Option D) can identify sac also be considered if diagnostic unccrlainty remains. Bkrod
roiliac inflammation even in the absence of radiographic pressure managemellt with labetalol would be inclicatecl lor
changes. However, 25')1, of healthy individuals can have MRI systolic blood pressure of. l60 nrnr Hg or grcater <lr diastolic
changes that meet criteria for sacroiliitis, and up to 47"/,, of bkxrd pressurc of 110 mm I lg or greater.
persons who regularly participate in impact loading athlet lntralenous cyclophospharnide (Option A) r'voulcl be
ics may have similar MRI changes. The extent of involvement a reasonable consiclcratiotl fbr treatmellt o1 a flirre of lupus
and presence of erosions on MRI increase the speciflcity of ncphritis in a nonpregt.tant patient. In screre cases. cyclophos
the result. However, plain radiography remains the corner phamide could be considcred itl pregnancy afler tlle first tri
stone of radiographic diagnosis in spondyloarthritis and nresler bLlt is zrssociated rvith an illcreasecl risk fbr fetal detnise.
'Ihe best treatmcnt fbr preeclampsia is clelivery (Option
has sufficient specificity to confirm a diagnosis. Sacroiliitis
shown by MRI is included in the ASAS classification criteria B) if the baby is adecluately n1atLlre. Al 26 r,t'eeks. this letus is
and would be useful ilthe plain radiograph was normal. In not adequatelv mature lbr delivcry I)elivery is lil<cly ttl lead
this case, checking HLA P27 antigen would be more cost- to little ur no improvemcnt in her lupus nephritis; other UI
6'
effective lor classiflcation as an axial spondyloarthritis if the therapies are rrcedecl.
ET
plain radiograph was normal. Plain radiography is the initial lntravenous magnesium sulfate (Option C) can be used
L.
imaging choice in patients suspected olhaving AS because of to pre.ucnt seizures in preeclanrpsia ancl rrigl.rt be appro (J
its relatively low cost and safety. priate trcatmer.rt if this patient's findings were related to
preeclampsiir rathcr thart SLfi. Ir-r sonle cllses, it rlll' be
t
=l
.E
XEY POIilTt t,l
lppl'opriate to treat lbr both cortditions.
. Spondyloarthritis accounts for BS'X, ofcases ofacute o
XEY POIf,IS B
tt
anterior uveitis in the United States, and most of these
patients will have ankylosing spondylitis. o In pregnant women, differentiating a flare of lupus
o Anteroposterior radiography of the pelvis is the
nephritis from preeclampsia can be difficult; a lupus
initial imaging examination for suspected ankylosing flare may be identified by rising anti-double-stranded
DNA antibody titers and falling complement levels.
spondylitis.
o In a pregnant patient with a life or organ threatening
Bibliography flare of systemic lupus erythematosus, first-line treat-
Mandl P, Nxvarro Compdn V, Terslev L, et alr l.)uropern League Agtinst ment is the addition of glucocorticoid therapy.
Rheumatism (tlULAR). IjULAR recomnrendations lbr the use of imalling
in the diagnosis rnd managenrcnt of spondvkr:rrthritis in clinical prac
tice. Ann Rheum Dis.2015;74:1327 39. IPMID:258371481 Bibliography
Lazzaroni MG. Dall'Ara H Fredi M. et al. A comprehensive review of the
clinicirl approlch to pregnancy and systemic lupus erythematosus.
J Autoinrmun. ')016;74:106 ll7. [PM]l): 273774fi1
\

tr Item 95 Answer:
Educational Objective: Treat
D
a flare of systemic lupus
erythematosus during pregnancy. Item 96 Answer: A
a
Educational Objective: Diagnose hemochromatosis as a
The lnost appropriatc marlagenrent is prednisonc (Option
cause of secondary osteoarthritis.
D). I rngrkgid, to trext a llare o( systernic lupus erythentirto
sus (SI-h-) durirrg pregnanc): An St-E fhre during pregnancy The most appropriate diagnostic test to perform next is HFE
carr be difficult to clistinguish fiorn precchntpsia. (lonrnron gene testing (Option A). Underlying disorders can predis
I'eatures inclucle proteinuria u'ith hl,perterrsion ancl ederna pose patients to the development ofsecondary osteoarthritis
as well ls heaclachcs and othcr nonspecific svntptolrs. (OA) and should be suspected when OA occurs at an early
Thronrbocy'topenia is ir relltively conlnlon feature oISLE but age or involves joints not typical for OA, such as the meta
is also sect.t with preeclampsia, cspecially with progression carpophalangeal (MCP) or shoulder joints. Hereditary hemo
to the HF.l.l.P (llcmoll'sis. Iilerratecl Lir:cr enz_r,mes. and l.on chromatosis can be associated with an arthropathy that
Platelets) syndrome. Clues to an Sl"Fl f lare include rnarkcrs causes symptoms of arthritis, arthralgia, and accompanying
of lupus activitli snch as rising anti ckruble strirnrled I)NA radiologic findings. These patients particularly develop OA in
antil)ody titers and falling serunt contplement levels. 'll.re the hands, where characteristic hook like osteophytes com
serunr urate level is often cler,ated ir-r preeclarlpsiu but not monly in the second and third MCP joints and sometimes
in Sl,E tlares. this patient's clinicirl features suggest a f lare the fourth MCP joint can be seen on radiographs (Figure [top
ol'lupus nephritis. l'reatmcnt oI SLL during pregnancl is of next pagel). Men are more commonly affected. and the
dillicult lrccausc many meclications that could otherwise symptoms usually appear in the lourth to fifth decades. This
be r:sed to trext a flare are colrtraindicrlted in ltrcgnanct. patient has hook shaped osteophytes of the second, third,
including cvclophosphamiclc, mvcophenolate rnolbtil. metl.r- and fourth MCP joints; onset of type 2 diabetes mellitus in
otrexate. and belimunrab. In this casc. first line treatnrcnt the last few years; abnormal aspartate and aminotransferase
would ir.rclude addition of high closc or "pulse" glucocor levels: and elevated serum ferritin and transf'errin saturation
ticoicl therap1,. Concurrent treatnrcnt fbr preeclanrpsia rlrr\' levels, all suggesting hereditary hemochromatosis. The HFE

173
tl:}t9tsgl{_c,t!iqy.:

Hlperparathyroidism can be associated with calcium


pyrophosphate deposition (CPPD) disease and can cause
secondary osteoarthritis. Calcifl cation of articular cartilage
(chondrocalcinosis), most commonly affecting the wrists
and knees. would be seen in a patient with CPPD and hyper
parathyroidism but is absent in this patient. Furthermore,
this patient has a normal serum calcium level, and measur
ing the serum parathyroid hormone level (Option C) is not
indicated.
Inflammatory arthritis, such as rheumatoid arthritis,
can lead to secondary joint damage and OA. This patient,
however. has no history physical examination, or laboratory
findings suggesting rheumatoid arthritis. Measuring rheu-
UI matoid factor (Option D) is not indicated.
E
.D IEY
Ut
o, . 'OIITI
Secondary osteoarthritis (OA) should be suspected
CL when OA occurs at an early age or involves joints not
a.l typical for OA.
gene mutation is diagnostic for this condition, and testing
It should be performed. o Hereditary hemochromatosis can be associated with
.D
Acromegaly due to growth hormone excess is a rare an arthropathy that causes arthritis, arthralgia, and
tr secondary cause of secondary OA. These patients present radiologic findings, such as characteristic hook-like
with enlarged hands, feet, jaw and other bones. Joint symp osteophytes in the second and third metacarpophalangeal
toms may be the presenting feature of the disease, and back joints.
pain with spinal kyphosis or scoliosis is frequently reported.
Testing for insulin-like growth factor 1 (Option B) would be Bibliography
appropriate if there were a high suspicion lor the disorder, Kiely PD. Haemochromatosis arthropathy a conundrum of the Celtic
but this patient has no findings of acromegaly. curse. J R Coll Physicians Edinb. 2018:48:233-238. IPMID:30191911]

174
lndex

Note: Page numbers followed by f and t denote figure and table, respectiveiy. Anti-Scl-70 antibody, 6t
Test questions are indicated by Q. Anti-Smith antibody, 6t, 48t
Anti-SRP antibody,6t
A Antisynthetase syndrcme, 54, 57
Abatacept, 141, 16t Anti TIF 1 Tantibody,6t
ACE inhibitors.64 Anti-U1-ribonucleoprotein, 6t, 48t
Acetaminophen, 10, 161, 29 Anxiety, fibromyalgia and, Q4o
Achilles tendon, 361 38 Aortic valve regurgitation, 36
Acromegaly, OA and, 25t Apremilast, 11t, 12, Q1
Acro-osteolysis, 62, 62f Arthritis. See olso Specific forms of
Acute cutaneous lupus erythematosus (ACLE), 43, Q31 Ieatures ol 1-2
Acute gouty arthritis, 67-68 Arthritis mutilans. 37
Acute monoarthritis, Q86 Arthrocentesis, 68 69, Q86
Adalimumab, l3t. Q36 Asymmetric oligoarthritis, 36
Adenocarcinoma, 63 Atherosclerotic heart disease, 21
Adult onset Still disease (AOSD), 3t, 90, Q39 Atrioventricular block, 36
Aerobic exercise for fibromyalgia, 32 Autoinflammatory diseases, 89-90, 89t
Alcohol, myopathy induced by, 54t Axial spine, sacroiliitis, 37
Aldolase levels, 57 Azathioprine
Alendronate, Q41 for AAV. 84
Alkaptonuria/ochronosis, 25t adverse effects, 49t
Allopurinol, 13-14, 161, 70 71, Q2l, Q49 characteristics of. 11t
Alternati\€ medicine, 17 [br enteropathic arthritis, 42
Amoxrcrllrn_ // /l{ for tgG4 related disease, 92, 92t
Amyopathic dermatomyositis, Q10 during pregnancy, 16t, 50
Anakinra,14t, 16t, Q23 toxicity of, 10-11
Analgesics for rheumatoid diseases, 9-10 use ol 10 11
ANcA-associated vasculitis (AAV), 4t, 83-85, 84t, Q33
Ankylosing spondylitis, 33-34, 39f, 40f B
diagnosis ol Q94 Baricitinib, 111, 16t
enteropathic arthritis and, 37 Barrett esophagus, 63
hip involvement, 36f Basic calcium phosphate (BCP),73
inflammatory bowel disease in, Qll Behqet syndrome, 87 88, 87f
management of, 41, 4lf criteria for, 88t
ocular manifestations of, 3t internal organ involvement in, 4t
radiologic findings, 7t ocular manil'estations of, 3t
renal amyloidosis in, Q72 patherry associated with, 88f
treatment oi Q14, Q78 ulcerations in, Q1
Anterior uveitis, ankylosing spondylitis and, 341, 35 Belimumab, 141, 161, 49t, 50
Antibody tests, 5, 6t Biologic disease-modifying antirheumatic drugs, 12 13,
Anticentromere antibody, 6t 12f,22
Anticoagulants, drug interactions with, 17 Biosimilar drugs, 13
Anti cyclic citrullinated peptide Bor re lia burgdorfer i, 7 3, 7 8t
diagnostic use o{ 19, Q44, Q70 Bouchard nodes, 26
specificity oftests, 5, 6t Bronchiectasis. 21
Anti-double-stranded DNA, 6t, 48t Bronchiolitis. 21, 64
Antihistone. 6t Budd Chiari syndrome, 87
Anti inflammatory agents, 8 9
Anti Jo 1 antibody,6t c
Anti-La/SSB antibody, 6t, 48t, 50, 52 Calcineurin inhibitors, 12, 50
Antimalarials, 54t Calcinosis,60, 62f
Anti-MDA-5 antibody, 6t Calcinosis cutis. 55
Anti Mi 2 antibody,6t Calcium phosphate-associated arthritis, Q35
Antimyeloperoxidase antibody, 6t Calcium pyrophosphate deposition (CPPD), Q13, Q27
Antinuclear antibodies (ANA) management of, 72t
in inclusion body myositis, 57 manifestations of, 71
in mixed connective tissue disease, 66 osteoarthritis secondary to, 25t,27 28
in Sjdgren syndrome, 52-53 physiolory of,71
in SLE, 42-43, 46-48, Q6 radiographic fi ndings, 7t
in systemic sclerosis, 6lt CampAlobacte r jejuni, 38
targets ol 5, 6t, 30-31 c ANCA antibody, 6t
Antiphospholipid antibodies, 45 Canakinumab, 14t,16t
Antiphospholipid syndrome, 3t, 4t, 45 Cancer in dermatomyositis, Q38
Antiproteinase 3 antibody, 6t Capsaicin for osteoarthritis, 29
Antiribosomal P antibody. 6t,48t Captopril for scleroderma renal crisis, Q15
Anti Ro/SSA antibody, Q60 Cardiovascular disease risk, Q19
in pregnancy,50 Carpal tunnel release, 17
in rheumatologic diseases, 6t Cauda equine syndrome, 36
in Sjogren syndrome, 52 Celuroxime axetil. 77 78
in SLE,48t, 49t Certolizumab pegol, 13t
in subacute cutaneous lupus erythematosus, 44 Cevimeline, Q26

175
lndex

Charcot joints, 25t Enthesis.2


Chikungunya virus, 76, Q66 Enthesitis, 35, 37, 87
Childbirth, SLE and, 50 Eosinophilic fasciitis, 61t
Chlamy dia trochomotis, 38, 42 Eosinophilic granulomatosis with polyangiitis (EGPA),
Chondroitin sulfate. 29 85
Chronic graft-versus-host disease, 6lt Epicondyliris, lateral, Q42
Chronic infantile neurologic, cutaneous, articular syndrome (CINCA), 89t Episcleritis, 21
Circinate balanitis, 38, 38f Epstein Barrvirus, 17
Citrulline, lT Erythema nodosum,9lf
Clostridtoides dfficile, 38 Erlthrocyte sedimentation rate (ESR), 2 4, 19
Cocaine, myopathy induced by, 54t Esophageal dysmotility, 63
Cognitive behavioral therapy, 32 Estrogen receptors, 17
Colchicine Etanercept, 13t
for acute gout, Q76 Exercise for osteoarthritis, 29, Q82
adverse effects of, 9 Eyes, rheumatologic disease and. 2, 20 2l
for Behqet syndrome, 88
for familial Mediterranean fever, Q2 f
function of. 9 Facet joints, synovial, 34-35
for gout, 69-70 Familial cold autoinflammatory syndrome (FCAS), 89, 89f
myopathy induced by, 54t Familial Mediterranean fever, 4t, 9, 89. 89t, Q2
in pregnancy, 16t Fatigue. rheumatologic disease and, 2
Complement,5,48 Febuxostat, Q21, Q61
Complementary medicine, 17 contraindications. 1l
Conjunctivitis, 37, 38 description o[ 14
Connective tissue disease,4t, 92 93 urate-lowering therapy with, 16t,70 7l
Constipation, treatment options, 65t Felty syndrome, 21, Q29
Corneal melt.21 Ferritin, serum, Q27
c-reactive protein (cRP),4-5, 19 Fever, rheumatologic disease and, 2
Creatine kinase, 53 Fibrillin 1, fibromyalgia and, 33
CREST syndrome,60 Fibromyalgia, Q81
Cricoarytenoid arthritis, 21 classifi cation of. 33-38
Cryoglobulinemia, 86 diagnosis of,30 31, 311,311, Q81
Cryoglobulinemic vasculitis, 86, Q56 epidemiolos/ ol 30
Cryoglobulins, 6t, 86 genetic factors, 33
crystal arthritis, 721 Q27 management ol 32
Crystal arthropathies, T, 67 -73 milancipran for, 10
Crystal deposition, T pain taxonomy for, 31f
cf,7, 40, s8, 64, 69 SLE and. 44
Cutaneous lupus erythematosus (CLE), Q31 treatment of, Q7, Q40
Cutaneous systemic sclerosis, Q69 Fungal infections, 75 7 6, 7 at
Cyclooxygenase, 9t
Cyclophosphamide G
adverse effects ol 11, 49t Gabapentin, 10,32
for Behqet syndrome, 88 Cabapentinoids, 10
characteristics ol 11, 11t Gastroesophageal reflux disease, 65t
for IlM. 59 Gastrointestinal dysmotiliry 65t
in prcgnancy, 16t Genital ulcerc, 87, 87f
for SSc. 64 Giant cell arteritis (GCA), Q16, Q85
Cyclosporine, 111, 161, 59 cardiac manifestations of. 4t
C)topenia, SLE and, 45 diagnosis of,79-80
epidemiolory ol 79
D management ol 80
1
Dactylitis, 2, 32, 33f , 37f ocular manifestations of. 3t
Dengue virus, 76 pathophysiolog/ of.79
Depression, Q40 Glucocorticoids :
Dermatomyositis, 54, 55, 58 for AAV Q33
amyopathic, Ql0 adverse effects. 8
cancer in, Q38 for enteropathic arthritis, 42 :
cutaneous manifestations of 561 56t immunosuppression by, 8-9
dermatologic manifestations of, 3t intra-articular iniections of, 29 30
treatment of, Q28 myopathy induced by, 54t
Diffuse cutaneous systemic sclerosis (DcSSc), 4t, 611, Q93 osteoporosis induced by, Q4l
Diffuse idiopathic skeletal hyperostosis (DISH), 7t, 25, 26f. Q8 during pregnancy, l6t, 50
Digital pitting, 63f RA management and, 22 - 23 i
Discoid lupus erlthematosus (DLE), 44, 44f for reactive arthritis. 42
Disease-modifuing antirheumatic drugs (DMARDs), 10-13, Q73 for rheumatologic diseases, 8-9
Distal interphalangeal joints (DIP), 36, 37f for SLE. 49
Doxycycline, 77 -78 , Ql2 tapering of, 50
DRESS (drug reaction with eosinophilia and systemic symptoms), Glucocorticoid-sparing agents, Q16
70-71 Glucosamine for osteoarthritis. 29
Drug-induced hypersensitivity syndrome (DIHS), 13 14,49t,70 77 Golimumab. 13t
Drug-induced lupus erythematosus (DILE), 48, 49t Gonococcal arthritis,73, 75t, Q63
Dryeye,20 27,52 Goodpasrure syndrome, 4t
Dry mouth, 20-27, 52, Q26, Q47 Gottron papules, 55, 56f
Duloxetine,10,29, Q40 Cottron sign, 55,56f
Dysbiosis,33 Gout,36,67 71
Dyspnea, SLE and,45 acute flare of, 69 70,69t
chronic recurrent, Q52
E intercritical. 68
Ehlers-Danlos syndrome, 92, 93t management of, 9, 13 15, 69 77
Electromyography, 58 radiologic findings, 7t
Enteropathic arthritis, 34t, 37 -38, 42 refractory Q23

176
lndex

tophaceous, Q21 osteoarthritis and, 25t


treatment ol Q76, Q89 rheumatoid arthritis, 20
Gouty arthritis, Q18
Granulomatosis with polyangiitis, 31,83-8s, Q4, Q33 l(
Granulomatous myocarditis, 21 Kawasaki disease, 4t, 83
Gullwing erosions, 24, 24f Keratitis, ocular manifestations of, 21
Guselkumab, l4t,16t Knees
Lyme arthritis,75f
H monosodium urate deposition, 70f
Hands osteoarthritis of, 27\ Q24, Q34
acro-osteolysis, 62f
osteoarthritis of 24f,26f, Q24 I
rheumatoid arthritis, l8t 19t 20f, Q83 Laboratory studies
systemic sclerosis, 62f in osteoarthritis, 27
Heart, RA and, 21 in rheumatoid arthritis, 19
Heberden nodes, 26 in rheumatologic disease, 2-5
Hemochromatosis, 251, Q96 in spondylitis,38
Henoch-Sch0nlein purpura, 4t, 86 in systemic lupus erJ,thematosus, 46
Hepatitis, SLE and, 46 Large granular lymphoryte leukemia, 21
Hepatitis B virus, 76, 81-82 Iarge granular lymphoclte syn&ome, 21
Hepatitis C virus,76 lateral epicondylitis, Q42
Herbal supplements, 17 Leflunomide, 10, 11t, 161, 50
Hip. anlVlosing spondylitis. 36f Leukopenia, SLE and,45
Histopatholos/ in IIM, 58 Libman Sacks endocarditis, 45
HlV, musculoskeletal disorders in, 76 Lidocaine for osteoarthritis, 29
H[A-B27 testing,37, Qs Limited cutaneous systemic sclerosis (LcSSc),41, 6ft, Q59
HI-A-B'58:01 allele, 14,70, Q49 Linear scleroderma, 61t
Hormones, RA and, U Livedo reticularis, 44, 82f
Hyaluronic acid, 29-30 Losartan, Q89
Hydrochlorothiazide, Q89 Lungs,21,36
Hydrorychloroquine Lupus er,'thmatosus, drug-induced, Q80
adverse effects,49t Lupus nephritis, 12,14t,44, Q46, Q77
characteristics of, llt Lupus pernio,9l, 91f
for nephritis in SLE, Q46 Lyme arthritis, 75, 751 Q12
during pregnency, 16t, 50 Lyme disease, 3t
for SLE, 46,49, Q75 Lymphomas
toxicity,49 RA and, 21
use of 10 in Sjogen syndrome, Q79
Hyperparathyroidism, 25t Lymphopenia,45
Hlpersensitivity vasculitis, 9, 87, Q48
Hlperuricemia, 36, 67, 67t ]t
MAGIC (mouth and genital ulcerations with inflamed cartilage), 88
I Malignanry, SLE and, 46
Idiopathic inflammatory myopathies olM), 53-59, 55t,58f Marfan syndrome, 92
IgA nephropathy,36 Measles, mumps, and rubella vaccine, Q43
IgA vasculitis, 86-87 Mechanic's hands, 3t, 55,551,56f
IgG4 related diseases, 3t, 92, 921, Q68 Meditation, 17
lmmune complex mediated vasculitis, 85-87, 86f Mental health screening, 8
Immune-mediated necrotizing myopathy, 57 Mepolizumab, 14t
Immunosuppression, 15 Metacarpophalangeal joints, I 8l'
Inclusion body myositis, 57, 59, Q45, Q65 Metalloproteinases, joint damage and, 20
Infectious arthritis, 73-78, 73f Methicillin-resistant S. oureus (MRSA), 78t
causes of,74-z Methicillin-sensitive S. oureus (MSSA), 78t
joint aspiration in,7 Methotrexate, l0
management of 77-78 for AAV, 84
pathogens, T8t adverse effects of, 10
risk factors, 74t characteristics of, llt
lnllammation for dermatomyositis, Q28
pain ol 1, 1t for enteropathic arthritis, 42
persistent, l9t for polymyalgia rheumatica, 80
signs of, I in pregnancy, 16t
Infl ammatory arthritis, 25t for psoriatic arthritis, 42
Inflammatory bowel disease, 3t, 37, Q1l for RA, 22, Q73
lnfl ammatory sacroiliitis, Q30 for reactive arthritis, 42
ankylosing spondylitis and, 7t for SLE, 50
radiographic imaging, 33 use of,10
Infliximab, l3t Methylsalicylate, 29
Intercritical gout, 68 Microscopic polyangiitis, 84-85
Interleukin-1 receptor antegonists, 891, Q23 Milnacipran, 10
Interleukin-17 inhibitors, 41, 42 Milwaukee shoulder, 73, Q35
Interstitial lung disease (lLD), 21,54, 64,651, Q17, Q62 Mind-body interactions, 17
Intestinal bacterial overgrowth, Q93 Mixed connective tissue disease (MCTD), 66-67, 66t, Q64
lxekizumab Monoarthritis, 1
characteristics of, l4t Monosodium urate deposition, 67-68
in pregnancy, 16t Morphea,6lt
MRI,7,20,40, s8, Q2s, Q30
J Muckle-Wells syndrome (MWS), 89t
Janus kinase inhibitors, 12 Muscle pain, evaluation ol 53
Joint replacement surgery Q50 Musculoskeletal examination, 1
Joints Mycobacteria, T3
aspiration ol 1,7-8 MAcobacterium marinum, 7 5
inflammation of, 7, 19t Ml.tcobacterium fuberculosis, 75, 78t

177
lndex

Mycophenolate mofetil Peripheral neuropathy, 45


adverse effects, 49t
characteristics ol 11t, 12
Periungual erythema, 55
Physical therapy
\
for IlM, 59 for ankylosing spondylitis, 41
for ILD, Q62 for inclusion body myositis, Q65
I
I
for nephritis in sLE, Q46 for osteoarthritis, Q82
in pregnancy, 16t in rheumatic disease, 15-17 \
for SLE, 50 Pleural effusions
I

for SSc, 64
Mucoplosmo spp., 17, 38
RA and.21 \It
SLE and. 45
Myelosuppression, 10, 11
Myocardial fibrosis, 63
SSc and, 64
Pleuritis, SSc and, 64 \
Myocardial infarction, 36 Poikiloderma,55, 60
Myocarditis, SLE and, 45 '1
Polyarteritis nodosa (PAN), 4t, 81 82, 821, esl
Myopathies Polyadhritis,2
differential diagnoses, s3t
drug-induced,54t
Polyarticular arthritis, 36 \
Polymyalgia rheumatica (PMR), 79, 80, Qss, Q8S )

T
Polymyositis, 54-58 \
Po rphgromonos gingiv alis, 17 I
Naproxen, Q78
Prednisone
Neisserio gonorrhoe ae, 7 4, 7 8t
adverse effects,49t
1
Neonatal lupus erlthematosus, 50 I
Neonatal onset multisystem inflammatory syndrome (NOMID), 89t
Nephrogenic systemic fibrosis, 6lt
long term use, 9
for nephritis in sLE, Q46 \
for polymyalgia rheumatica, Q55
Neutrophilic dermatoses, 20
Nintedanib.64
for SLE, 44, Q75
Pregabalin, 10, 32, Q7
\l
Nonbiologic disease modirying antirheumatic drugs, lO-t2, 22
Noninflammatory pain, l, lt
Pregnancy !
medications in, 15, 16t I
Nonselective COX inhibitors, 9t
Nonsteroidal antiinJlammatory drugs (NSAIDs), S
preconception planning, Q88
RA in,23 \
adverse effects. 49t i
RA treatment during, Q57
for anlgilosing spondylitis, Q78
for osteoarthritis, 29, Q24, Q54
SLE and, 50 \I
SLE flares in, Q95
for parvovirus B19 arthropathy, Q53
in pregnancy, 16t
RA management and, 22
SSc in.65
Primary angiitis ofthe central nervous system (PACNS),4t. 82-83
i:
Probenecid, 161, 7l 1
for reactive arthritis, Q67 Prosthetic ioint infections, 77, Q74 I
topical,9 Proteinuria, 36, 44, 45, 50
toxicities, 9t Proton pump inhibitors, 29
Pse udomonos oeruginoso, 78t :
0
Obesity, osteoarthritis and, 23 Psoriasis, Q18, Q58 \
Occupational therapy, l5 17 Psoriatic arthritis. 36 37
Oligoarthritis, fearures of, 2 dermatologic manifestations of, 3t
Opiates in pregnancy, 16t description ol 2 I
Opioids for osteoarthritis, 29 features ol 34t
gouty arthritis in patients with, Q18
Optic neuritis, 87 l
Oral ulcers,38,87
joint pain in, Q58
Osteoarthritis (OA) radiologic flndings, 7t
classification of. 24-25 risk factors,36
sex and. 36
i
diagnosisof,25 27,Q2O
differential diagnosis of, 27-28 subtypes, Q3 \
epidemiolory, 23-25 treatment of, 42,42t
Pulmonary arterial hypertension (PAH), 64, 651, Q69 \I
erosive, 24-25
hand,24f,27f Pulmonary veno occlusive disease, 64
joint pain in, Q58 Pyoderma gangrenosum, 20 \
knee,27f, Q34 Pyrin mutations, 89
management of 28 30
NSAIDs for. 29, Q54 o :
pathophysiolory ol 23 Quinacrine,50
primary 24 \
radiologic findings, 7t R I
risk factors,23 25 Radiography
secondary 25,251, Q96 in calcium pyrophosphate deposition disease, 71 :

in gout, 69
treatment of, 281, Q24, Q82
Osteogenesis imperfecta, 92 in infectious arthritis. 73 l
Osteonecrosis, 7, 44, Q25 in osteoarthritis, 27
Osteoporosis, drug-induced, Q41 in rheumatoid arthritis, 19, Q83, Q90
in rheumatologic diseases, 5-6, 7t
P in sarcoidosis. 90f
Pain in spondyloarthritis, 39, Q94
inflammatory vs. noninflammatory I Raynaud phenomenon, 44, 46, 62 63
pathway modulators, 9-10 Reactive arthritis, 3t, 34t, 38, 42, Q67
soft tissue abnormalities and. 2 Receptor activator of nuclear factor kappa B ligand (RANKL), 20
p ANCA antibody,6t Relapsing polychondritis, 3t, 88-89, 88f, Q92
Panuveitis,87 Renal amyloidosis, Q72
Parvovirus 819, 17,76, Q53
Patient history
Repetitive motion, OA and, 23
Retiform purpura,82f
\:
musculoskeletal examination and, 1, Q2o Retinal vasculitis. 87
Pegloticase, 15, 161,71, Q61 Rheumatic disease ..

Pencil-in-cup deformity, 40f MMR vaccine in, Q43


Pericarditis, SLE and, 45, Q91 physical therapy,15 u
Periodontal disease, RA and, 17 therapeutics, S 17 ;

l
178
\
lndex

Rheumatic fever. 3t ocular manifestations of, 3t


Rheumatoid arthritis (RA) pathophysiolos/,32 33
approach to, 22f Staphylococcus oureus, 74
classification criteria, 18t Staphylococcus epidermidis, 74 , Q74
diagnosis of, 18-20, Q32, Q37, Q44 Statins, myopathy induced by, 54t
evaluation of, Q83 Stiffness, morning,2
hands, 18f Still disease, adult-onset, 3t, 90, Q39
imaging studies, 19-20, Q90 Subacute cutaneous lupus erythematosus (SCLE), 43, Q84
inflammatory cascade in, 12f Sulfasaiazine, 10, 111, 161, 42
internal organ involvement in,4t Surgery
interstitial lung disease in, Q17 ioint replacement, Q50
laboratory studies, 19, Q70 for osteoarthritis, 30
management of, 21-23 for RA, 23
monoarticular pain, Q87 Sweet syndrome, 20
ocular manifestations o[, 3t Synovial fluid, 1, 8, 8t, 73
patholory of, 17 Synovitis, 7, 20
radiologic findings, 7t Systemic lupus erythematosus (SLE)
risk factors, 17, Q9 autoantibodies in, 48t
treatment during pregnancy, Q57 cardiovascular involvement. 45
treatment of, Q36 ciassiflcation of. 47t
Rheumatoid factor, 5, 61, 19, Q44, Q7o clinical manifestations, 43-46
Rheumatoid nodules. 21f dermatologic manifestations of, 3t
Rheumatologic disease diagnosis ol 46 48
extra-articular features of 2 epidemiolog/ of, 42 43
laboratory studies in, 2 5 flares in pregnancy, Q95
Rilonacept, 14t,16t hematologic involvement, 45 46
Rituximab internal organ involvement in, 4t
for AAV 84, Q33 ioint pain in, Q25
characteristics of, 14t kidney involvement, 44 45
for IgG4 related diseases, 92, 92t, Q68 lupus nephritis, Q77
for IIM. 59 medications tirr. 49t
in pregnancy, 16t musculoskeletal in\,0lvement, 44
for RA. 22 nephritis in, Q46
for SLE. 50 neuropsychiatric involvement, 45
for SSc. 64 65 ocular manifestations of. 3t
Rotator cufftendon repair, 17 osteonecrosis and. 4'l
Rubella, T6 pathophysiolog/ of l2-43
patient assessment, Q19
5 pericarditis in, Q91
Sacroiliac joints, 391 pregnancy and, Q88
Sacroiliitis, 37, 39f prognosis, 51
Saliva, artificial, 52 pulmonary involvement. 45
Solmonello,38 subcutaneous. 43f
Sarcoidosis. 3t, 4t, 90 92, gof, Q71 treatment of, Q75
Sariiumab, 14t, 16t Systemic sclerosis (SSc), 59 65
Sausage digits, 2 capillary loops in. 63f
Sclerederma, 61t classiflcation ln.60t
Scleritis, 21 cutaneous, Q59
Scleroderma spectrum disorders, 611, 63 64, Q15 digital pitting. 63f
Scleromyxedema, 61t internal organ involvement in, 4t
Seagull erosions, 24, 24f interstitial lung disease in, Q62
Secukinumab, l4t, l6t manifestations of, 3t, 60-64
Sensory neuropathy, 25t treatment of, 65t
Serotonin-norepinephrine reuptake inhibitors (SNRIs), Systemic sclerosis sine scleroderma, 60, 61t
10,32 Systemic vasculitis. 3t, 78-87
Shawl sign, 56f
Shigello,38 T
Sicca, management of, 52 Tacrolimus, fbr llM, 59
Sjogren syndrome Tai chi, 17, Q34
diagnosis of, Q60 Takayasu arteritis, 81.811. Q22
epidemiolory, Sl Telangiectasias, mat like. 60
Iymphomas in,51, Q79 Tendonitis, ultrasonography of, 7
ocular manifestations of, 3t, 21 Thalidomide for Behqet syndrome, 88
oral dryness in, Q26 Thrombocytopenia. SLE and. 45
prognosis,52 53 Tissue biopsies, 8
Skin Tocilizumab, 141, 161, 22. 88
rheumatoid arthritis, 20 Toes
rheumatologic disease and, 2 dactylitis, 37f
Sleep hygiene, 9 gout,68,68f
Small vessel cutaneous vasculitis, 21 Tofacitinib,
11t, 16t
Smoking, inflammation and, u Tophaceous gout, Q21, Qbl
Smoking cessation, Q9 Tophi,68
Soft-tissue abnormalities, 2 Totalioint arthroplasty. 17
Soft tissue iniuries, mechanical, Q42 Tramadol, 10, 161. 29. 32
Spine, ossification ol 35 TRAPS (tumor necrosis factor receptor-associated periodic fever
Spondyloarthritis, 2 syndrome), 89,891
assessment of 35t Tricyclic antidepressants, 32
features of. 34t Tumor necrosis factor inhibitors, 13, 13t, 161, 41, 42t, Q14
HLA-B27 testing, QS
imaging studies, 39-41 U
laboratory studies in, 38 39 Ulcerations in Behqet syndrome, Q1
MRI.7 Ultrasonography, 7. 79 20, 69

179
lndex

Undifferentiated connective tissue disease (ICTD), 66t Viral infections, T6-z


Upadacitinib, 111, 16t Vision loss, 2
Urate-lowering therapy, 13-15, 70, Q52 Vitamin D, intake of 50
Uric acid, gout and, 67 Voclosporin, Ut
Uricosuric agents, 15, 71
Ustekinumab, 141, 16t w
Uveitis, treatment of, 41 Weight loss for osteoarthritis, Q82

U
x
Vaccination in immunosuppression, 15 Xanthine oxidase inhibitors, 70-71
Vasculitis Xerostoma, medication-induced, Q47
derrnatologic manifestations of, 3t Y
differential diagnoses, 79t Yamaguchi criteria, 90
hypersensitivity, 9 Yersinio spp., 38
ocular manifestations of, 3t Yoga,77
RA and,21
secondary 79t z
skin manifestations, 20 Zidorudine, myopathy induced by, 54t
Vertebral fractures, 36 Zika virus.76

:
180 a

i
i

You might also like